Dies ist ein Archiv der Seite Wikipedia:Auskunft. Es enthält alle Abschnitte, die in der Kalender-Woche 15 im Jahr 2016 begonnen wurden.

Möchtest du in einer neuen Diskussion auf dieses Archiv verweisen? Nutze folgenden Link und ersetze ________ durch den Namen des Abschnittes. Die Nummer im Inhaltsverzeichnis gehört nicht dazu:

[[Wikipedia:Auskunft/Archiv/2016/Woche 15#________]]


← vorherige Woche Gesamtarchiv nächste Woche →

Weniger Bandbreite mit Repeater

Beim Einsatz eines WLAN-Repeaters sinkt ja, wie man oft liest, die zur Verfügung stehende Bandbreite um die Hälfte. Gilt das nur für die Geräte, die mit dem Repeater verbunden sind? Ich probiere das gerade aus und habe bei einer VDSL50-Leitung der Telekom beim Speedtest der ComputerBild eine Downloadrate von rund 48.000 kbit/s, wenn der Repeater eingeschaltet ist, sich der Computer aber direkt mit dem Speedport verbindet. Wenn ich den Computer aber mit dem Repeater verbinde, sinkt der Download auf rund 13.000 kbit/s. Kann ich also davon ausgehen, dass die reduzierte Geschwindigkeit nur für die Geräte gilt, die mit dem Repeater verbunden sind? --2003:76:E5B:41C3:54E3:D9AD:96D7:BAF0 00:59, 11. Apr. 2016 (CEST)

Es kommt auf das (W)LAN-interne MAC-basierte Routing an. Wenn der Repeater anhand der MAC-Adressen der Netzwerkpakete erkennt, dass die Pakete nicht von bzw für einen seiner nachgeschalteten Clients sind, dann repetiert er sie nicht. Es werden also nur Pakete verdoppelt, die ohne Repeater nicht ankommen würden. --Rôtkæppchen₆₈ 01:04, 11. Apr. 2016 (CEST)
Das heißt also für den Laien ausgedrückt, die Bandbreite wird nur dann reduziert, wenn Geräte mit dem Repeater verbunden sind und dort ein Datenaustausch stattfindet? --2003:76:E5B:41C3:54E3:D9AD:96D7:BAF0 01:10, 11. Apr. 2016 (CEST)
Dann wird aber auch die für den Rest des WLANs zur Verfügung stehende Bandbreite halbiert, denn in der Zeit, in der der Repeater die Pakete verdoppelt, können direkt mit dem Accesspoint verbundene Geräte auch nicht senden. --Rôtkæppchen₆₈ 01:17, 11. Apr. 2016 (CEST)
Ok, ich denke, dann habe ich es kapiert. Ich danke Dir. --2003:76:E5B:41C3:54E3:D9AD:96D7:BAF0 01:19, 11. Apr. 2016 (CEST)

Google-Karte in VIAF-Suche

Um die Suche von Virtual International Authority File (VIAF) unter https://viaf.org/ benutzen zu können muß man Java-Skript aktiviert haben und die Google-Weltkarte mit Standorten von Bibliotheken anzeigen lassen. M.E. ist diese Weltkarte an dieser Stelle nicht hilfreich, optisch störend und überflüssig.

Handelt es sich um versteckte Schleichwerbung von Google in der vom offiziel gemeinnützigen Online Computer Library Center betriebenen VIAF-Datenbank? Rosenkohl (Diskussion) 01:19, 11. Apr. 2016 (CEST)

Archivierung dieses Abschnittes wurde gewünscht von: Keine Frage Benutzer:Eingangskontrolle Schlagt den Boten und die Welt wird besser 18:04, 12. Apr. 2016 (CEST)

Der Prozess useractivityd wurde unerwartet beendet.

Nie von diesem Prozess gehört. Wofür ist er und wie werde ich diesen dreck auf dem mac los? Ich habe einen Spyware scanner mal über den mac laufen gelassen, der hat aber nur conduit und ein paar tracking cookies gefunden, kein useractivityd. Der dumme Name hört sich an wie ein Programm dass die "User activity" überwachen soll?--Ip80.123 (Diskussion) 05:44, 11. Apr. 2016 (CEST)

Ah, du hast nie davon gehört, aber du weißt sofort, daß es Dreck ist und einen dummen Namen hat … Ich bin auch kein Maxperte, aber auf -d enden in der unixoiden Welt üblicherweise Systemdienste (daemons). --Kreuzschnabel 08:56, 11. Apr. 2016 (CEST)
[1] -- Janka (Diskussion) 11:14, 11. Apr. 2016 (CEST)
Wenn dir von irgendeinem Prozess der Name nicht gefällt: killall -9 irgendeinprozess --Eike (Diskussion) 11:33, 11. Apr. 2016 (CEST)
Unter Mac OS X können Prozesse im Activity Monitor, Button links oben "Force a process to quit" deletiert werden. Wahrscheinlich aber auch mit Eikes Command im Terminal-Fenster. Kann natürlich in die Hose gehen wenn ein Systemdienst / etc :o( --Cookatoo.ergo.ZooM (Diskussion) 12:00, 11. Apr. 2016 (CEST)

Scannen mit KonicaMinolta magicolor 1690MF

Ich kann drucken, ich habe den offiziellen Scannertreiber installiert, aber es wird in den Systemeinstellungen kein Scanner angezeigt und im Scanprogramm kein Scanner zur Auswahl angeboten.

Ich dachte, dieses Gerät hat SANE und deswegen testweise ein telnet auf Port 6566 gemacht. Ich erhalte Connection refused. Auf die Ports 80, 515 und 9100 kann ich problemlos zugreifen.

--° (Gradzeichen) 10:03, 11. Apr. 2016 (CEST)

[2] [3] [4] -- Janka (Diskussion) 11:53, 11. Apr. 2016 (CEST)

Das Problem damit ist, dass ich - da ich auf die Schnelle nichts gegenteiliges gefunden habe - davon ausgehen muss, dass dies eine Umsetzung von Windows-Protokoll nach SANE ist (was auch erklären würde, warum ich keine Verbindung auf 6566 bekomme - langfristig kann das eine Lösung sein, aber leider nicht sofort. Kurzfristig wäre es toll, den Herstellereigenen Treiber (der installiert ist, aber nicht zur Anzeige des KonMIn in den Systemeinstellungen führt) zum Laufen zu bekommen, nur welches Protokoll/Port verwendet der? (auf 9100 ist ein "RAW"-Zugang, aber was ist das?) --° (Gradzeichen) 12:57, 11. Apr. 2016 (CEST)
Was willst du nun erreichen? Dass dein Scanner mit SANE funktioniert? Dann gehe so wie auf der ersten verlinkten Seite unter dem Punkt "HOW TO SCAN" beschrieben vor. Der Scanner funktioniert mit dem magicolor-backend sowohl per USB als auch per IP. Welchen Port der Scanner in seinem nativen Protokoll benützt steht auf der dritten verlinkten Seite, 4567/tcp. Und das native Protokoll ist dort auch beschrieben. Port 9100 hat damit gar nichts zu tun, das ist die Druckfunktion in der HPJetDirect-Printerserver-Emulation (einfach Druckdaten im Binärformat mittels netcat, socat o.ä. an diesen Port schicken und schon druckt er). -- Janka (Diskussion) 14:23, 11. Apr. 2016 (CEST)

Der offizielle Treiber geht immer noch nicht, aber mit xsane geht es nun. (erl.) --° (Gradzeichen) 14:28, 11. Apr. 2016 (CEST)

Warum Kopenhagen?

Bis auf USA wird so ziemlich jede geografische Bezeichnung eingedeutscht. Warum sagen wir nicht Kopenhafen, zumal die Dänen es fasst genau so nennen - København?--Wikiseidank (Diskussion) 07:22, 11. Apr. 2016 (CEST)

Auch wenns blöd klingt: Das haben wir schon immer so gemacht. --Benutzer:Eingangskontrolle Schlagt den Boten und die Welt wird besser 08:45, 11. Apr. 2016 (CEST)
Das kannst Du eigentlich im Artikel nachlesen. Linkhilfe: Kopenhagen#Ortsname und Exonym_und_Endonym#Exonym --mw (Diskussion) 09:05, 11. Apr. 2016 (CEST)
Hab ich doch, deswegen ja. "Heißt Hafen, die deutschen sagen Hagen", wohl nicht, weil die Deutschen das Wort Hafen nicht kennen/anders verwenden?--Wikiseidank (Diskussion) 10:38, 11. Apr. 2016 (CEST)
ein Hagen ist ein mit einem Zaun, einer Hecke o.ä. eingefriedeter, also befestigter Ort. Durchaus was anderes als Hafen. Vielleicht kamen die Teutonen eher von der Landseite aus angerückt...? --Edith Wahr (Diskussion) 11:00, 11. Apr. 2016 (CEST)
Erstmal kling 'havn' nicht wie deutsch 'Hafen' sondern eher wie hawn. Hör dir die Sprachdatei an. OHne Ahnung der dänischen Sprache kann man da durchaus ein Köpenhagen raushören. Das Deutsche Exonym kommt jedenfalls aus dem Niederdeutschen von den hansischen Kaufleuten. Aber einen genauen Grund wieso aus -havn dann -hagen wurde, weiß keiner. Da gibt es keine befriedigende Antwort die alles eindeutig klärt. --2A02:2028:640:3101:74BD:A6FE:99DC:B747 11:07, 11. Apr. 2016 (CEST)
vielleicht ging es den hansischen Kaufleuten ja wie mir, ich zumindest will mir, wenn Dänisch gesprochen wird, oder auch nur Deutsch oder Englisch mit dänischem Akzent, am liebsten die Ohren zuhalten (oder davonlaufen, oder meinem Gegenüber die Zunge mit einer rostigen Machete herausschneiden). Jedenfalls ist die Beobachtung schon ganz richtig: auch in dieser Schwarte wird festgestellt, dass da eine "Umwandlung" vonstatten ging und es also eigentlich Kopenhaven/-hafen heißen müsste und ein -hagen was ganz anderes ist. Das warum lässt sich aber wohl nicht mehr klären. --Edith Wahr (Diskussion) 11:27, 11. Apr. 2016 (CEST)
Und der erste Satz ist schon mal falsch. Es wurden (mit Betonung auf der Vergangenheit) nur ganz wenige Orte in wenigen Ländern eingedeutscht. In Frankreich praktisch keiner, außer Elsass und teilweise in der Aussprache. --195.200.70.39 11:17, 11. Apr. 2016 (CEST)
In Norddeutschland wird Kopenhagen wie Kobmhaagn mit verschlucktem b und g ausgesprochen. Als das irgendwann mal aufgeschrieben wurde, muss der Schreiberling angenommen haben, der Sprecher hätte gerade eine Kartoffel im Mund und meinte Kopenhagen. Ähnlich gelagert: Apenrade vs. Åbenrå vs. Affenråe. -- Janka (Diskussion) 11:35, 11. Apr. 2016 (CEST)
Der erste Satz meinte: Rom/Roma, Prag/Praha, Nizza/Nice, Brüssel/Bruxelles, Havanna/La Habana, Peking/Beijing usw.--Wikiseidank (Diskussion) 13:01, 11. Apr. 2016 (CEST)
Bei Peking/Beijing hat sich glaub ich nur die Transliteration geändert. --Eike (Diskussion) 13:04, 11. Apr. 2016 (CEST)
(Einschub) Naja, so richtige Parallelen zu Kopenhagen-/hafen bestehen da aber nicht, hat doch keiner der Namen einen Bestandteil, der per Übersetzung (wie havnHafen) ins Deutsche übertragbar wäre. Ähnliches gilt auch für Warschau, Bukarest, Moskau usw. Da ist jeweils lediglich die Aussprache (und entsprechend die Schreibweise) der deutschen Zunge angepasst worden. --Gretarsson (Diskussion) 15:19, 11. Apr. 2016 (CEST)
Peking und Beijing sollen regionale Aussprachevarianten desselben Zeichens sein, ebenso Birma, Burma und Myanmar. --Rôtkæppchen₆₈ 14:18, 11. Apr. 2016 (CEST)
Quetsch - Peking war m.E. einfach eine unwissenschaftliche grob-akustische Verwestlichung. Allerdings ist das erste B von "Beijing" in der Tat kein richtiges B gemaess der Aussprache hierzulande, sondern ein unaspiriertes p, das aber vom im chinesischen existierenden "richtigen" P zu unterscheiden ist. Hingegen kann ich beim J in "Beijing" in der Aussprache nichts finden, was lautlich mit dem k zu tun hat. Das ist wohl einfach, wie vorher beschrieben, grob-akustisch. (Siehe auch Sinkiang, Tsingtau etc.). -- 83.167.34.67 17:34, 11. Apr. 2016 (CEST)
jing wurde historisch während der Tang-Dynastie/Song-Dynastie als giæng gesprochen, was sich noch in den südchinesischen Dialekten z.B. Kantonesisch ging erhalten hat (bzw. auch sino-japanisch kin oder sino-vietnamisch kinh). --Mps、かみまみたDisk. 20:16, 11. Apr. 2016 (CEST)
Hier gibt es noch drei Antworten zu der Frage (dort bitte bis ganz unten lesen). In da:Københavns navn wird weiter ausgeführt, dass die Ersterwähnung von 1043 (als Höfn) aus der Knytlinga saga stamme. Da letztere erst um 1260 schriftlich fixiert wurde, hat die dortige Erwähnung aber die Schriftform von um 1260 oder noch später, falls kein Original, sondern nur spätere Abschriften überliefert wären. Die Ersterwähnung Wismars (als Vizmar Höfn) in derselben Saga gilt laut Geschichte der Hansestadt Wismar#Namensgeschichte als sagenhaft. --Pp.paul.4 (Diskussion) 15:07, 11. Apr. 2016 (CEST)
(BK)Interessant sind hier die deutsche und niederländische Bezeichnung. Im Niederländischen lautet der Ort auch Kopenhagen, obwohl auch hier dasselbe zutrifft wie für das Deutsche: "De naam Kopenhagen gaat terug op Købmandshavn, dat "Koopmanshaven" betekent." Die Nähe ist hier sogar noch größer, da der erste Namensbestandteil auch noch gedeutet werden könnte: kopen = kaufen.
Schaut man sich die mittelniederdeutschen Quellen an (es ist naheliegend, daß der Name aus dem Niederdeutschen vermittelt wurde) findet sich auch die Schreibweise "Kopenhaven", vgl. z. B. aus einem Hanserezess von 1477: Item Schelme [Personenname] hefft genomen und to Kopenhavene geforet sodane twee grote tunne talges [...]" (Hanserezesse, III,1, S. 38). 1513 schreibt ein Kaufmann wiederum: "Alse gy uns in juwen vorbreven screven van dem herendaghe in Kopenhagen" (Haserezesse III, 6, S. 446). Zeitlich ist das also alles sehr nah.
Kommen wir zunächst zum Wort "Hafen". Es wurde aus dem Niederdeutschen entlehnt. In dieser Gestalt ist das Wort im Hochdeutschen seit dem 16. Jh. üblich, mittelhochdeutsch lautete es "hap/habe". Die Ähnlichkeit zu dän. havn war damit zu dieser Zeit im Hochdeutschen nicht unbedingt ersichtlich.
Die mittelniederdeutschen Quellen belegen nun beides: Kopenhaven und Kopenhagen. Hierüber kann man nun spekulieren. Was wohl angenommen werden kann, ist eine Verdunklung der Namensbedeutung, denn sonst hieße es ja "Kopmannshaven". Ein Erklärungsansatz wäre folgender - allerdings ist das pure TF meinerseits: Im östlichen Nordniedersächsischen trat schon zu mittelniederdeutscher Zeit der Wandel /w/ zu /g/ ein, besonders bei /uw/, Agathe Lasch nennt in ihrer Mittelhochdeutschen Grammatik (S. 185) u. a. huswrughe frau, Garz 1410, [...] de grage monnyck [= der graue Mönch] Lüneburg 16. Jh. Spätere belege für grâge (Schlesw.-Holst. 1627, Mecklenburg 1732) usw. sind häufig. Pâgel > Pâuwel [= Paul], Klâges > Clâuwes [= Klaus]; pagelûn pfau im 17. jh. bei N. Gryse." Im Mecklenburgischen heute noch üblich sind z. B. grag 'grau', blag 'blau' und eben Pageluun 'Pfau' (dekliniert: mnd. blawe - mecklb. blage, vgl. grawe vs. grage, vgl. Schiller/Lübben, Bd. 1, S. 349, der als Formen aufzählt: bla, blae, blaw, blawe, blauwe). Teuchert zählt für das Mecklenburgische weiterhin auf (Jahrbuch des Vereins für nd. Sprachf. 82 [1959], S. 211): trugen 'trauen', sick grugen 'sich grauen', Frug 'Frau', jug 'euch' und zitiert Laschs Stralsunder Beispiel "pagellunshern (1562) Herren der Pawelunes broderschop". Allerdings stammen viele Belege aus dem 17. Jh., daneben finden sich selbst zu dieser Zeit auch noch Formen mit /w/ bzw. /v/. Da die frühesten Belege jedoch bis ins 14. Jh. hineinreichen, wäre der Wandel -haven > -hagen durchaus im Bereich des Möglichen, zumal, wenn die Namensbedeutung zunehmend verblaßte. Auch im Dänischen änderte sich der Name ja. Problem wäre hier nur, wie die niederländische Form zustande gekommen ist.--IP-Los (Diskussion) 15:13, 11. Apr. 2016 (CEST)
Bemerkenswert ist auch die Ähnlichkeit von dän. havn für "Hafen" und dän. have, was ursprünglich dem deutschen Hag oder Hagen (= eingefriedeter Bereich) entsprach und heute die dänische Bezeichnung für "Garten" ist (während sich das deutsche Wort "Garten" wohl aus gard = Hof, Einflussbereich entwickelte). Es gibt auch heute noch das dänische Wort "flyve" für dt. "fliegen", was vom Alt-Dänischen "flyghæ" abstammen soll. - TF1: Der Ursprüngliche Dänische Name war im ursprünglichen Niederdeutsch ein "falscher Freund". - TF2: Die damalige Einfriedung (Siedlung) stand auch für die Dänen anfänglich im Vordergrund und wurde erst mit der Wandlung von g zu v auf den Hafen übertragen. --Optimum (Diskussion) 18:11, 11. Apr. 2016 (CEST)

Youtube Lautstärke

Seit ein paar Wochen starten Youtube-Videos bei mir immer mit voller Lautstärke auf dem Laptop. Ich benutze Firefox (alle updates) und Kubuntu 14.04. Bei Videos von anderen Seiten passiert das nicht. Hat jemand von euch eine Ahnung warum das so ist/wie man das abstellen kann? Welche Lautstärke ich vor dem Aufruf einstelle macht keinen Unterschied. Danke für jeden Tipp. --MathiasNest (Diskussion) 16:16, 13. Apr. 2016 (CEST)

Werden die Videos mit dem Flash-Player oder mit HTML5 abgespielt? Jedenfalls werden diese Voreinstellungen in Cookies oder ggf. in Flash-Cookies gespeichert. Wenn die nicht mehr zugelassen sind, dann kommt es zu solchen Problemen. --87.123.50.238 17:10, 13. Apr. 2016 (CEST)
Hmm, cookies hab ich erlaubt, aber das Problem besteht auch, wenn ich mehrere Videos direkt nacheinander kucke. Der Lautstärkeregler springt jedesmal wieder auf 100%. Woran erkenn ich denn, ob es der flash oder html5-player ist?--MathiasNest (Diskussion) 17:39, 13. Apr. 2016 (CEST)
Bei Youtube sieht man das am Kontextmenü (Maus über das Video halten und rechte Maustaste drücken.) Wünsche noch einen schönen Tag. Gruß 217.251.198.253 17:43, 13. Apr. 2016 (CEST)
Also ist es der html5-player, aber das hilft mir noch nicht weiter ... --MathiasNest (Diskussion) 18:26, 13. Apr. 2016 (CEST)
Das heißt, dass es an Flash-Cookies nicht liegen kann.
Lässt du Cookies nicht nur von youtube.com, sondern auch von den anderen Seiten, die youtube.com nutzt, zu? --87.123.50.238 19:54, 13. Apr. 2016 (CEST)
Ja, ich lass alle Cookies zu. Nach einem kleinen Test gerade eben scheint es so zu sein, dass alle Webseiten, die html5 benutzen mit voller Lautstärke loslegen, auch bei mehreren Videos nacheinander, während sites die flash benutzen, mit normaler Lautstärke anfangen. Seltsam ... --MathiasNest (Diskussion) 20:23, 13. Apr. 2016 (CEST)
Tritt das seit einem spezifischen Update von Firefox auf? Vielleicht haben die was am HTML5-Abspieler geändert. --87.148.90.191 21:10, 13. Apr. 2016 (CEST)
Scheint ein Firefox Bug zu sein. An einer Lösung wird wohl nicht gearbeitet. Workaround 1 (Javascript), Workaround 2 (Firefox config). Gruß, --Nfreaker91 21:48, 13. Apr. 2016 (CEST)
Hey, workaround2 hat funktioniert! Danke! --MathiasNest (Diskussion) 22:28, 13. Apr. 2016 (CEST)
Gern geschehen. Gruß, --Nfreaker91 22:34, 13. Apr. 2016 (CEST)
Archivierung dieses Abschnittes wurde gewünscht von: Gruß, --Nfreaker91 22:34, 13. Apr. 2016 (CEST)

Wartungsmission Weltraumteleskop?

Das Keplerteleskop ist offenbar defekt. Gibt es keine Möglichkeit dahin zu fliegen und das zu reparieren? Angenommen Orion (Raumschiff) wäre einsatzbereit, könnte man es nutzen um Kepler zu erreichen und wieder zur Erde zurückzukommen? Gibt es ein anderes Raumfahrzeug das dies könnte? --89.144.206.96 14:27, 11. Apr. 2016 (CEST) --89.144.206.96 14:27, 11. Apr. 2016 (CEST)

Die Space Shuttles damals haben Hubble eingefangen und in funktionstuechtigen Zustand gebracht, nachdem es nur partiell funktionstuechtig im Weltraum ankam. --192.91.60.10 15:37, 11. Apr. 2016 (CEST)
Ja, das Hubble-Weltraumteleskop wurde fünf Mal von Space Shuttles besucht. Beim Kepler geht das nicht. Dieses Teleskop befindet sich nicht in der Erdumlaufbahn, sondern in einer Bahn um die Sonne, genau wie die Erde selbst. Um zum Kepler zu kommen, muss ein Raumflugkörper erst einmal den Bereich der Erdanziehungskraft verlassen, also Fluchtgeschwindigkeit erreichen. Für unbemannte Raumsonden ist das mit entsprechendem Aufwand möglich, für bemannte Raumschiffe gibt es derzeit keine Rakete, die das leistet. Eine Reparatur vor Ort und eine Rückkehr zur Erde macht die Sache so komplex wie einen Flug zum Mars (ohne Landung). Also: keine Chance. --Asdert (Diskussion) 15:42, 11. Apr. 2016 (CEST)
Es dauert nicht mehr lange bis der Betrieb des James Webb Space Telescope verwirklicht wird - wenn alles gut geht. Damit bekommen die Astronomen ein ganz neues "Superspielzeug" welches der Astronomie einen gewaltigen Sprung verspricht. Ist zwar kein Ersatz für die spezielle Funktion des Keplerteleskops aber selbiges hat genau genommen seine geplante Aufgabe (bis 2012 Planeten suchen) längst glänzend erfüllt. --Kharon 22:28, 11. Apr. 2016 (CEST)
Geht wieder [5] :D --Kharon 08:46, 12. Apr. 2016 (CEST)
na ja ;) --just aLuser (Diskussion) 09:01, 12. Apr. 2016 (CEST)

Wie hießen die Leute die vor den Aborigines in Australien gelebt haben?

Ich interessiere mich vor allem für die Felszeichnungen die diese hinterlassen haben. Kann nur nicht danach googeln weil ich die Bezeichnung für diese Kultur nicht kenne. --2003:66:8936:4C51:2CEF:7AE2:7372:22A8 09:56, 12. Apr. 2016 (CEST)

Soweit ich weiß, stammen die Felszeichnungen von den Aborigines. Also von denen, die damals da gelebt haben - mithin den Vorfahren der heutigen Aborigines. --Snevern 10:08, 12. Apr. 2016 (CEST)
Aborigines, Felsmalerei, Petroglyphe. --Vsop (Diskussion) 10:16, 12. Apr. 2016 (CEST)
Wie oben, die Aborigines waren die ersten (siehe Mungo Man).
Aber du könntest nachsehen, welcher der Stämme der Aborigines was gemalt hat (sie hatten voneinander unterschiedliche Vorlieben für Symbole). Also Suche mit => Stammesname rock painting <= o.ä. Play It Again, SPAM (Diskussion) 10:25, 12. Apr. 2016 (CEST)
Ich habe einen Journalisten von National Geographics in einem Gespräch locker drüber sprechen hören. Der hat erwähnt das von einer Vorkultur der Aborigines Bilder in Australien auf Felsen sind die so alt sind das man nur noch die Schatten sieht wo einst die Bilder waren. Die Datierung schätzt die Bilder auf mindestens 30.000 Jahre eventuell aber auch deutlich älter. --2003:66:8936:4C51:2CEF:7AE2:7372:22A8 10:38, 12. Apr. 2016 (CEST)
"Aborigines" ist keine einheitliche Ethnie, sondern eine (deutsche) Sammelbezeichnung für alle indigenen Populationen vor der Ankunft der Europäer. Die ältesten Funde sind ca. 40.000 Jahre alt (Mungo Man im Mungo National Park). Die ältesten datierten Felszeichnungen stammen aus Arnhem Land von der Nawarla Gabarnmung-Fundstelle. Das Gebiet wird heute von den Jawoyn besiedelt, aus deren Sprache auch die Bezeichnung der Fundstelle stammt.--Gonzo.Lubitsch (Diskussion) 11:05, 12. Apr. 2016 (CEST)
Es gab – vor den Europäern – nur eine Besiedlungswelle vor mindestens 40.000 Jahren. Natürlich haben sich die Menschen in unterschiedliche Stämme aufgeteilt und im Laufe der Zeit unterschiedliche Kulturepochen enwickelt. Aber alle sind relativ nah miteinander verwandt. Rainer Z ... 12:16, 12. Apr. 2016 (CEST)
Es gibt Hinweise auf Kontakte der australischen Ureinwohner mit Bewohnern Südasiens vor etwa 4 bis 6tausend Jahren, Indizien sind eine neue Steinwerkzeug-Kultur, die Einführung des Dingo und (begrenzte) genetische Daten für Intogression. Neuere Daten, etwa zum Genom des Y-Chromosoms, zeigen aber klar, dass Australien (oder genauer, der Urkontinent Sahul, unter Einschluss von Neuguinea) vor Ankunft der Europäer nur einmal von Menschen besiedelt wurde, vor etwa 50.000 Jahren. Alle Felszeichnungen stammen also von den direkten Vorfahen der heutigen Ureinwohner.--Meloe (Diskussion) 15:16, 12. Apr. 2016 (CEST)

Stierkampf

In Spanien ist wieder mal ein Stierkämpfer in den Hintern gespickt worden.

Sind die Stierkämpfe nicht vor einiger Zeit eingestellt worden? Es soll einen letzen Stierkampf gegeben haben. Wieso gibt es jetzt erneut Stierkämpfe? Jubiläumsausgaben? 88.73.188.233 18:57, 11. Apr. 2016 (CEST)

Genau lesen, was die Presse schreibt! Meist heißt es dann "der wahrscheinlich letzte ..." oder so. --Heletz (Diskussion) 19:18, 11. Apr. 2016 (CEST)
Es gibt Regionen in Spanien, in denen kein Stierkampf mehr stattfindet.
Das zitierte Beispiel zeigt, dass der Stier zumindest eine Chance hatte (ich stelle es mir schmerzhaft vor, in den Tuches gepiekst und dann auch noch hochgehoben zu werden...). Wer kennt ein Beispiel, wo Ähnliches (die Kreatur gewinnt) in einem Schlachthaus passiert ist? 213.169.163.106 19:30, 11. Apr. 2016 (CEST)
In Katalonien herrscht seit einigen Jahren ein Verbot. In Rest-Spanien wurde nichts eingestellt. --Blutgretchen (Diskussion) 19:36, 11. Apr. 2016 (CEST)
Nein, der Stier hat keine Chance, denn er stirbt auch, wenn er gewinnt. --Snevern 20:00, 11. Apr. 2016 (CEST)
In Schlachthöfen kommt es gelegentlich vor, dass ein Schwein ausbüxt und es eine Weile dauert, bis man es wieder eingefangen hat. Auch die Schweine (und Rinder) haben aber keine Chance ("Möge der bessere gewinnen!"), sondern sterben nur ein bisschen später (oder - sehr viel häufiger - deutlich früher, zum Beispiel auf dem Transport). --Snevern 20:06, 11. Apr. 2016 (CEST)
Wir alle sterben irgendwann, auch Schweine und Stiere. Die Fälle, in denen ein Kampfstier "begnadigt" wurde, sind sehr, sehr selten, aber sie sind vorgekommen. So wurde Murciélago nach seinem gewonnenen Kampf sogar zu einem Zuchtstier befördert. Anders als Schweine und Rinder im Schlachthof werden Kampfstiere in der Arena allerdings zu nichts gezwungen, was sie nicht wollen. Dass die Stiere die Menschen angreifen, weil sie nicht wissen, wie schlau und gefährlich Menschen sind, kann man nicht den Menschen zur Last legen. Geoz (Diskussion) 20:40, 11. Apr. 2016 (CEST)
Zu Murciélago sagt die englische Wikipedia was anderes. Wer hat da nun die besseren Quellen? --192.91.60.11 13:51, 12. Apr. 2016 (CEST)
Der Artikel der englischen Wikipedia hat überhaupt keine Quellen, von daher ist die Frage leicht zu beantworten.
Auf den im deutschsprachigen Artikel zitierten Text habe ich keinen Zugriff. In den beiden Artikeln über die jeweiligen Lamborghini-Modelle steht aber zum einen, der Stier sei verschenkt (und nicht verkauft) worden, zum anderen, der Stier sei von Miura gezüchtet worden (und nicht von del Val de Navarra). Unabhängig davon, was hier jeweils richtig und falsch ist (in beiden Fällen kann ja höchstens eine der zwei Darstellungen richtig sein) ist die gesamte Geschichte höchst fragwürdig: Vieles davon ist vermutlich einfach nur eine Legende. --Snevern 22:35, 12. Apr. 2016 (CEST)

Selbst wenn der Stier überleben könnte, wäre das keine Lösung, die jedem Stier offen stehen würde. Auf der anderen Seite scheint es doch etwas fairer, wenn der Stier die Gelegenheit zur Gegenwehr bekommt, auch wenn er nicht intelligent genug ist zu verstehen, gegen wen er sich wehren muss. Letztendlich ist es ein Spiel der menschlichen Fähigkeiten gegen die Kognition des Stieres. Er erkennts halt meistens nicht, von dem die Gefahr ausgeht. Und ja, Geoz hat Recht. Der Stier greift von allein an. Darin liegt ja eine Metapher des Spieles, dass ein aggressives und übermächtig starkes Wesen bösartig wird und versucht einen körperlich Schwächeren umzubringen und dann durch dessen Eleganz und Intellekt unerwartet besiegt wird. --88.73.188.233 20:43, 11. Apr. 2016 (CEST)

...und nach diesen Beiträgen wundert sich hoffentlich keiner mehr, dass der Stierkampf noch immer nicht ausgestorben ist. --Snevern 21:26, 11. Apr. 2016 (CEST)
Der Mensch an sich ist "aggressiv" und "neugierig" und "sozial" (ein Gruppenwesen). Es gibt Gruppen, die gewisse Aspekte kulturell ausleben (wollen). Der Umsatz und die Anzahl der Arbeitsplätze bezüglich Stierkampf liegen in der Grössenordnung der ThyssenKrupp Metallurgie GmbH (Katalonien hat mehr und andere Einkommensmöglichkeiten als die südlicheren Regionen). Die Sterberate von Profi-Wrestlern liegt weit über dem Normalwert - aber die Tode sind nicht so spektakulär wie beim Stierkampf - sie schleichen eher dahin (abgehalfterte Heroen, die irgendwann umfallen). Wie die Hunderttausende zu Tode geliebten und gefütterten Kleinhunde.
Vielleicht könnte man sich so aus der moralischen Schlinge ziehen: Wir machen mit unserer Gruppe das, was wir schon immer gerne gemacht haben - und ihr macht mit eurer Gruppe das, was ihr schon immer gerne gemacht habt. Und wir versuchen ernsthaft, uns nicht ins Gehege (!) zu kommen - was bei immer mehr Menschen, die immer dichter aufeinanderhocken immer schwieriger wird....
Suum quique statt non licet bovi. Play It Again, SPAM (Diskussion) 08:39, 12. Apr. 2016 (CEST)
LOL! "Suum cuique" ist hierzulande ja nicht mehr so beliebt - aber mit "non licet bovi" hast du meinen Tag gerettet! --Snevern 10:02, 12. Apr. 2016 (CEST)

Poststell in Schwerin - Lankow

Wann und von wem wurde die Poststelle in Schwerin - Lankow betrieben? Sie lag der alten Schule gegenüber, links neben dem alten Umspannwerk. Die Poststelle wird in den Ausführungen über Schwerin - Lankow nicht erwähnt. Fritz Körner hat mit seiner Frau diese Poststelle lange Jahre geführt, nach meinem Wissen. --93.204.214.73 11:23, 12. Apr. 2016 (CEST)

Untere Grenze: 1895 gab es sie noch nicht. 213.169.163.106 12:42, 12. Apr. 2016 (CEST)
GoogleBooks sagt: „1936 oder kurz davor."
Der Nächste !! 213.169.163.106 12:50, 12. Apr. 2016 (CEST)

frag doch beim Gewerbeamt/ Ordnungsamtt in Schwerin per Email nach...--Ip80.123 (Diskussion) 17:18, 12. Apr. 2016 (CEST)

Yardstickzahl

--79.215.69.156 18:19, 12. Apr. 2016 (CEST) Hallo, ich segle einen Finn Baujahr 1962 formverleimt mit einem Alumast, Aluschwwert und Alupinne mit Ruderkloben und Senkruderblatt. Ich möchte an den Revierregatten teilnehmen und benötige eine Yardstickzahl dafür. Wer kann mir weiter helfen. Mit sportlichem Gruß H-J Gerlach

Finn (Bootsklasse)...je nach Ausf. 110, 112 oder 114 bzw. Die DSV-Yardstickzahlen 2016 (Finn Carbonmast mit beliebigem Segel 110, Finn Alumast mit beliebigem Segel 112, Finn Holzrigg mit Dacronsegel 114) - die in den Artikeln angegebenen Yardstickzahlen sollten eigentlich alle auf diese Tabelle des DSV zurückgehen. --Btr 18:40, 12. Apr. 2016 (CEST) PS: Yardstick

Chinesisch-Sprecher gesucht

Diese Aussage in Diclofenac ist mit einem Artikel in einem chinesischen Journal belegt. Kann sie jemand mit entsprechenden Sprachkenntnissen überprüfen? --Leyo 14:19, 13. Apr. 2016 (CEST)

Koennt ja jemanden fragen. Aber gibts denn den Artikel ueberhaupt irgendwo? Hier fehlt entsprechende Ausgabe, selbst hier reicht das Archiv nicht lang genug zurueck, hier muss man sich einloggen zum runterladen. Laut Abstract beschreibt das Paper wohl den Syntheseweg, und eher nicht "heute ausschliesslich nach diesem Verfahren" - das Paper ist von 1998... ok, hab mich auch durchgeklickt. Eingefuegt wurde es hier: [6]. Ich denke die Ref kann man wohl zusammen mit dem Satz streichen. --Nurmalschnell (Diskussion) 19:30, 13. Apr. 2016 (CEST)
+1 Suche zeigt, dass Diclofenac heute in ca. 16 Ländern hergestellt wird.
Dass Medikamente vor dem Ablaufen der Patentrechte "woanders" hergestellt werden, ist (auch) nicht substanzspezifisch. Ich meine, der Satz kann weg. Play It Again, SPAM (Diskussion) 19:40, 13. Apr. 2016 (CEST)
Zwischen Plagiat und Generikum ist eine riesige Grauzone, die unter anderem Afrika und Indien umfasst. --Rôtkæppchen₆₈ 02:35, 14. Apr. 2016 (CEST)
Genial. Man sucht einen Chinesisch-Sprecher und bekommt Informationen über Afrika und Indien. --62.202.182.81 05:14, 14. Apr. 2016 (CEST)

Von mir aus kann man den Satz samt Einzelnachweis entfernen, falls keine gegenteiligen Meinungsäusserungen mehr kommen. Oder vielleicht ist ja jemand direkt mutig. ;-) --Leyo 12:06, 14. Apr. 2016 (CEST)

Entfernt. --Leyo 00:28, 15. Apr. 2016 (CEST)
Archivierung dieses Abschnittes wurde gewünscht von: Leyo 00:28, 15. Apr. 2016 (CEST)

Suche Horrorfilm-Titel

Kann mich nur noch grob an den Trailer erinnern. Es war glaube ich eine Parkhausszene. Ich kann mich noch in Auszügen an den dazugehörigen Spruch erinnern, der in etwa lautete: „Es ist unheimlich, wenn du allein bist, aber noch unheimlicher, wenn du nicht allein bist.“ So ähnlich zumindest. Dürfte noch nicht soo lange her sein, vermutlich 2 - 5 Jahre. Wäre klasse, wenn hier einer den Titel wüsste. Grüße --all apatcha msg 16:09, 14. Apr. 2016 (CEST)

Hm, P2 – Schreie im Parkhaus hat natürlich Parkhausszenen, aber auch Drag Me to Hell. --Magnus (Diskussion) 16:11, 14. Apr. 2016 (CEST)
Plakat spricht für ersteren. --Magnus (Diskussion) 16:14, 14. Apr. 2016 (CEST)
Ja, genau den meinte ich. Danke! --all apatcha msg 16:16, 14. Apr. 2016 (CEST)
e. Schade, dass wir da keinen Artikel zu haben. --all apatcha msg 16:17, 14. Apr. 2016 (CEST)
Dann ändere das doch. --Kreuzschnabel 20:20, 14. Apr. 2016 (CEST)
leider nicht so mein gebiet --all apatcha msg 20:37, 14. Apr. 2016 (CEST)
Archivierung dieses Abschnittes wurde gewünscht von: --all apatcha msg 20:37, 14. Apr. 2016 (CEST)

Trockenanzug

Was ist der Temperaturgewinn bei einem Nass(!)anzug versus Normaltemperatur? Und ist dann der erste Wasserkontakt sowieso gleich kalt? Diese Infos fehlen mir hier: Tauchanzug

Es gibt gar keinen Temperaturgewinn bei Trockis (nur bei den aktiv geheizten, die Berufstaucher tragen), lediglich der Temperaturverlust wird damit verlangsamt. Das Ausmaß hängt von den Parametern Wassertemperatur, Körperoberfläche und sonstigen Isolatoren (wie z.B. dem Unterzieher und dem Füllgas) ab. --80.219.124.55 18:15, 11. Apr. 2016 (CEST)
Sorry, der Nassanzug (Schwimmanzug) ist gemeint. --Muroshi (Diskussion) 20:57, 11. Apr. 2016 (CEST)
Gase sind sehr schlechte Wärmeleiter. So schlecht das sie in thermischen Betrachtungen schlicht als Isolator bezeichnet werden. Wasser ist dagegen ein sehr guter Wärmeleiter. So ein Trockenanzug ist also immer besser isoliert als ein normaler Neoprenanzug bei dem die Restluft zwischen Anzug und Haut bei Bewegung und mit dem Wasserdruck schlicht rausgepresst und durch Wasser ersetzt wird. --Kharon 21:06, 11. Apr. 2016 (CEST)
Nein, der erste Wasserkontakt ist auch bei einem Nassanzug nicht so kalt, wie wenn du gar keinen Tauchanzug trägst ("Normaltemperatur)" sondern merklich abgemildert. --Neitram  18:05, 12. Apr. 2016 (CEST)
Das Wasser wärmt sich so schnell auf? Sind keine Triathleten unter uns, die das in Grad beziffern können?--178.199.98.114 10:06, 13. Apr. 2016 (CEST)

Farbe die verrostet

Wie heißt die Farbe, mit der man Autos anpinseln kann und die dann verrostet? Das Auto sieht dann total verrostet aus, ist aber in Wirklichkeit nicht verrostet. Hält diese Farbe permanent auf Lack? --88.73.188.233 21:16, 11. Apr. 2016 (CEST)

http://www.lmdfdg.com/?q=rostfarbe+auto --JD {æ} 21:28, 11. Apr. 2016 (CEST)
Rostfarbe ist ganz einfach Eisenpulver in einem offenen Bindemittel, also z.B. Acryl in Wasser. Allerdings werden die Autos zumeist durch Abschrubben des alten Lacks und Behandeln des blanken Eisens mit Kochsalzlösung zum Verrosten gebracht. -- Janka (Diskussion) 01:18, 12. Apr. 2016 (CEST)
Hat das Ganze eigentlich auch einen Sinn? Brünieren ist ja wohl was Anderes. --Pölkkyposkisolisti 12:35, 12. Apr. 2016 (CEST)
Disneyfizierung des Objekts. Nicht vergessen, auf die Rostfarbe Klarlack aufzutragen, damit der Rost nicht rostet. -- Janka (Diskussion) 14:06, 12. Apr. 2016 (CEST)
Disneyfizierung?? Schutz vor Diebstahl? Beim Fahrrad heißt das Boruttisieren. --Pölkkyposkisolisti 14:10, 12. Apr. 2016 (CEST)
 
The Rat Look...
Rostfarbe am Auto (Rostlaube) scheint ein Aspekt der Kunstrichtung/Selbstdarstellung zu sein: The Rat Look, Rat Style, Ratting, Faux Patina. Da dürften dann Damen aussteigen, deren Jeans ripped slits haben? Es geht weiter zurück als man denkt - und hat in Filmen eine Rolle gespielt. Play It Again, SPAM (Diskussion) 17:14, 12. Apr. 2016 (CEST).
P.S. Der VW und der Bully hier ganz unten ... mit den glitzernden Felgen ... das hat schon was.
Rat Look? In meiner Jugend hieß das Derelicte. It is a fashion, a way of life inspired by the very homeless, the vagrants, the crack whores that make this wonderful city so unique. --Edith Wahr (Diskussion) 21:20, 12. Apr. 2016 (CEST)
Klingt wie eine liebevolle Beschreibung von Ankh Morpork...
Homeless - maybe; car(e)less - no. ( punk cars - habe ich mittlerweile gelernt - ist wieder etwas anderes... Artikel ohne Ende ...) Play It Again, SPAM (Diskussion) 09:11, 13. Apr. 2016 (CEST)

alternative für Zuckerwatte?

ist es möglich, Zuckerwatte aus braunem Zucker vom Zuckerrohr, stevia, aspartam, honig, xucker oder irgend einem anderen dieser Ersatzstoffe für weißen Raffineriezucker aus Rüben zu machen? --Ip80.123 (Diskussion) 17:17, 12. Apr. 2016 (CEST)

Ja. --Rôtkæppchen₆₈ 17:26, 12. Apr. 2016 (CEST)
Siehe Artikel Zuckerwatte: Haushaltszucker oder Isomalt – in Nordamerika wird auch oft Ahornsirup verwendet. --Buchling (Diskussion) 17:27, 12. Apr. 2016 (CEST)
Stevia, Aspartam: Nein.
Hier das Prinzip. Es muss eine Substanz sein, die man verflüssigen kann und die dann wieder kristallisiert (grob gesagt).Play It Again, SPAM (Diskussion) 19:51, 12. Apr. 2016 (CEST)
Für Aspartam und Steviosid sind in den Infoboxen der Artikel Schmelzpunkte angegeben. Ich bin bei meinem obigen Ja deswegen davon ausgegangen, dass sich auch Aspartam und Steviosid zersetzungsfrei schmelzen und zu Fäden spinnen lassen. Ob es schmeckt ist eine gaaanz andere Frage. --Rôtkæppchen₆₈ 21:54, 12. Apr. 2016 (CEST)
Ganz unmöglich ist es nicht: Man kan sie darin "einbauen". Hmmmm ... Solloide <Homerdrool> Play It Again, SPAM (Diskussion) 08:43, 13. Apr. 2016 (CEST)
Reines Steviosid kann man als Pulver kaufen. Abgesehen von den Kosten ist das aber so furchtbar süß, dass man Zuckerwatte daraus nicht ertragen könnte. --Magnus (Diskussion) 08:47, 13. Apr. 2016 (CEST)
Offtopic: Wer mal weinen möchte ... ;-) --> http://www.kraftfuttermischwerk.de/blogg/ein-waschbaer-mit-zuckerwatte-das-vielleicht-traurigste-gif-der-welt/ VG --Apraphul Disk 08:55, 13. Apr. 2016 (CEST)

Seit wann galt Rauchen als gesundheitsschädlich?

Ich suche Belege, seit wann es anerkannt ist, dass Tabak Rauchen gesundheitsschädlich ist. Ich frage deswegen, weil ich gelesen habe, dass es im 3. Reich umfangreiche Antiraucherkampagnen gab, vom Gefühl her würde ich aber behaupten, dass klare Belege über die Gesundheitsgefahr erst nach dem Krieg gab. Entweder war die Forschung im 3. Reich zum Thema Tabak weiter als alle anderen Länder oder die anderen Länder waren besonders ignorant. Vorausgesetzt natürlich, es gab diese Kampagnen, was die "Zeit" im Jahr 2001 behauptet. [7] --95.112.187.135 20:19, 12. Apr. 2016 (CEST)

Siehe Maßnahmen gegen das Rauchen im NS-Staat. --87.123.28.129 20:41, 12. Apr. 2016 (CEST)
Ich habe das Jahr 1926 im Kopf, da soll das erste Paper veröffentlicht worden sein das Rauchen in Zusammenhang mit Lungenkrebs brachte. Klare Belege folgten in den 1950ern in den USA, 1964 wurde ein erster großer Untersuchungsbericht vom Surgeon General of the United States dazu veröffentlicht.--Antemister (Diskussion) 21:22, 12. Apr. 2016 (CEST)
Weils mich auch grade interessiert hat, etwas gegoogelt: Habe mich um nur drei Jahre verschätzt, es war 1929. Fritz Lickint hieß der Mann. Lt. S. 112, mit dessen Hilfe ich das gefunden hatte, gab es Vermutungen aber schon noch deutlich länger.--Antemister (Diskussion) 21:33, 12. Apr. 2016 (CEST)
Ich habe einen etwas anderen Beleg und zwar früher. Karl May veröffentlichte 1909-10 seinen Roman „Winnetou IV“, später auch als „Winnetous Erben“ erschienen. Unter teilweise Verwendung der Strukturen und des Personals der alten Abenteuer-Romane werden hier in symbolischer Form religiöse und grundsätzliche Themen behandelt; zugleich enthält das Werk etliche autobiographische Elemente (Reise von 1908); May erzählt in erster Person.
Nach dem Essen rauchen einige Mitglieder der Reisegruppe; es wird gesagt, wer das nicht tut und warum nicht: „Was mich betrifft, so weiß man, daß ich sehr, sehr stark rauchte. Ich gestehe sogar ein, daß ich der stärkste von allen Rauchern war, die ich kennengelernt habe." (May war tatsächlich ein sehr starker Raucher gewesen.) "Jetzt bin ich es nicht mehr. Es sind nun fünf Jahre her, da bat mich das Herzle, nicht mehr soviel zu rauchen. Sie meinte, ich habe meinen Lesern noch außerordentlich viel zu sagen und müsse also trachten, solange wie möglich zu leben. Da legte ich die Zigarre, die ich im Mund hatte, weg und sagte: "Das ist die letzte gewesen im Leben, ich rauche nie wieder!" Warum hätte ich meiner Frau nicht gehorchen sollen? Sie hatte doch recht!" Das Herzle ist Klara, seit 1903 Mays zweite Frau. Mays Ansicht, er müsse nun, mit etwa achtundsechzig, seinen Lesern noch viel geben, ja, sein eigentliches Werk erst beginnen, ist belegt. Es ist gut möglich, dass dieser konkrete Verzicht aufs Rauchen auch autobiograhisch ist. Es spielt aber keine Rolle. May war kein Arzt, er hatte aber eine weitreichende Allgemeinbildung und verschlang Zeitungen, Enzyklopädien usw. Er hatte also 1908/09 ein Wissen darüber, dass das (starke) Rauchen das Leben eines Menschen verkürzen würde. Und dieses Wissen muss er irgendwo aufgeschnappt haben; es muss also in jenen Jahren schon zugänglich gewesen sein. NfdA (Diskussion) 22:43, 12. Apr. 2016 (CEST)
Christoph_Wilhelm_Hufeland: "„Das Rauchen ist einer der unbegreiflichsten Genüsse. Etwas Unkörperliches, Schmutziges, Beißendes, Übelriechendes kann ein solcher Lebensgenuß, ja ein solches Lebensbedürfnis werden, dass es Menschen gibt, die nicht eher munter, vergnügt und lebensfroh werden, ja, die nicht eher denken können, als bis sie Rauch durch den Mund und die Nase ziehen"--Wikiseidank (Diskussion) 07:07, 13. Apr. 2016 (CEST)
Hier eine ganz nette historische Zusammenstellung mit Jahreszahlen und Studien. Play It Again, SPAM (Diskussion) 08:26, 13. Apr. 2016 (CEST)

Sehtest für urteilsunfähige Personen

Hallo. Gibt es eigentlich eine Art die Sehschwäche von urteilsunfähigen Personen zu bestimmen? --Muroshi (Diskussion) 11:59, 11. Apr. 2016 (CEST)

Ja, das gibt es. Das wird mit einem Gerät gemacht, das die Abbildungseigenschaften der Linse ohne Zutun des Patienten vermisst. --Rôtkæppchen₆₈ 12:09, 11. Apr. 2016 (CEST) Näheres per Google optische Biometrie. --Rôtkæppchen₆₈ 12:13, 11. Apr. 2016 (CEST)
Autorefraktometer... aber dazu muss der Pat entspannt in die Ferne sehen, was mir schon schwer fällt, obwohl ich n guter Pat sein mag... irgendwie kommt da scheinbar dauernd direkt was auf mich drauf zu, was ich unwillkürlich genauer zu betrachten versuche... --Heimschützenzentrum (?) 12:18, 11. Apr. 2016 (CEST)
Die Zukunft? --2003:45:464B:1500:511C:4F5A:E02:DECE 20:55, 11. Apr. 2016 (CEST)
Welcher Optiker bietet so etwas an? --Muroshi (Diskussion) 12:26, 11. Apr. 2016 (CEST)
Häufig Optiker, die sich auf (kleine) Kinder spezialisiert haben. Einem Unter-Einjährigen z.B. kann man ja auch nicht sagen, er solle mal da und da hinschauen und das auf diese oder jene Art tun. Trotzdem kriegen die es hin, die Sehschwäche zu bestimmen. --87.123.34.118 12:48, 11. Apr. 2016 (CEST)
Ist das nicht sowas wie bei Fielmann mit dem Ballon? Zumindest dachte ich immer, dass damit zumindest schon mal ein Grobabgleich gemacht wird. --192.91.60.11 13:16, 11. Apr. 2016 (CEST)
Richtig, gerade bei solchen unkommunikativen Patienten (Kleinkind, urteilunfähige Person) ist es schon hilfreich, wenn es in etwa stimmt. Soll heissen für den Patienten selber ist eine grob bestimmte Brille immer noch besser als gar keine Brille. Denn wenn man solchen Patienten überhaupt eine Brille verpasst, muss ein arger Sehfehler liegen. --Bobo11 (Diskussion) 13:25, 11. Apr. 2016 (CEST)
Lieber zum Augenarzt statt zum Optiker. Der kann dann auch feststellen, ob nicht eine Krankheit vorliegt statt lediglich Kurz- oder Weitsichtigkeit. Zyklopen gehen natürlich zum Augearzt. 194.25.103.254 14:08, 11. Apr. 2016 (CEST)
Aha. Deshalb sagt man auch "Gitarrenkonzert" und "Oboenkonzert", weil da immer mindestens zwei Solisten vor dem Orchester stehen. Und der Zahnarzt kann gleich zumachen, da er in der Konkurrenz zum Zaehnearzt fast keinen Kunden hat. Und dann noch der Hals-Nase-Ohrenarzt. scnr --83.167.34.67 17:28, 11. Apr. 2016 (CEST)
goldig :-) --91.3.24.37 21:00, 11. Apr. 2016 (CEST)
 
Semmel(n)knödel(n)
Lesetipp: Fugenlaut – oder muss das Fugelaut heißen? Dazu dann Semmelnknödeln. --Rôtkæppchen₆₈ 23:05, 11. Apr. 2016 (CEST)
Äpfelsäure. --178.4.110.174 23:35, 13. Apr. 2016 (CEST)

Russen im Untegrund?

Was bedeutet es, dass auf diesen Gullys russische Inschriften sind? (Richtige Gullys sind es nicht, es steht darauf etwas mit Glasfaser). Fotografiert in Jerusalem. (Oder hat das mit dieser Invasion zu tun :-) ?) - -jkb- 20:04, 12. Apr. 2016 (CEST)

"Vorsicht, optisches Kabel" steht da auf Russisch, genau wie in anderen Sprachen. Warum auf Russisch? Hm, wäre es nicht in Jerusalem, sondern etwa in Petach Tikwa, wäre die Antwort naheliegend (eine Stadt mit sehr vielen Russen). Aber so... Es gibt viele Russen in Israel, es gab sogar Bestrebungen, Russisch zur dritten Amtssprache zu erheben, doch dass es tatsächlich dreisprachige Aufschriften gibt (englisch als vierte Sprache für Touristen), ist mir neu. Werde bei meiner nächsten Israelreise darauf achten --Alexmagnus Fragen? 20:24, 12. Apr. 2016 (CEST)
Siehe auch en:Russian language in Israel.--Alexmagnus Fragen? 20:30, 12. Apr. 2016 (CEST)
Hm, a ja. Klar, man hört dort auf der Straße russisch sehr viel (teils kam ich mir vor wie auf der türkischen Riviera), aber dass es so ausartet, da war ich überrascht. -jkb- 20:33, 12. Apr. 2016 (CEST)
Die hätten auch noch mehr Sprachen draufgeschrieben, wenn es gepasst hätte. Damit der geneigte Kanalarbeiter nicht den Gully verwechselt und mal flugs halb Jerusalem vom Internet trennt. -- Janka (Diskussion) 21:19, 12. Apr. 2016 (CEST)
Bei Geldautomaten in Jerusalem ist Russisch auch eine der 4 Sprachen, die angeboten werden, ich hab das Bild nur noch nicht hochgeladen. Russisch ist in Israel sehr präsent. --Pölkkyposkisolisti 23:12, 12. Apr. 2016 (CEST)
Lieber Alexmagnus , warum sind und waren die Russen so interessiert daran in Israel präsent zu sein? Ich dachte die Sowjetunion und das heutige Russland ignoriert den Staat Israel, einfach weil das für die ein Westliches Natoland ist mit Atomwaffen. Ich hätte gedacht, die beziehungen zwischen Israel und Südafrika wären deutlich stärker als wie zu Russland... --Ip80.123 (Diskussion) 02:31, 13. Apr. 2016 (CEST)
„Russisch ist in Israel sehr präsent“ ist etwas anderes als „die Russen sind in Israel sehr präsent“. Denn nicht nur „die Russen“ sprechen Russisch. Ich vermute, das liegt an der großen Zahl jüdischer Einwanderer, die lange in Rußland gelebt haben und deshalb vorwiegend Russisch sprechen, bis sie ausreichend Ivrit gelernt haben :-) --Kreuzschnabel 07:27, 13. Apr. 2016 (CEST)
Nicht "die Russen" (du denkst an Staat und Regierung der Russischen Föderation) sind in Israel präsent, sondern Juden mit russischer Muttersprache, die im 20. Jahrhundert aus Russland bzw. der Sowjetunion nach Israel ausgewandert sind. Die russische Beschriftung der Straßenkappen ist keine politische Entscheidung, sondern eine pragmatische. In der NATO ist Israel übrigens nicht. --::Slomox:: >< 07:25, 13. Apr. 2016 (CEST)
Da kann doch der böse Putin nichts dafür. Ben-Gurion, geboren in Russland. Golda Meir, geboren in Kiew, damals Russland. Menachem Begin, geboren in Russland. Shimon Peres, geboren in Wischnewa. --62.202.182.81 10:05, 13. Apr. 2016 (CEST)
Bei Ben-Gurion, Begin und Peres geht es eher um den Osten des alten Polen-Litauen, der bis zum Zweiten Weltrkrieg großräumig jüdisch besiedelt war, mit Russland und Russisch jedoch relativ wenig zu tun hat. --j.budissin+/- 10:26, 13. Apr. 2016 (CEST)
Wie kommst Du auf die Idee mit "altem Polen-Litauen"? Für die Juden gab und gibt es immer 2 Hauptsprachen, Hebräisch, und die Amtssprache des jeweils herschenden Systems. Selbst bei Wechseln wird häufig noch daran festgehalten, wie zB. die Deutschkenntnisse vieler ab 1918 auf einmal "polnischen Juden". Die genannten wurden im Russischen Reich geboren, also hatten sie Russisch als zweite Muttersprache, und diese neben der Kultur mit nach Palästina/Israel genommen. Was sich hier eher zeigt ist die scheinbare oder tatsächliche westjüdische Dominanz, welche Israel als Fortsetzung der jüdischen Kultur Westeuropas zeigt, während es tatsächlich von Anbeginn eine multikulturelle Gesellschaft war, wo neben der urbanen jüdischen Bevölkerung genauso die ostjüdischen Aschkenasi und die Sepharden des Mittelmeerraums Platz hatten (ergänzt um Einwanderer aus Indien, Jemen, Uganda oder USA).Oliver S.Y. (Diskussion) 10:38, 13. Apr. 2016 (CEST)
Zunächst mal ist der Grund, warum es diese großen jüdisch geprägten Gebiete dort gab, nicht Russland, sondern Polen-Litauen mit seiner liberalen Religions- und Ansiedlungspolitik. Zweitens halte ich es für ein Gerücht, dass in Brest-Litowsk, Płońsk und Wischnewa zu den betreffenden Zeiten großartig Russisch gesprochen wurde. Zweitsprache der jüdischen Bevölkerung war zumeist die im jeweiligen Gebiet vorherrschende Mehrheitssprache, und das war entweder Polnisch oder Weißrussisch. Auch die übrige Bevölkerung dort sprach in aller Regel nicht Russisch. Wenn man da mal über die älteren Friedhöfe spaziert (nicht die jüdischen), wird das ziemlich deutlich. --j.budissin+/- 10:48, 13. Apr. 2016 (CEST)
Was ist Weißrussisch, außer dass es auch mal mit arabischen Schriftzeichen geschrieben wurde, anderes als ein russischer Dialekt? --62.202.182.81 12:02, 13. Apr. 2016 (CEST)
Erwartest du darauf von einem Slawisten ernsthaft eine Antwort oder möchtest du dir zunächst einmal den Artikel und die dort genannte Literatur zu Gemüte führen? --j.budissin+/- 12:56, 13. Apr. 2016 (CEST)
Die Frage war rhetorisch gemeint. Ich bin nun nicht hochobergelehrter Slavist, habe aber Russisch gelernt und verstehe deswegen Weißrussisch problemlos, mündlich und schriftlich. --62.202.182.81 15:03, 13. Apr. 2016 (CEST)
Das ist nun bei slawischen Sprachen auch nicht weiter ungewöhnlich. Meine Empfindlichkeit rührt daher, dass hier durchaus immer wieder Leute herumspringen, die gewissen Sprachen ihre Eigenständigkeit absprechen. Sorry, falls du nicht dazugehörst. Grüße, j.budissin+/- 16:39, 13. Apr. 2016 (CEST)
Ich gehöre dazu und spreche der weißrussischen Sprache die Eigenständigkeit ab, nicht aber dem weißrussichen Staat und dem weißrussischen Volk, so wie ich dem deutschländischen Hochdeutsch die Eigenständigkeit abspreche (obwohl dort so völlig schräge Formulierungen wie "parken", "grillen", "die Tram" oder "Rindfleischetikettierungsüberwachungsaufgabenübertragungsgesetz" vorkommen, was von der Mehrheit der deutschsprachigen Staaten (österreichisches und Schweizer Hochdeutsch) aber ebenso problemlos verstanden wird. --62.202.182.81 17:23, 13. Apr. 2016 (CEST)
Weißrussische Sprache ist eigenständig. Gegenseitig verständlich mit Russisch - na und. So sind auch Dänisch und Norwegisch. Und ja, langsam gesprochenes Polnisch versteht man als Russe auch (geschriebenes sowieso). Gegenseitige Verständlichkeit zweier Sprachen ist noch kein Grund, diese Sprachen nicht anzuerkennen. Übrigens, es ist eher Russisch ein Dialekt - es gibt eine Menge Wörter, deren Wurzeln es von allen slawischen Sprachen nur im Russischen gibt. Nicht im Weißrussischen, nicht im Polnischen, nicht im Mazedonischen :D--Alexmagnus Fragen? 20:26, 13. Apr. 2016 (CEST)
Ihr sprecht alle über Russen aus dem ehemaligen Russischen Reich, man soll aber die Russen, die nach dem Zerfall der Sowjetunion ausgewandert sind, nicht vergessen. Die große jüdische Auswanderung - nach Israel und Deutschland - zwischen 1990 und etwa 2005. Dadurch hat Israel (s. den englischen Artikel, den ich verlinkt habe) die drittgrößte russischsprachige Community eines nicht-postsowjetischen Landes (Nach Deutschland - wo es neben Juden noch ethnische Deutsche sind - und den USA - wohin auch viele Juden auswanderten, oft über Zwischenstation Israel/Deutschland).--Alexmagnus Fragen? 20:19, 13. Apr. 2016 (CEST)

Wirkungsgrad Waffen

Bei Welt der Wunder wird gesagt, ein Pfeil von einem Bogenschützen hat so in etwa 30% Wirkungsgrad, ein Armbrustschütze kann spitzenwerte bis 45% erreichen. Also das was von 100% Energie die zum Abfeiern des Pfeils. Wie sieht es bei Pistolen, Panzerfäusten, Maschinenpistolen, Sturmgewehren, Flinten (von Jägern) und Vorderschaftreptilflinten (Pumpguns) und Scharfschützengewehren aus? Wieviel % der Energie geht verloren / kommt durch? --Ip80.123 (Diskussion) 04:41, 13. Apr. 2016 (CEST)

Man kann jedenfalls einen Maximalwert für reine Projektilwaffen berechnen über die Impulserhaltung. Dieser Maximalwert gilt nicht für Waffen mit selbst angetriebenen Geschoßen (Raketen), also z.B. nicht für Panzerfäuste. Da die Patrone und die Waffe (siehe Rückstoß) sich in die entgengesetzte Richtung bewegen (die Waffe natürlich nur, wenn man sie nicht festhält, aber der Einfachheit halber betrachte ich diesen Fall), kann man das Problem eindimensional betrachten. Wir wählen das Bezugssystem so, dass die Waffe und die Patrone vor dem Schuss ruhen. Nach dem Schuß gilt folgende Impulsgleichung (Der Index W bezieht sich jeweils auf die Waffe ohne das Projektil, P auf das Projektil, vW und vP sind die Beträge der Geschwindigkeiten):
 mW*vW - mP*vP = 0
Daraus bekommt man:
 vW = mP*vP/mW
somit ist die gesamte kinetische Energie:
 mW*vW² + mP*vP² =  mW*(mP*vP/mW)² + mP*vP² = mP²*vP²/mW + mP*vP² = vP²*(mP²/mW + mP)
Die kinetische Energie der Patrone beträgt natürlich mP*vP². Der Wirkungsgrad ist allgemein kinetische Energie der Patrone/zugeführte Energie. Wegen dem Energieerhaltungssatz muss die zugeführte Energie jedoch mindestens so hoch sein wie die gesamte kinetische Energie nach dem Schuss, also gilt:
 Wirkungsgrad < mP*vP²/(vP²*(mP²/mW + mP)) = mP/(mP²/mW + mP*mW/mW) = mP*mW/(mP² + mP*mW) = mW/(mP + mW)
Rechnung ohne Gewehr Gewähr. Dieser Wert ist allerdings meist relativ hoch, da die Waffe normalerweise viel schwerer ist als das Projektil. Der tatsächliche Wirkungsgrad wird wegen Reibungsverlusten + eventuell ineffizient genutzter chemischer Energie noch deutlich geringer sein. --MrBurns (Diskussion) 05:32, 13. Apr. 2016 (CEST)

Irgendwie eine Prozentangabe wäre was tolles gewesen...--Ip80.123 (Diskussion) 05:57, 13. Apr. 2016 (CEST)

Was zahlst du, wenn ich das für dich durchrechne? ;-) --Kreuzschnabel 07:35, 13. Apr. 2016 (CEST)
In der Überschrift steht "Wirkungsgrad Waffen". Und in Wirklichkeit geht es dann doch nur um deren Unterkategorie Schußwaffen und nicht auch um andere. Das moniere ich! --Heletz (Diskussion) 08:11, 13. Apr. 2016 (CEST)
Beispiel AK-47, erster Schuss: In dem Fall gilt mW = 4,3kg (dass das abgeschossene Projektil eigentlich nicht dazuzählt kann man vernachlässigen), Geschossmasse 10g (ist innerhlab des möglichen Bereichs für 7,62 × 39 mm). Dann gilt mW/(mP + mW) = 4,3/4,31 ~ 99,77%. Der tatsächliche Wirkunsgrad wird wie erwähnt deutlich darunter liegen. --MrBurns (Diskussion) 16:25, 13. Apr. 2016 (CEST) Nachtrag: ich bin mir jetzt nicht sicher, für welches Geschossgewicht die 4,3 kg Gesamtmasse sind, aber auch wenns für 6,61g oder 12,5g ist, macht das kleinen großen Unterschied insofern, dass der die Obergrenze für den Wirkungsgrad durch den Impuls nahe bei 100% liegt. Ich hab halt 10g angenommen, weils eine runde Zahl ist. --MrBurns (Diskussion) 16:33, 13. Apr. 2016 (CEST)
Noch ein Beispiel: Zarenkanone, Masse 39312 kg, Geschossmasse 1970 kg. In dem Fall gilt: mW/(mP + mW) = 95,23%. --MrBurns (Diskussion) 16:37, 13. Apr. 2016 (CEST)

Mit Parkscheibe 20 Stunden parken

Auch wenn ich es nicht wirklich glauben konnte, es gibt durchaus Parkplätze, wo man mit Parkscheibe bis zu 20 Stunden parken darf (z. B. da). Nur: Wie sieht das praktisch aus? Eine übliche Parkscheibe hat ja ein 12-Stunden-Format, sodass nicht direkt ersichtlich ist, ob da jemand erst 9 Stunden steht oder schon 21. Ich kann mir auch nicht vorstellen, dass die Ordnungskräfte in einem komplizierten Verfahren regelmäßig dokumentieren, wer seit wann wo parkt, um nachweisen zu können, dass da jemand länger als die erlaubte Parkzeit an einem Platz stand ohne sein Fahrzeug zwischendurch zu bewegen. Hat man auf solchen Parkplätzen wirklich ein Bußgeld wegen Falschparken zu befürchten, oder ist das einfach nur ein Schildbürgerstreich? --132.230.195.196 09:52, 13. Apr. 2016 (CEST)

Es gibt die Technik die Reifen mit Kreide zu markieren. --Zulu55 (Diskussion) Unwissen 10:04, 13. Apr. 2016 (CEST)
Und die Kreide ist nach 20 Stunden runtergewaschen...? --Magnus (Diskussion) 10:06, 13. Apr. 2016 (CEST)
@Magnus: Nein, eben nicht, daher kann der Politeur dann ja auch sehen, dass der Wagen vor 21 Stunden schon da war. --Zulu55 (Diskussion) Unwissen 13:14, 13. Apr. 2016 (CEST)
Totally oldschool, die Kreide. Der Überwacher des ruhenden Verkehrs notiert die "Uhrzeit", die das Ventil des rechten Vorderrades anzeigt. Cool, wa?! 79.204.223.180 19:23, 13. Apr. 2016 (CEST)
Schaue bitte nochmal genau hin, meist haben diese Schilder ergänzende Regelungen. Im Streitfall hat der Fahrer/Halter die Ankunftszeit nachzuweisen. Und was "Schildbürgerstreiche" angeht, manchmal auch einfach nur Pfusch, der unbemerkt bleibt. Der Wert ist so unüblich, daß es eine Individualanfertigung sein müßte. Regeln auf Privatgelände, wie Parkplätzen von Supermärkten sind was anderes, aber auch da mußt Du nachweisen, und die Kontrollfirmen fotografieren mit Zeitstempel.Oliver S.Y. (Diskussion) 10:07, 13. Apr. 2016 (CEST)
Frager, kannst du eine offizielle Seite verlinken, auf der dieses 20-Stunden-Parkscheiben-Parken beschrieben ist? Play It Again, SPAM (Diskussion) 10:09, 13. Apr. 2016 (CEST)

Es gibt ab und zu Parkplätze, auf denen man 16 oder 20 Stunden parken darf, aber eben nicht länger (z.B. um das Übernachten von LKW-Fahrern zu ermöglichen, aber längere Standzeiten auszuschließen). Mit Parkscheibe kenne ich das allerdings nicht, läuft wohl eher auf Fotodokumentation o.ä. raus.--141.76.20.173 15:15, 13. Apr. 2016 (CEST)

Excel 2010: Fragen

Ich beschäftige mich gerade gezwungenermaßen ein wenig mit Excel und habe dazu nachfolgende Fragen, bei denen ich momentan nicht so recht weiter weiß. Vielleicht ist hier ja jemand so nett und mag helfen.

Frage 1: Ich möchte (rein grafisch, also ohne hinterlegte Zellfunktion) eine Zelle einen Zeitverlauf darstellen lassen. Dazu soll oberhalb der Zelle - am besten auf den jeweiligen Zellbegrenzungen - eine Uhrzeit stehen. Ein Beispiel ist diese Grafik, am unteren Ende die Datumseinträge. Es soll allerdings kein Diagramm sein. Gibt es die Möglichkeit?
Frage 2: Ich möchte möglichst DAU-sicher zwei Spalten der Tabelle aus dem Druck ausnehmen und nur den Rest drucken. Bisher kenne ich die Funktion Druckbereich (macht zwar prinzipiell, was ich will, teilt mir dann allerdings die Tabelle auf zwei Seiten auf, was ich nicht möchte) und die Funktion Gruppieren (nicht zu 100% DAU-sicher, da die Gruppe vor dem Druck manuell geschlossen werden muss und ich zudem in den Spalten Kontrollkästchen habe, die beim Schließen der Gruppe komischerweise nicht ganz mitverschwinden). Geht sowas vielleicht über Makros und einem "Drucken"-Button in der Tabelle?

Danke bis hierher einmal, eventuell folgen noch weitere Fragen. -- Pionic !? 12:17, 13. Apr. 2016 (CEST)

Zur zweiten Frage: Du kannst eine Mehrfachauswahl als Druckbereich festlegen. Wähle den ersten Teilbereich aus, dann drücke und halte die Strg-Taste und wähle die weiteren Teilbereiche aus. Dann lasse die Strg-Taste los. Alternativ könntest Du die beim Druck unerwünschten Spalten einfach ausblenden, indem Du einen Rechtsklick auf den Tabellenkopf machst und dann ausblenden klickst. --Rôtkæppchen₆₈ 12:29, 13. Apr. 2016 (CEST)
Ja, diese Möglichkeit habe ich auch schon in Betracht gezogen. Problem dabei ist, dass sie von jedem User auf's Neue durchgeführt werden muss, oder? Genau das wollte ich nämlich vermeiden. -- Pionic !? 13:05, 13. Apr. 2016 (CEST)
Nimm das von Rotkäppchen beschriebene als Makro auf. Und mach dir irgendwo ins Excel-Dokument einen Formularknopf hin, den betitelst du mit "für Druck vorbereiten" oder so und in dessen OnClick-Ereignis startest du besagtes Makro. Klingt kompliziert, ist aber eigentlich ganz einfach. Und jeder Benutzer klickt vor'm Druck nur auf diesen Button. --Susad 1978 (Diskussion) 13:12, 13. Apr. 2016 (CEST)
(BK)Du könntest In Deinem Dokument ein weiteres Blatt „Druckansicht“ einrichten, wo Du Dir per Formel =Rechenansicht!A1 die Zellinhalte aus dem Blatt „Rechenansicht“ holst. Die Formatierungen kannst Du mit dem gelben Pinsel holen. Dann einfach die unerwünschten Zellen aus der Druckansicht löschen. --Rôtkæppchen₆₈ 13:15, 13. Apr. 2016 (CEST)
Zur ersten Frage: Warum kein Diagramm? Da gibts bestimmt schon was fertiges im Äxxl. --Susad 1978 (Diskussion) 13:18, 13. Apr. 2016 (CEST)
Das Erste geht bestimmt mit automatischem Ausfüllen. Erste und zweite Uhrzeit eintragen, dann beide Zellen markieren und am Punkt rechts unten an der Markierung ziehen, bis der gewünschte Bereich gefüllt ist. Oder erste und letzte Uhrzeit eintragen, kompletten Bereich markieren und dann Reihe ausfüllen, Trend anhaken. --Rôtkæppchen₆₈ 13:30, 13. Apr. 2016 (CEST)

@Susad: Ja, das mit dem Makro habe ich mir so in etwa vorgestellt. Es gibt nur zwei kleine Haken (oder ich habe etwas übersehen): Ich kann erstens das Druckmenü nicht direkt ansteuern (oder?) und zweitens muss ich im Druckmenü dann die Option "Nur markierte Bereiche drucken" auswählen. Übersehe ich das, drucke ich wieder alles. Also nicht DAU-sicher.

Hmm doch. Irgendwie so könnte man das machen:

Selection.PrintOut Copies:=1, Collate:=True, IgnorePrintAreas:=False

da müsstest du mal die VB-Hilfe konsultieren, welche Optionen man da übergeben kann.--Susad 1978 (Diskussion) 14:08, 13. Apr. 2016 (CEST)

Zur ersten Frage, warum kein Diagramm: Weil die Zellen darunter keine Diagrammwerte enthalten, sondern anderweitigen Inhalt. Oder ist es möglich, quasi nur eine Diagrammachse mit der Beschriftung zu erstellen?! @ Rotkaeppchen: Die Zeiten sind schon eingetragen, das war nicht das Problem. Mir geht es mehr um die grafische Formatierung, da jetzt im Moment der Wert (Beispielsweise 6 Uhr) mittig in der Zelle steht und nicht auf der Zellenbegrenzung, wo er eigentlich hinsollte. -- Pionic !? 13:56, 13. Apr. 2016 (CEST)

Mit dem von dir geschriebenen Befehl geht es leider nicht, obwohl er sich gut liest. Ich bin aber leider zu wenig Profi, als dass ich den debuggen könnte. -- Pionic !? 14:21, 13. Apr. 2016 (CEST)
Bei mir hat das gerade geklappt. Es muss natürlich was markiert sein, was ausgedruckt werden soll. Ansonsten:
  • Copies:= ist klar oder?
  • Collate:=True/False (Ausgbabe sortieren)

Hier findest du weiterführende Hinweise --Susad 1978 (Diskussion) 14:44, 13. Apr. 2016 (CEST)

Mein Makro sieht derzeit so aus:
    Range("A1:D22,F1:I22").Select
    Range("F1").Activate
    Selection.PrintOut Copies:=1, Collate:=True, IgnorePrintAreas:=False
Da bringt er mir aber den Laufzeitfehler '1004'. -- Pionic !? 14:58, 13. Apr. 2016 (CEST)
Dann probier mal folgendes:
Range("A1:D22,F1:I22").PrintOut Copies:=1, Collate:=True

--Susad 1978 (Diskussion) 15:08, 13. Apr. 2016 (CEST)

So weit, so gut. :-) Er druckt, allerdings ist die Ausgabe genau wie bei den Druckbereichen - er stückelt die einzelnen Bereiche auf und druckt jeden auf einer eigenen Seite. Kann man das unterbinden? -- Pionic !? 15:31, 13. Apr. 2016 (CEST)
Man könnte das Druckergebnis skalieren, vielleicht bringt das was? --Susad 1978 (Diskussion) 15:41, 13. Apr. 2016 (CEST)

Hmm. Ich habe zum Test einfach einmal zwei farbige Bereiche erstellt. Diese nehmen jede für sich nicht einmal ein Drittel der Seite ein, es sollte also locker draufpassen. Abgesehen davon müsste das Skalieren ja wieder von Hand erfolgen... -- Pionic !? 16:10, 13. Apr. 2016 (CEST)

Bitte auch das heutige xkcd dazu beachten. --87.148.90.191 18:41, 13. Apr. 2016 (CEST)

Problem mit IE 11

Guten Abend! Mit einer speziellen Software habe ich jedem Mitarbeiter eine eigene Seite auf meinem Server eingerichtet. Nun heißt der eine Mitarbeiter Beispiel und der andere Mitarbeiter Baispiel. Was in Opera oder FF problemlos funktioniert, hakt im IE 11. Dort ist wahrscheinlich MA Beispiel als Erster aufgerufen und im Cache hinterlegt worden, sodaß beim Aufruf von MA Baispiel immer der MA Beispiel erscheint. Ich habe schon alle mir bekannten Tricks versucht, den MA Baispiel aufzurufen, damit er auch im Cache gespeichert wird und von dort dann richtig geladen werden kann. Aber ich schaff es ums Verr.... nicht, den Baispiel direkt oder über die Software aufzurufen. Wie könnte ich ihn holen? MfG Laubert --79.225.113.172 17:52, 13. Apr. 2016 (CEST)

Hast Du schon Auto-Complete abgeschaltet? --Rôtkæppchen₆₈ 21:11, 13. Apr. 2016 (CEST)
Yes! Plus Direkteingabe über Eingabefeld, plus willentlich Fehleingabe, damit Server eine Auswahl ähnlich geschriebener HTMLs vorschlägt, schlägt auch die entsprechende Baispiel.html vor, nach Klick darauf kommt aber wieder Beispiel.html (ich kann aber aus betrieblichen Gründen deshalb den Cache nicht leeren, aber dann wäre es vermutlich anders rum, dann kommt halt statt B. der B.) --79.225.113.172 22:29, 13. Apr. 2016 (CEST)
Problem gelöst! Über InPrivate aufgerufen, unter Favoriten gespeichert und in normalem Fenster über Favoriten aufgerufen. Dadurch im Cache und wird jetzt auch korrekt angezeigt. MfG Laubert--79.225.113.172 23:15, 13. Apr. 2016 (CEST)

An-Bürotür-Klopfen, keiner sagt was

Ich bin etwas ratlos bei der Etikette bzgl. an-Bürotür-Kopfen und keiner reagiert (ich höre auch keine Stimme[n]).

So heute hatte ich den Fall, da habe ich bei meinem Ober-Chef angeklopft (ohne Termin) und niemand hat reagiert. Dann habe ich die Tür geöffnet und prompt war mein Chef in einer Besprechung mit meinem Oberchef. Etwas ungut, habe ohne etwas zu sagen gleich wieder die Tür geschlossen. Nun will ich mal fragen, ob es allgemeine ungeschriebene Gesetze hierzu gibt, wie man sich verhält, wenn eine Bürotür zu ist, man anklopft, sich aber keiner meldet.

a) Ich will mit jemandem sprechen > keiner sagt was > ich gehe nicht rein (evtl. Besprechung oder Telefonat oder ...)
b) Ich will etwas hinlegen > keiner sagt was > einfach reingehen (könnte ja auch keiner drin sitzen)
bb) Ich will etwas hinlegen > keiner sagt was > es ist zugesperrt > ich sperre auf (könnte aber auch delikat werden, wenn gerade ein Stelldichein stattfindet)

Ist das so OK = Usus in Mitteleuropa? --93.133.107.89 20:58, 11. Apr. 2016 (CEST)

Wenn anklopfen erwünscht ist, dann ist "herein" erforderlich. Sagt keiner was, bleibt die Tür zu. So und nicht anders halte ich es und kenne ich es. --Snevern 21:02, 11. Apr. 2016 (CEST)
1. bei „Ein Käfig voller Helden“ gab es mal eine Szene, wo Hogan richtig heftig gegen die Tür boxt und dann sofort die Tür öffnet... das war tierisch witzig... 2. an nem Sekretariat stand mal außen an der Tür „Ohne Klopfen eintreten“ (von innen stand da „Man muss nich verrückt sein, um hier zu arbeiten, aber es hilft“)... 3. ansonsten klopf ich auch, wenn klopfen nich ausdrücklich gewünscht ist, gern an, bevor ich die Tür aufmach... damit alle wissen, dass gleich die Tür aufgeht... n MSCE saß nämlich mal Stunden-lang an seinem Tisch und klagte über Nasenbluten, nachdem ich die Tür aufgemacht hab (aber ich glaub, s war n kleiner Münchhausener, weil r dauernd so Geschichten erzählte...)... --Heimschützenzentrum (?) 21:19, 11. Apr. 2016 (CEST)
(BK)Wenn es draussen einen Schieber Besetzt/Frei gibt. Dann ist es bei Frei möglich, ohne Antwort rein zugehen. Aber auch wenn auf frei, ist aus Höflichkeit anklopfen angesagt, nach dem Anklopfen trotzdem immer kurz warten (Die Chance für ein "ein Moment bitte, bin am Telefon" usw. geben), dann rein gehen. Wenn auf besetzt, 1. nur in ganz wichtigen Fällen klopfen, 2. ganz sicher nicht ohne Aufforderung eintreten. Ob jetzt eintretten nach Anklopfen ohne Antwort zulässig ist, kommt wirklich auf Branche und deren Bürokodex an. Und logischer weise auch denn "Rang" in der Bürohriarchie, der Chef kann sich beim Abteilungsleiter sicher mehr heraus nehmen als der Praktikant. In einer Autowerkstatt wird das ganz sicher anders gehandhabt, als bei einer Versicherung/Bank. Hier geht es ganz klar auch um die Vertraulichkeit der möglichn Gesprächthemen. In der Personalabteilungs-Büro ist mehr Vorsicht angebracht, als in Büro der AVOR (Wo eh meist ein kommen und gehen ist). Aufschleissen ist nur dann angebracht, wenn du genau für diesen Zweck -weshalb du jetzt rein willst-, auch denn Schlüssel gekrigt hast. --Bobo11 (Diskussion) 21:26, 11. Apr. 2016 (CEST)
Was sind das denn für hilflose Gestalten? Entweder man wurde gerufen, oder man hat etwas wichtiges. Wenn man nichts wichtiges hat, geht man nicht zum Chef. Wenn man zum Chef geht und Türe zu, klopft man an. Denn man wurde gerufen oder hat etwas wichtiges. Wenn keine Reaktion, geht man rein. Wenn besetzt, geht man wieder raus. Falls der Mensch hinter der Türe so doof ist, sein von dir angetöntes kompromittierendes Verhalten im Büro auszuleben und dann erst noch mit nicht verriegelter Türe, ist das nicht mein Problem. Wenn man völlig keinen Plan hat über das eigene Anklopf- und Türöffnungsverhalten, fragt man den Chef über das empfehlenswerte Verhalten, und sicher nicht die Wikipedia. Wenn man Angst vor dem eigenen Chef hat, geht man entweder meditieren und versucht es am nächsten Tag nochmals, oder fragt in der Personalabteilung. --2A02:1206:45B4:A2E0:F137:154:FC7B:8550 22:34, 11. Apr. 2016 (CEST)
Das würdest du bei mir mit deinem wichtigen Anliegen genau ein mal tun: Anklopfen und reinkommen, ohne dass ich "Herein!" sage. Ansonsten würdest du das beim Sachbearbeiter des Arbeitsamts ausprobieren dürfen - den stört's vielleicht nicht. Es hat einen Grund, wenn ich die Türe schließe - sonst steht sie offen.
Aber ich gebe zu, es hängt wohl sehr von der Branche und von den jeweiligen Gepflogenheiten vor Ort ab. --Snevern 08:17, 12. Apr. 2016 (CEST)
Ach so, du bist Wikipedianer. Na Scheff, dann komme ich halt eben nicht in dein Büro wenn ich das "Herein" nicht höre ("Herein, bitte" wäre besser, es gibt Leute denen muss man sowas sagen), und löse die Probleme selber. Womit du dich überflüssig gemacht hast. Grüss mir den Sachbearbeiter, unbekannterweise. --2A02:1206:45B4:A2E0:F137:154:FC7B:8550 10:04, 12. Apr. 2016 (CEST)
Leute mit Humor haben jedenfalls schon mal überdurchschnittlich gute Chancen bei mir, Leute mit mangelndem Respekt dagegen unterdurchschnittliche. Respekt und Höflichkeit auch im Umgang mit Untergebenen sind bei mir so selbstverständlich, dass da keiner ein "bitte" hinter dem "herein" braucht - du bekämst es also auch nicht.
Wenn du als mein Angestellter dein Problem selbst löst, habe ich mich nicht überflüssig gemacht, sondern du hast deinen Job gemacht und kannst ihn dann vielleicht auch behalten. Da du dein Problem ja auch ohne mich lösen konntest, hast du offenbar nur aus Faulheit geklopft und es erstmal auf die bequeme Weise versucht; da ist es vermutlich besser, dass ich das gar nicht mitgekriegt habe. --Snevern 10:34, 12. Apr. 2016 (CEST)
Es gibt da keine eindeutigen Regeln und hängt von dem jeweiligen Bürogebilde ab. Wenn man nicht durch Versuch & Irrtum gehen will, fragt man einfach nach, wie die Büroinsassen es gerne hätten.--Wikiseidank (Diskussion) 06:54, 12. Apr. 2016 (CEST)
Es spricht nichts dagegen, einzutreten, wenn man niemand antworten hört. Es gibt Leute, die sprechen sehr leise und Bürotüren, die dämmen einfach gut. Wenn aber drinnen eine Besprechung stattfindet, sagt man: "Oh, Verzeihung, ich wollte nicht stören!" oder etwas Ähnliches und schließt die Tür wieder. Ohne etwas zu sagen zu gehen, geht gar nicht! --Heletz (Diskussion) 08:15, 12. Apr. 2016 (CEST)
Also dann ist der Mensch hinter der Türe wirklich doof: Wenn er sich nicht darüber im Klaren ist, dass man ihn angesichts seiner eigenen Lautstärke und der Dicke der Tür draußen womöglich nicht hören kann. Ansonsten tut eigentlich der Chef gut daran, im Büro zu kommunizieren, wie er's gerne hätte. Hat der Neue das noch nicht mitgekriegt, kann er ja fragen - oder eben seine eigenen Erfahrungen machen: Keiner kriegt ernsthafte Probleme, wenn er eine Regel verletzt, die er nicht kennen kann (oder er ist dort auf jeden Fall verkehrt). --Snevern 08:21, 12. Apr. 2016 (CEST)
Das sind doch elementarste Höflichkeitsregeln. Wenn keiner antwortet, ist keiner drin oder will nicht gestört werden. Dann geht man wieder. Mehr gibts dazu nicht zu wissen und über mehr muss man auch nicht nachdenken. Einfach mal schauen ob doch jemand da ist, ist grob unhöflich. Nur wenn man einen Termin hat kann man davon ausgehen das einen der andere nicht gehört hat oder man seine Antwort nicht gehört hat. Dann klopft man nochmal. Türen sind nicht ohne Grund geschlossen. --2003:66:8936:4C51:2CEF:7AE2:7372:22A8 10:03, 12. Apr. 2016 (CEST)
Es ist ganz normal einzutreten, auch wenn man kein „Herein!“ vernimmt. Denn mal ist drinnen Lärm (offenes Fenster, Kaffeemaschine, was auch immer), aber oftmals einfach auch auf dem Flur. Wenn der eintretende dann selbst feststellt, dass er gerade stört (z.B. die von dir angesprochene Besprechung mit dem Chef), kann er sich entschuldigen und wieder gehen. Hat der „Büroinhaber“ gerade noch was zu erledigen, kann er darauf hinweisen und sagen, dass er zurückruft/vorbeikommt/die Person kurz drinnen oder draußen warten soll. Das hat alles auch mal überhaupt nichts damit zu tun, dass „Türen nicht ohne Grund geschlossen sind“. Bei mir laufen täglich hunderte Personen am Büro vorbei, die allesamt nicht hier arbeiten. Es ist einfach zu laut und zu viel los, als dass ich dauernd die Tür offen stehen habe. --KayHo (Diskussion) 10:08, 12. Apr. 2016 (CEST)
(BK) Super Diskussion hier! Aber die Vorredner-IP hat nicht recht: Türen schließt man auch, um störenden Lärm abzuhalten und in Ruhe arbeiten zu können. Oder es zieht sonst. Das heißt aber nicht, dass nicht ein Gespräch mit Kollegen oder Mitarbeitern trotzdem möglich wäre. Es wurde schon darauf hingewiesen, dass viele Bürotüren so dick sind, dass man keine Rückmeldung wahrnimmt. Insofern ist es durchaus passend, laut zu klopfen, nach ein paar Sekunden des Innehaltens hineinzugehen, und dann ggf. nach "Entschuldigung" wieder rauszugehen, falls besetzt. Und die ganz wichtigen Leute haben eh' ein Vorzimmer mit Wach-Drachen. --muns (Diskussion) 10:11, 12. Apr. 2016 (CEST)
Bin schwerhörig, höre gelegentlich das Klopfen nicht, fast immer das "Herein" durch die geschlossene Tür nicht. Als ich Chef war (und noch nicht schwerhörig) galt die Regel, daß man bei offener Tür (Regelfall) einfach 'reinkommen konnte, bei geschlossener Türe nur bei sehr (!) dringenden Sachen. Mein jetziger Chef hält es ähnlich. Da er ohnehin ein Stockwerk über mir residiert, rufe ich zu den Zeiten, zu denen er üblicherweise die Türe geschlossen hat, kurz an, ob er Zeit hat. Wenn er nicht abhebt, hat er keine.--80.129.129.56 14:54, 12. Apr. 2016 (CEST)
Ich kann nur aus meiner Erfahrung sprechen, aber in so einem Fall würde ich überhaupt gar nicht gestört werden wollen - egal, ob die Tür auf oder zu ist. --217.84.80.73 13:49, 14. Apr. 2016 (CEST)

Sakramente für "nicht ordentlich verheiratete" Katholiken

Nachdem durch das neue päpstliche Rundschreiben etwas Bewegung in das Problem der Wiederverheiratung geschiedener Katholiken gekommen ist, habe ich eine Frage zu einer weiteren Personengruppe: die nur durch Zivilehe Verheirateten. Welche Regelungen gelten für diese Gruppe, sind sie auch vom Empfang der Sakramente ausgeschlossen (bisher) und wenn ja, wird sich da auch künftig etwas ändern? --84.135.134.195 22:42, 11. Apr. 2016 (CEST)

Also vom Sakrament der Ehe sind sie schon mal nicht ausgeschlossen, das spenden sie sich gegenseitig. --WolfD59 (Diskussion) 22:47, 11. Apr. 2016 (CEST)
Die Frage ist falsch gestellt.
Der Papst ist nicht irgendein Bundestag oder ALDI-Vorstand, der heute beschließen kann, dass ab nächste Woche eine Änderung im Strafgesetzbuch oder im Butterpreis eintritt. Er könnte beinahe alles beschließen, weiß aber sehr wohl, dass die Kirche dabei nicht mitkäme. Er weiß, was die Kirche ist, nämlich ein Monstrum, ein überdimensionaler Ozeandampfer, ein weißer Berg wie Moby Dick, der von jeder noch so kleinen Revolution zerrissen würde. Revolutionären ist sowas gemeinhin wurscht; er fühlt sich aber verantwortlich für jede einzelne Seele und darf keine verlieren (es sind weit über eine Milliarde, aber irgendwie scheint er die andere sechs Milliarden auch auf dem Schirm zu haben, stärker als jeder seiner Vorgänger).
Der Mann steht in einem ICE am offenen Fenster und rast mit dreihundert Sachen durch die Lande; bisher hängt er sich gerade soweit raus, dass er nicht am nächsten Signal oder Tunnelmund zerschellt. Ein Millimeter mehr und – persönliche Angst vor Tod oder Rücktritt ist es wohl nicht, was ihm Vorsicht gebietet; aber er traut seinem Nachfolger (noch lange) nicht alles zu, was er tun möchte...
Daher die jetzige Strategie, von der er aber nicht zugeben darf, dass es eine Strategie auch nur sein könnte: Als oberster Gesetzesbrecher oder Revolutionär wäre er sofort absolut angreifbar. Als oberster Seelenhirte kann man ihm nicht viel sagen, wenn er ganz vorsichtig darauf hinweist, man könne doch eventuell in einigen Fällen auch mal nicht ins Gesetzbuch schauen, sondern den Einzelfall bewerten.
Dabei ist er aber Jesuit. Er kann also mehrdimensional um eine nennenswerte Zahl von Ecken denken und dabei freundlich lächeln, als schaue er nur bis zum nächsten Straßenbaum.
Mal als kleines Beispiel: Der Vorgänger (der viel revolutionärer war, als er von Norden gesehen wirkte) hat (endlich) den Limbus abgeschafft. Wie lange hat das gedauert? Wie formal fundiert musste er daran arbeiten (lassen)? Und doch ging es nicht ohne das Hintertürchen, man dürfe sowas natürlich doch denken! Und das war eine Kleinigkeit, eine theologische Spitzfindigkeit, die keinen großen Rückhalt unter den Laien hatte, keine großen praktischen Auswirkungen.
NfdA (Diskussion) 00:30, 12. Apr. 2016 (CEST)
Ich wollte mit meiner Frage hier keine polemische Diskussion lostreten. Ich möchte nur ganz einfach wissen: Darf ein Katholik, der nur zivilrechtlich verheiratet ist, nach geltendem Kirchenrecht die Kommunion erhalten? (Ich hoffe, die Frage ist jetzt klar genug und richtig gestellt.) --84.135.134.195 08:43, 12. Apr. 2016 (CEST)
Keine Polemik. Von mir nur eine Andeutung dazu, wie und warum vorsichtige Andeutungen Bergoglios nicht gleich Änderungen im Kirchenrecht bedeuten.
Aber ist denn bisher standesamtlich, aber nicht kirchlich verheirateten Katholiken die Kommunion verboten? Man müsste mal einen Katholiken fragen - aber ich glaube, das wird als Sünde, aber nicht als schwere Sünde gesehen. NfdA (Diskussion) 08:56, 12. Apr. 2016 (CEST)
s. Bertrand Russell, An Outline of Intellectual Rubbish (1950): Although there are many kinds of sin, seven of which are deadly, the most fruitful field for Satan's wiles is sex. The orthodox Catholic doctrine on this subject is to be found in St. Paul, St. Augustine, and St. Thomas Aquinas […] Marriage is nominally indissoluble, but many people who seem to be married are not. In the case of influential Catholics, some ground for nullity can often be found, but for the poor there is no such outlet, except perhaps in cases of impotence. Persons who divorce and remarry are guilty of adultery in the sight of God. The phrase "in the sight of God" puzzles me. One would suppose that God sees everything, but apparently this is a mistake. He does not see Reno, for you cannot be divorced in the sight of God. Registry offices are a doubtful point. I notice that respectable people, who would not call on anybody who lives in open sin, are quite willing to call on people who have had only a civil marriage; so apparently God does see registry offices. --Edith Wahr (Diskussion) 10:29, 12. Apr. 2016 (CEST)
Sakramente werden nacheinander vergeben. Firmung setzt die Taufe voraus, Heirat die Firmung. Die Kommunion setzt aber nicht die Ehe voraus oder wird hinfällig weil man gesündigt hat. Vorher muss man nur gebeichtet haben. --2003:66:8936:4C51:2CEF:7AE2:7372:22A8 11:00, 12. Apr. 2016 (CEST)
Laß es lieber… An den Fragesteller: IANAC, aber die Gültigkeit einer Ehe ist nicht davon abhängig, ob sie etwa nur standesamtlich geschlossen wurde.--Turris Davidica (Diskussion) 11:04, 12. Apr. 2016 (CEST)
Natürlich ist die Gültigkeit der (kirchlichen) Ehe davon abhängig, siehe Formmangel_(Kirchenrecht)#Folgen_eines_Formmangels. Wer ohne Segen des kath. Geistlichen mit Frauen Unzucht treibt (das ist es dann nämlich, egal wie alt), darf nicht zu den Sakramenten zugelassen werden. -- Ian Dury Hit me  12:18, 12. Apr. 2016 (CEST)
Ich schickte ja „I am not a canonist“ voraus, es handelt sich um einen Formmangel. Mir war erinnerlich, daß eine rein standesamtlich geschlossene Ehe nicht automatisch ungültig sei. Warum allerdings jemand, der sich um die Form des einen Sakraments nicht schert, sich nun intensiv mit der Form und dem Zustandekommen anderer Sakramente beschäftigen sollte, steht auf einem ganz anderen Blatt. --Turris Davidica (Diskussion) 12:39, 12. Apr. 2016 (CEST)
Wer sich um die Form des einen Sakraments nicht schert (= zivilverheirateter Katholik) könnte trotzdem den Wunsch haben, an der Kommunion teilzunehmen. Praktisch kann er das natürlich. Es geht nur darum, ob das kirchenrechtlich erlaubt ist. Benutzer:Ian Dury (bisher einzige konkrete Antwort) zufolge wäre es das nicht. Weitere Ansichten? --84.135.134.195 13:22, 12. Apr. 2016 (CEST)
Hinweis: Diese Seite ist für allgemeine Wissenfragen gedacht, weniger bis gar nicht für Ansichten. Die Frage ist aus meiner Sicht beantwortet. Die Regelungen gelten nach wie vor. Wie damit umzugehen ist, hat Franz ja neulich erst in aller Vorsichtigkeit kundgetan. -- Ian Dury Hit me  14:22, 12. Apr. 2016 (CEST)
Die IP fragt doch gerade nach konkreten Antworten wie deiner (ob man die dann nun "Ansichten" oder sonstwie nennt). --Eike (Diskussion) 14:30, 12. Apr. 2016 (CEST)
Den zuständigen Priester und oder Bischof fragen, vor allem, wenn man den als aufgeschlossen kennt. Man muss dabei auch nicht darauf hinweisen, dass doch der Papst neulich - das weiß der selber. NfdA (Diskussion) 14:35, 12. Apr. 2016 (CEST)
Gilt das auch für Frauen? Und für Pfarrköchinnen? --62.202.183.131 13:48, 12. Apr. 2016 (CEST)

Ich habe nur deshalb gefragt, weil das ein relativ häufiger Fall sein dürfte, und ich daher vermute, dass das schon einmal irgendwo geregelt wurde. Mich interessiert diese Regelung. Falls eine verbindliche Regelung (Kommunion erlaubt: ja oder nein) allerdings nicht existieren sollte, wäre das auch interessant. Es könnte ja sein, dass sich hier jemand in solchen formaltheologischen Fragen auskennt, keinesfalls wollte ich persönliche Ansichten erfragen. --84.135.154.92 17:54, 12. Apr. 2016 (CEST)

Auf Diskussion:Formmangel_(Kirchenrecht) werden sogar die Hausnummern der Regeln genannt. Den Codex Iuris Canonici könnte der Pfarrer deiner Wahl zur Hand haben. 85.178.101.25 19:15, 12. Apr. 2016 (CEST)
Es wurde nach Antworten gefragt, diese wurden gegeben. Wenn man natürlich nur an dem interessiert ist, was einen in der eigenen Auffassung bestätigt, fragt man vielleicht gar nicht erst. Immerhin steht hier allerdings „Auskunft“ obendrüber. Eine dieser Auskünfte lautete, dass sich die Frage nach der Form sich durchaus auch in Bezug auf andere Sakramente stellen kann, etwa die Kommunion, an der man gerne „teilnehmen“ möchte. --Turris Davidica (Diskussion) 10:22, 13. Apr. 2016 (CEST)
Ich bin nochmal der Fragesteller. Dies letzte Statement habe ich inhaltlich nicht ganz verstanden. Meine Frage (12. April, 8:43) hatte nicht die Absicht, inhaltlich irgendetwas "bestätigt" zu bekommen, sondern eine simple Wissenslücke bei mir zu füllen.
Ich habe den empfohlenen Link: Diskussion:Formmangel_(Kirchenrecht) konsultiert, habe in der dortigen komplizierten Sachdiskussion auch keine klare Antwort gefunden. Auf der entsprechenden Artikelseite Diskussion:Formmangel_(Kirchenrecht)#Folgen des Formmangels steht die Antwort im zweiten Satz. Wenn ich diesen richtig verstehe, ist der nur zivilverheiratete Katholik, egal ob vorher geschieden oder nicht, vom Kommunionempfang ausgeschlossen. Vielen Dank an die IP für diesen Seitenhinweis. Die Empfehlung "Pfarrer oder Bischof fragen" halte ich allerdings für unpraktikabel, es handelt sich bei der nachgefragten Fallgestaltung um etwas sehr Häufiges und keinen abwegigen Spezialfall. --84.135.154.92 12:36, 13. Apr. 2016 (CEST)
Vielleicht ein Mißverständnis: ich hatte den Eindruck, diejenigen, die geantwortet hatten, hätten sich bemüht, diese Wissenslücke zu füllen. Dann kam die angebliche Unterscheidung zwischen „konkreten Antworten“ und „Ansichten“ in Spiel, daraufhin war ich leicht angefressen. Den jeweiligen Seelsorger zu fragen, hielte ich tatsächlich für eine Möglichkeit (wieso sollte das eigentlich „nicht praktikabel“ sein? Immerhin käme ja auch die Heilung in der Wurzel [Sanatio in radice] in Frage). Eherecht der katholischen Kirche hilft hier womöglich weiter.--Turris Davidica (Diskussion) 12:48, 13. Apr. 2016 (CEST)
In der Praxis gilt, dass jede Person, die zur Kommunion vorgeht und die Hände aufhält, eine Hostie bekommt. Egal, ob sie evangelisch, wiederverheiratet, geschieden, in Zivilehe lebend oder ein bekannter Drogenhändler ist. Es wird auch nicht gefragt, ob die Person vorher mal beichten war oder nicht. Das Gewissen der Person entscheidet. Das hat man uns jedenfalls im Kommunionhelferkurs so beigebracht. Was das Lehramt dazu sagt, ist natürlich was anderes, nämlich, dass die Person in Sünde lebt und vorher beichten und ihr Leben ändern muss, bzw. als evangelischer Christ erst formal konvertieren muss. Aber nicht erst seit Franziskus gibt es einen Unterschied zwischen Dogmatik und Pastoral. --Sr. F (Diskussion) 17:28, 13. Apr. 2016 (CEST)
Wie wahr! Die dogmatische Frage wäre soweit geklärt. Dass die Praxis anders läuft, ist auch klar. Ich kenne noch aus dem Religionsunterricht der 60er Jahre für den "unwürdigen" Kommunionempfang den Begriff "Gottesraub", ob der noch aktuell verwendet wird, weiß ich nicht. Damals wurde die Dinge weitgehend dogmatisch gesehen. Jesus scheint auch nicht so streng gewesen zu sein, immerhin ließ er den Judas zum Abendmahl zu. --84.135.156.173 20:04, 13. Apr. 2016 (CEST)
Als Ministrant habe ich hingegen durchaus schon Rückfragen bzw. ausdrückliche Belehrungen von der Kommunionbank gehört („Sind Sie getauft?“ Nun, dann lassen Sie sich taufen, dann können Sie auch die Kommunion empfangen“). Manche Antwort des Kommunikanten („Danke!“) legt solche Fragen auch irgendwie nahe. Das Gewissen des einzelnen spielt eine wichtige Rolle, d'accord. Der Gesichtspunkt, der am Rande berührt wurde, ist jedoch, wieso das Gewissen in der wichtigen Frage, bin ich überhaupt disponiert, die heilige Kommunion zu empfangen?, bei manchem offenbar seine Stimme kaum oder gar nicht erhebt. --Turris Davidica (Diskussion) 09:49, 14. Apr. 2016 (CEST)

"Freiheit der Kunst"

Zur Zeit wird Deutschland weit über J. Böhmermann und sein "Schmähgedicht" debattiert und dabei immer wieder die Freiheit der Kunst beschwören. Das scheint jedoch nicht für alle zu gelten. Es gibt nicht wenige Spielfilme, die auf Grund ihrer Gewaltdarstellung in Deutschland beschlagnahmt sind. Der Regisseur eines solchen Films (und auch potenzielle Kunden der DVDs) sehen diese Filme als Kunst an. Es sind Spielfilme mit Schauspielern, die Darstellungen sind "gespielt" und entsprechen nicht der Realität. Es sind SPIELFILME. Man mag das mögen oder nicht, die Filme sind trotzdem "gespielt" und für die Macher und Zuschauer Kunst. Wieso darf eine Kommission entscheiden, was Kunst ist und was nicht bzw. diese Kunst beschlagnahmen? Wenn Kunst im Auge des Betrachters gilt, wie können dann einzelne Personen entscheiden, was Kunst ist und was nicht? Wenn die Freiheit der Kunst gilt, wieso gilt diese Freiheit nicht für alle? Danke. --87.184.154.65 10:16, 13. Apr. 2016 (CEST)

Freiwillige Selbstkontrolle der Filmwirtschaft enthält alle notwendigen Elemente.
Merkwürdig finde ich aber den Zusatz "Freiwillige", da man ja keine "Unfreiwillige" hat... "Selbstkontrolle der Filmwirtschaft" sollte reichen. 213.169.163.106 11:05, 13. Apr. 2016 (CEST)
Das heißt also, der Vertrieb der "Kunst" bittet um Freigabe und diese hängt dann von der Meinung einer Minderheit ab? Bei Freigaben für Filme im Zuge des Jugendschutzes durchaus verständlich. Aber nicht für Erwachsene. Also kann man vereinfacht sagen, es gibt keine Freiheit der Kunst, da diese (auch für volljährige Personen) von einer Kommission bewertet wird (wenn auch freiwillig, was dann aber dem finanziellen Gedanken des Vetriebs zu Grunde liegt) und dann eventuell aus dem Verkehr gezogen wird? Was dann wiederum den Artikel "Zensur" ins Spiel bringt, oder...;-)?--87.184.154.65 11:33, 13. Apr. 2016 (CEST)
Neuer Begriff. Kunstfreiheit und Zensur enthalten die nötwendigen Informationen. 213.169.163.106 11:50, 13. Apr. 2016 (CEST)
BK Der Kunstbegriff ist leider eine Hure, auch noch eine vergewaltigte, denn jeder nutzt den Begriff. Die Sendung mit dem Herrn Bömermann ist eigentlich auch keine Kunstsendung, sondern dient eher der Unterhaltung und die Machwerke sind nach meinem Empfinden auch keine Kunst. Das mag der Urheber anders sehen, ich habe ein anderes Kunstempfinden.--2003:75:AF0F:9100:2908:2045:2A8E:A9D8 11:59, 13. Apr. 2016 (CEST)
Deine Empfdindung hat auf Begriff und Wesen der Kunst sowie Art. 5 Abs. 3 GG keinen Einfluss. --Rôtkæppchen₆₈ 12:02, 13. Apr. 2016 (CEST)
(BK)Filme für Erwachsene brauchen keine Freigabe. Die dürfen aber nur Erwachsenen gezeigt oder an sie verliehen werden. Auch zieht die FSK keine Filme aus dem Verkehr. Meistens werden von der FSK beanstandete Filme so geschnitten, dass sie Jugendlichen gezeigt werden können. Das Z-Wort ist hier vollkommen unangebracht, da Z. durch den Staat geschieht. Die FSK ist aber eine von einem e.V. betriebene GmbH, also etwas vollkommen Privates. Dias von Dir als Minderheit bezeichnete Gremium wird von 1100 Unternehmen der Film-, Fernseh- und Videowirtschaft betrieben. Wieviele Unternehmen der Film-, Fernseh- und Videowirtschaft gibt es denn tatsächlich, dass diese 1100 Unternehmen eine Minderheit darstellen? --Rôtkæppchen₆₈ 12:00, 13. Apr. 2016 (CEST)
1. Wer oder was ist BK?
2. Ich rede nicht von "Filmen für Erwachsene" (Pornos), sondern von Spielfilmen.
3. "Auch zieht die FSK keine Filme aus dem Verkehr." Wer reicht die Filme beim Gericht zur Beschlagnahme ein?
4. "Meistens werden von der FSK beanstandete Filme so geschnitten, dass sie Jugendlichen gezeigt werden können." Ich beziehe mich ausdrücklich nicht auf geschnittene Filme und auch nicht auf Jugendliche, sondern auf Kunst für die Allgemeinheit.
5. "Das Z-Wort ist hier vollkommen unangebracht, da Z. durch den Staat geschieht. " Durch den Staat? Das sieht im Artikel "Zensur" aber anders aus.
6. "von Dir als Minderheit bezeichnete Gremium wird von 1100 Unternehmen der Film-, Fernseh- und Videowirtschaft betrieben." Ich rede nicht von Betreibern, sondern von Entscheidern. Laut Wikipedia: "Etwa 250 Prüfer sind ehrenamtlich für die FSK tätig."
Ich könnte noch einiges aus deinem Kommentar hier anführen, aber das wird mir doch zu anstrengend. --87.184.154.65 12:24, 13. Apr. 2016 (CEST)
1. Bearbietungskonflikt. 2. Das ist unerheblich. 3. Die Bundesprüfstelle für jugendgefährdende Medien 5. Du hast sicher Belege für eine Zensur in Deutschland im Sinne des Artikels a) durch staatliche und b) durch nichtstaatliche Stellen. 6. Beschwer Dich bei der FSK. --Rôtkæppchen₆₈ 12:40, 13. Apr. 2016 (CEST)
Ganz tolle Reaktion auf meine Bemerkungen. Danke für Punkt 1 (ehrlich). Und natürlich für Punkt 6.--87.184.154.65 12:52, 13. Apr. 2016 (CEST)
Kannst du mal Beispiele nennen für in Deutschland generell verbotene Filme? Rainer Z ... 12:38, 13. Apr. 2016 (CEST)
Wenn du verboten mit beschlagnahmt gleich setzten magst, dann unter Kategorie:Beschlagnahmter Film. Sorry, Kategorien lassen sich wohl nicht verlinken. Ich kann es zumindest auf die Schnelle nicht...--87.184.154.65 12:52, 13. Apr. 2016 (CEST)
Siehe Verbreitungsverbot. Wie viele Filme in D davon betroffen sind kann man aus dieser Übersicht ableiten. Relevant sind dabei die Listen B und E. Da Du ein beispiel wolltest: "A Serbian Film" in der ungeschnittenen oV. --Joyborg 13:00, 13. Apr. 2016 (CEST)
Danke. Es werden im Artikel jedoch Film der Liste A, B und E zusammen gefasst ("Filme (2193 Titel)". Oder habe ich etwas übersehen?--87.184.154.65 13:49, 13. Apr. 2016 (CEST)
Das stimmt. Die genauen Zahlen für die einzelnen Listen konnte ich auf die Schnelle auch woanders nicht finden. Die BPjM veröffentlicht aber im Bundesanzeiger monatlich alle Neuaufnahmen und Umschreibungen, nach Listen getrennt ([8], dort nach "Bundesprüfstelle für jugendgefährdende Medien" suchen, Direktlink geht nicht). --Joyborg 14:14, 13. Apr. 2016 (CEST)
(nach BK) @87.184.154.65: Sorry, warum sollen nur Pornos „Filme für Erwachsene“ sein, und warum sind Action-/Horrorfilme Kunst und keine Unterhaltung? Bist du etwa der Meinung, Kinder sollten sich Filme mit extremen Gewaltdarstellungen ansehen (dürfen)? Übrigens gibt es bei Fernsehsendungen einen engen Zusammenhang zwischen Altersgerechtigkeit und Sendezeit. „Extra 3“, die Sendung, in der Böhmermanns „Schmähkritik“ ausgestrahlt wurde, läuft nicht ohne Grund erst spät am Abend, denn auch sie ist eher für Erwachsene gedacht. Auch das Fernsehen unterwirft seine Eigenproduktionen einer Art FSK. Deine Argumentation geht m.E. am Thema vorbei, denn die aktuelle Debatte dreht sich um die Freiheit der Kunst vor Strafverfolgung, nicht um eine totale Freiehit der Kunst in jedweder Hinsicht einschließlich einer Abschaffung des Jugendschutzes. --Gretarsson (Diskussion) 13:08, 13. Apr. 2016 (CEST)
Sorry, Böhmermanns "Schmähkritik" lief im "Neo Magazin Royale" auf ZDF Neo. In Extra3 (NDR) lief der Song "Erdowie, Erdowo, Erdogan", der mit Böhmermann nichts zu tun hat. --Joyborg 13:47, 13. Apr. 2016 (CEST)
Ah, OK, danke, ich hatte mir das auf YouTube angesehen und in dem Video ist eine Einblendung „X3.de“ zu sehen, ich dachte daher, es wäre ein Gastbeitrag bei „extra 3“ gewesen, zumal ich im Hinterkopf hatte, dass die (auch) einen Schmähsong gegen Erdogan gemacht hatten. Auf YT steht ja auch nochmal „NeoMagazinRoyale“ dabei (wenngleich auf solche Angaben auch nicht immer verlass ist). Nichtsdestoweniger läuft auch das NMR erst spät abends bzw. nachts, spielt also hinsichtlich meiner Argumentation keine Rolle, ob NMR oder X3. --Gretarsson (Diskussion) 14:21, 13. Apr. 2016 (CEST)
Lies' doch bitte meine Texte richtig durch. Wo schreibe ich denn, dass sich Kinder Filme mit "extremen Gewaltdarstellungen ansehen" sollen/können? In Kommentar 2 schreibe ich ausdrücklich "Bei Freigaben für Filme im Zuge des Jugendschutzes durchaus verständlich." Verdreh' mir doch nicht die Worte im Mund. Und ob Action-/Horrorfilme Kunst sind oder nicht, hängt von der Intention der Macher ab. Die einen sehen es als schlichte Unterhaltung, andere sehen es als Kunst an. Meine Frage war, wo beschlagnahmte Filme in Bezug auf die Freiheit der Kunst stehen. Und "Deine Argumentation geht m.E. am Thema vorbei, denn die aktuelle Debatte dreht sich um die Freiheit der Kunst vor Strafverfolgung" ist erst recht nicht mein Thema, denn meine Fragen beziehen sich nicht auf die konkrete Causa "Böhmermann", sondern auf die Relation von Kunst (oder halt nicht) und Freiheit von Kunst im allgemeinen. Dieses ganze Brimborium um Böhmermann hat mich nur veranlasst, allgemein über die Freiheit der Kunst (anhand von beschlagnahmten Filmen) Fragen zu stellen. Mehr nicht. --87.184.154.65 13:32, 13. Apr. 2016 (CEST)
OK, danke für die Klarstellung, es geht also nur um tatsächlich (auch für Erwachsene) verbotenes Material. Aus dem oben schon verlinkten Artikel Verbreitungsverbot geht m.E. relativ eindeutig hervor, dass Kunstfreiheit dort aufhört, wo die „Kunst“ strafrechtlich relevante Elemente enthält. Im Einzelfall kann die Lage da recht eindeutig sein (bei Kinder- oder Tierpornos bspw.), manchmal aber eben weniger eindeutig und fast schon wieder geschmacksabhängig (und das ist wahrscheinlich die Grauzone, auf die du dich beziehst). Kunst ist halt ein sehr dehnbarer Begriff (oder, wie oben jemand treffend sagte, „der Kunstbegriff ist leider eine Hure, auch noch eine vergewaltigte“) und über Geschmack lässt sich eh trefflich streiten. Deine Eingangsfrage geht also über eine reine Wissensfrage deutlich hinaus... --Gretarsson (Diskussion) 14:08, 13. Apr. 2016 (CEST)
P.S. Fälle, in denen die Kunstfreiheit (außerhalb der Filmkunst) mehr oder weniger breit und mehr oder weniger Kontrovers diksutiert wurden sind zum Beispiel Gunter von Hagens „Körperwelten“ (Leichenfledderei) oder die „Arschficker“-Skulpturen (Beleidigung) von Günter Schumann. --Gretarsson (Diskussion) 14:38, 13. Apr. 2016 (CEST)
Ja, genau. Es ging mir darum, wer, wann, wie und wieso definiert, was freie Kunst ist. Bei eindeutig geregelten gesellschaftlichen Taboos, wie die von dir genannten, braucht man sicherlich nicht zu diskutieren. Es geht mir tatsächlich um die "Grauzone" und ob wir als erwachsene Gesellschaft uns von Kommssionen vorschreiben lassen sollen oder können, was sie für relevant und gesund für unseren Geist halten. Andererseits fühle ich mich als mündiger Bürger natürlich angesprochen, wenn das "Gedicht" eines Komikers (von dem ich nichts halte, weder vom Gedicht noch von Böhmermann, aber das ist natürlich meine persönliche Meinung) als Kunst empfunden wird, andere Kunstrichtungen wie z.B. Film oder auch Musik/Texte dem mündigen Bürger vorenthalten werden. Es geht auch nicht darum, dass ich jetzt etwaige Filme haben muss, sondern es geht mir um den Umgang mit etwas, was Kunst ist oder sein mag und das wiederum von einigen Leuten für Erwachsene als nicht zumutbar abgestempelt wird. Als Beispiel soll mal der Film Blutgericht in Texas gelten, der in Deutschland Jahrzehnte lang (wegen vermeintlicher Gewaltdarstellung) beschlagnahmt war und erst seit ein paar Jahren ungeschnitten verfügbar ist. Viele Leute haben schon in den 1980er darüber diskutiert, die wenigsten haben den Film wirklich gesehen. Als Psycho-Thriller entspricht er meinem Verständnis (und dem vieler anderer) von Kunst. Jeder Tatort mit T. Schweiger zeigt mehr vordergründige Gewalt. Es ist diese Diskrepanz zwischen dem Anspruch auf eine freie Kunst und die Entscheidungen von Kommissionen etwas als Schund (und ist Schund nicht auch Kunst?) und nicht gesellschafts-relevant zu bezeichnen und für den Interessenten unzugänglich zu machen. Und ja, meine ursprüngliche Frage geht über eine Wissensfrage hinaus. Aber wenn schon öffentliche rhetorische Tricks (wie die Einleitung von Böhmermann vor seinem Gedicht) die Kunstfreiheit rechtfertigt, darf ich doch einige simple rhetorische Fragen stellen...;-) Es passt auch in die Zeit, interessiert mich aber auch wirklich...--87.184.154.65 16:33, 13. Apr. 2016 (CEST)
Es gibt auch noch die Vorbehaltsfilme, durch die man sofort Nazi wird, wenn man sie ansieht. Die Frage ist ja auch: sollte der "mündige Bürger" alles dürfen? Bei harten Drogen muss er dann eben selbst sehen, dass er nicht abhängig wird, bei Schrottfimen, dass er keinen Psycho-Schaden erleidet. Oder hat der Staat eine Schutzpflicht gegenüber seinen Bürgern? Und kann die dann wiederum ausgehebelt werden, wenn man sagt "Das ist Kunst"? --Optimum (Diskussion) 20:26, 13. Apr. 2016 (CEST)
Berechtigte und interessante Fragen. Vor allem die letzte. Wobei "harte Drogen" per se verboten sind (es sei denn man bekommt sie verschrieben). Ich würde "harte Drogen" aber auch nicht unbedingt als Kunst definieren...;-) Nun ja, wer weiß? ;-)--87.184.154.65 21:41, 13. Apr. 2016 (CEST)
Ob dir vom Staat als mündigem Büger nun Filme oder Drogen verboten verden, ist prinzipiell eigentlich garkein so großer Unterschied. Aber wie gesagt, das ist ein Themenkomplex, über den man ausgiebig philosophieren kann und wo es auch stark von der Weltanschaung abhängt, welche Meinung man dazu jeweils vertritt. Eigentlich nichts für die WP:Auskunft. --Gretarsson (Diskussion) 13:29, 14. Apr. 2016 (CEST)

Warum für Mord nur 15 Jahre??

Ich frage mich, warum ein Mensch der einen anderen umbringt dafür nur 15 Jahre sitzen muss? Und danach weiter machen kann? Genau so weiterleben wie vorher? Ich meine im heutigen Knast hast du ja Fernsehen auf der Zelle und Einzelzelle und Schwimmbad und darfst sogar Geld verdienen, wo ist da noch die Bestraftung gibt doch sogar Hafturlaub habe ich vor kurzem gelesen, dass du 21 Tage raus darfst, während am Beispiel des Herborner Polizistenmörder die vier Kinder lebenslang auf ihren Vater verzichten müssen, muss der Täter maximal 15 Jahre sitzen und kann dann entspannt weiter leben? Warum gibt es nicht für einen Mord wirkliches lebenslang?--Instaerar (Diskussion) 10:20, 13. Apr. 2016 (CEST)

Lebenslange_Freiheitsstrafe hast du gelesen? Und verstanden? Dann verstehe ich deine Schreibe nicht ganz. Lebenslange Freiheitsstrafe heißt eben nicht „nur 15 Jahre sitzen“, sondern auf unbestimmte Zeit, mindestens aber 15 Jahre. Das ist etwas anderes, oder? Den Rest deines Stammtischgesabbels („danach weiter machen kann“) möchte ich am liebsten ignorieren. Bei der Freiheitsstrafe geht es doch nicht darum, es einem Täter unmöglich zu machen, eine entsprechende Tat nochmals zu begehen (sonst müsste ja auch auf kleinen Ladendiebstahl schon „lebenslänglich“ stehen), sondern darum, ihm eine bestimmte Auszeit aufzuzwingen, um seine Persönlichkeit und Lebensführung zu überdenken (das ist meine laienhafte Formulierung und keinesfalls juristisch wasserdicht, aber es erklärt, wieso das beliebte Argument „der kommt nach 15 Jahren raus, aber die Opfer leiden lebenslang“ keines ist, im Strafrecht geht es nicht um Ausgleich von Leid!). Wenn nach den 15 Jahren ein Fachmann der Ansicht ist, der Mensch stelle keine Gefahr mehr da und könne wieder ein nützliches Glied der Gesellschaft sein, dann wird der Rest der Strafe auf Bewährung ausgesetzt. Wenn nicht, kommt der Mensch in Sicherheitsverwahrung. Er kann also in keinem Fall „weitermachen“. Daß diese Gutachter sich in Einzelfällen irren können, steht außer Frage, es sind auch nur Menschen. Aber es ist nur im Fernsehkrimi und im Kino so, daß ein nach 15 Jahren entlassener Mörder laut auflacht und gleich wieder straffällig wird. Tatsächlich ist, soweit ich weiß, die Rückfallquote eher klein (Achtung, von den nicht Rückfälligen steht nichts in der BILD). --Kreuzschnabel 10:33, 13. Apr. 2016 (CEST)
(nach BK)Wer die Vorfahrt missachtet und dadurch einen anderen tötet, kommt meist mit einer Geldstrafe davon (sofern keine besondere Fahrlässigkeit, Vorsatz oder Wiederholungstat vorliegt). Das Opfer ist trotzdem tot. Auge um Auge, Zahn um Zahn ist seit Jahren nicht mehr sonderlich modern.--Vertigo Man-iac (Diskussion)
Was ist denn daran modern, einem wie den Schaffner der in Bad Aibling 9 Menschen getötet und 9 Familien zerstört hat, nur weil sein Smartphone wichtiger war, für den Rest des Lebens weg zu sperren? Ist es Human immer ein auge zu zudrücken? Der wurde nach vier Monaten erst verhaftet warum nicht umgehend?--Instaerar (Diskussion) 10:39, 13. Apr. 2016 (CEST)
Was ist daran human, die Todesstrafe zu exekutieren? Sind dadurch die Mörder ausgestorben? War das etwas erfolgreich? --Heletz (Diskussion) 10:46, 13. Apr. 2016 (CEST)
Go to WP:Café. :-) VG --Apraphul Disk 10:46, 13. Apr. 2016 (CEST)
Das kann man ja langsam nicht mehr ernst nehmen. Du möchtest dich bitte über die Unterschiede zwischen Mord, Totschlag und fahrlässiger Tötung schlau machen. Hinweis: Hatte der Fahrdienstleiter (kein Schaffner) die Absicht, die Züge kollidieren und die elf (nicht neun) Menschen sterben zu lassen? Hat er eine bewußte Entscheidung getroffen: „Nein, ich stelle das Signal jetzt nicht um, sollen die Züge ruhig zusammenkrachen, das Handyspiel ist mir wichtiger“? Das sind die Fragen, mit denen sich Gerichte herumschlagen müssen, und das zu Recht! Wie oft schaust du denn beim Autofahren mal für zwei Sekunden nicht nach vorn? Reines Glück, daß dir bislang wohl noch nie genau dann ein Kind vors Auto gelaufen ist. Wenn dir das passiert wäre, wärst du dann auch für eine superharte Strafe, die mindestens so lange dauert wie die Eltern auf ihr totes Kind verzichten müssen? Denk da mal drüber nach. --Kreuzschnabel 10:49, 13. Apr. 2016 (CEST)

Die Moderation sollte den Unsinn hier beenden! Der Vorschlag mit dem Cafe ist wirklich gut. --Heletz (Diskussion) 10:56, 13. Apr. 2016 (CEST)

Diesen Unsinn find ich aber an einem langweiligen Arbeitstag, wie heute, recht unterhaltsam ;o) --Susad 1978 (Diskussion) 12:11, 13. Apr. 2016 (CEST)

+1: Mit Wikipedia kriegt man die Arbeitszeit besser rum als mit jedem Handyspiel … -- HilberTraum (d, m) 12:53, 13. Apr. 2016 (CEST)
Die ersten zwei Fragen des Fragestellers beantwortet die im Artikel Lebenslange Freiheitsstrafe verlinkte Entscheidung des Bundesverfassungsgerichts BVerfGE 45, 187. --Rôtkæppchen₆₈ 12:34, 13. Apr. 2016 (CEST)

Über jedes Verbrechen kann man diskutieren, wie groß die Strafe sein soll. Jeder Mensch hat seine eigene Meinung darüber wie hart ein Mord bestraft werden soll, letztendlich sind die mindestens 15 Jahre ein Mittelwert, ein Kompromiss zwischen vielen Vorschlägen. Andere Länder haben höhere oder niedrigere Strafen und auch dort wird diskutiert, ob es richtig sei. Letztendlich gibt es nicht die ideale Strafe für Mord bzw. jedes andere Verbrechen. --84.139.200.32 13:04, 13. Apr. 2016 (CEST)

Ja, genau, Knast ist eigentlich Urlaub. Man wünscht es ja eigentlich keinem, aber viellleicht solltest du mal im Bau sitzen, nur ein, zwei Wochen, dann wüsstest du, was Freitsentzug tatsächlich bedeutet, was es bedeutet, nicht mal eben einfach irgendwo hingehen zu können, wo man gerade Lust hat hinzugehen, ein im Prinzip fremdbestimmtes Leben führen zu müssen. Im Übrigen sind Fernsehen und Arbeit Privilegien, die längst nicht jeder Insasse genießt bzw. genießen darf. Der Arbeitslohn liegt irgendwo zwischen 1 und 2 Euro die Stunde. Sex und Alkohol gib’s auch (offiziell) keinen. Wenn du mich fragst, ein Scheißleben ist das. --Gretarsson (Diskussion) 15:18, 13. Apr. 2016 (CEST)

Ergänzend: Wir haben hier auch den ganz passablen Artikel Strafzwecktheorie. Die (falsche) Vorstellung, nach 15 Jahren käme ein lebenslänglich Verurteilter frei (weil die Begriffe „Gesetzliche Mindesverbüßungsdauer“ und „Strafrestaussetzung“ nicht wahrgenommen oder verstanden werden) und die Unkenntnis darüber, dass es sich beim Freikommen grundsätzlich um Aussetzung auf Bewährung handelt, ist weit verbreitet. Über die tatsächliche Strafdauer gibt mit vielen Statistiken z.B. Auskunft: Axel Dessecker: Lebenslange Freiheitsstrafe und Sicherungsverwahrung. Dauer und Gründe der Beendigung im Jahr 2010. KrimZ, Kriminologische Zentralstelle e.V., Wiesbaden 2012, ISSN 1869-4764. Für das Berichtsjahr wird bei der Vollzugsdauer ein arithmetisches Mittel von 17½ Jahren angegeben (S. 15) Dass das arithmetische Mittel durch grundlegend unterschiedliche Fallgruppen zustande kommt wird in diesem Zitat deutlich: „Während die Strafrestaussetzungen überwiegend nach 15- bis 20-jähriger Verbüßungszeit erfolgten (Tabelle A.11), fällt auf, dass die gesetzliche Mindestdauer von 15 Jahren in einem Fall deutlich unterschritten wurde. Dieser Gefangene wurde nach einer Verbüßungsdauer von 7½ Jahren im Alter von 64 Jahren entlassen. Das Absehen von weiterer Strafvollstreckung aufgrund aufenthaltsrechtlicher Entscheidungen erfolgte in zwei Dritteln der Fälle vor Erreichen der 15-Jahres-Grenze, die nach der Rechtsprechung bei den Fällen des § 456a StPO nicht eingreift. Frühzeitigere Abschiebungen bei lebenslangen Freiheitsstrafen werden durch Richtlinien der Landesjustizverwaltungen teilweise deutlich erleichtert.“ (S. 17) Tatsächlich sind die verbüßte Haftstrafen, wenn man die Fälle der Aussetzung wegen Krankheit, Abschiebung oder das Ende der Haftzeit durch Suizid herausrechnet, generell für den Großteil der Verurteilten wesentlich länger als die als arithmetisches Mittel errechnete Haftdauer von 17½ Jahren. (Mehr dazu bei Dessecker 2012, a.a.O.)
Ein weiterer Aspekt, wenn man vom Strafzweck absieht und ein durchaus berechtigtes Schutzbedürfnis berücksichtigt, wäre die Rückfallquote bei Mordfällen. Unter dem etwas sperrigen Titel Legalbewährung nach strafrechtlichen Sanktionen veröffentlicht das Bundesministerium der Justiz und für Verbraucherschutz Rückfallstatistiken. Während 2007 von der Gesamtzahl der Straffälligen 35% rückfällig wurden, ist bei dem Delikt Mord und Totschlag die Rückfallquote mit 18% deutlich niedriger (Spiegel Online 14. Februar 2014) Ausführlichere internationale Zahlen und Verweis auf entsprechende Studien lese ich bei Gregor Groß: Deliktbezogene Rezidivraten von Straftätern im internationalen Vergleich, Diss. 2004
Bei etwa 300 Mordopfern im Jahr (statista.com), und in etwa der gleichen Anzahl von Totschlagopfern (wir haben da auch für den Vergleich den Artikel Tötungsrate nach Ländern) also einer Tötungsrate (im Jahr 2011) von 0,8 Tötungsdelikten pro 100.000 Einwohner würde ich erwarten, dass man sich über andere Todesarten im gleichen Verhältnis mehr aufregen sollte. Die Bundesanstalt für Arbeitsschutz und Arbeitsmedizin registriert für 2013 insgesamt 3.542 Verkehrstote. Darüber könnte man sich also mindestens 10fach so stark erregen wie über die Mordopfer. Wie heißt es so schön: Ermordet werden ist Teil des Lebens. Aber Totgefahrenwerden ist auch Teil des Lebens. Und: „Mit 9.765 Menschen starben im Berichtsjahr 2006 mehr als 1 % aller Menschen in Deutschland durch eine Vorsätzliche Selbsttötung/-vergiftung und damit fast doppelt so viele wie durch in der Todesursachenstatistik ausgewiesene Verkehrsunfälle.“ (Stefan P. Rübenach: Todesursache Suizid, Statistisches Bundesamt, Wirtschaft und Statistik 10/2007)
Jenseits der Diskussion um Gerechtigkeitsgefühl und um den Strafzweck und den fehlenden Willen der Verantwortlichen, die Resozialisierungsbemühungen zu optimieren haben wir also durchaus problematischere gesellschaftliche Felder der Aufregung und Betroffenheit als die wenigen Mordfälle, mit denen Straftäter nach der Strafrestaussetzung rückfällig werden. (Die Rückfallquote ist generell übrigens stark altersabhängig: Junge tun es eher, Alte tun es weniger.) Eine Statistik über die Opferzahlen für diesen Deliktfall konnte ich (vermutlich wegen der geringen Zahl) nicht finden. Sicherlich: Mordfälle sind nichts Schönes (auch nicht im Fernsehen), aber sie sind ein normaler Teil des gesellschaftlichen Lebens und im internationalen Vergleich gesehen stehen wir (auch wenn das für Betroffene kein Trost sein kann) recht gut da. (Im Artikel Tötungsrate nach Ländern kann man die Tabelle nach Tötungsrate sortieren und da nimmt Liechtenstein den ersten und bei gleichen Werten wie Dänemark (19.) und Spanien (20.) nimmt Deutschland den 21. Platz ein.) --2003:45:4648:FD00:94B5:A7E2:6BAB:55DA 17:19, 13. Apr. 2016 (CEST)
NB: Zu Wiederholungsgefahr bei Tötungsdelikten siene auch unter Sozialtherapeutische Anstalt. -- Zerolevel (Diskussion) 18:25, 13. Apr. 2016 (CEST)
Und dann gäbe es noch in Relation an die 15.000 Opfer von Krankenhauskeimen im Jahr (Quelle), wo auch dann Kinder oder Eltern trauern...---90.146.53.23 (23:48, 13. Apr. 2016 (CEST), Datum/Uhrzeit nachträglich eingefügt, siehe Hilfe:Signatur)
Ja. Ich möchte da allerdings auch nicht mißverstanden werden. Man kann Tote nicht gegeneinander aufrechnen. Jeder Tod, ob natürlich oder unnatürlich, ob alt oder jung, ist eine dramatische Sache und führt in der Regel zu viel Leid, Schmerz und Trauer. Wenn wir aber über unsere Ängste reden, über unser Bedürfnis nach Schutz und darüber, was wir in unserer Gesellschaft nicht gut geregelt sehen gibt es meines Erachtens eine Ebene von Verhältnismäßigkeit. Wir müssen, vor allem wenn es um unsere Freiheitsrechte und die Liberalität der Gesellschaft geht, weg von der Aufregung, weg von der Panik und der Panikmache. Die Chance, von einem Terroranschlag betroffen zu sein ist geringer als die Chance auf einen Sechser im Lotto. (Und wenn man sich normalerweise nicht in einer Großstadt aufhält noch wesentlich geringer.) Dennoch gibt es da viel Angst. Wir müssen – und das gilt auch für den Strafvollzug – mit den Füßen auf dem Boden bleiben.
Ich hatte oben nicht umsonst auf den Artikel Strafzwecktheorie verwiesen. Dadurch sollte deutlich werden, dass die Dinge komplexer verstanden werden als das „immer ein auge zu zudrücken“, das der Fragesteller unterstellt. Neben der Frage der Strafe muss jede Gesellschaft zweifellos überlegen, wie sie sich schützt und dennoch nicht in die Inhumanität zurückfällt. Einfach nur Wegsperren ist dabei der teuerste und ineffektivste Schutz. Und davor steht die sorgfältige Ermittlung der Umstände und der Schuldfrage. Der Fragesteller wendet im Fall Bad Aibling ein: „Der wurde nach vier Monaten erst verhaftet warum nicht umgehend?“ Die Antwort ist einfach: Weil erst ermittelt werden muss. Die Information der Ablenkung durch Spielen mit dem Handy ist relativ neu. Es liegt auf der Hand, dass dafür erst der Nachweis erbracht werden musste, der Mann hat das ja nicht von Anfang an eingestanden.
Das alles macht die Toten von Bad Aibling aber auch nicht lebendig und die vielen verletzten Opfer (an die man da auch denken sollte) nicht gesünder. Und es gibt im Leben halt auch Ebenen, in denen eine völlige Gerechtigkeit, ein völliger Ausgleich, eine völlige Genugtuung einfach nicht möglich sind, wo wir den Fußtritte des Lebens einfach nur ausgeliefert sind und sie aushalten müssen und nach Trost suchen müssen. Terroranschläge, betrunkene Autofahrer, mordende Erbschleicher und gewalttätige Totschläger sind ebenso Teil einer modernen Industriegesellschaft wie ein mit dem Handy spielender Fahrdienstleiter (und nicht Schaffner, wie der Fragesteller schreibt). Wir sind von diesem Elend weitaus weniger betroffen als andere Menschen in anderen Teilen der Welt. Das mag kein Trost sein, aber vielleicht ein Grund für etwas mehr gelassenenes Nachdenken.
Vielleicht ist ja eine Lehre aus Bad Aibling auch, dass man Fahrdienstleiter von eingleisigen Strecken mit Gegenverkehr mit ihrer ungeheuren Verantwortung möglichst nicht und möglichst wenig alleine läßt. Die Technik, dass Züge in solchen Situationen von alleine „merken“, dass auf dem Gleis ein Zug entgegenkommt, ist entwickelt und verfügbar. Das kann man kaufen. (Wenn man den Betrieb nicht gerade kaputtspart, damit die Rendite für den Börsengang möglichst hoch und attraktiv ist.)
Menschen machen Fehler, Menschen sind bösartig und Menschen sind krank. Damit müssen wir leben. Und daran ändern Strafen (fast) nichts. Wenn man die oben verlinkten Arbeiten zu Rückfallstatistiken genauer ansieht wird deutlich, dass die Strafe kein wesentlicher Faktor ist, der vom Rückfall abhält. --2003:45:4648:FD00:94B5:A7E2:6BAB:55DA 00:37, 14. Apr. 2016 (CEST) Nachsatz: Die Opfer von Krankenhauskeimen sind auch die Opfer von Betriebskalkuationen und Gewinnerwartungen in der Gesundheitsindustrie. Die Niederlande machen uns schon lange vor, wie man durch konsequentes Hygiene-Management diese Bedrohung effizient minimieren kann. --2003:45:4648:FD00:94B5:A7E2:6BAB:55DA 00:56, 14. Apr. 2016 (CEST)

IP mit ellenlangen Beiträgen, Off-Topic mäandernd. Welcher derzeit gesperrte User könnte das nur sein?--2003:75:AF10:2600:A1EF:D764:CCFD:EB48 07:35, 14. Apr. 2016 (CEST)

Ist Herr Haase gesperrt?--84.187.141.121 07:58, 14. Apr. 2016 (CEST)
Jo, scheinbar einen Monat. --192.91.60.10 08:18, 14. Apr. 2016 (CEST)
Mich mit Herrn Haase in Verbindung zu bringen grenzt aus meiner Sicht an grobe Beleidigung. --2003:45:4645:9B00:31AE:7891:C73:B5F9 13:18, 14. Apr. 2016 (CEST)

Text in PDF aus LibreOffice lässt sich neuerdings nur sehr zäh markieren

Hallo, ich habe mit LibreOffice 4 ein kleines Dokument erstellt und als ein PDF exportiert. Nun ist es so, dass man innerhalb des PDFs den Text nur sehr zäh markieren kann (es hat so eine gewisse Verzögerung, als wäre der PC überfordert, daran kann es aber kaum liegen, sind nur ein paar Sätze normaler Text). Woran mag das liegen? Ich habe es verglichen mit älteren PDFs, da ging das Markieren von Text so ganz normal wie man es eben kennt... --87.140.193.3 19:08, 13. Apr. 2016 (CEST)

Und in diesen älteren PDFs geht das auch jetzt noch?
Ist die Dateigröße dieser PDF-Datei sehr groß? --87.123.50.238 19:50, 13. Apr. 2016 (CEST)
Ja eben, die älteren PDFs sind gut, da kann man das ganz normal immer noch markieren. Die Datei-Größe ist nicht groß, ein paar kB. --87.140.194.22 20:01, 13. Apr. 2016 (CEST)
Also geht es mit älteren Dateien noch, bei neuen Datei ist das Problem aber auch bei kleiner Dateigröße schon da.
Mit welchem Programm hast du dieses Problem? Acrobat Reader? Einem Webbrowser? Dem Reader-Plugin in einem Browser? Eine der betroffenen Dateien in einem der genannten anderen Programme zu nutzen, geht problemlos? --87.123.50.238 11:12, 14. Apr. 2016 (CEST)

Nicht arbeitslos gemeldet

Diese Reportage stellt einen Mann vor, der sich "aus falschem Stolz" nach dem Arbeitsplatzverlust nicht arbeitslos gemeldet und dadurch obdachlos wurde. Jetzt die Frage, geht das denn heute überhaupt, sich nicht arbeitslos zu melden? Spätestens nach einem Monat fällt doch auf das keine Krankenversicherungsbeiträge mehr gezahlt werden, und nachdem Krankenversicherungspflicht besteht müsste da doch die KV aktiv werden. Dass der Mann massive psychische Probleme fällt ja nach wenigen Sätzen auf (immerhin hat die Obdachlosenveraltung richtig reagiert und ihn ins Betreute Wohnen verbracht), denkbar das er auch Briefe nicht beantwortet hat. Was passiert in solch einem Fall?--Antemister (Diskussion) 22:49, 13. Apr. 2016 (CEST)

Wenn man privat oder freiwillig gesetzlich versichert ist, zahlt man seine Beiträge selbst. --BlackEyedLion (Diskussion) 23:01, 13. Apr. 2016 (CEST)
Um das Zahlen kommt man nicht herum, denn die KV wird nicht gezahlte Beiträge nachfordern. --Rôtkæppchen₆₈ 23:22, 13. Apr. 2016 (CEST)
Nach zwei Monaten Zahlungsverzug fällt man in die Notversorgung. Dann bezahlt die Krankenkasse nur noch akute Notfälle, aber keine chronischen Krankheiten (ein Diabetiker z. B. müsste dann sterben, wenn er sein Insulin nicht selbst zahlen kann). Die Krankenkasse wird selbstverständlich auch versuchen die Beiträge per Zwangsvollstreckung einzutreiben und solange nicht alles bis auf den letzten Cent bezahlt ist muss man ohne ärztliche Versorgung leben, es sei denn man kommt in den Sozialleistungsbezug. -- 217.236.175.4 23:32, 13. Apr. 2016 (CEST)
Das trifft doch primär Selbständige, die ihre Krankenversicherung ganz selbst zahlen (müssen) und dies wegen zu geringer Einnahmen evtl. zeitweise nicht können. Aber hier geht es um einen offensichtlich Arbeitslosen, bei dem auch nichts zu holen wäre. Hier müsste sich doch die Krankenkasse ans Arbeitsamt richten, denn das zahlt ja für Arbeitslose.--Antemister (Diskussion) 23:57, 13. Apr. 2016 (CEST)
Arbeitslosengeld, egal ob I oder II, gibt es nur auf Antrag. Solange der nicht gestellt ist, hat auch die Krankenkasse keine Ansprüche gegenüber dem Amt und muss sich an den Versicherten wenden. -- 217.236.175.4 00:01, 14. Apr. 2016 (CEST)
Es stimmt nicht, dass die ärztliche Versorgung ausgesetzt bleibt, bis der letzte Cent an Beitragsrückständen bezahlt ist: Sobald man eine Ratenzahlungsvereinbarung mit der Krankenkasse geschlossen hat, lebt der Versicherungsschutz im normalen Umfang wieder auf.
Der Sachbearbeiter der Krankenkasse, der für die rückständigen Beiträge zuständig ist, interessiert sich nicht für Diagnosen; selbst wenn einem Laien nach wenigen Sätzen auffiele, dass das Mitglied psychische Probleme hat, weiß er das vermutlich nicht, es würde ihn aber auch nicht interessieren, und es würde ihn auch nichts angehen. --Snevern 06:55, 14. Apr. 2016 (CEST)
Ohne Versicherg kommt offenbar gar nicht so selten vor. Hier ein paar Zahlen für D und da für A.--89.144.207.8 (14:23, 14. Apr. 2016 (CEST), Datum/Uhrzeit nachträglich eingefügt, siehe Hilfe:Signatur)

Mitgesprochene Lizenz

Hallo, mir ist aufgefallen, dass bei gesprochenen Wikipediaartikeln immer oder fast immer die Lizenz mitgesprochen wird und fast immer der komplette Artikel vorgelesen wurde (gut ich fände einzelne abschnitte Praktischer). Ich weiß, dass die gesprochene Wikipedia irgendwann in grauer Vorzeit mal für Blinde eingesprochen wurde und diese Lizenzhinweise nicht lesen können, aber zwingend nötwendig dürfte dieses Lizenzvorlesen doch gar nicht sein, oder kenne ich da irgendeine besonderheit der CC Lizenzen nicht? ... bei Bildern wird ja auch nicht noch extra ins Bild die Lizenz eingefügt, damit die korrekte Lizenz bloßnicht verloren geht. --178.12.205.91 23:54, 13. Apr. 2016 (CEST)

Die Lizenz muß erreichbar sein. Bei Bildern in der visuellen WP bringt dich ein Klick auf die Dateibeschreibungsseite, dort steht auch die Lizenz des Bildes. Nutzt du ein Bild aus Commons mit einer CC-Lizenz außerhalb der damit verbundenen Online-Medien, mußt du die Lizenz eigens verlinken (beim Bildquellenhinweis). In jedem Fall muß eindeutig erkennbar sein, unter welcher Lizenz das Bild steht. --Kreuzschnabel 07:39, 14. Apr. 2016 (CEST)

Suche nach Geschichte / Buch / Maerchen

Hallo zusammen,

ich suche nach einer Geschichte die in etwa so geht:

  • Der Raeuberhaeuptling sagt zu seinen Kindern "Ich gehe stehlen. Passt mir auf das Essen/die Kueche auf damit wir heute Abend essen koennen."
  • Die Kinder verwuesten unter Tags die Kueche.
  • Sagt der Raeuberhaeuptling "Macht nichts, dann klau ich eben eine neue."
  • Der Raeuberhaeuptling sagt zu seinen Kindern "Ich gehe stehlen. Passt mir auf die Moebel auf."
  • Die Kinder verwuesten unter Tags die Moebel.
  • Sagt der Raeuberhaeuptling "Macht nichts, dann klau ich eben neue."
  • Der Raeuberhaeuptling sagt zu seinen Kindern "Ich gehe stehlen. Passt auf euch selbst auf."
  • Die Kinder verletzen sich unter Tags schwer.
  • Sagt der Raeuberhaeuptling "Macht nichts, dann klau ich eben neue."

Kann mir da wer helfen?

Danke, --2001:608:0:1:AE87:A3FF:FE15:66A0 14:20, 14. Apr. 2016 (CEST)

Windows Update schlägt fehl mit Fehler 0x80070490

Wenn ich versuche, auf meinem Windows-7-System mit Windows Update nach neuen Updates zu suchen, passiert für über 30 Minuten scheinbar nichts.

Ich hab in WindowsUpdate.log nachgesehen und folgende Einträge gefunden: Erst prüft der Update-Client, ob ein Self-Update nötig wäre. Ergebnis: Ist es nicht. Dann folgen für mehrere Minuten gar keine Einträge. Und dann kommt das hier:

2016-04-12	23:36:38:284	1040	ccc	Handler	FATAL: UH: 0x80070490: EvaluateApplicability failed in CCbs::EvaluateApplicability
2016-04-12	23:36:38:643	1040	ccc	Agent	WARNING: Failed to evaluate Installed rule, updateId = {9B3E0557-1739-409A-AFE2-CE51B1CC706F}.110, hr = 80070490
2016-04-12	23:36:50:643	1040	ccc	Handler	FATAL: UH: 0x80070490: EvaluateApplicability failed in CCbs::EvaluateApplicability
2016-04-12	23:37:26:877	1040	834	AU	###########  AU: Uninitializing Automatic Updates  ###########
2016-04-12	23:37:26:987	1040	ccc	Agent	  * WARNING: Exit code = 0x8024000B

Dieses Fix-It hab ich ausprobiert. Es erkennt den Fehler 0x80070490, führt auch Reparaturen durch, aber der Fehler bleibt.

Welches Update ist das mit der ID 9B3E0557-1739-409A-AFE2-CE51B1CC706F?

--87.123.28.129 21:25, 12. Apr. 2016 (CEST)

komisch... Manjaro Linux sagt sowas nie zu mir... hier gab's n ähnlichen Fehler: [9] & [10]... --Heimschützenzentrum (?) 00:42, 13. Apr. 2016 (CEST)
CBS.log enthält zu dieser Fehlermeldung die folgenden Zeilen:
2016-04-12 23:46:02, Info                  CBS    Appl: Partial install Status testing, package: Package_for_KB976933~31bf3856ad364e35~x86~~6.1.7601.17514, partially installed (true/false), 0
2016-04-12 23:46:02, Info                  CBS    Failed to find package from aggreated XML string, package: Package_for_KB976933~31bf3856ad364e35~x86~fr-FR~6.1.7601.17514 [HRESULT = 0x80070490 - ERROR_NOT_FOUND]
2016-04-12 23:46:02, Info                  CBS    Failed to find package from deployment update [HRESULT = 0x80070490 - ERROR_NOT_FOUND]
2016-04-12 23:46:02, Error                 CBS    Failed to check whether package is partially installed, package: Windows7SP1-KB976933~31bf3856ad364e35~x86~~6.1.1.17514 [HRESULT = 0x80070490 - ERROR_NOT_FOUND]
2016-04-12 23:46:02, Info                  CBS    Failed to evaluate external applicability for package update: 976933-0_neutral_PACKAGE [HRESULT = 0x80070490 - ERROR_NOT_FOUND]
2016-04-12 23:46:02, Error                 CBS    Failed to call external evaluate applicability on package: Package_for_KB976932~31bf3856ad364e35~x86~~6.1.1.17514, Update: 976933-0_neutral_PACKAGE [HRESULT = 0x80070490 - ERROR_NOT_FOUND]
KB976932 ist offenbar Windows 7 SP1. Der ist installiert. Und nu? --87.123.28.129 00:45, 13. Apr. 2016 (CEST)
hier steht, wie man den Fehler repariert... --Heimschützenzentrum (?) 00:46, 13. Apr. 2016 (CEST)
Das Tool ist in der Nacht durchgelaufen, aber die Fehlermeldung ist immer noch (oder wieder?) da. Wieder dieselbe Update-ID und wieder derselbe Fehlercode. Aber im Laufe der Nacht hat er zumindest die aktuellen Updates installiert - und zwar 7 Stunden nachdem ich das Tool angestoßen hatte. --87.123.28.129 12:39, 13. Apr. 2016 (CEST)
7h für ne Zero-Day-Attacke... *yay* hast du mal die von Microschrott angegebene Liste abgearbeitet? oder hört auf die eh keiner mehr? --Heimschützenzentrum (?) 13:07, 13. Apr. 2016 (CEST)
Ich hab die Liste wie es sich gehört abgearbeitet, sozusagen von klein nach groß: Erst den Update-Client aktualisiert (was nicht nötig war), dann Windows Update teilweise zurückgesetzt, dann Windows Update komplett zurückgesetzt, schließlich das MSU-Paket installiert und der Fehler war nach jedem dieser Versuche immer noch da.
Jetzt im Moment geht das Suchen nach Updates allerdings sehr schnell; dauert weniger als eine Minute. Ist nur die Frage, ob das auch so bleibt, wenn es wieder neue Updates gibt. Denn: Wenn es keine neuen Updates gab, war das auch die letzten Male recht schnell. Und wenn es neue Updates gab, dann ging es auch irgendwie, nach ellenlanger Wartezeit. Wäre halt schön, wenn das flott gehen würde, so wie auf anderen PCs auch... --87.123.28.129 14:12, 13. Apr. 2016 (CEST)
ich meinte „meine“ Liste: [11] (also die, die nur ich und ich selbst allein gefund'n hab'...)... oder isses die Gleiche? :) --Heimschützenzentrum (?) 14:34, 13. Apr. 2016 (CEST)
Diese "Liste" ist nur ein kleiner Teil dessen, was ich versucht habe. Davor hab ich den Update-Client aktualisiert (was nicht nötig war), dann Windows Update teilweise zurückgesetzt und dann Windows Update komplett zurückgesetzt. Erst danach hab ich die von dir angesprochene, noch umfangreichere Option gewählt. Wo ich jetzt auch nicht weiß, ob es geholfen hat.
Es wäre halt schön, wenn das Suchen nach neuen Updates flott gehen würde, so wie auf anderen PCs auch. --87.123.50.238 14:57, 13. Apr. 2016 (CEST)
Vielleicht das hier? https://support.microsoft.com/de-de/kb/958044 --Eike (Diskussion) 15:04, 13. Apr. 2016 (CEST)
Das ist das Gleiche oder zumindest ein Teil dessen, was ich schon probiert habe. sfc /scannow ergab ebenfalls keine Fehler.
Was ich noch nicht probiert habe ist, das problemtische Update, also den Servie Pack 1, neuzuinstallieren. Ich befürchte, danach fliegt mir das System dann noch um die Ohren... --87.123.50.238 16:00, 13. Apr. 2016 (CEST)
lieber ein Ende mit Schrecken, als Schnecken ohne Ende? „frisch von vorn“ ist auch mal ganz lustig... --Heimschützenzentrum (?) 10:03, 14. Apr. 2016 (CEST)
Unangemessen drastisch. --87.123.58.223 18:51, 14. Apr. 2016 (CEST)

E-Ink-Bildschirm

Gibt es derzeit eigentlich einen Bildschirm der wie Elektronisches Papier funktioniert? --178.199.98.114 17:39, 13. Apr. 2016 (CEST)

So zum Anschließen an den Computer? Mit entsprechend langsamem Umschalten zwischen zwei Bildern? Also, ich wollt' es nicht haben... Die Tendenz geht in die andere Richtung, immer schnelleres Umschalten. --Eike (Diskussion) 18:18, 13. Apr. 2016 (CEST)
Ich fände es für Onlinezeitungen oder Wiki sehr praktisch. Die ordentlichen Bildschirme sind einfach nicht zum Lesen geeignet. --178.199.98.114 18:37, 13. Apr. 2016 (CEST)
Gibt es nicht. Sony bietet einen großen E-Ink-Reader mit 13.3 Zoll an, aber der ist nicht als Monitor gedacht, kostet einen Haufen Geld und ist in Europa nicht zu haben. Ich fürchte, das wird auch so bleiben, solange es keine reflexive, schnelle Bildschirmtechnik mit subtraktiver Farbmischung gibt. Und die ist noch nicht mal am Horizont. Rainer Z ... 19:41, 13. Apr. 2016 (CEST)
Mit heutiger E-Ink-Technik wären übliche Websites und Betriebssysteme überfordert. Du würdest also einen speziellen E-Ink-Modus für jede Website und jedes Betriebssystem brauchen. Da kannst Du gleich einen E-Book-Reader nehmen. --Rôtkæppchen₆₈ 20:40, 13. Apr. 2016 (CEST)
Auch bei Onlinezeitungen will man ja blättern oder scrollen. Und Bildschirme, auf denen man nicht "wischen" kann, sind heutzutage wahrscheinlich unverkäuflich :) --Optimum (Diskussion) 20:55, 13. Apr. 2016 (CEST)
Zum Lesen mag es toll sein, aber du könntest darauf einen Mauszeiger nicht gezielt bewegen... --Eike (Diskussion) 21:07, 13. Apr. 2016 (CEST)
Das ist auf dem durchschnittlichen Smartphone doch auch kein Problem. --87.148.90.191 21:12, 13. Apr. 2016 (CEST)
Dort hast Du z.B. animierte Buttons oder Scrollbars und auch sonst viele Wischfunktionen, die auf einem E-Ink-Display undarstellbar wären. E-Ink eignet sich nur für statische Anzeigen in wenigen Helligkeitsstufen. Laut Artikel ist bei E-Ink ja selbst die schatten- und geistfreie Darstellung einer Uhrzeit ein Problem. --Rôtkæppchen₆₈ 21:21, 13. Apr. 2016 (CEST)

Also auf einem E-Ink-Bildschirm kann man schon wischen. Es sieht einfach verrüttelt aus, während die Seite sich bewegt, aber grundsätzlich geht das. Theoretisch fände ich das gut, so als Zweitbildschirm um etwas längeres zu lesen. Aber da man ohnehin auf andere Displays angewiesen ist und ein anständig grosser E-Ink-Screen wahrscheinlich schweineteuer wäre, wird es das wohl in absehbarer Zeit für den normalen Konsumenten nicht geben. --King Rk (Diskussion) 08:19, 14. Apr. 2016 (CEST)

Also, dass da nur statische Anzeigen möglich sind, stimmt auf jeden Fall nicht: https://www.youtube.com/watch?v=3n2xxqMQyfY (das Video ist von 2008) --178.199.98.114 11:33, 14. Apr. 2016 (CEST)
Hast Du Dir mal den Seitenübergang angeschaut? Das erinnert mich irgendwie an die Frühzeit von CGA. Epileptiker würden sich über so einen Bildschirm in Schreibtischgröße sicher freuen. --Rôtkæppchen₆₈ 12:06, 14. Apr. 2016 (CEST)
Es ging bei der Frage um Anwendungen mit viel Text, nicht um Zuckeluhren oder Zappelklickibuntis und scrollen mussen auch nicht, dafuer gibt es die Seitenumblaetterfunktion, wie bei einem echten EReader. Mir ist also unklar warum darauf rumgeritten wird. Jeder modern eReader mit eInk sollte eigentlich keine groesseren Probleme mit der Darstellung der mobilen Version der Wikipedia oder einer Text-Nachrichtenseite in seinem nativen Browser haben. --192.91.60.11 14:11, 14. Apr. 2016 (CEST)
Nein, es ging um Computerbildschirme. Die Frage war, ob es sowas gibt, die Antwort ist, nein, des gibt es soweit wir wissen nicht, und zwar, weil an einen Computerbildschirm ganz andere Anforderungen gestellt werden. Der muss mehr können als Text. Wörter wie "Zappelklickibuntis" helfen dagegen nicht. Ich kaufe keinen Computerbildschirm, der davon überfordert ist, einem Mauszeiger flüssig anzuzeigen. --Eike (Diskussion) 14:25, 14. Apr. 2016 (CEST)
Also doch, das gibt es schon: Dasung Paperlike 13.3" (Demo, Rezension). Kostet aber 850 Euro. --King Rk (Diskussion) 20:49, 14. Apr. 2016 (CEST)
…und zeigt im zweiten verlinkten Video genau die oben angesprochenen Probleme. Außerdem mag ich keinen Bildschirm, dessen Anschlussstecker seitlich übersteht. --Rôtkæppchen₆₈ 22:27, 14. Apr. 2016 (CEST)

wie stoppe ich Win 10?

Die letzten paar Monate habe ich das ständig aufpoppende Win-10-upgrade-Angebot immer weggeklickt. Ich bin glücklich mit Win 8.1. Kann sein, dass ich mich einmal verklickt habe, denn jetzt lässt es mir nur noch die Wahl, das Upgrade sofort durchzuführen oder einen unveränderlichen Upgrade-Zeitpunkt einzutragen. Das Fenster lässt sich nicht mehr schließen. Für mich grenzt das alles an Nötigung. Kann ich das ganze noch stoppen? Wie? --Ratzer (Diskussion) 23:00, 13. Apr. 2016 (CEST)

Nach einem Upgrade hast Du zwei Wochen Zeit, wieder zum alten Betriebssystem zurückzukehren. Danach erscheint diese ziemlich nervige Upgradeaufforderung allerdings wieder. Finde die KB-Nummern der GWX-Komponenten raus und deinstalliere und deaktiviere sie im Windows Update. --Rôtkæppchen₆₈ 23:06, 13. Apr. 2016 (CEST)
Versuche dieses Update. --Rôtkæppchen₆₈ 23:11, 13. Apr. 2016 (CEST)
Und wo ist das Problem? Ich bin direkt von XP auf Windoof.10 umgestiegen und - trara - erstaunlicherweise funktioneirt es. --Informationswiedergutmachung (Diskussion) 23:09, 13. Apr. 2016 (CEST)
Das ist eher weniger empfehlenswert. Da es von Windows XP auf 10 sowieso kein automatisches Upgrade gibt, ist das eine passende Gelegenheit, sämtliche Benutzerdaten zu sichern und den Rechner komplett neu aufzusetzen, mit UEFI, AHCI und GPT statt Bios, IDE und MBR. Das habe ich am Montag und Dienstag mit einem Vista-Rechner gemacht. Aktueller Anlass war übrigens die Einstellung von Google Chrome für Windows XP und Vista. Dummerweise ist Chrome aber der Lieblingsbrowser des PC-Besitzers. Windows 10 kennt er schon von seinem Laptop und findet es leichter zu bedienen als XP, Vista oder 7. Auf meinen virtuellen XP- und Vistarechnern hab ich Google Chrome einfach deinstalliert und durch Firefox ersetzt. --Rôtkæppchen₆₈ 23:29, 13. Apr. 2016 (CEST)
Nachdem ich der Datenschleuder Chrome eh abhold bin und mein Rechner sich ins elektronische Nirvana verabschiedet hat, war es egal, alldieweil ich einen neuen Rechner brauchte und eh Firefox bevorzuge. Wer benutzt eigentlich Chrome? --Informationswiedergutmachung (Diskussion) 23:32, 13. Apr. 2016 (CEST)
 
grün=Chrome, orange=Firefox, rot=Opera
In allen grün markierten Ländern ist Google Chrome der beliebteste Browser. --Rôtkæppchen₆₈ 00:03, 14. Apr. 2016 (CEST)
Das ist eine ziemlich sinnlose Karte, denn die Mobilgeräte sind mitgezählt. Und da wird wohl in den seltensten Fällen ein anderer Browser benutzt als der vorinstallierte. Damit gibt die Karte nicht die Beliebtheit eines bestimmten Browsers beim Nutzer an, sondern welche Mobilfunkfirma mit welchem Tarif und welchen Geräten das Rennen gemacht hat. -- Janka (Diskussion) 00:34, 14. Apr. 2016 (CEST)
 
nur Desktop: grün=Chrome, orange=Firefox, blau=IE
Diese Karte gibt es auch noch nur für Desktopgeräte. --Rôtkæppchen₆₈ 00:39, 14. Apr. 2016 (CEST)
Findet es noch jemand witzig, dass Nordkorea grün ist?--84.187.141.121 07:30, 14. Apr. 2016 (CEST)
Das war mir auch aufgefallen. Aber irgendwomit müssen die Bonzen das unzensierte en:Internet in North Korea ja nutzen. --Rôtkæppchen₆₈ 11:59, 14. Apr. 2016 (CEST)
Das ist ja ´ne lustige Karte. IE ist nur in Japan, im Iran und auf einigen Bohrinseln beliebt? --Optimum (Diskussion) 22:52, 14. Apr. 2016 (CEST)
Soso, die Schweiz ist also eine Bohrinsel. Wissen die Schweizer das? --Rôtkæppchen₆₈ 01:38, 15. Apr. 2016 (CEST)
GWX Control Panel oder Never10 könnten behilflich sein. -- MonsieurRoi (Diskussion) 16:40, 14. Apr. 2016 (CEST)

Ich müsste mich jetzt sehr täuschen, wenn ich sie mit jemandem verwechsele; hatte sie am Anfang ihrer Karriere nicht einen dreiteiligen Namen? In einem Bestandteil war glaube ich auch noch ein Ü. Kann jemand Auskunft geben? --87.178.29.45 16:09, 14. Apr. 2016 (CEST)

Helga Trüpel-Rüdel. --Slökmann (Diskussion) 16:46, 14. Apr. 2016 (CEST)
Waren es doch nur 2 Namen? Jedenfalls Danke!--87.178.6.202 18:54, 14. Apr. 2016 (CEST)
Wievielle Bekannte mit drei Nachnamen gibt es denn? Klar, Simone Greiner-Petter-Memm, aber sonst? --Hachinger62 (Diskussion) 22:10, 14. Apr. 2016 (CEST)
Rosa Müller-Graf-Kleditsch. --Rôtkæppchen₆₈ 22:18, 14. Apr. 2016 (CEST)
Elisabeth Noelle-Neumann-Maier-Leibnitz. Maier-Leibnitz muss als Geburtsname von Heinz Maier-Leibnitz als einteilig gezählt werden. --BlackEyedLion (Diskussion) 23:40, 14. Apr. 2016 (CEST)

Rumänien Visum nötig?

Ich plane, in den Sommerferien mit dem Nachtzug durch Osteuropa zu reisen, unter anderem auch durch Rumänien. Die Seiten vom Auswärtigen Amt und von der rumänischen Botschaft sind nicht ganz eindeutig. Muss ich jetzt ein Visum oder sonst was beantragen, oder reicht ein EU-Perso am Grenzbahnhof (falls da überhaupt kontrolliert wird)? Ich bin etwas verwirrt… --87.123.168.92 17:05, 14. Apr. 2016 (CEST)

Kommt auf deine Staatsangehörigkeit an. --Komischn (Diskussion) 17:28, 14. Apr. 2016 (CEST)
@Komischn, Deutsch. -- 87.123.168.92 17:39, 14. Apr. 2016 (CEST)
(BK)Rumänien ist noch nicht Mitglied des Schengener Abkommens. Es reicht, einen gültigen Personalausweis mitzuführen. Rest siehe hier. --Rôtkæppchen₆₈ 17:28, 14. Apr. 2016 (CEST)
Abgesehen von Weißrussland und Russland ist für die Länder Osteuropas seit Jahren kein Visum mehr nötig. --j.budissin+/- 18:34, 14. Apr. 2016 (CEST)
Glückwunsch jedenfalls zur Urlaubswahl: Wir haben vor ein paar Jahren eine Südosteuropa-Nachtzug-Tour gemacht (mit den Etappen München-Budapest-Belgrad-Bukarest-Sofia-Saloniki) und es war ein sehr netter und interessanter Urlaub. Nachtzüge sind toll :-) --Rudolph Buch (Diskussion) 23:27, 14. Apr. 2016 (CEST)

Wappenschilde mit Knollennase

 

Was sind das für Wappen unten im Bild, könnte eine Karikatur sein (Knollennasen), bloß bezüglich wem und was hat das mit Gottvater zu tun resp. mit wem sonst? --Mattes (Diskussion) 21:06, 14. Apr. 2016 (CEST)

Ich nehme an, das ist ein Mohr. Wird wohl das Wappen des Autoren oder Auftraggebers sein. --King Rk (Diskussion) 21:37, 14. Apr. 2016 (CEST)

Das sind Wappen, deren Felder aus den Eltern und Ehefrauen Schedels zusammengesetzt sind: Der Holzschnitt am Beginn der Chronik zeigt Gottvater sowie zwei Wappen mit den Zeichen der Familie Schedel ("Mohrenkopf"), Grabner ("gekreuzte Schaufel", Schedels Mutter: Anna Grabner, gest. 1445), Heugel ("gekreuzte Dreschflegel?"; Schedels erste Frau: Anna Heugel, gest. 1485), Haller ("3 Sterne am Stab mit Ring?"; Schedels zweite Frau: Magdalena Haller, gest. 1505). (Aus dem Privatexemplar des Hartmann Schedel) (Bayerische Staatsbibliothek Rar 287, fol. 1v) Quelle: https://www.historisches-lexikon-bayerns.de/Lexikon/Schedelsche_Weltchronik --WolfD59 (Diskussion) 22:21, 14. Apr. 2016 (CEST)

Viele Wohls

Z.B. Kochen mit Pflanzenöl kann wohl das Herz schädigen, Augsburger Allgemeine, 14. April 2016

könnte bedeuten:

  • trotziges "wohl kann Kochen mit Pflanzenöl das Herz schädigen"
  • empörtes "wer jetzt immer noch mit Pflanzeöl kocht will wohl das Herz schädigen"
  • "Kochen mit Pflanzenöl kann Wohl des Herz schädigen"?
  • "Kochen mit Pflanzenöl kann anscheinend das Herz schädigen"; der Artikel geht jedoch gar nicht soweit, sogar eine Schädigung zu vermuten, sondern es wird lediglich vermutet: "Pflanzenöle beim Kochen senken das Risiko für Herzkrankheiten wohl nicht"

Weitere Beispiele alleine von heute: Abdeslam sammelte wohl Infos über deutsche Atomanlage, Das Zika-Virus löst wohl wirklich Schädelfehlbildungen aus, wie inzwischen bekannt ist (ist es jetzt bekannt oder "nur" "wohl bekannt"?), EU plant wohl Kooperation mit afrikanischen Despoten (Wohltuende Kooperation?) etc.

Auskunftsfrage: Hat ein "Wohl"täter die Schaltstelle einer Presseargentur übernommen oder wie kommt die in den letzten Wochen zu beobachtende plötzliche Welle der "Wohls" in Zeitungsschlagzeilen wohl zustande? --Rosenkohl (Diskussion) 21:07, 14. Apr. 2016 (CEST)

In der Tat, Modalpartikeln in Verbindung mit Intonation sind eine ganz, ganz große Herausforderung beim Erlernen der deutschen Sprache als Zweitsprache, zumal da man bei geschriebenen Texten die intendierte Intonation erraten muss. Grüße Dumbox (Diskussion) 21:18, 14. Apr. 2016 (CEST)
Service: in diesem Fall: Intonation = Intonation (Sprachwissenschaft) --Jmv (Diskussion) 22:20, 14. Apr. 2016 (CEST) (nach bk)
ähnlich doppeldeutig/widersprüchlich wie das mal empathisch bejahende, mal in den Zweifel ziehende wohl ist das attributive gewiss. „Ein gewisser Herr Müller“ will sagen, dass man über Herrn Müller nichts weiß: Aus der Bedeutung ‘bestimmt’ (eine gewisse Stunde) entwickeln sich im Dt. Verwendungen [...], bei denen man das genaue Maß bzw. eine nähere Bestimmung nicht angeben will oder nicht angeben kann (Wolfgang Pfeifer, Etymologisches Wörterbuch des Deutschen.) --Edith Wahr (Diskussion) 22:59, 14. Apr. 2016 (CEST)

Schulferien in Deutschland

Moin aus dem Norden. Gibt es irgentwo eine Liste im Netz, die Schulferien der vergangenen Jahre (natürlich Bundesland spezifisch, {{Deutschland bezogen}} ;-) darstellt?

  1. Ich brauche für eine private Recherche das offizielle Ende des Schuljahres 1969/1970 in Hamburg. (war im März 1970, aber das genaue Datum ist wichtig)
  2. Aber ich denke das gewesene Schulferien auch für andere zum nachschlagen (in Listenform (als Artikel)) interessant sein könnten (gern dann auch für Österreich & Schweiz, Lichenstein ....)

Ist ja auch "Wissen" im wikipedianischen Sinn. --Jmv (Diskussion) 22:22, 14. Apr. 2016 (CEST)

bei mir zweiter google-treffer für "schulferien 1970" -- southpark 22:27, 14. Apr. 2016 (CEST)
Danke, da hab ich wohl falsch gesucht :( --Jmv (Diskussion) 22:33, 14. Apr. 2016 (CEST)
OK: der link ist so gut, das Wp da keine eigenen Listen pflegen muß! :-) --Jmv (Diskussion) 23:29, 14. Apr. 2016 (CEST)
Eine Schulferienliste würde möglicherweise wp:WWNI Punkte 7 und 8 widersprechen, also das Ziel von Löschanträgen. --Rôtkæppchen₆₈ 23:38, 14. Apr. 2016 (CEST)

Rundfunkbeitrag fällig?

Guten Morgen,

folgendes Szenario: Ich führe ein eigenes Unternehmen, bei dem das Elternhaus die Elternwohnung gleichzeitig der Geschäftssitz ist. Die eigentliche Arbeit wird von einer anderen eigenen Wohnung aus verrichtet. Für das Elternhaus die Elternwohnung wird bereits privat von meinen Eltern die Rundfunkgebühr der Rundfunkbeitrag bezahlt. Nun habe ich eine Zahlungsaufforderung von der GEZ des Beitragsservices erhalten, dass ich ebenfalls für den unternehmerischen Geschäftssitz eine Rundfunkgebühr einen Rundfunkbeitrag bezahlen soll. Frage: Wäre es dann nicht doppelt, da für das Gebäude ja bereits eine Rundfunkgebühr ein Rundfunkbeitrag (und zwar privat von meinen Eltern) verrichtet wird? Ich habe etwas in diese Richtung gehört, dass es seit dem Rundfunkbeitrag, nur noch darauf ankommt, ob für ein Gebäude die Gebühr bezahlt wird und es somit einmalig ausreichend ist.

Vielen Dank --95.223.244.70 10:41, 12. Apr. 2016 (CEST)

Die Gebühr für Privatpersonen wird pro Wohnung (nicht pro Haus) erhoben, für Gewertetreibende pro Betriebsstätte (nicht pro Haus). --Snevern 10:56, 12. Apr. 2016 (CEST) Der nach mir weiß es besser.
§ 5 Abs. 5 Rundfunkbeitragsstaatsvertrag: "Ein Rundfunkbeitrag nach Absatz 1 ist nicht zu entrichten für Betriebsstätten, [...] die sich innerhalb einer beitragspflichtigen Wohnung befinden, für die bereits ein Rundfunkbeitrag entrichtet wird." Das heißt, du musst natürlich nicht noch ein zweites Mal Rundfunkgebühr zahlen. -- 217.236.175.4 10:58, 12. Apr. 2016 (CEST)
eine „Zahlungsaufforderung von der GEZ“ würde ich mangels Vorlage einer Vollmacht ablehnen... zumal es gar keine GEZ gibt... --Heimschützenzentrum (?) 11:02, 12. Apr. 2016 (CEST)
Ich denke, es ist klar, was gemeint war. --95.223.244.70 11:06, 12. Apr. 2016 (CEST)
Spontan war ich geneigt, dir zuzustimmen. Dann jedoch kam mir der Gedanke, daß die Gebühr nicht für ein Gebäude sondern für einen Haushalt erhoben wird, und deine geschäftlichen Tätigkeiten sind schwerlich mit der privaten Haushaltsführung deiner Eltern vereinbar; immerhin könntest du ja rein rechtlich sogar Mietkosten (z.B. für ein Büro dort) steuerlich absetzen. Aber auch hier kamen mir erneut Zweifel. Ich wohne in einer WG mit getrennten Haushalten, und hier reicht auch eine Gebühr. Es bleibt also durchaus eine spannende Frage. Gruß Timm Thaler (Diskussion) 11:22, 12. Apr. 2016 (CEST)P.S.:Nachtrag: Der von IP 217... gepostete § scheint es wohl zu klären.Timm Thaler (Diskussion) 11:24, 12. Apr. 2016 (CEST)
Die differenzierungstermini des Beitragsservice lauten Wohnung und Betriebsstätte, nicht Gebäude, Haus oder Haushalt. --Rôtkæppchen₆₈ 11:56, 12. Apr. 2016 (CEST)
hätt' ja auch ne Betrugsmasche sein können... z. B. von jmd, der/die noch an die GEZ gläubet... :) --Heimschützenzentrum (?) 14:09, 12. Apr. 2016 (CEST)
Bestehen denn überhaupt Geschäftsräume? Also zahlst du Miete für Geschäftsräume bzw. setzt du einen Teil deiner Mietkosten über die Steuer als Miete für Geschäftsräume ab? Falls nicht bezweifle ich das du für die reine Adresse GEZ zahlen musst. --2003:66:8936:4C51:F9CF:8E08:889:8BC1 11:57, 12. Apr. 2016 (CEST)
Nein, es gibt im Geschäftsitz (Elternwohnung) keine Büroräume. --95.223.244.70 12:04, 12. Apr. 2016 (CEST)
(BK)Auch für Dich noch einmal langsam und zum Mitschreiben: Rundfunkgebühr und GEZ gibt es nicht mehr. Dia ÖRR-Finanzierung durch Rundfunkbeitrag und Beitragsservice unterscheidet sich grundlegend vom vorherigen Modell. --Rôtkæppchen₆₈ 12:06, 12. Apr. 2016 (CEST)
In Deutschland wird sicher auch keine Tierhaut auf Straßen ausgelegt und trotzdem sagen die wenigsten Fußgängerüberweg. Und einen Fishmac nennt auch kein Mensch einen Filet-o-fish oder wie das Teil jetzt heißt. Es weiß also jeder was mit GEZ gemeint ist. Oder soll er wirklich schreiben, dass „er eine Zahlungsforderung vom ARD ZDF Deutschlandradio Beitragsservice“ erhalten habe? --KayHo (Diskussion) 12:39, 12. Apr. 2016 (CEST)
Ist doch vollkommen egal wie man das gerade wiedermal umbenannt hat. Der Staat sammelt zwangsweise Geld für einen Service ein den keiner bestellt hat. Daran hat sich nichts geändert außer dass man mit der neuen Regelung auch noch das Geld von denen nimmt die es nicht mal nutzen können oder wollen. Ob er das GEZ, Beitraggservice oder Fiedens und Freiheitsabgabe nennt ist dabei unerheblich. Wenn man von GEZ spricht weiß wenigstens jeder wer und was gemeint ist. --2003:66:8936:4C51:F9CF:8E08:889:8BC1 12:54, 12. Apr. 2016 (CEST)
Dumm ist nur, dass der ÖRR nicht dem Staat zuzurechnen ist. Vielleicht solltest Du mehr AfD wählen oder für eine Verstaatlichung des ÖRR vor dem Bundesverfassgungsgericht klagen. Dann ändert sich das vielleicht. --Rôtkæppchen₆₈ 13:02, 12. Apr. 2016 (CEST)
Ich find auch immer seltsam, dass sich fast niemand daran zu stören scheint, für was Steuern alles ausgegeben werden, was sie weder bestellt haben noch nutzen. Da scheint die Abstraktionsanforderung einfach zu groß zu sein... --Eike (Diskussion) 13:13, 12. Apr. 2016 (CEST)
Das hier ist einfach eine zweckgebundene Abgabe bei der man die Unredlichkeit der Begründung wie und von wem Sie erhoben wird überdeutlich sieht. --2003:66:8936:4C51:F9CF:8E08:889:8BC1 14:19, 12. Apr. 2016 (CEST)
Ich sehe das ganz anders: Über den Beitrag werden sinnvolle, m.E. auch notwendige Insitutionen finanziert. Allerdings wäre eine Finanzierung über Steuern (oder zumindest analog einer Steuer) besser, da dann die Zahlungspflicht sich nach der wirtschaftlichen Leistungsfähigkeit richtet und nicht der Geringverdiener genauso viel zahlen müsste wie jemand mit 10.000 Netto im Monat. Aber selbstverständlich ist das jetzige System viel besser als das vorherige, da nicht alle Ehrlichen die Anteile der Betrüger mitfinanzieren müssen. Denn dass eine Person weder Radio, noch Fernseher, noch Computer, noch internetfähiges Handy/Smartphone besitzt, aber trotzdem über eine Wohnsitz verfügt, ist extrem selten und kein Grund, es den Betrügern so leicht zu machen, wie es im alten System war.--141.76.98.133 16:39, 12. Apr. 2016 (CEST)
Nein! Der Staat hat weder mit der GEZ noch mit dem Beitragsservice auch nur das Geringste zu tun. Der Beitragsservice ist keine Behörde und keine staatliche Agentur, sondern eine Einrichtung des öffentlich-rechtlichen Rundfunks zur abgabenseitigen Finanzierung desselben. Ist nicht ganz unwichtig. --Kreuzschnabel 13:37, 12. Apr. 2016 (CEST)
ist es nich so, dass der ÖRR und somit auch der nämliche Beitragsservice durch den Staat geschaffen wurden? durch Gesetze und Staatsverträge? dann kommen noch diverse Beeinflussungsmöglichkeiten durch den Staat dazu... z. B. Monitor (Fernsehmagazin) klagte einmal selbst über Kürzung der Sendezeit durch die Politik... --Heimschützenzentrum (?) 19:47, 12. Apr. 2016 (CEST)
Der Staat kann durchaus Entitäten schaffen, auf die er selbst keinen Einfluss haben kann und darf. Gegenteilige Versuche wurden bisher immer vom Bundesverfassungsgericht gestoppt, Stichwort Adenauer-Fernsehen. Auch der (westdeutsche) ÖRR wurde zwar von den Siegermächten des Zweiten Weltkriegs verordnet und installiert, aber ohne einen politischen, wirtschaftlichen, finanziellen oder inhaltlichen Einfluss. Über das Wesen von Deutschlandradio, der Deutschen Welle, Radio Andernach, diverser staatlich finanzierter Studentensender gab es höchstrichterliche Urteile. Bei den teilweise von irgendwelchen Bundesländern finanzierten privaten Spartensendern Xxhome und Timm gab es Diskussionen über eine fehlende Staatsferne, die aber ergebnislos blieben. Umgekehrt gab es öffentliche Diskussionen, weil B.TV zeitweise als Haussender des baden-württembergischen Staatsministerium galt. Das war aber lediglich vorauseilender Gehorsam, ohne irgendeinen tatsächlichen Einfluss des Landes Baden-Württemberg. Zur Erinnerung: Die baden-württembergische Landesanstalt für Kommunikation ist wie die anderen Landesmedienanstalten ebenfalls öffentlich-rechtlich verfasst, staatsfern und aus den Rundfunkgebühren oder seit neustem -beiträgen finaziert. --Rôtkæppchen₆₈ 00:55, 13. Apr. 2016 (CEST) Das Parlamentsfernsehen des Deutschen Bundestages#Kritik habe ich vergessen. Das darf aufgrund def fehlenden Staatsferne nicht rundfunkmäßig verbreitet werden. --Rôtkæppchen₆₈ 03:14, 13. Apr. 2016 (CEST)
der ÖRR und die Organisationen um ihn drum herum sind eindeutig vom Staat gesteuert/geknebelt... *lol* also nich nur durch das Strafrecht, denn: wenn die wie Lois&Clark wären, gäb es so verrückte Fälle wie Mollath und Arnold nich... so! :) --Heimschützenzentrum (?) 10:35, 13. Apr. 2016 (CEST)
Da hast Du sicher Belege für… --Rôtkæppchen₆₈ 11:41, 13. Apr. 2016 (CEST)
wenn der ÖRR jahrelang zusieht, wie einer ohne Grund quasi in Haft ist, und dann erst nach Ablauf irgendeiner Frist aktiv wird, dann ist das für mich Beleg genug... man glaubt ja auch nich (mehr), dass derselbe Fahrer jede Woche einen gleichartigen Unfall haben kann... --Heimschützenzentrum (?) 16:37, 13. Apr. 2016 (CEST)
Ein zusammenhanglos hingeworfener wirrer Satz ist sicher kein Beleg im Sinne der Wikipedia. --Rôtkæppchen₆₈ 20:59, 13. Apr. 2016 (CEST)
? --Heimschützenzentrum (?) 01:33, 14. Apr. 2016 (CEST)
mal anders: 1. wielange hätte der ÖRR denn versagen dürfen, bevor du seine Abhängigkeit für nachgewiesen hälst? 2. gibt es denn umgekehrt den Nachweis der Unabhängigkeit? Böhmermann? lol --Heimschützenzentrum (?) 02:14, 14. Apr. 2016 (CEST)
Volkswagen wurde nicht nur vom Staat "geschaffen", sondern gehört bis heute zum Teil dem Land Niedersachsen. Zahlt man nun den Kaufpreis für einen PKW von VW "an den Staat"? Dabei kann das Geld vom Autokauf über die Dividende tatsächlich beim Staat landen, anders als beim Rundfunkbeitrag, bei dem das Geld nicht direkt ohne Gegenleistung an Gebietskörperschaften fließen kann.--141.76.20.173 15:21, 13. Apr. 2016 (CEST)
gibt es n VW-Gesetz? oder n VWStV? nö, oda? --Heimschützenzentrum (?) 16:37, 13. Apr. 2016 (CEST)
Doch, gibt es. --Rôtkæppchen₆₈ 20:59, 13. Apr. 2016 (CEST)
das Beispiel passt immer weniger... durch das Gesetz gibt der Staat ja gerade Kontrolle an Einzelne ab... --Heimschützenzentrum (?) 01:33, 14. Apr. 2016 (CEST)
Eine Kontrolle, die er beim öffentlich-rechtlichen Rundfunk gar nicht erst hat. "Der Staat" bist übrigens auch du...--141.30.184.25 10:22, 14. Apr. 2016 (CEST)
1. gibt es einen Nachweis, dass der ÖRR unabhängig ist? außer den Beteuerungen des Staates? 2. auf „den Staat“ habe ich offensichtlich keinerlei Einfluss... der setzt auch verrücktesten Schwachsinn gegen meinen erklärten Willen durch (nicht einmal beteiligten Beamten droht dabei ernsthaft Strafe... das ist beim ÖRR sogar noch schlimmer...)... --Heimschützenzentrum (?) 12:28, 14. Apr. 2016 (CEST)
Wie jetzt, das Wahlrecht ist dir auch entzogen? Und eine Partei darfst du auch nicht gründen? --Eike (Diskussion) 12:33, 14. Apr. 2016 (CEST)
1. verspottet mich da etwa jmd? 2. welche Partei ist denn z. B. gegen „Psych“iatrie, wie sie heute betrieben wird (also: künstliche Syphilis, Epilepsie und sonstige Schweinereien)? irgendwie antworten die mir darauf nie... vllt ist auch mein Fax gaputt? --Heimschützenzentrum (?) 14:40, 14. Apr. 2016 (CEST)
Das sollte jetzt nicht (übermäßig) spöttisch sein. Was ich rüberbringen wollte, ist, dass du vermutlich dein 80-Millionstel an Mitspracherecht hast, das alle anderen auch haben. Wenn du Dinge willst, die aus völlig unverständlichen Gründen sonst fast keiner will, gehört dazu auch das Recht, eine eigene Partei zu gründen. Es könnte aber natürlich auch sein, dass du dir manchmal Dinge nur einbildest. --Eike (Diskussion) 14:55, 14. Apr. 2016 (CEST)
zu glauben, man hätte ein Mitspracherecht ist wohl eher die Einbildung... :) nich mal aus meiner Wohnung kann ich Rauchwarnmelder und so raushalten... --Heimschützenzentrum (?) 15:25, 14. Apr. 2016 (CEST)
Wie gesagt: ein Achtzigmillionstel. Das ist natürlich wenig, aber du wirst ja nicht für dich beanspruchen, mehr Mitspracherecht zu haben als andere. Inwieweit diese Macht in den privaten Bereich reinragen darf, ist natürlich diskussionswürdig. Andererseits ist ein Feuer in deiner Wohnung ganz schnell keine Privatsache mehr. Aber das hat nicht mehr viel mit dem Rundfunkbeitrag zu tun. --Eike (Diskussion) 16:17, 14. Apr. 2016 (CEST)
1. ich würd bekanntlich mich von den anderen viel stärker trennen... aber das will ja irgendwie niemand außer mir... deswegen quält mich der ganze Unfug vermutlich auch so sehr... 2. das hatten wir doch schonmal: n Rauchwarnmelder ist höchstens nur zum Schutz derjenigen geeignet, die das laute Signal auch richtig laut hören können... es bleibt also Privatsache... --Heimschützenzentrum (?) 16:49, 14. Apr. 2016 (CEST)
zu 1: Die Menschheit ist nicht durch Vereinzelung zu dem geworden, was sie ist... zu 2: Auch die Nachbarn profitieren davon, wenn deine Wohnung nicht abbrennt - es macht ihre Wohnung sicherer. --Eike (Diskussion) 10:12, 15. Apr. 2016 (CEST)
1. eben drum! 2. das sagt ja noch nichma die Rauchwarnmelder-Mafia, denn: was soll denn das Feuer bremsen, wenn ich mich vor die Tür rette? oder denkst du ich hol' dann erstmal mein dickes Strahlrohr raus? *lol* außerdem bin ich n Freund der Prävention... --Heimschützenzentrum (?) 14:17, 15. Apr. 2016 (CEST)
Vielleicht reicht noch die Löschdecke, vielleicht holst du schnell dein Rohr raus, und wenn nicht, kaannst du immer noch früher die Profis rufen - die haben ein noch viel größeres Rohr! Aber hier genug davon, wenn du weiterquatschen willst, lass uns eine Benutzerdiskussionsseite nehmen. --Eike (Diskussion) 14:26, 15. Apr. 2016 (CEST)
Durch die Hintertür kommen die Damen und Herren vom "Beitragsservice" (fast) immer rein: Private Kfz sind beitragsfrei. Bei Betriebsstätten ist ein Kfz beitragsfrei - aber nur wenn für die Betriebsstätte der Rundfunkbeitrag gezahlt wird. Ist die Betriebsstätte in einer Wohnung und damit als Betriebsstätte beitragsfrei, wird aber ein eigener privater Pkw auch für den Betrieb genutzt, besteht für diesen Pkw die Beitragspflicht, unabhängig vom Umfang der betrieblichen Nutzung. 91.54.42.184 12:07, 12. Apr. 2016 (CEST)

Witwenrente und Krankenversicherung

Ist eine Bezieherin einer großen Witwenrente nur mit dem Anteil auf die Rente gesetzlich krankenversichert, oder werden auch andere Einkommen/Vermögen dafür angerechnet, sodaß ggf. ein höherer Beitrag wie bei einer freiwillig gesetzlichen Krankenversicherung besteht? Beispiel - der Verstorbene bezog eine EU-Rente von 900 Euro. Als Altfall werden dann 540 Witwenrente gezahlt. Für den Toten bestanden 3 Lebensversicherungen über 750.000 Euro, von denen die Witwe nun samt der Zinsen lebt. Selbst bei 1% Verzinsung liegen diese Einnahmen allein also deutlich über der Witwenrente.85.178.66.31 09:46, 15. Apr. 2016 (CEST)

Das hängt davon ab, ob die Rentnerin Pflichtmitglied oder freiwillig versichert ist. Die Pflichtmitglieder zahlen keine Beiträge auf Zinseinkünfte, die freiwillig Versicherten dagegen schon. --Snevern 10:01, 15. Apr. 2016 (CEST)
Sry, war noch ausgeloggt. Also sie ist bislang pflichtversichert durch eigene Arbeit, und würde angesichts dieses Vermögens gerne aufhören zu arbeiten, ohne sich mit ALG 1 befassen zu müssen. Eigene Rente steht noch nicht an. Die Witwenrente wäre ihr einziges Einkommen. Oliver S.Y. (Diskussion) 10:09, 15. Apr. 2016 (CEST)
Müsste das nicht auf dem Rentenbescheid stehen, ob sie über den Rentenbezug pflichtversichert ist? Hier ist wohl die 9/10-Regelung relevant. -- 217.236.175.4 10:16, 15. Apr. 2016 (CEST)
Zum Zeitpunkt des Rentenbescheids wäre sie ja über ihre Arbeit parallel pflichtversichert, und die Witwenrente würde sogar noch anteilmäßig gekürzt. Gibt da irgendwie ne gefühlte Gerechtigkeitslücke beim Vergleich beider Aspekte für mich. Ob existent, oder nur angenommen, darum die Frage hier.Oliver S.Y. (Diskussion) 10:21, 15. Apr. 2016 (CEST)
Die zwei Unbekannten in dieser Gleichung sind "wie hoch lägen die Zinsen" und "würde durch Anrechnung der Zinsen auf die Witwenrente diese komplett entfallen", denn dann wäre es natürlich auch Essig mit der Krankenversicherung. Ich bin aber kein Rentenexperte. -- 217.236.175.4 10:32, 15. Apr. 2016 (CEST)
Nehm einfach an 1%, also 7500 Zinsen und 6480 Witwenrente. Da das Geld ja aktuell angelegt wird, gäbe es ja auch in den ersten Monaten gar keine Zinseinnahmen.Oliver S.Y. (Diskussion) 11:03, 15. Apr. 2016 (CEST)
Maßgeblich ist wohl § 97 SGB VI. Grundlage für die Einkommensanrechnung ist wohl der aktuelle Rentenwert, der sich zwischen West- und Ostdeutschland unterscheidet. Auch §§ 18a-18e SGB IV könnte hier wichtig sein. -- 217.236.175.4 12:15, 15. Apr. 2016 (CEST)
Nachtrag: bei bestimmten Altfällen bleiben nach § 114 SGB IV Zinsen als Einkommen völlig unberücksichtigt. Vielleicht trifft das ja hier zu? -- 217.236.175.4 12:32, 15. Apr. 2016 (CEST)

Webseite gesucht welche Chemie Elemente anzeigt.

Hallo, ich wollte gerne wissen ob es eine Seite gibt, die mir aus einer Summenformel anzeigen kann, um welches Element es sich handelt. Ich suche zum Beispiel ein Elment für: 8H+S+2F+2N+6C . Liebe Grüße--Ip80.123 (Diskussion) 05:59, 11. Apr. 2016 (CEST)

Du meinst: um welchen Stoff. Offensichtlich handelt es sich ja um eine Verbindung, kein Element :-) Ob es so eine Seite gibt, weiß ich nicht, aber ganz eindeutig kann so eine Ermittlung wegen der Isomerie nicht sein. --Kreuzschnabel 08:53, 11. Apr. 2016 (CEST)
Das Handbook of Chemistry and Physics, das wie die Bibel in jeden guten Haushalt gehört, hat eine Sektion (Formula index of organic compounds), in der Summenformeln organischer Verbindungen (aber Elementanordnung in der richtigen Reihenfolge, junger Freund!) angegeben sind und mit Referenznummer auf die Verbindung(en) vorne im Buch verweisen (dort Name, Eigenschaften wie Schmelzpunkt, Siedepunkt etc.). Da kann eine Summenformel schon mal für 15 oder 20 verschiedende Verbindungen stehen (C6H14O ist beispielsweise die Summenformel von (mindestens) 32 (!) verschiedenen (!) Verbindungen - Einige davon haben wir bereits...). Ob es das im Web gibt, weiss ich nicht. Play It Again, SPAM (Diskussion) 09:35, 11. Apr. 2016 (CEST)
Hier ist es: Hier das Ergebnis für C6H6. (Klappt nur, wenn nach dem Hill-System eingegeben wird!) Play It Again, SPAM (Diskussion) 09:43, 11. Apr. 2016 (CEST)
Du kannst auch in der deutschen Wikipedia oder bei Google nach der Summenformel in Hill-System-Reihenfolge ohne Tief- und Hochstellungen volltextsuchen. Deswegen steht diese Formel in jeder Chemieinfobox. C6H8F2N2S --Rôtkæppchen₆₈ 11:22, 11. Apr. 2016 (CEST)[12]


Bitteschön. Die Seite kennt viel, aber nicht alles. Deine Verbindung ist klein und vermutlich ein Aromat, weil das C/H-Verhältnis nahe 1:1 ist. Anwesenheit von Schwefel und Stickstoff spricht für ein Molekül mit biologischer Funktion, Anwesenheit von Fluor für ein Medikament. -- Janka (Diskussion) 11:21, 11. Apr. 2016 (CEST)
Ich habe die Formel in dieses Tool eingegeben. Sie bringt aber Ergebnisse für ganz andere Elementarverhältnisse. Ein wirklich funktionierendes Tool wäre durchaus interessant, bezweifle aber, ob es das gibt. Grundsätzlich kann man das bei so einem kleinen Molekül auch ausprobieren (H und F: nur eine Einfachbindung, C und N ein- bis dreibindig, N maximal drei, C maximal vier Bindungen, außerdem Edelgasregel beachten). --84.135.142.46 14:16, 11. Apr. 2016 (CEST)
Deshalb wurde oben doch auch auf Google verwiesen: Hill-Formel eingeben und gucken, was das Web liefert.
Das garantiert aber nicht, dass damit alle möglichen Moleküle gefunden werden. Play It Again, SPAM (Diskussion) 19:40, 11. Apr. 2016 (CEST)

Bei dieser Suche kann man noch einige Parameter mit einbeziehen. [13]. --Gonzo.Lubitsch (Diskussion) 10:57, 12. Apr. 2016 (CEST)

Bitteschön Das sind schon einmal 2780 Kandidaten. Da kannst du dann je zwei beliebige Wasserstoffatome durch Fluor ersetzen und hast schon einmal eine (sehr lange und sicher nicht vollständige) Kandidatenliste von etwa 125.000 Molekülen. Ich stimme Janka zu; Fluor im Molekül ist smoking gun eines synthetischen Moleküls. Es ist nichts, was mir schon einmal über den Weg gelaufen ist. Pubchem kenn für die konkrete Summenformel 47 Kandidaten. --Foreade (Diskussion) 16:13, 15. Apr. 2016 (CEST)

Domain kaufen?

Hallo, kann man Domains kaufen? Oder doch eher nur mieten? So für 20 Euro im Monat eine einzige Domain?

Und wenn ich dann noch Webspace möchte, gehe ich besser zu einem anderen Anbieter. Und Webspace kostet dann ein paar Euro im Monat, so vielleicht 5 Euro?

Hintergrund: möchte mir gerne eine Website zulegen, vor allem ein wenig probieren und so. Aber gerne schon quasi mit einem angenehmeren Hintergrund und nicht unter einer Subdomain oder mit einem werbefinanzierten Webspace.

Wo finde ich Empfehlungen zu Domain und Webspace? Möglichst aus Deutschland kommend, solide und vertrauenswürdig und seriös? --87.140.193.7 21:43, 13. Apr. 2016 (CEST)

Denic.de verkauft die Domains nicht direkt an Endkunden. Du musst also einen Hostprovider bemühen, der die Domain für Dich kauft. Dieser Hostprovider macht auch den Rest, der zu einer Website notwendig ist. --Rôtkæppchen₆₈ 21:55, 13. Apr. 2016 (CEST)
Einer von vielen Anbietern (ich bin mit ihm zufrieden): http://all-inkl.com/webhosting/paketvergleich/ Domain ist im Preis enthalten. --Eike (Diskussion) 22:01, 13. Apr. 2016 (CEST)
Aber: Du kannst jederzeit zu einem anderen Provider umziehen mit "deiner" Domain. Eine .de-Domain mit Webspace bekommt man - ohne Werbung etc. - schon für ca. 10 bis 13 Euro pro Jahr. Beispiel: https://www.webspace4all.eu/ --2003:45:4648:FD00:94B5:A7E2:6BAB:55DA 01:23, 14. Apr. 2016 (CEST)
Man kann Domains nicht "kaufen", auch nicht "mieten", genausowenig wie man ein Patent oder eine Marke kaufen kann, eine Domain ist ein Immaterielles Gut. Bei einer Domainregistrierung wird der Domaininhaber zum "materiell Berechtigten", d.h. er verfügt über die Eigenschaften, die mit der Domain registriert sind (insbesondere ihre Nameserver-Einträge, die im Endeffekt zu Inhalten und Diensten dieser Domain führen. Die monatliche/jährliche Registrierungsgebühr ist im wesentlichen ein Entgelt für die Dienstleistung der (möglicherweise mehreren hintereinandergeschalten) Domain-Verzeichnis-Verwalter. Doch, Denic bietet bietet Registrierungen für Endbenutzer an: https://direct.secure.denic.de/ , aber diese Leistung ist nicht dazu gedacht, mit den Mitgliedern der Denic e.G. (preislich) zu konkurrieren. Wie schon gesagt, es gibt unzählige Dienstleister, die Domains für sehr geringe Gebühren (monatlich/jährlich) anbieten - einen wesentlichen Preisunterschied macht die Wahl der Top-Level-Domain aus. Zum Experimentieren taugt jede TLD, da kann man also gut nach dem Preis gehen. --Krukrus (Diskussion) 12:08, 14. Apr. 2016 (CEST)
Domains an sich kosten nicht viel (sofern sie noch frei sind), für reines "Parken" geht's je nach TLD und Anbieter bei pi mal Daumen 1 € / Monat los, wobei .de und .eu-Domains meist am günstigsten sind. Teu(r)er wird's erst, wenn du die Domain auch tatsächlich nutzen möchtest, also beispielsweise E-Mail-Postfächer brauchst, eine Weiterleitung auf bereits vorhandenen Webspace einrichten oder Webspace und Traffic vom Provider "mieten". Spätestens hier stellt sich die Frage, was du genau online realisieren willst. Von daher würde ich genau anders herum vorgehen: Nicht erst irgendein beliebiges Domain-und-Website-Komplettpaket kaufen und damit herumspielen, sondern zunächst mal (offline) herumspielen und erst dann, wenn deine Website "steht", schauen, welches Hosting-Paket (mit wieviel Traffic, welchen Datenbanken, Sicherheitsmerkmalen usw.) du dafür brauchst. Wenn du allerdings mit all dem noch gar keine Erfahrung hast, könnte für den Start ein Komplettanbieter mit "Baukasten" wie Jimdo das richtige sein. --Mangomix 🍸 17:18, 15. Apr. 2016 (CEST)

Welches Studienfach lernt Flora-Fauna-"Gutachten" zu erstellen?

Nach der "Lektüre" (wie sagt man eigentlich beim Film schauen?) des Filmes Die Vermessung der Welt wollen wir (ohne schulischem Auftrag!) auf den Spuren Alexander von Humboldts wandeln, uns ein abgrenzbares Gebiet in der Umgebung vornehmen und dessen Zusammensetzung ermitteln. Insbesondere bezüglich Baumbewuchs und anderen großen Pflanzen sowie auch die Bestimmung, welche Tiere in diesem Gebiet leben (anhand von Spuren/Hinterlassenschaften usw., alles größer, als Insekten und Spinnen). Unsere Ergebnisse würden wir gerne mit "Fachleuten" abgleichen. Dazu jedoch ein Gutachtenbüro (was für eins eigentlich?) zu beauftragen, ist wohl zu hoch gegriffen, trotzdem wäre es mir Geld wert und daher besteht die Idee, einen Studenten, der vielleicht so etwas (Feldstudie) im Rahmen seines Studiums sowieso machen müsste, zu involvieren. Da wir hier (Berlin) gut mit Hochschulen ausgestattet sind, müsste ich nur noch wissen, welcher Fachbereich/Studiengang dafür zielführend wäre?--Wikiseidank (Diskussion) 07:11, 15. Apr. 2016 (CEST)

Links, Liste der Sachkundigen. VORHER anrufen, fragen. Play It Again, SPAM (Diskussion) 07:40, 15. Apr. 2016 (CEST)
Danke (guter Impuls der Verwaltung). Der Link verweist auf den Bereich der gebäudebewohnenden Arten. So etwas für Freilandartenschutz? Wir spekulieren auf die Untersuchung eines "Waldstückes", eher so ein untypisches Waldstückchen zwischen Feldern und Garten-/Wohnbebauung. Die Berliner Forsten sind wohl eher für die (richtigen/) großen Waldgebiete zuständig?--Wikiseidank (Diskussion) 08:04, 15. Apr. 2016 (CEST)
Sowas machen Ökologen, also spezialisierte Biologen, und entsprechend qualifizierte Landespfleger, entweder als Freiberufler oder, öfter, in entsprechend spezialisierten Planungsbüros. Ein Gutachten durch so einen Profi wäre aber ggf. sehr teuer (geht von Stundensätzen von 50 Euro aufwärts, als Minimum, aus). Die müssen halt davon leben können. In Berlin gibt es das Institut für Landschaftsarchitektur und Umweltplanung an der Technischen Universität, die sind aber eher weniger auf sowas spezialisiert. Eher euer Fall wäre das Institut für Ökologie der FU oder die Arbeitsgruppe Ökologie am Institut für Biologie der Humboldt-Universität, wo sich eventuell ein paar mitleidige Studenten erbarmen könnten. Realistischer ist es sicher, bei den Naturschutzverbänden BUND und NABU mal nachzufragen. Möglicherweise gibt es eine Arbeitsgruppe, die einen "Geo-Tag der Artenvielfalt" für 2016 organisiert (gab´s immer in den vergangenen Jahren), das wären die richtigen Leute, die da zusammenkommen. Macht euch nicht zuviel Hoffnungen, eine verlässliche lokale Artenliste für ein Gebiet ist weitaus anspruchsvoller, als es sich zunächst anhört. Einfach rauslaufen und Gucken führt nicht weit. Pflanzen blühen nicht immer dann, wenn sie sollen, Tiere tendieren zum Heimlichen und Misstrauischen und sind nie da, wann und wo sie sein sollen, Fledermäuse kommen nur Nachts raus, usw. ...--Meloe (Diskussion) 08:56, 15. Apr. 2016 (CEST)
Danke! Humboldt konnte sich ja auch nicht ewig überall aufhalten und hat Erstaunliches zusammengetragen. Vielleicht schätzt man seine Leistung auch noch höher, wenn man es selbst mal probiert hat.--Wikiseidank (Diskussion) 12:34, 15. Apr. 2016 (CEST)
Google nach "Naturschutzplanung Gutachter", da findest Du Leute - fast immer Biologen - , die sowas ("Ökologische Voruntersuchung", "Ökologische Ressourcenanalyse") gewerblich machen. Wie es gemacht wird, findest Du ziemlich detailliert hier (allerdings konkret eher für Wiesen und Ackerflächen als für Wald). --Joyborg 15:33, 15. Apr. 2016 (CEST)

Wer kann mir bei einem Parlamentsdiagramm helfen?

Hallo Wikipedianarinnen und Wikipedianer, ich bräuchte für meine gewerkschaftliche Tätigkeit ein klassisches Parlamentsdiagramm. Gibt es hier irgendwo eine Anwendung (Tool), die so etwas leisten kann? Direkt hier bei Wikipedia oder gibt es so etwas auch bei Excel? Was ist es was ich genau will? Ich möchte zwei Gruppen darstellen in Form eines halben Kreises, so wie an den Wahlabenden, wenn nach den Prozentdarstellungen noch die Sitzverteilung im Parlament dargestellt wird. (Falls mir jemand besonders intensiv helfen will: Die verdi hat 8 Sitze und die komba hat 3 Sitze. Der Personalrat hat also insgesamt 11 Sitze. Die verdi müsste in rot und die komba in violett dargestellt werden.) Wer weiß was? Danke. Le Duc de Deux-Ponts (Diskussion) 08:55, 15. Apr. 2016 (CEST)

Kann man da nicht mit Excel/Tabellenkalkulation deines Office-Pakets ein Diagramm machen? Da gibt es doch eine Auswahl an Vorlagen. Farblich anpassen kann man die auch. --Susad 1978 (Diskussion) 09:48, 15. Apr. 2016 (CEST)
Du kannst mich auch anschreiben, da schick ich dir ein Beispiel --Susad 1978 (Diskussion) 09:51, 15. Apr. 2016 (CEST)
Sitzverteilung
8
3
Insgesamt 11 Sitze
  • verdi: 8
  • komba: 3

Die Vorlage:Sitzverteilung kann alles! Gruß, --Nfreaker91 18:26, 15. Apr. 2016 (CEST)

Excel:
  • Tabelle erstellen: verdi 8, komba 3, freier Bereich 20.
  • Die Daten markieren.
  • Einfügen → Diagramme → Kreisdiagramme → 3D-Kreis.
  • Auf den Teil, der den freien Bereich darstellt, zweimal (mit gewissem zeitlichem Abstand) klicken, um nur ihn zu markieren.
  • Klick mit der rechten Maustaste auf diesen Bereich → Datenpunkt formatieren → in der erscheinenden Spalte das Symbol mit dem Farbeimer auswählen → Füllung → keine Füllung → Rahmen → keine Linie, damit dieser Teil unsichtbar wird.
  • In der Spalte auf das Symbol mit den drei Säulen klicken → Winkel des ersten Segments, so das verbliebene Diagramm wunschgemäß drehen.
  • Durch eine Anpassung des Wertes für den freien Bereich in der Tabelle kann das verbliebene Diagramm geeignet gestaucht oder gespreizt werden. --BlackEyedLion (Diskussion) 09:53, 15. Apr. 2016 (CEST)
Vielen Dank schon mal für die guten Tipps. Leider verstehe ich den zweitletzten Punkt mit den Drei Säulen nicht. Wo ist dieser Knopf? (Wie drehe ich meinen Halbkreis?) Übrigens schon erstaunlich, wie viele verschiedene Möglichkeiten der Darstellung Excel bietet, aber das prompt die Parlamentsdarstellung fehlt! Le Duc de Deux-Ponts (Diskussion) 13:08, 15. Apr. 2016 (CEST)
In den neuesten Excel-Versionen (denen mit der Ribbon-Darstellung anstelle von Menüs und Symbolleisten) erscheint nach „Datenpunkt formatieren“ eine Spalte (üblicherweise rechts), die die Eingabemöglichkeiten zur Einstellung enthält. Sie ist in drei Bereiche gruppiert: Farbeimer für die Gestaltung, nach meiner Erinnerung Quadrat mit Bemaßungen für die Positionierung und drei Säulen (Säulendiagramm) für diagrammspezifische Einstellungen. --BlackEyedLion (Diskussion) 00:25, 16. Apr. 2016 (CEST)

Dumme Frage: Zu viel RAM einbauen

Bevor ich es ausprobiere und einen gut erhaltenen Computer grille: Das Chip-Set eines alten Computers unterstützt max. 256 MB RAM verteilt auf 2 Slots zu max. 128 MB. Verbaut ist ein 128-MB-Modul. Ich habe 2 1-GB-Module da. Was würde passieren, wenn ich eines der Module in den zweiten Slot einbaue, was würde passieren, wenn ich beide Module einbaue? Hätte ich dann einen funktionierenden 256-MB-Computer, oder Sperrmüll? --° (Gradzeichen) 15:46, 15. Apr. 2016 (CEST)

Ich vermute, er startet einfach nicht. Ist aber m. E. keinen Versuch wert. --Eike (Diskussion) 15:49, 15. Apr. 2016 (CEST)
mit viel glück hast du dann einen 256MB-rechner, hochwahrscheinlich passiert aber das, was eike vermutet. --JD {æ} 15:50, 15. Apr. 2016 (CEST)
Bei so einem Rechner vermute ich mal das da noch der alte SDRAM verbaut ist, gab es da überhaupt 1GB Riegel? --Be11 (Diskussion) 15:56, 15. Apr. 2016 (CEST)

Ich habe mal gehört, bei windows wäre es dann so, dass nur so und soviel GB arbeitsreicher gelesen werden können und der Rest ignoriert wird. Bei Linux könnte es sein, dass alles gelesen wird. Versuch es also doch mal einfach nur guck, ob die 1 GB genausoviel mA haben wie deine anderen 256 MB sticks. --Ip80.123 (Diskussion) 15:59, 15. Apr. 2016 (CEST)

Es kommt darauf an. Manchmal wurde das vom Hersteller einfach nicht getestet, weil es damals keine so großen RAM-Riegel gab. Manchmal liefert der Chipset eine Größenbeschränkung, sogar je nach Bitness des Betriebssystems. Es kann dann vorkommen , dass der Rechner gar nicht mehr geht oder den zusätzlichen Speicher nicht erkennt. --Rôtkæppchen₆₈ 16:08, 15. Apr. 2016 (CEST)


Stelle erst einmal sicher, dass die Kerben in deinem RAM-Modul überhaupt mechanisch in die RAM-Slots des Boards passen. Da gibt es subtile Unterschiede zwischen DDR-SDRAM, was das Board vermutlich benötigt, und den Nachfolgern DDR2 und DDR3. -- Janka (Diskussion) 16:15, 15. Apr. 2016 (CEST)

Na ja, wenn es mechanisch nicht passt, merk ich das ja beim Einbau. Auch wenn nach dem Einschalten nichts geht, ist das noch nicht problematisch. Die Frage ist halt, ob ich das dann wieder zurückbauen kann, oder ob es da Hardwareschäden geben kann? (Falls es klappt, dachte ich an Antix, habe da aber keine Erfahrungen) --° (Gradzeichen) 16:36, 15. Apr. 2016 (CEST)
In der Dunkelheit des Gehäuses siehst du es nicht. Man kann DDR1 und DDR2-Module sehr leicht falsch herum "halb" einbauen. Danach sind das Modul und der Slot gegrillt. Prüfe das erst einmal. Es ist nälich sehr wahrscheinlich, dass dein Board DDR1 benötigt und deine Module sind vermutlich DDR2 oder gar DDR3. -- Janka (Diskussion) 16:40, 15. Apr. 2016 (CEST)
Auf so etwas bin ich auch mal reingefallen, allerdings war glücklicherweise nur der Riegel und nicht der Slot gegrillt. --Be11 (Diskussion) 16:44, 15. Apr. 2016 (CEST)
Poste mal Typ des Motherboards und der Speicherriegel. Die Experten® der Auskunft können Dir dann dabei helfen, herauszufinden, ob es sinnvoll oder möglich ist, die Riegel in den Rechner einzubauen. --Rôtkæppchen₆₈ 16:56, 15. Apr. 2016 (CEST)
Ich hab mal ein bisschen recherchiert. 128 MiB gibt es im Desktopbereich in den Bauformen PS/2-SIMM, SDR SDRAM und DDR1. 1 GiB gibt es als SDR SDRAM, DDR1, DDR2 und DDR3. Die Tauschaktion kann also nur klappen, wenn Board und Ersatzriegel entweder SDR oder DDR1 sind. AT- und PS/2-SIMM sind zu alt, DDR-2, -3 und -4 zu neu. --Rôtkæppchen₆₈ 21:38, 15. Apr. 2016 (CEST)

3D-tauglicher HDMI 5.1 Analog-Audio-Renderer

Liebe Heimkinogemeinde, ich hab bei mir zuhause eine 5.1 Soundanlage (alte Logitech), die zwar gut klingt aber nur analoges Audio entgegennimmt. 5.1 Audiorenderer gibts ja wie Sand am Meer, aber ich find leider bei keinem ne Angabe ob die auch mit FullHD 3D via HDMI klarkommen und ob bei Stereosignal via HDMI auch Upsampling auf 5.1 beherrscht wird... wer kennt da was? Und ich bräuchte auch noch irgendeine Lösung, die es mir erlaubt, an die Soundanlage per Bluetooth ein Handy anzuklemmen... natürlich wäre da eine Lösung am besten, die Audio per HDMI ausgeben kann. Selbstbau mit 'nem RPi 3 oder gibt es auch hier schon was fertiges?

--2001:A61:115F:4701:701B:2D9B:7B44:1FF0 20:45, 15. Apr. 2016 (CEST)

Ich werde nicht ganz schlau, was du vorhast, aber es gibt von allen relevanten Herstellern AV-Receiver mit 3D-Unterstützung und Bluetooth. Und natürlich können die auch alle auf unterschiedliche Arten 2 auf 5(.1) Kanäle verteilen. Suchhilfe: http://geizhals.de/?cat=hifirec&xf=327_3D-Ready~5771_Bluetooth+%28integriert%29#xf_top --Magnus (Diskussion) 21:24, 15. Apr. 2016 (CEST)

Hohe Strafe fürs Fahren mit Gangschaltungsauto

welche nennenswerte schöne Strafe erwartet jemanden, der einen Führerschein hat in dem verzeichnet ist, dass er nur Automatikfahrzeuge fahren darf, jedoch in einem Gangschaltung Fahrzeug unterwegs ist und nicht rausgewinkt wird, weil er zu dumm zum Schalten war, sondern wegen allgemeiner Verkehrskontrolle? 2 Jahre Gefängnis? 5 Jahre Bewährung? Und wenn er Fahrzeughalter ist, Stilllegung und Beschlagnahmung des Fahrzeuges? Wenn er nicht Halter ist sondern das Auto geliehen ist, was dann? Keine Stilllegung? Und das Paradoxon, dass es wohl in 40-50 Jahren keine Gangschaltungsfahrzeuge mehr geben wird, weil das Benzin ausgeht, sei mal auch noch dahin gestellt. --Ip80.123 (Diskussion) 02:25, 13. Apr. 2016 (CEST)

Siehe Fahren ohne Fahrerlaubnis und dort verlinkte Straf- und Ordnungswidrigkeitstatbestände. --Rôtkæppchen₆₈ 02:28, 13. Apr. 2016 (CEST)
Wenn im Behindertenführerschein eine Einschränkung zum Auto drin steht, ist die grundsätzlich im eigenen und im Interesse der Allgemeinheit einzuhalten. In der Regel sind diese Eintragungen Einzelfallentscheidungen von medizinischen und technischen Sachverständigen und auch begründet, vom Behinderten auch beim Erwerb des Führerscheins nicht weiter widersprochen, also im Umfang akzeptiert. Es ist also nicht nur ein juristisch-technisches Problem, sondern auch ein juristisch-medizinisches. Wenn Jemand die Einschränkungen nicht einhält, wie oben auch noch uneinsichtig und sachfremd verteidigt, wird auch zu entscheiden sein, ob der Führerschein nicht eingezogen werden muss. Wenn sich deine Behinderung soweit verbessert hat, du der Meinung bist Fahrzeuge ohne technische Einschränkungen zu lenken, kannst du eine Neubegutachtung beantragen. Das Kind jetzt liegt im vorliegenden Fall allerdings schon im Brunnen. Viel Erfolg und bedenke auch die Allgemeinheit hat ein Recht.--2003:75:AF0F:9100:9CDA:CF4A:3C7F:AD22 03:58, 13. Apr. 2016 (CEST)
mir wäre nicht bekannt, dass aus gesundheitlichen Gründen jemand nur Automatik fahren kann, das macht sehr wenig sinn. Soviel nimmt Schlaten und Kupplung drücken nciht in Anspruch, wenn einer kein Pedal drücken kann, kann er überhaupt GAR NICHT fahren, weil beide Varianten ein Gaspedal haben... In den verlinkten Links stand nichts zu fahren mit einem Gänge Auto obwohl man nur einen Führerschein für Automatik Fahrzeuge hat... Noch eine Frage wegen Mofa: Muss man Schutzkleidung tragen so ähnlich wie Motorradfahrer? --Ip80.123 (Diskussion) 05:11, 13. Apr. 2016 (CEST)
Für Behinderte gibt es auch Autos ohne Pedale.[14][15] Wenn man nur die Hände zur Verfügung hat, kann man max. 2 Sachen gleichzeitig machen (außer man verwendet verschiedene Finger an einer Hand gleichzeitig, für was aber mehr Koordination braucht als man beim normalen Fahren braucht), also nicht gleichzeitig lenken, schalten und kuppeln, also braucht man zumindest eine Auto, bei dem man nicht selbst kuppeln muss. Dafür braucht man nicht unbedingt eine volle Automatik, es würde auch gehen, wenn nur die Kupplung automatisiert ist. --MrBurns (Diskussion) 05:54, 13. Apr. 2016 (CEST)

Also heißt das auf deutsch, das sind Menschen mit behinderung die ausweise besitzen, wo drinnen steht, die dürfen ausschließlich Automatik fahren? ich dachte ein kerngesunder Mensch hat ebenfalls die Möglichkeit, so einen Führerschein zu haben auf dem Vermerkt wurde, dass die Fahrprüfung in Automatikfahrzeugen stattfand...--Ip80.123 (Diskussion) 05:59, 13. Apr. 2016 (CEST)

Wenn im Ausweis steht, dass du nur Fahrzeuge mit Automatik fahren darfst (Code 15.03), dann ist dieser Ausweis nur für Fahrzeug mit Automatik gültig. Für geschaltete Fahzeuge besitzt du in diesem Fall keine Fahrerlaubniss, entsprechend kommt der Straffenkatalog für "Fahren ohne Fahrerlaubnis " zum Einsatz.--Bobo11 (Diskussion) 06:11, 13. Apr. 2016 (CEST)
Die Automatik/Gangschaltungdebatte ist völlig überholt, ist vom Gesetzgeber nicht angepasst worden und meiner Kenntnis nach noch nie Prüfgegenstand durch die Verkehrsüberwachungsbehörde - und das aus guten Grund. Es ist doch keine hohe Kunst, eine Gangschaltung zu bedienen. Ein 14jähriger lernt das in einer Ausfahrt. Bei der Verkehrssicherheit kommt es auf ganz andere Fähigkeiten an, als permanent das "Motoröl umzurühren". Diese Fähigkeiten erwirbt man (oder genauso wenig wie bei einer Gangschaltungfahrprüfung) auch bei einer Automatikfahrschule. Wenn wir uns ernsthaft Gedanken um Verkehrssicherheit machen wollen, dann gibt es da unzählige, gravierende Themen.--Wikiseidank (Diskussion) 07:02, 13. Apr. 2016 (CEST)
Auch Auflagen, die man selbst für überholt hält, sind selbstverständlich solange zu beachten, wie sie im Dokument stehen. --Heletz (Diskussion) 12:35, 13. Apr. 2016 (CEST)
BK
Ob es eine hohe Kunst ist, lassen wir mal dahingestellt. Tatsächlich ist die Koordination von Fußarbeit und Schaltung ein Problem für Fahranfänger. Dein Vierzehnjähriger wird vielleicht kaum Gelegenheit haben, sofort mit einem Schaltwagen durch die Gegend zu fahren. Aber es geht auch nicht um Vierzehnjährige.
Wer seit Jahrzehnten nur Automatik fährt (und inzwischen auch schon älter ist), hat nicht die nötige Routine. Es ist auch vorwiegend die Klientel, die Bobo11 anspricht, eher Damen als Herren, eher nicht dem Vierten Stand zugehörig, eher kein Führerschein mit 20, sondern eher mit 40 oder 50 und das auch schon vor 40 Jahren („Straffen“ lassen die sich heute eher nicht mehr; da sind sie moderner als beim Autofahren, da kommt Botox zum Einsatz). Das Problem ist einerseits, dass die den Gang nicht wechseln können, ohne den Blick mehrere Sekunden lang intensiv auf den Schaltknüppel zu richten, währenddessen aber auf der Straße auch Interessantes geschehen kann.
Vor allem aber werden instinktiv die Pedale verwechselt; letzteres gilt auch für „Schalter“, die seit Jahrzehnten keinen Automatikwagen gefahren haben; die vergessen dann auch das Halten der Bremse. Dadurch passieren (weil es sich um instinktives Verhalten handelt) Unfälle. Und selbstverständlich schaut der Polizist bei der Routinekontrolle in den Führerschein und auf die Schaltung; da hat Bobo11 wiederum recht. Mit Behinderten hat das nichts zu tun. NfdA (Diskussion) 13:10, 13. Apr. 2016 (CEST)
Bis Dezember habe ich fast 1½ Jahre lang einen Automatik gefahren. Die Umgewöhnung auf einen Schalter hat schon bißl gedauert. Es ist einfach sinnvoll, da zu unterscheiden. --Heletz (Diskussion) 14:07, 13. Apr. 2016 (CEST)
Dann sollte man auch Zusatzprüfungen abhalten, wie man mit einem Tempomat umgeht, mit einem ABS richtig bremst, sich vom automatisch ausfahrenden 10 Zoll Monitor nicht ablenken lässt, Linksverkehr gewohnte auf Rechtsverkehr loslässt vv., Provinzler in der Stadt fahren lässt vv., wass auf der Fahrbahn eingezeichnete Radwege bedeuten - insbesondere unter Berücksichtigung von Überholsicherheitsabstand usw. usf. Der "Koordinationsaufwand" einer Gangschaltung ist ein Ammenmärchen.--Wikiseidank (Diskussion) 15:43, 13. Apr. 2016 (CEST)
Oder auch Gebirgsfahrten für Flachländer (wird zwar gerne gemacht, indem man z.B. in Wien mit dem Fahrschüler auf die Höhenstraße fährt, ist aber soviel ich weiß weder in Österreich noch Deutschland zwingend vorgeschrieben), Kurvenfahren mit Anhänger (leichte Anhänger darf man derzeit wohl fahren, ohne das jemals in der Fahrschule gemacht zu haben), etc. Das mit den Radwegen muss man aber jedenfalls in Österreich in der Theorieprüfung können (falls diese Frage bei den zufällig ausgewählten Fragen dabei ist), wies in Deutschland ist weiß ich nicht. Mir hat auch der Fahrlehrer gesagt, dass man beim Überholen von einspurigen Fahrzeugen immer ich glaub min. 1m Abstand halten muss und dass man wenn sich das nicht ausgeht hinterher fahren muss, ist wohl klar. Ich bin mir sicher, wenn ich bei der Fahrprüfung mit 0,5m Abstand einen Radfahrer überholt hätte, hätte es dafür einen Punkteabzug gegeben. Im Moment herrscht halt eher die Philosophie, dass man in der Fahrschule nur die Grundlagen lernt und bei der Prüfung wird geprüft, dass man ins einer Gegend halbwegs Autofahren kann, der Rest ergibt sich aus Übung. Es wäre wohl auch zu viel Aufwand, wie bei einem Piloten fast jede denkbare Situation zu unterrichten, also gehts wohl nicht anders, im Vergleich zu anderen Staaten dürften die Anforderungen in Deutschland und Österreich aber doch recht hoch sein, in vielen US-Bundesstaaten muss man gar keine Fahrstunden in der Fahrschule machen, man kann auch alle mit dem Papa machen und die Fahrprüfung besteht wohl teilweise nur aus einmal um den Block fahren. --MrBurns (Diskussion) 16:06, 13. Apr. 2016 (CEST)
"wies in Deutschland ist weiß ich nicht. Mir hat auch der Fahrlehrer gesagt, dass man beim Überholen von einspurigen Fahrzeugen immer ich glaub min. 1m Abstand halten muss" In Deutschland beträgt (Richterrecht) der einzuhaltende minimale Seitenabstand beim Überholen von Radfahrern (was etwas andere als "Vorbeifahren" ist) "auch unter günstigsten Umständen" 1,50 m; diese Vorschrift ist allerdings nicht sehr bekannt.--78.50.149.93 18:37, 13. Apr. 2016 (CEST)

Ich weiß nicht, warum hier über Behinderungen und Krankheiten diskutiert wird. Es ist viel einfacher: Wenn man früher (keine Ahnung, wie das heute geregelt ist) Fahrunterricht und Prüfung ausschließlich mit Schulungsfahrzeugen mit Automatikgetrieben absolviert hat, hat man eine Beschränkung der Fahrerlaubnis auf solche Fahrzeuge eingetragen bekommen. Um das zu vermeiden waren zwei oder drei Pflichtstunden auf Schalterfahrzeugen notwendig. Benutzerkennung: 43067 21:59, 13. Apr. 2016 (CEST) PS, da kommt mir auch eine interessante Fragestellung in den Sinn: Darf man mit einer Fahrerlaubnis mit Automatikgetriebebeschränkung Fahrzeuge mit Doppelkupplunggetriebe fahren, wenn man per Schaltwippen am Lenkrad manuell schaltet?

Die Diskussion um Behinderungen ist schon richtig. Ich hatte einen Kollegen, der genau diese Beschränkung – nur Automatikfahrzeuge – aufgrund eines nicht vollständig einsetzbaren Armes hat. Man bescheinigt ihm also, mit seinen anderthalb Händen das Lenkrad sicher bedienen, aber nicht gleichzeitig noch schalten zu können (halb nutzbar ist der linke Arm). Er sagte mal, er könne auch Schaltgetriebe fahren, dürfe aber halt nicht. Über mögliche Strafen haben wir uns nicht unterhalten. eryakaas | D 22:28, 17. Apr. 2016 (CEST)
Archivierung dieses Abschnittes wurde gewünscht von: --Dansker 22:44, 17. Apr. 2016 (CEST)

Zusatzfrage

Weil irgendwie ignoriert hier jetzt als eigene Frage: Lieber Rôtkæppchen₆₈, danke für den Artikel, darin heißt es: Wer mit einem versicherten, unfrisierten Mofa fährt, ohne im Besitz der ggf. nötigen Mofa-Prüfbescheinigung zu sein (nur Personen, die nach dem 1. April 1965 geboren sind), begeht nur eine Ordnungswidrigkeit, die mit 20.- € (§ 5 Abs. 1, § 75 FeV; § 24 StVG; Nr. 168 BKat)[1] geahndet wird. Stimmt das wirklich? Und kann ich diese Ordnungswidriegkeit so oft begehen wie ich will, ich werde niemals genötigt und gezwungen, den Roller auszuhändigen und eine Mofa Fahrprüfung für 700 Euro zu machen?--Ip80.123 (Diskussion) 04:45, 13. Apr. 2016 (CEST)

IANAL – ich halte §25 StVG für anwendbar. --Rôtkæppchen₆₈ 12:40, 15. Apr. 2016 (CEST)
Es kommt darauf an, wie oft man beanstandet wird und mit wem (Polizist und Sachbearbeiter der Führerscheinstelle) man es zu tun hat. Es kann durchaus sein, dass man eine Fahrerlaubnis-Sperrfrist wegen fehlender charakterlicher Eignung bekommt. Außerdem kann es sein, dass das Mofa eingezogen wird (Bescheid des Straßenverkehrsamtes). Es kann auch sein, dass das Fahrzeug von der Polizei zur Gefahrenabwehr sichergestellt wird. Außerdem kann es passieren, dass ein Gericht (sofern man Einspruch einlegt) den Regelsatz wegen Vorsatz verdoppelt. Des weiteren ist eine Anordnung zur Teilnahme am Verkehrsunterricht denkbar. Und vor allem ist es rechtlich möglich, dass das Straßenverkehrsamt den "Schwarzfahrer" von der Verkehrsteilnahme als Mofa-Fahrer ausschließt, die Zulassung von Personen zum Straßenverkehr wird demjenigen bezüglich des Mofa-Fahrens abgesprochen (Zuwiderhandlung zieht dann ein Ordnungsgeld resp. Ersatzfreiheitsstrafe nach sich).

--93.133.118.230 10:48, 17. Apr. 2016 (CEST)

Archivierung dieses Abschnittes wurde gewünscht von: --Dansker 22:42, 17. Apr. 2016 (CEST)

Strafe für Wikipedia Admin / User

Angenommen ein Azubi bei einer uns unbekannten Bank hat aus Langeweile manuell Verwendungszwecke durchsucht und erhält die Meldung vom Computer, dass ein Konto sehr viele Zahlungen erhält, mit dem Verwendungszweck "Wikipedia". Es kommt ihm Spanisch/Dänisch/Griechisch vor, und er postet einen Screenshot davon für 2 Stunden in einem Snapchat Account der sich mit Problemen der Wikipedia befasst. Bei weiterer Recherche von Wikipedia Usern die es hinbekommen haben, weitere Artikelverfälschungen gegen Bezahlung rauszufinden stellt sich heraus dass z.B. eine Zahnpasta Firma einen Administrator - oder einfach nur einen Wikipedia User, dafür bezahlt hat, dass er alle Artikel die mit Fluorid und dessen schädlicher giftiger Wirkung zu tun haben, "entschärft" und schreckliche Sachen trotz Belege rauslöscht, damit Fluorid einen besseren angesehenen Standard in der WIkipedia genießt, aber man keine Ahnung hat, in welchem europäischen Land der User lebt, wo kann man das zur anzeige bringen? (Vor allem wenn der Wikipedia Email support darauf nicht reagier).

Es geht hier nicht nur um Fälschung und Verfälschung, sondern aufgrund von Zahlungseingängen sehe ich auch ein illegales Gewerbe und Steuerhinterziehung gegeben. Der Azubi möchte mit der Sache nichts zu tun haben aus Angst, seine Arbeit zu verlieren und hat Banknamen und alles andere im Dokument geschwärzt. Droht auch Wikipedia Administratoren, die Hinweise per Email erhalten oder auf ihre Diskussionsseite - und rein gar nichts dagegen tun, sondern sogar die Emails einfach löschen, ebenfalls Probleme - innerhalb der Wikipedia? Oder genießen die Admins absolute Immunität?--Ip80.123 (Diskussion) 06:13, 13. Apr. 2016 (CEST)

Ist das eine ernst gemeinte Frage oder Beschäftigungstherapie? Mal eben die Kontodaten nach dem Verwendungszweck zu durchsuchen, dürfte für einen normalen Bankmitarbeiter unmöglich sein - schon aus Datenschutzgründen. Das geht maximal für einzelne, konkrete Konten - und wird darüber hinaus vom System geloggt. Aber das war ja nicht die Frage. Ob Steuerhinterziehung vorliegt kann nur beurteilen, wer die Steuererklärung kennt. Es gibt aber auch keine Pflicht für Admins, irgendwann tätig zu werden. Die WP ist ein Projekt, das auf Freiwilligkeit basiert. Aber Admins genießen keine absolute Immunität. Solle ein Admin Seiten "verfälschen" oder anderweitig seine Knöpfe missbrauchen, dann war er/sie die längste Zeit Admin. --Vertigo Man-iac (Diskussion) 07:37, 13. Apr. 2016 (CEST)
Dass Bankangestellte sowas nicht lesen (dürfen) dachte ich auch, bis mich meine Bankberaterin angerufen und mir zu meinem neuen Job gratuliert hat... Ab wann eine Bank wirklich illegale Vorgänge anzeigen darf oder muss, von denen sie Kenntnis erlangt, weiß ich aber nicht, ein Azubi sollte aber wissen, wie er sich in einem solchen Fall verhalten muss, Snapchat ist da mit Sicherheit keine Option. --Ailura (Diskussion) 08:49, 13. Apr. 2016 (CEST)
Bankmitarbeiter können und dürfen die Kontobewegungen einsehen, soweit das für die Kundenbeziehung erforderlich ist ("need to know"). Sie sollen das auch, wenn es besondere Vorfälle gibt. Ein ungewöhnlich hoher Geldeingang kann solch ein Fall sein. Nicht zulässig ist aber das systematische Hinterherschnüffeln, vor allem, wenn der Kunde einem anderen Berater zugeordnet ist.--Vertigo Man-iac (Diskussion) 10:30, 13. Apr. 2016 (CEST)
*reinquetsch* Beschäftigungstherapie - im wahrsten Sinne der letzten drei Silben, befürchte ich. Wenn solche Fragen wenigstens im WP:Café landen würden ... VG --Apraphul Disk 08:35, 13. Apr. 2016 (CEST)
Ich glaube, du hast etwas eigenartige Vorstellungen davon, was die Admins hier darstellen. Das sind keine Oberaufpasser oder verantwortliche Geschäftsführer, sondern das sind ganz normale User mit noch einigen Sonderrechten („Knöppen“), was die Arbeit an der Wikipedia angeht (sie können Artikel komplett löschen, unkooperative Benutzer sperren etcetera) – das heißt: ihnen wird von der Community ein verantwortlicher Umgang mit etwas Dreckarbeit zugetraut, die man nicht gleich jedem freigeben möchte. Lesetip: Wikipedia:Administratoren. Und wenn irnkein Benutzer unqualifiziert Fakten über Fluorid rauslöscht, wird das wahrscheinlich drei Minuten später jemandem auffallen, der dann auf „Rückgängig“ drückt. Für Vandalismusüberwachung werden die Admins nicht gebraucht, das macht die Community. Im übrigen sind die Fakten damit nicht „weg“, sondern in der Versionsgeschichte über die Vorversion weiterhin abrufbar. Nur ein Admin könnte etwas, was einmal in der WP stand, vollständig daraus entfernen. --Kreuzschnabel 07:54, 13. Apr. 2016 (CEST)
+1 Ließ mal WP:Konflikte und WP:Administratoren und verabschiedet ich dann von der Vorstellung, dass ein einzelner Admin hier so etwas unter den Tisch kehren könnte, ohne das irgendwer davon mitbekommt oder interveniert. Wenn vermeintliche Wahrheiten nicht in der WP stehen, dann liegt das i.d.R. nicht daran, dass sich hier irgendwelche sinistren Nutzer dagegen verschworen hätten, sondern daran, dass diese vermeintlichen Wahrheiten selbst ein Verschörungstheorie sind,die bei näherer wissenschaftlicher Betrachtung so nicht haltbar ist.
Falls es hier um einen konkreten Fall gehen sollte,würde ich Dich bitten die Karten (z.B. die OTRS-Ticket-Nr) auf den Tisch zu legen und die entsprechenden Konfliktlösung Stellen einzuschalten. //Martin K. (Diskussion) 08:02, 13. Apr. 2016 (CEST)
Liliana ist dank einer korrupten Bande Admins tot, es lebe IP80.123 als Rächer der Unterdrückten. Ich werde auf die Beiträge dieses Users nicht mehr reagieren. Verarschen kann ich mich selbst.--2003:75:AF0F:9100:2908:2045:2A8E:A9D8 09:23, 13. Apr. 2016 (CEST)
Ich war doch partout nicht auf den Namen gekommen, danke! Dumbox (Diskussion) 09:30, 13. Apr. 2016 (CEST)

wenn irnkein Benutzer unqualifiziert Fakten über Fluorid rauslöscht, wird das wahrscheinlich drei Minuten später jemandem auffallen, der dann auf „Rückgängig“ drückt. Lieber Kreuz, zeig mir doch mal einen Wikipedianer der den Mut hat, gegen einen Administrator irgendwas zu machen, Meldungen auf VM gegen Administratoren sind nach 3 Minuten geschlossen und der User, der Meldet, gelöscht. Und der Artikel bleibt dann in der verfälschten Version, weil sich außer dem einen user meistens keine Sau dafür interessiert, was ein Administrator löscht. --Ip80.123 (Diskussion) 05:35, 15. Apr. 2016 (CEST)

Weiß man, Diskussion, wie man it dieser Liliana in Kontakt treten kann hat die eigentlich eine eigene Homepage oder sowas ähnliches ? und wie alt war die, so eine Benutzerin liliana60 behauptete die wäre 15 aber diese antworten auf der Seite https://de.wikipedia.org/wiki/Wikipedia:Auskunft/Archiv/2015/Woche_48 sprechen eher dagegen --Ip80.123 (Diskussion) 05:41, 15. Apr. 2016 (CEST)
Ich persönlich würde gar nichts davon merken, wenn ich gegen einen Administrator irgendetwas machte. Ich kenne die Liste der Admins ja nicht auswendig. Und wenn mir eine zweifelhafte Bearbeitung auffällt, dann tue ich was, ohne vorher nachzuschauen, ob der Bearbeiter zufällig auch noch Administrator ist. --Kreuzschnabel 08:58, 15. Apr. 2016 (CEST)
@ Kreuz Du mußt die Admins auch nicht auswendig kennen, es reicht, wenn du unter Eintellungen/Helferlein ein Häkchen bei "markAdmins" machst - dann werden Admins neben ihren Signaturen mit (A) gekenntzeichnet. Nur so nebenbei. --Maresa63 Talk 21:58, 15. Apr. 2016 (CEST)
@ Kreuzschnabel: Nachschauen ist entbehrlich. Falls es interessiert, ein Tipp: Geh auf Deinen "Einstellungen" auf den Reiter "Helferlein" und setze unter "Veränderung der Oberfläche" beim 8. Kästchen ein Häkchen: "markAdmins markiert [ ... ] Benutzer mit erweiterten Rechten." Bei jedem Benutzernamen eines Admins wird dann ein fettes A angezeigt. -- Zerolevel (Diskussion) 21:57, 15. Apr. 2016 (CEST)
Warum sollte ich? Ich will doch gar nicht wissen, wer Admin ist und wer nicht :-) --Kreuzschnabel 22:13, 15. Apr. 2016 (CEST)
Dein gutes Recht - darum schrob ich: Falls es ihnteressiert. (Dein Posting konnte auch so gelesen werden, als seiest Du prinzipiell interessiert und wolltest nur nicht jedesmal extra nachschauen.) Gruß -- Zerolevel (Diskussion) 22:31, 15. Apr. 2016 (CEST)

Danke dir Zerolevel, die funktion ist echt toll. Jetzt kann ich mir bei WP:VM die comedy und den Zirkus anschauen, wo ein Admin User melden muss, anstatt gleich zu löschen. Ist der selbe Witz wie Inkasos die Ihre Forderung an einen Anwalt übergeben, weil die alleine sich nicht trauen, einen Rechtsweg zu gehen. Abschließend war jetzt eiegntlich immer noch nicht geklärt, wo man das jetzt anzeigt, einfach in ausgedruckter Form zu Interpol schicken, alle Beweise die man hat? Denn der Benutzer macht ja munter weiter. Die letzte Meldung über ihn wurde kommentarlos entfernt. --Ip80.123 (Diskussion) 15:15, 17. Apr. 2016 (CEST)

Archivierung dieses Abschnittes wurde gewünscht von: --Dansker 22:41, 17. Apr. 2016 (CEST)

Hamburg Blankenese

Warum genau hat Hamburg zu wenig Wohnraum, weshalb soviele Menschen hin und her pendeln müssen oder Wohnungen nur gegen viel zu hohe Mietbeiträge erhalten? (quelle extra 3) Wieso baut man nicht zum Beispiel in Blankenee irgendwelche Sozialbauten oder neuen Wohnraum wenn es in Hamburg soviel Mangel an Wohnraum gibt? Platz für ein Flüchtlingsheim hat man in Blankenese doch auch gefunden, also werden sich doch auch Mietwohnungen einrichten lassen... --Ip80.123 (Diskussion) 07:20, 15. Apr. 2016 (CEST)

Du weisst schon, was extra 3 ist...? Gruß --Mikered (Diskussion) 08:20, 15. Apr. 2016 (CEST)
Gute Idee - wenn man zu wenig Wohnungen hat, warum baut man dann nicht einfach welche? --Optimum (Diskussion) 09:32, 15. Apr. 2016 (CEST)
Fang bei dir selbst an, Ip80: Warum baust du keine? Nächste Frage: Angenommen, du hättest genug Geld, um es zu investieren, warum würdest du keine bauen? Nächste Frage: Angenommen, du hättest zusätzlich auch noch Ahnung von der Materie? Gut, das wird jetzt dann doch zu theoretisch, das sehe ich ein, aber letztlich führt es da hin, dass keine gebaut werden, weil es sich schlicht nicht lohnt. Luxusappartements lohnen sich (und deswegen gibt es die auch!), aber Wohnungen für Familien mit Kindern oder für Studenten oder andere Habenichtse lohnen sich unter'm Strich nicht, machen aber viel Arbeit und Ärger, und solange man sein Geld anderweitig bequemer, sicherer und rentabler anlegen kann, wird sich an der Wohnungsnot in den Städten (und an den Mietpreisen) nicht grundsätzlich etwas ändern. --Snevern 09:48, 15. Apr. 2016 (CEST)
Respektive solange es die Kommunen nicht wieder selbst in die Hand nehmen. Aber kommunaler Wohnungsbau ist ja mittlerweile verpönt in Deutschland. --j.budissin+/- 10:26, 15. Apr. 2016 (CEST)
Das nennt sich Marktwirtschaft. Es gibt diese Wohnungen, die man sich leisten kann. Willst du mehr Wohnraum, musst du mehr arbeiten. --62.202.182.81 14:40, 15. Apr. 2016 (CEST)

Immobilien dienen zunehmend als Wertanlage, mit Tendenz zur Blasenbildung. Man braucht nur zu lesen, seit Jahren hämmert z.B. die Welt:

Rosenkohl (Diskussion) 15:27, 15. Apr. 2016 (CEST)

Warum muss ich die Wohnung bauen? es gib doch Baugenossenschaften in Hamburg, warum bauen diese einfach nicht Wohnungen, wenn die in Hamburg so gebraucht werden? Gibts in Hamburg überhaupt Wohnungsmangel? --Ip80.123 (Diskussion) 16:00, 15. Apr. 2016 (CEST)

Um ein Haus (oder Wohnungen) zu bauen, braucht man ein Grundstück, einen Architekten, eine Baugenehmigung, einen Bauunternehmer und Geld. Wer ausreichend Geld hat, bekommt die übrigen Punkte auch irgendwie gelöst. Wer zu wenig Geld hat, braucht Kredit. Und den gibt es nur, wenn die Banken glauben (lat., von credere), dass sie das Geld auch zurückbekommen. Jeder, der einen Investitionsplan vorlegen kann, der die Banken überzeugt, wird auch das nötige Geld bekommen.
Problem: Der Investionsplan überzeugt nicht, wenn die Grundstücke zu teuer sind (→ Blankenese), die Mieteinnahmen zu gering (→sozialschwache Mieter) oder die Bauunternehmen zu teuer (→Neubauboom). Auch die Baugenossenschaften sind normalerweise Privatunternehmen und können vom Staat nicht gezwungen werden, Mietwohnungen zu bauen. Solange es sich nicht rechnet (oder andere Projekte attraktiver sind), wird es in der freien Marktwirtschaft keinen sozialen Wohnungsbau in Villenvierteln geben. --Vertigo Man-iac (Diskussion) 16:25, 15. Apr. 2016 (CEST)

Ich weiss nicht, wer von Euch schon mal in Blankenese war. Der Ort liegt tlws. am Hang an der Elbe, ist teilweise schwer zugänglich und bebaut. Auch ist der Ort zum Teil bewaldet. Ich kann mir schwer vorstellen, wo man in Blanenese noch Platz für Sozialbauten hätte. Das ändert aber nichts daran, dass es in Deutschland eine deutlich grössere Vermischung der Stadtteil geben müsste, um eine Angleichung der Sozialstrukturen in Schulklassen zu ermöglichen. Das ist sicher ein Schlüssel zur besseren Integration. 90.184.23.200 10:21, 16. Apr. 2016 (CEST)

Der Bundesetat für Sozialwohnungsbau wurde jetzt endlich wieder mal erhöht, aber noch nicht auf das frühere oder gar erforderliche Niveu. Wenigstens ein Schritt in die richtige Richtung. Rainer Z ... 16:29, 16. Apr. 2016 (CEST)
Archivierung dieses Abschnittes wurde gewünscht von: --Dansker 22:39, 17. Apr. 2016 (CEST)

10 mal sim karte raus, 10 mal sim karte rein

trotzdem behauptet das amazon fire fone, "keine Sim karte". Davor hat er sie gelesen. Jetzt nicht mehr. Ich habe die Sim karte das erste mal raus getan, weil es zu erst hieß "ungültige Sim" und auch nach handy Neustart stand da "ungültige sim" also dachte ich, karte raus wäre die Lösung, jetzt behauptet er, keine Karten zu haben. Hilfe...--Ip80.123 (Diskussion) 15:57, 15. Apr. 2016 (CEST)

Dann ist der Chip der Sim-Karte hinüber. Hatte ich selbst schon mehrfach. --Be11 (Diskussion) 16:00, 15. Apr. 2016 (CEST)
kann man dann einfach eine neue Sim-Karte mit der alten Nummer kaufen? Gruß 79.224.209.79 16:18, 15. Apr. 2016 (CEST)
Ja, kann man. Einfach an den Anbieter wenden, manche machen es sogar kostenlos.--Be11 (Diskussion) 16:37, 15. Apr. 2016 (CEST)
Mir ist mal ähnliches passiert, aber nachdem ich die Karte mehrfach um Milimeterbruchteile verschoben habe, ging es wieder. --° (Gradzeichen) 16:43, 15. Apr. 2016 (CEST)
Das könnte auch sein, insbesondere wenn sie per Hand zugeschnitten wurde. --Be11 (Diskussion) 16:45, 15. Apr. 2016 (CEST)
Hast Du schon die Gegenprobe gemacht und die SIM-Karte in ein anderes mobilgerät und eine andere SIM-Karte in Dein mobilgerät ewingebaut? Wenn nein, hole das unbedingt sofort nach und berichte über das Ergebnis, denn ansonsten kann Dir hier nur mit unverbindlichen Mutmaßungen geholfen werden. --Rôtkæppchen₆₈ 16:58, 15. Apr. 2016 (CEST)

Hab beim Nachbarn nebenan gefragt ob ich die karte mal reinschieben darf. Sein Handy hat die Sim ohne Probleme erkannt. (kein "sim ungültig".) Hab die karte wieder in mein handy. "keine Sim". Habe meine Wohnung auf den Kopf gestellt. Irgend eine Sim karte gefunden - reingetan - "Sim gesperrt". Also andere sim erkennt er, hab nach dieser Karte, wieder diese Sim was nicht gehen will rein, "keine sim", danach wieder diese Sim karte zu der ich die Pin nicht mehr kenne (ist auch 2 Jahre alt) und die hat er ein zweites mal erkannt. --Ip80.123 (Diskussion) 18:37, 15. Apr. 2016 (CEST)

Das könnte auch ein Problem einer abweichenden SIM-Betriebsspannung sein. Im Laufe der Jahre gab es verschiedene Betriebsspannungen für SIMs. Wende Dich an den Kundendienst Deines Mobilfunkanbieters. Wenn Du Glück hast, bekommst Du Deine SIM-Karte sofort kostenlos getauscht. --Rôtkæppchen₆₈ 18:47, 15. Apr. 2016 (CEST)
Wieso gibt es unterschiedliche Spannungen und warum konnte er erst die karte lesen, die ersten 10 tage--Ip80.123 (Diskussion) 18:50, 15. Apr. 2016 (CEST)
Du weisst auch, dass Du die PIN zurücksetzen kannst, wenn Du die PUK kennst? --° (Gradzeichen) 18:52, 15. Apr. 2016 (CEST)
Sorry dass ich dich enttäuschen muss Rotkaeppchen, aber an der Spannung wird es sicherlich nicht liegen. Er schreibt ja das die SIM-Karte anfangs problemlos vom Gerät erkannt wurde. Heutzutage ist das alles genormt, außer es ist eine uralte SIM-Karte. Aber selbst da sind mir keine Probleme bekannt. --Be11 (Diskussion) 19:04, 15. Apr. 2016 (CEST)
„Wieso gibt es unterschiedliche Spannungen?“ In den 1970er-Jahren wurden integrierte Logikschaltungen mit 5 oder gar 12 Volt betrieben. Bis Mitte der 1990er-Jahre überwiegte dann 5 Volt. Später wurden integrierte Logikschaltungen so schnell, dass die Abwärme durch eine Verminderung der Betriebsspannung zuerst auf 3,3 Volt und später sogar noch weiter gesenkt wurde. Je nachdem, aus welcher Zeit die SIM-Karte stammt, kann sie daher 5, 3,3 oder 1,8 Volt, mache SIM-Karten können auch 3,3 bis 5 oder 1,8 bis 3,3 Volt. --Rôtkæppchen₆₈ 13:39, 16. Apr. 2016 (CEST)
Daran habe ich auch nicht gezweifelt, dass es unterschiedliche Spannungen gibt. Das aber jemand noch eine SIM-Karte von Anfang der Neunziger hat, dass ist ziemlich selten (und selbst damit hatte ich keine Probleme) und wenn, dann hätte das Firephone sie am Anfang ja nicht erkannt. --Be11 (Diskussion) 09:34, 17. Apr. 2016 (CEST)
Aus eigener Erfahrung, das hat wenig mit der Karte zu tun, sondern dann ist wohl leider der Sim-Kartenleser defekt und eine Reparatur übersteigt oft den Wert des Handys. War bei meinem Sony Xperia 10 mini jedenfalls leider der Fall und nach Auskunft der Techniker ein häufiges Problem. --Label5 (L5) 18:43, 17. Apr. 2016 (CEST)
Es deutet aber nach der Schilderung oben nichts auf einen Defekt des Kartenlesers sondern alles auf einen Defekt der Karte. Alle anderen Karten haben darin doch funktioniert wie sie sollten. --Paestaweary (Diskussion) 19:18, 17. Apr. 2016 (CEST)
Woher nimmst du diese Annahme. Oben wurde nicht dargestellt dass andere Karten funktionieren. --Label5 (L5) 20:26, 17. Apr. 2016 (CEST)
Archivierung dieses Abschnittes wurde gewünscht von: --Dansker 22:38, 17. Apr. 2016 (CEST)

Kartenbuch

hallo ich habe hier ein Kartenbuch mit Straßenkarten aus Papier mit Seiten. Wie kriegt man da raus wo man gerade ist? momentan gehts noch aber wenn man los fährt bleibt das Buch doch gleich. Außerdem sind oben die Straßen am Rand zu ende. wie kriegt man raus wo es im buch weiter geht und ob man da schon ist bzw wann. schaut man da was in der nähe ist oder gibt es einen trick? Und wie kriegt man raus welcher Weg der kürzeste ist? (nicht signierter Beitrag von 88.73.181.236 (Diskussion) 10:26, 17. Apr. 2016 (CEST))

Vor langer Zeit lernte man in der Schule, dass so ein Kartenbuch "Atlas" heißt. Narürlich sagt es Dir nicht, wo du gerade bist. Das erfährst Du z.B. durch Dein Blicksignal zu einem Ortseingangs- bzw. einem Straßenschild, ergänzend mit Hausnummernschildern. Damit kannst Du Dich dann auf der Straßenkarte verorten (so genau das nach jeweiligem Maßstab möglich; Das wort "Maßstab" wurde früher auch in Schulen erklärt.) Das Verorten dafst Du natürlich nur als Beifahrer machen, sonst muss gehalten werden. Navis funktionieren im Prinzip genau so, statt Blicksignal eben GPS-Signal. Die Benutzung eines Kartenwerks ist allerdings für den Benutzer intellektuell komplexer als die Navi-Benutzung. --84.135.133.155 11:07, 17. Apr. 2016 (CEST)
Aber fahr dort, wo die Straßen am Rand zu Ende sind, vorsichtig, sonst geht's dir wie den mittelalterlichen Schiffen oder auf manchen Autobahnen in Sizilien. Hans Urian | 11:19, 17. Apr. 2016 (CEST)
@„wie kriegt man raus wo es im buch weiter geht“: Die Anschlussseitenzahl ist meist mit aufgedruckt, ansonsten hilft die Übersichtskarte auf den ersten oder letzten Seiten des Atlas. Bei manchen Plänen (Falkplan) blättert man einfach nur um, auch in Nod-Süd-Richtung. --Rôtkæppchen₆₈ 12:02, 17. Apr. 2016 (CEST)
(BK)Wo es weiter geht steht häufig am Seitenrand: kleine Pfeile mit der Seitennummer, auf die Du wischen blättern musst. Um die kürzeste Strecke zu ermitteln sind entlang der Straßen Entfernungsangaben eingetragen, diese für die möglichen Stecken addieren und vergleichen. Wenn man dann noch die Durchschnittsgeschwindigkeit kennt, mit der man fährt (abhängig davon ob Du mit Auto, Fahrrad etc unterwegs bist; aber auch ob Autobahn, Landstraße, Ortsdurchfahrten) kann man ausrechnen, wann man wo ist. --TheRunnerUp 12:03, 17. Apr. 2016 (CEST)
Entfernungsangaben sind die zahlen in blau oder die roten? Sind das die zwischen zwei Orten? Die muss man dann zusammen zählen und dann sehen was kürzer ist. Gibt es nicht irgend wo eine Liste wo man Anfang und Ende raus suchen kann und die fertige Zahl sieht? Und zum Beispiel oben bei 38 geht es nicht weiter. Die Seite ist dort zu ende. Es gibt keine Seite wo es da weiter geht, rechts kommt die mit Cotbus und so. Wenn man blättert kommt irgendwann Bayern. (nicht signierter Beitrag von 88.73.181.236 (Diskussion) 19:39, 17. Apr. 2016 (CEST))
Je nach Atlas gibt es unterschiedlich feine Kilometrierungen. Die blauen sind die Entfernungen zwischen den blauen Markierungen, die roten die zwischen den roten. Es gibt eine Übersicht der Entfernungen zwischen Städten als Tabelle (an den Schnittpunkten stehen die Zahlen, manchmal nach Bahn- und Straßenkilometern getrennt). Für alle Orte gibt’s das nicht, dafür ist das Druckformat zu klein. Wenn du einfach weiterblätterst, kommst du irnkwann auf die nächste „Zeile“ der Kartenabdeckung, also die nächste südliche Leiste (britische Atlanten fangen ybrinx mit der südlichsten Zeile an und arbeiten sich nach Norden vor, das funktioniert auch). Und jetzt stell dir vor, nach so was haben wir bis Ende der 90er im Auto navigiert – und uns tatsächlich zurechtgefunden! --Kreuzschnabel 19:56, 17. Apr. 2016 (CEST)
So umständlich ist es gar nicht. Kartenlesen lernt man als Kind besser, ähnlich wie Sprachen, Tennis, Schwimmen und so. Und man erwirbt dabei das nötige Minimum an fotografischem Gedächtnis für Landkarten, mit dem man dann als Erwachsener auch ohne Beifahrer zurechtkommt. Man schaut sich den geplanten Weg an, merkt sich die Stellen, wo man abbiegen muss und fährt dann los, indem man während der Fahrt die nunmehr mentale Karte konsultiert. NfdA (Diskussion) 14:10, 17. Apr. 2016 (CEST)
Und zur Gedächtnisstütze kann man sich noch ein kleines Post-It auf das Lenkrad kleben, auf dem die Straßennummern und die zu passierenden Orte aufgelistet sind. Geoz (Diskussion) 17:08, 17. Apr. 2016 (CEST)

Das bringt überhaupt nichts. Zum Beispiel wenn man runter fährt ist es die andere Richtung und wenn man rechts fährt muss man eigentlich nach links. Es ist nur gültig wenn man hin will. (nicht signierter Beitrag von 88.73.181.236 (Diskussion) 19:39, 17. Apr. 2016 (CEST))

Stell dein Navi auf „Norden oben“, dann kannst du das Umdenken üben. Es geht. --Kreuzschnabel 19:58, 17. Apr. 2016 (CEST)
Ich frage mich gerade, ob die Anfrage ein Scherz sein soll, oder ob es inzwischen wirklich Leute gibt, die mit einer papiernen Landkarte oder einem Atlas gar nicht mehr umgehen können. Rainer Z ... 16:59, 17. Apr. 2016 (CEST)
Du hast übrigens das Recht, die Karte so zu drehen, wie es dir gefällt... --Eike (Diskussion) 20:03, 17. Apr. 2016 (CEST)
Ich frage mich [...] ob es inzwischen wirklich Leute gibt, die mit einer papiernen Landkarte oder einem Atlas gar nicht mehr umgehen können. Also ich würde mich eher fragen, ob es immer noch Leute gibt, die mit einer Landkarte nicht umgehen können. Das ist mir nämlich vor allem bei älteren Leuten aufgefallen, die wenig mobil sind und auch mit Karte und Kompass völlig aufgeschmissen sind. Da geht es zwar weniger um Fahrten über Landstraßen (die ja meist gut beschildert sind) als vielmehr die Orientierung in der Stadt.--Antemister (Diskussion) 23:10, 17. Apr. 2016 (CEST)
Archivierung dieses Abschnittes wurde gewünscht von: --Dansker 22:36, 17. Apr. 2016 (CEST)

Am Strahlflugzeug während des Fluges aus der Kanzel aussteigen

 
Dassault Rafale
  1. Ist das Foto echt? Wenn ja, wieso wird der Mann nicht vom Fahrtwind weggedrückt?
  2. Warum steigt die Person aus der Flugkanzel?
    Schönes Rest-WE --Mattes (Diskussion) 11:09, 17. Apr. 2016 (CEST)
Das Foto dürfte echt sein. Er wird nicht vom Fahrtwind weggedrückt, weil das abgebildete Flugzeug dafür fliegen müsste. Die Person steigt aus, weil so eine Flugzeugkanzel auf Dauer zu eng ist und man nach der Landung irgendwann nach Hause möchte. --Sitacuisses (Diskussion) 11:15, 17. Apr. 2016 (CEST) <<< …Clown gefrühstückt? Bist wohl Du auf dem Foto oder hast Du mit dem Menschen gesprochen? Dieser Jet auf der Abb. schwebt nicht, sondern er fliegt (oder es ist ein Einzelstück mit abgeänderter Technik, was äußerst unwahrscheinlich ist). Im Weltall befindet er auch nicht und ein Parabelflug wäre zu kurz, um die Zeit der Schwerelosigkeit für so eine Aktion nutzen zu können. --Mattes (Diskussion) 12:24, 17. Apr. 2016 (CEST)
@Mattes: Falls das ernstgemeint sein sollte: Du siehst schon die Leiter über die der Pilot das Flugzeug besteigt und die Hügel und dieses Versorgungsfahrzeug im Hintergrund? Oder schwebt fliegt das auch alles? // Martin K. (Diskussion) 12:34, 17. Apr. 2016 (CEST)
Wieso kommst du auf die Idee, es sei ein Mann? --62.202.182.81 11:35, 17. Apr. 2016 (CEST)
Weil es so aussieht (Körperbau). --Mattes (Diskussion) 12:24, 17. Apr. 2016 (CEST)
Faule Ausrede. Da wo es darauf ankommt, sieht man wegen der Unterbelichtung sowieso keine Details. --62.202.182.81 13:53, 17. Apr. 2016 (CEST)
Ich frage mich, wie man das Foto derart fehlinterpretieren kann. Die Maschine steht eindeutig am Boden und der Pilot steigt gerade aus. (laut Bildbeschreibung, sonst könnte man auch denken, daß er einsteigt.) Am Flugzeug ist eine Leiter angebracht. Die wäre nicht da, wenn es sich in der Luft befände.Hinter dem Flugzeug, ziemlich dunkel unten rechts, ist ein LKW, wahrscheinlich ein Tankwagen, und dahinter ein Gebäude, wohl ein Hangar. -- Gerd (Diskussion) 12:36, 17. Apr. 2016 (CEST)
Erledigt, hatte nicht genau geschaut... --Mattes (Diskussion) 12:53, 17. Apr. 2016 (CEST)
For allem lautet eine der wichtigsten Metaphern des Fliegens „über den Wolken“. Das Foto wurde aber eindeutig unter den Wolken aufgenommen. <scnr> --Kreuzschnabel 20:00, 17. Apr. 2016 (CEST)
Archivierung dieses Abschnittes wurde gewünscht von: Lidius (Diskussion) 17:08, 17. Apr. 2016 (CEST)

Stift zum Nachladen bei der MP5

Der Stift zum Nachladen bei der MP5 verursacht wirklich schmerzen, vor allem wenn man so ein Gerät hat wo man die Einzelschussfunktion üben muss. Gibts da eine Möglichkeit, sich nicht dauernd den Finger einzuklemmen? so locker | wie der hier es zeigt geht es dann doch nicht. Wieso ist dieser Stift zum nachladen überhaupt nur so klein? höchstens 3,5 cm steckt das Teil raus. --Ip80.123 (Diskussion) 15:19, 17. Apr. 2016 (CEST)

Wenn es dir nicht gefällt, kannst du für deine Nachbarschaftsstreitigkeiten auf Handgranaten umsatteln. --62.202.182.81 15:58, 17. Apr. 2016 (CEST)
Sag mal, hat dir dein Ausbilder das denn nicht erklärt!? Was ist denn das bloß für eine Lusche!!
Ich hatte jedenfalls mit der MP5 noch nie ein Problem! --Snevern 16:35, 17. Apr. 2016 (CEST)
Was sind jetzt euere Vorschläge was man tun kann um sich da nicht den Finger einzuklemmen? Nie Probleme? nicht mal schmerzende Finger? --Ip80.123 (Diskussion) 16:43, 17. Apr. 2016 (CEST)
Ich bin deswegen auf die MP7 umgestiegen.--2003:75:AF7E:9500:89A4:8C48:EFC7:BBDD 17:11, 17. Apr. 2016 (CEST)
Was soll an meinem HG-Vorschlag nicht in Ordnung sein? Dein Nachbar ist schmerzloser und nachhaltiger hinüber, als wenn er an Schussverletzungen verbluten muss. Kollateralschäden wegen Querschlägern gibt es auch keine. Und wenn du den Bügel einsammelst, bleibt nichts zurück was Fingerabdruck- oder DNA-Spuren von dir aufweisen könnte. Zudem ist so eine 18er-Kiste HG beim Pizzabäcker deines Vertrauens deutlich billiger zu kriegen als eine MP5. --62.202.182.81 17:22, 17. Apr. 2016 (CEST)
Nach Allem, was da so die Benutzerin xxxxxxxx auf Commons losgelassen hat, will sich die mit einem großen Knall verabschieden, da bleibt zwar auch noch einiges zurück. Wie komm ich nur auf xxxxxxxx?--2003:75:AF7E:9500:89A4:8C48:EFC7:BBDD 18:37, 17. Apr. 2016 (CEST)
Könnte man nicht bitte dieses thread versionslöschen, da es um ein Gerät zur Tötung von Nachbarn geht sowie um einen Nutzer, der hier schon etliche Male eine Selbsttötung angekündigt hat? Danke, NfdA (Diskussion) 18:43, 17. Apr. 2016 (CEST)
Archivierung dieses Abschnittes wurde gewünscht von: --Dansker 22:33, 17. Apr. 2016 (CEST)

Kosten für Biosiegel

Warum hat z.B. Red Bull Cola kein Biosiegel? Kümmert sich Red Bull nicht darum, Kokablätter, Süßholz und Kaffeebohnen aus biologischen nachhaltigen Anbau zu verwenden - oder kostet es irgendwie ein Unternehmen etwas, ein Biosiegel zu verwenden und zu erhalten und Red Bull will es nicht auf die Cola drucken? Was kostet so ein Siegel?--Ip80.123 (Diskussion) 16:41, 17. Apr. 2016 (CEST)

ist die frage jetzt ernst gemeint? natürlich kosten biologisch zertifizierte zutaten mehr als ohne. -- southpark 16:50, 17. Apr. 2016 (CEST)
Kokablätter sind da schonmal gewiss nicht drin, auch wenn man sowohl Cola, als auch Energiedrinks bis heute einen Kokaingehalt nachsagt, vielmehr ist es die Colanuss, die für die Cola verwendet wird, wobei sich deren Geschmack sicherlich auch inzwischen synthetisch nachbauen lässt. Auch Kaffee ist meines Wissens nicht in Redbull und den meisten anderen Energiedrinks, es gibt auch andere Koffeinquellen und zur Not setzt man künstlich hergestelltes zu, da unterliegen dann die Endprodukte nicht mehr den Gehaltsschwankungen, die bei Naturbelassenen Produkten die Regel sind, allenfalls nimmt man als natürliches Produkt Guaraná. Und ja, ein Biosiegel zu beantragen kostet geld und insbesondere solch riesenunternehmen wie Cocacola und Redbull versuchen bis auf die x-te nachkommastelle einzusparen, da sind Biosiegel und Bioprodukte schlicht lästig und "teuer" --178.12.205.91 18:18, 17. Apr. 2016 (CEST)
@„künstlich hergestelltes“: Coffein ist ein Nebenprodukt der Entcoffeinierung von Kaffee. Es wird soviel entcoffeinierter Kaffee produziert, dass das daraus entfernte Coffein für die Cola- und Energydrinkproduktion ausreicht. --Rôtkæppchen₆₈ 18:34, 17. Apr. 2016 (CEST)
Halten wir fest: Es gibt Getränke mit Fliegeln und Getränke mit Sügeln. NfdA (Diskussion) 18:39, 17. Apr. 2016 (CEST)
Mir ist immer noch rätselhaft, wie Softdrinks und Klaviervermietung zusammenpassen. --Rôtkæppchen₆₈ 19:13, 17. Apr. 2016 (CEST)
Archivierung dieses Abschnittes wurde gewünscht von: --Dansker 22:34, 17. Apr. 2016 (CEST)

Die Werbung hat ihr Ziel erreicht, wenn der Verbraucher glaubt(!), dass sich hinter einer (künstlich erzeugten) Marke ein Qualitätsversprechen befindet.--Wikiseidank (Diskussion) 13:38, 18. Apr. 2016 (CEST)

Böhmermann-Gedicht - § 103 StGB

§ 103 Abs. 1 lautet: Wer ein ausländisches Staatsoberhaupt oder wer mit Beziehung auf ihre Stellung ein Mitglied einer ausländischen Regierung, das sich in amtlicher Eigenschaft im Inland aufhält, oder einen im Bundesgebiet beglaubigten Leiter einer ausländischen diplomatischen Vertretung beleidigt, wird mit Freiheitsstrafe bis zu drei Jahren oder mit Geldstrafe, im Falle der verleumderischen Beleidigung mit Freiheitsstrafe von drei Monaten bis zu fünf Jahren bestraft.

Wie kommt man nun dahin, dass dies für Böhmermann Probleme bereiten könnte? Erdogan war meines Wissens nicht in Deutschland zu der Zeit. Oder bezieht sich das im Inland nur auf ein Mitglied einer ausländlischen Regierung, aber nicht auf das ausländische Staatsoberhaupt? --87.140.195.2 18:32, 12. Apr. 2016 (CEST)

Letzteres. --Komischn (Diskussion) 18:39, 12. Apr. 2016 (CEST)
Der Paragraf stammt aus uralten Zeiten und sollte grenzübergreifende "Majestätsbeleidigungen" bestrafen. Eigentlich ist er nicht mehr aktuell, aber er ist halt noch da. Der Böhmermann wusste das, tat es trotzdem, also vorsätzlich. --2003:75:AF12:700:7DE0:4464:5002:4FEA 19:42, 12. Apr. 2016 (CEST)
Vermutlich ist dem türkischen Staatsoberhaupt auch nicht bewußt, dass diese von Böhmermann präferiere literarische Form auf eine lange, länderübergreifende Tradition zurückblickt. --95.119.100.176 19:54, 12. Apr. 2016 (CEST)

Was ich mich frage, ist warum nicht einfach das Strafverfahren eingeleitet wird. Soll doch das Gericht entscheiden. Böhmermann war sich doch der Konzequenzen bewusst. Bei Facebook-Hasskommentaren gegen Flüchtliche, war die öffentliche Meinung genau aus der anderen Richtung. Wo bitte ist Merkel in der Klemme? Strafverfahren einleiten und fertig. Am besten gleichzeitig dieses Gesetz abschaffen, so dass das Verfahren wieder eigestellt wird.--84.179.141.124 20:04, 12. Apr. 2016 (CEST)

Man muss sich allerdings auch mal fragen, warum Böhmermann das gemacht hat. Und dann kommt man schnell darauf, dass der einzige Grund nicht etwa die Verteidigung der Satirefreiheit o.ä. ist, sondern die Erhöhung von Böhmermanns eigenem Bekanntheitsgrad. Wenn er angeklagt wird. prima, dann kann er seinen Bekanntheitsgrad noch weiter in die Höhe treiben und steht gleichzeitig als Märtyrer da, für die Westliche Zivilisation, die Freiheit der Presse und was weiß ich noch alles. Selbst wenn er verurteilt wird - Knast ist ja als Ersttäter eher unwahrscheinlich - zahlt er eben ein paar tausend Euro. Das hätte er freiwillig gezahlt, wenn dann auch so viele Medien über ihn berichtet hätten. Und alle lassen sich vor seinen Karren spannen. Echt traurig. --Optimum (Diskussion) 20:12, 12. Apr. 2016 (CEST)
Der Justizminister ist für die Justiz zuständig, also die Richter (auch wenn er ihnen nicht fachlich weisungsberechigt ist), der Innenminister für die Staatsanwaltschaften (und die Polizei, die ja nicht unwesentlich an der Strafverfolgung beteiligt ist). Auch die Strafvollstreckung fällt in die Zuständhttps://de.wikipedia.org/wiki/Beleidigung_%28Deutschland%29igkeit der Staatsanwaltschaften und damit des Innenministeriums.
<quetsch>Die Staatsanwaltschaften unterstehen nicht dem Innenminister sondern dem Justizminister --Digamma (Diskussion) 18:37, 14. Apr. 2016 (CEST)</quetsch>
Die Ermächtigung wird in der Tat nur für die qualifizierte Beleidigung nach § 103 StGB gebraucht, nicht für die normale nach § 185 StGB. Eine Anzeige reicht nicht, und die braucht auch nicht von Erdogan zu kommen - es braucht dafür aber einen Strafantrag, und den kann nur Erdogan als der Verletzte des Delikts stellen (hat er ja nun auch getan). Ich bin in praktischen Strafrechtsverfahren ohne Recherche nicht beschlagen genug, ob tatsächlich trotz Vorliegens der Voraussetzungen des § 103 nach dem normalen Beleidigungsparagraphen verfolgt werden könnte (mit dem dann geringeren Strafrahmen, versteht sich). --Snevern 22:46, 12. Apr. 2016 (CEST)
Ich persönlich bin ja dafür, die Sache vor Gericht zu bringen. Böhmermann sollte auch nicht straffrei da rauskommen. Ich halte eine symbolische Strafe von max. 10 Tagessätzen (zu zahlen an einen Verein, der sich für türkisch-kurdische Versöhnung einsetzt *grins*) für angemessen. Damit wäre klargestellt, dass Presse- und Meinungsfreiheit eine Sache, Beleidigungen eine andere Sache sind. Ansonsten sollte das hohe Gericht dem Herrn Ministerpräsidiktator klar machen, dass er in seinem Land vielleicht Urteile einklagen kann, aber nicht bei uns. Aus diesem Grund kann ich aber auch damit leben, wenn das Gericht zu dem Ergebnis kommt, dass im konkreten Fall keine Beleidung vorliegt oder das Verfahren wg. Geringfügigkeit (*grins*) einstellt. --Vertigo Man-iac (Diskussion) 20:21, 12. Apr. 2016 (CEST)

Ich bin auf Böhmermanns Seite. Alleine, weil ich Erdogan nicht mag. Aber mal als Gedankenexperient. Man stelle sich vor, Betarix von Storch hätte das Gedicht geschrieben und in der selben Form geäußert. Steigerung: Als Adressat der oberste deutsche Grüßaugust. Was dann wohl los wäre. Vermutung: Die Presse würde sich vor Ekel kaum mehr halten können und die Intelektuellen, die jetzt alle Pro Böhmermann sind (Weil Erdogan uns unsymphatisch ist) würden sich überschlagen was die Verurteilungswünsche angehen würde. Nur weil Erdogan für unsere Maßstäbe ein politischer Unsymphat ist, sollten deswegen nicht weniger "Schutz" für ihn gelten, als sagen wir, für den Gauck, Merkel oder den Papst. --95.112.187.135 20:43, 12. Apr. 2016 (CEST)

Dann mach dir doch mal Gedanken über dich selbst, liebe IP. Würdest du schreiben "Ich bin auf Beatrix von Storchens Seite", wenn die Intellektuellen und die Zeitung mit den vier Buchstaben plötzlich eben mal kurz auf Erdogans Seite wären? --MannMaus (Diskussion) 20:51, 12. Apr. 2016 (CEST)
Hä? Wenn Erdogan ein Gedicht über Beatrix von Storch machen würde? --Expressis verbis (Diskussion) 20:58, 12. Apr. 2016 (CEST)
Nein, wenn B. von Storch ein Gedicht - nein, das Gedicht - über Erdogan gemacht hätte, und ihn jetzt "alle" bedauern würden. --MannMaus§ 104a

Voraussetzungen der Strafverfolgung

Straftaten nach diesem Abschnitt werden nur verfolgt, wenn die Bundesrepublik Deutschland zu dem anderen Staat diplomatische Beziehungen unterhält, die Gegenseitigkeit verbürgt ist und auch zur Zeit der Tat verbürgt war, ein Strafverlangen der ausländischen Regierung vorliegt und die Bundesregierung die Ermächtigung zur Strafverfolgung erteilt. ] (Diskussion) 21:03, 12. Apr. 2016 (CEST)

Ich glaube, Böhmermann wird jetzt so breit unterstützt, weil er politisch korrekt und immer staatstragend die politisch "richtige" Meinung hatte. Gegen jemanden, den wir alle nicht wirklich mögen. Es geht mir darum, dass, hätte ein unsymphatischerer oder unangepasster (Böhmermann ist politisch absolut auf Linie) ein ähnliches Gedicht verfasst gegen jemanden, der nicht so abgelehnt wird, dann wäre die Debatte eine andere. ::Der Justizminister ist für die Justiz zuständig, also die Richter (auch wenn er ihnen nicht fachlich weisungsberechigt ist), der Innenminister für die Staatsanwaltschaften (und die Polizei, die ja nicht unwesentlich an der Strafverfolgung beteiligt ist). Auch die Strafvollstreckung fällt in die Zuständhttps://de.wikipedia.org/wiki/Beleidigung_%28Deutschland%29igkeit der Staatsanwaltschaften und damit des Innenministeriums.
Die Ermächtigung wird in der Tat nur für die qualifizierte Beleidigung nach § 103 StGB gebraucht, nicht für die normale nach § 185 StGB. Eine Anzeige reicht nicht, und die braucht auch nicht von Erdogan zu kommen - es braucht dafür aber einen Strafantrag, und den kann nur Erdogan als der Verletzte des Delikts stellen (hat er ja nun auch getan). Ich bin in praktischen Strafrechtsverfahren ohne Recherche nicht beschlagen genug, ob tatsächlich trotz Vorliegens der Voraussetzungen des § 103 nach dem normalen Beleidigungsparagraphen verfolgt werden könnte (mit dem dann geringeren Strafrahmen, versteht sich). --Snevern 22:46, 12. Apr. 2016 (CEST)Anderes Extrembeispiel: Nazi-Satiriker vs. Zentralrat der Juden. Also anstatt "Erdogan fickt Ziegen" "Der Jude, der verspeist kleine Kinder" oder so etwas. --95.112.187.135 21:14, 12. Apr. 2016 (CEST)
Nur zur Frage nach dem Gesetzestext: "Wer ein ausländisches Staatsoberhaupt oder wer mit Beziehung auf ihre Stellung ein Mitglied einer ausländischen Regierung, das sich in amtlicher Eigenschaft im Inland aufhält..." ausländisches Staatsoberhaupt oder ein "Ausländer" im Inland.--Wikiseidank (Diskussion) 21:11, 12. Apr. 2016 (CEST)
Obwohl ich Böhmersmanns völlig dämliches, geschmackloses und unsatirisches "Gedicht" unterste Schublade fand (das zudem ein gutes Beispiel für die gesunkene Qualität unserer öffentlich-rechtlichen Medien ist), finde ich Gefängnis jetzt etwas stark übertrieben. Eine Verbannung aus dem öffentlich-rechtlichen Fernsehen hätte (mir) gereicht. Es reicht mir schon das ich sowas noch finanzieren muss. Es ist natürlich immer sehr lustig, wenn man, ohne jemals in der Türkei gelebt zu haben, so einen Mist verzapft. Die Mehrheit der Türken findet das Gedicht geschmacklos und steht völlig hinter ihrem Präsidenten. Es geht mir nicht darum einen spezifischen Präsidenten zu schützen, hätte Böhmermann ein anderes Staatsoberhaupt verteidigt hätte ich das gleiche gesagt. --Be11 (Diskussion) 21:22, 12. Apr. 2016 (CEST)
Über Geschmack lässt sich streiten, aber nicht über die Freiheit der Kunst. IMHO hat sich Erdogan diese Reaktion auf seine selbstherrliche Medienpolitik und unterdrückte Meinungsfreiheit selbst zuzuschreiben. Ihm wurde durch diplomatische Kanäle mehrfach klargemacht, dass es in Deutschland keine Verhältnisse wie in der Türkei geben kann, wo beliebige dem Präsidenten missliebige Äußerungen politische Staftaten sind und Medien, die dem Präsidenten nicht gefallen, quasi auf Zuruf verstaatlicht oder geschlossen werden können. Insofern erwarte ich einen Freispruch oder eine milde Strafe für Böhmermann. --Rôtkæppchen₆₈ 21:47, 12. Apr. 2016 (CEST)
Weiter oben stellte IP 84.179.141.124 die Frage, warum nicht einfach ein Strafverfahren eingeleitet wird. Die Antwort findet sich in § 104a StGB: "Straftaten nach diesem Abschnitt werden nur verfolgt, wenn die Bundesrepublik Deutschland zu dem anderen Staat diplomatische Beziehungen unterhält, die Gegenseitigkeit verbürgt ist und auch zur Zeit der Tat verbürgt war, ein Strafverlangen der ausländischen Regierung vorliegt und die Bundesregierung die Ermächtigung zur Strafverfolgung erteilt." Der letzte Halbsatz ist der, der Frau Merkel nun in Bedrängnis bringt, denn zum ersten Mal kostet es sie womöglich etwas, wenn sie sich für die Pressefreiheit einsetzt. --Snevern 22:01, 12. Apr. 2016 (CEST)

@Snevern: OK, das ist wohl das entscheidende dieser Posse... Aber: Staatsanwaltschaften sind ja formell weisungsgebunden, wozu denn diese Klarstellung?--Antemister (Diskussion) 22:20, 12. Apr. 2016 (CEST)

Die Staatsanwaltschaften sind in der Tat hierarchisch aufgebaut und unterstehen letztlich dem Innen Justizminister des jeweiligen Bundeslandes, nicht aber der Bundesregierung. --Snevern 22:28, 12. Apr. 2016 (CEST)
Ich hatte das so verstanden, dass die Bundesregierung bei §103 ihr OK geben muss, während für die "normale" Beleidigung Erdogans Anzeige ausreicht. --Optimum (Diskussion) 22:33, 12. Apr. 2016 (CEST)
Wirklich Innenministerium? Wozu gibt es dann die Justizministerien? --Rôtkæppchen₆₈ 22:35, 12. Apr. 2016 (CEST) Im Abschnitt Staatsanwaltschaft (Deutschland)#Weisungsrecht steht: „Für die Dienstaufsicht und sämtliche Verwaltungsangelegenheiten im Bereich der Staatsanwaltschaften ist das jeweilige Landesjustizministerium zuständig.“ --Rôtkæppchen₆₈ 22:40, 12. Apr. 2016 (CEST)
Du hast natürlich recht - Justiz-, nicht Innenminister. Die Polizei, die die meisten Hilfsbeamten der Staatsanwaltschaft stellt, untersteht dem Innenminister.
Die Ermächtigung wird in der Tat nur für die qualifizierte Beleidigung nach § 103 StGB gebraucht, nicht für die normale nach § 185 StGB. Eine Anzeige reicht allerdings nicht, und die braucht ja auch gar nicht von Erdogan zu kommen - es braucht aber einen Strafantrag, und den kann nur Erdogan als der Verletzte des Delikts stellen (hat er ja nun auch getan). Ich bin in praktischen Strafrechtsverfahren nicht beschlagen genug, um ohne Recherche sagen zu können, ob tatsächlich trotz Vorliegens der Voraussetzungen des § 103 nach dem normalen Beleidigungsparagraphen verfolgt werden könnte (mit dem dann geringeren Strafrahmen, versteht sich). --Snevern 22:46, 12. Apr. 2016 (CEST)
Wie läuft das in der Praxis? Schickt da das Bundeskanzleramt der rheinland-pfälzischen Staatskanzlei einen Brief, die rheinland-pfälzische Staatskanzlei dem rheinland-pfälzischen Justizministerium einen Brief und das dann der Staatsanwaltschaft Mainz? --Rôtkæppchen₆₈ 22:57, 12. Apr. 2016 (CEST)
Das kann ich dir nicht sagen, wie das in der Praxis läuft. Ich habe so eine Ermächtigung noch nie gesehen und war noch nie an einem Verfahren beteiligt, in dem eine gebraucht worden wäre. --Snevern 23:31, 12. Apr. 2016 (CEST)

Mancher scheint immer noch nicht verstanden zu haben, was Böhmermann gemacht und gemeint hat. Als Reaktion auf die völlig inakpzeptable Einbestellung des deutschen Botschafters wegen einer wirklich harmlosen Extra-3-Satire und das allgemeine gegenwärtige politische Wegsehen im Hinblick auf Erdogan hat er mal mit vorhergehender Erläuterung vorgeführt, was ein wirklich beleidigender Beitrag wäre. Dieses Gedicht ist nur ein Demonstrationsobjekt. Nicht nur Erdogan scheint darauf hereingefallen zu sein, das ist auch jeder, der dessen Geschmacklosigkeit kritisiert. Rainer Z ... 22:54, 12. Apr. 2016 (CEST)

Das ist natürlich sein vorab ausgebrachter Rettungsanker. Aber mit der gleichen Logik könnte man auch sagen: "So, jetzt zeige ich Dir mal, was eine verbotene Körperverletzung ist, die man nicht machen darf." und wäre dann gegen eine Strafverfolgung abgesichert. --Optimum (Diskussion) 23:01, 12. Apr. 2016 (CEST)
(Quetsch) Nein, das ist es nicht. Böhmermann wird Erdogan sicher nicht für einen Ziegenficker halten. Er hat vorgeführt, dass das eine Schmähung wäre (anhand eines gängigen Vorurteils gegenüber Türken, übrigens). Und dabei weniger Edogan im Visier gehabt. Was er macht, ist so eine Art Meta-Satire, die nicht darauf zielt, wonach es auf den ersten Blick ausssieht. Das scheint aber wirklich nicht so ganz leicht zu verstehen zu sein. Vielleicht müssen das wirklich ein paar Richter den Deutschen erklären. Was dann wirklich lustig wäre. Rainer Z ... 00:39, 13. Apr. 2016 (CEST)
Richtig. Das scheint aber mancher nicht verstanden zu haben.
Geschmacklos finde ich's trotzdem.
Und dennoch fände ich es geradezu grotesk, wenn der jetzt plötzlich so berühmt gewordene Herr Böhmermann dafür belangt würde.
Mein Tipp an Frau Merkel: Erteile die Genehmigung und überlasse es dann der Justiz, Böhmermann freizusprechen. Das Risiko für die Kanzlerin ist dann am geringsten - selbst wenn Böhmermann verurteilt würde: Das wäre ja nicht ihr Problem, sondern der Richter hätte ihr ja sogar nachträglich bestätigt, dass es eine strafbare Handlung war und die Ermächtigung damit nachträglich gerechtfertigt. Eine Ablehnung der Ermächtigung wäre politisch unklug, und wozu muss man die Justiz des eigenen Staats denn überhaupt fürchten, wenn man doch glaubt, der beleidigende Text sei von der Satire-Freiheit gedeckt!? --Snevern 23:26, 12. Apr. 2016 (CEST)
Jetzt bleibt nur noch die Frage, wann Frau Merkel das Beraterhonorar an STT überweist (STT = Snevern Think Tank). --Snevern 18:57, 15. Apr. 2016 (CEST)
schick' ihr doch schnell noch n Brief, dass sie bis die Sache durch ist, die Volljuristen aus dem Fall Horst Arnold (Justizopfer) nach Afghanistan schicken soll... auf ne Pfeffer-Plantage... auf mich hört sie nämlich immer nich... --Heimschützenzentrum (?) 10:01, 16. Apr. 2016 (CEST)
Horst Arnold (Justizopfer)...? --Heimschützenzentrum (?) 00:16, 13. Apr. 2016 (CEST)

<räusper> Beim Blättern im StGB ist mir der § 188 StGB über den Weg gelaufen - nanu, gibt's das wirklich noch? Wäre Herr Erdogan ein deutscher Politiker (AfD? NPD? CSU?), so wäre er anscheinend auch hier strafrechtlich besonders geschützt. Da klingt für mich auch noch einiges mit von wegen "Schutz der Herrschenden vor demokratischen Demagogen". Ach übrigens, wenn man im "Ziegenficker" eine Tatsachenbehauptung sieht, könnte sogar eine verleumderische Beleidigung mit erhöhtem Strafrahmen vorliegen - Böhmi ab in den Bau! ((Ich schicke ihm auch zum Trost eine schönt Ansichtskarte mit Solidaritätsbekundung.) Ob das TV-Programm dann wirklich besser wird, kann bezweifelt werden. -- Zerolevel (Diskussion) 23:47, 12. Apr. 2016 (CEST)

„und ist die Tat geeignet, sein öffentliches Wirken erheblich zu erschweren“ ist hier wohl offensichtlich nicht erfüllt. Was juckt es die stolze türkische Eiche, wenn sich der deutsche Eber an ihr reibt? --Rôtkæppchen₆₈ 00:24, 13. Apr. 2016 (CEST)
So ist es, wenn man Zitate aus dem Zusammenhang reißt oder wie es das Böhmermannteam macht, Zitate in den Raum stellt, die jeder mit seiner "Phantasie" unterschiedlich interpretiert. RainerZ hat das oben eigentlich schon erklärt.--Wikiseidank (Diskussion) 07:15, 13. Apr. 2016 (CEST)
Und Optimum hat es als das entlarvt, was es ist: ein billiger rhetorischer Trick, den schon die alten Römer kannten und nutzten: "Ich will mich nicht beteiligen an der Hetze derjenigen, die behaupten... (hier die gewünschte Lüge/Beleidigung/Unverschämtheit o.ä. einsetzen, die man verbreiten, von der man sich aber zugleich distanzieren möchte)". Man setze also eine verbale Klammer, und dann kann man innerhalb der Klammer bar jeder Verantwortung für den Inhalt wüten so viel man will. So jedenfalls stellt sich der kleine Max den Krieg vor. --Snevern 12:49, 13. Apr. 2016 (CEST)
Das gibt es natürlich auch, Klassiker. Aber hier sehe ich eine ganz andere Intention. Rainer Z ... 17:19, 13. Apr. 2016 (CEST)
Worin genau siehst du denn die Intention? Aufklärung über die Grenzen der Straffreiheit? --Snevern 18:00, 13. Apr. 2016 (CEST)
Ich gehe davon aus, dass Böhmermann vor dem Hintergrund der zunehmend restritiven bis agressiven türkischen Politik, dem Flüchtlingsabkommen und der Extra-3-Posse einerseits Erdogan mal vorführen wollte, was eine solide Beleidigung wäre, andererseits durchaus darauf spekuliert hat, dass Erdogans im Prinzip erwartbare Reaktion die deutsche Regierung in einen peinlichen Konflikt verstrickt. Sie verhält sich ja gegenüber der haarsträubenden Entwicklung in der Türkei ausgesprochen handzahm (um die Flüchtlinge vor der Tür zu halten), kann das aber natürlich nicht durchhalten, wenn es um Meinungsfreiheit im eigenen Land geht. Böhmermann hat eine Zwickmühle aufgebaut, angesichts derer sich alle nur blamieren können. Dass das Spiel sooo garstig geworden ist, hat er vermutlich nicht erwartet. Man muss den Mann nicht mögen, aber blöd ist er nicht – dieses dumpfbackige »Schmähgedicht« ist sicher nicht Ausdruck seiner Meinung über Erdogan. Es war der Köder in einer Versuchsanordnung. Rainer Z ... 19:29, 13. Apr. 2016 (CEST)
Na ja, dann könnte ich im Zuge meiner oben angedeuteten Körperverletzung ja den türkischen Botschafter vermöbeln und sagen, dass das eine Performance (Kunst) sei, mit der ich auf die Fragilität menschlichen Lebens hinweisen und die Erpressbarkeit der Bundesregierung aufdecken wollte. Aber die Kunst ist ja nicht ganz frei, sondern im gleichen Artikel 5 GG wird die Freiheit schon wieder eingeschränkt: 2) Diese Rechte finden ihre Schranken in den Vorschriften der allgemeinen Gesetze, den gesetzlichen Bestimmungen zum Schutze der Jugend und in dem Recht der persönlichen Ehre.--Optimum (Diskussion) 20:45, 13. Apr. 2016 (CEST)
Der Vergleich hinkt gewaltig. Wenn man den türkischen Botschafter vermöbelt, ist er hinterher eindeutig verletzt. Ob er beleidigt wird, wenn man ihm sagt: »Wollte ich Sie beleidigen, würde ich Sie ›Ziegeficker‹ nennen.«, ist nicht eindeutig, sondern eine Interpretationssache. Anders als bei einer Körperverletzung (Auge blau, klarer Fall) kommt es da auch noch auf den Kontext an. Rainer Z ... 22:58, 13. Apr. 2016 (CEST)
Ich bezweifle stark, daß es zu einem Verfahren aufgrund von § 103 Abs. 1 StGB kommen wird, nachdem die Bundesregierung gleichzeitig mit der Ermächtigung zur Strafverfolgung die Absicht der Streichung der Vorschrift angekündigt hat. Nach § 2 III StGB müßte nämlich die zuständige StA beim Justizministerium anfragen, ob die Streichung tatsächlich zeitnah geplant ist, und bei der erwartbaren positiven Antwort ist das Verfahren dann aufgrund der oa Vorschrift einzustellen. War das jetzt listig oder unbedacht von Mutti?--80.171.153.232 15:06, 16. Apr. 2016 (CEST)

Könnte Böhmermann bei 2000 Beleidigungsklagen ein psychiatrisches Gutachten erwirken? --95.119.66.227 21:17, 14. Apr. 2016 (CEST)

Was ist Hochgeschwindigkeitsstahl und warum heißt diese Stahlsorte so?

--90.146.53.23 23:34, 13. Apr. 2016 (CEST)

Hast du den Artikel Schnellarbeitsstahl gelesen? Gruss --Nightflyer (Diskussion) 23:43, 13. Apr. 2016 (CEST)
Hat sich einfach als Standard bei "normalen" Schneidwerkzeugen durchgesetzt und sowas bekommt dann eben typisch einen Namen, obwohl es eigentlich genaugenommen einige Varianten gibt. Ähnlich wie z.B. "Werkzeugstahl" oder "Nylon". --Kharon 03:48, 14. Apr. 2016 (CEST)
Der Fachmann und gehobene Baumarktbesucher sagt HSS (High Speed Steel) dazu und kauft auch nur HSS-Bohrer (für Arbeiten mit Metall). --2003:75:AF10:2600:A1EF:D764:CCFD:EB48 07:23, 14. Apr. 2016 (CEST)
HSS hieß es schon in meiner (ersten) Ausbildung, noch zu DDR-Zeiten. Aus meiner Erinnerung hieß dieser Stahl so, weil er hohe Geschwindigkeiten (rotierend) überstehen konnte, ohne zu schmelzen (wurde trotzdem gekühlt, bspw. mit "Bohrmilch" Kühlschmiermittel)--Wikiseidank (Diskussion) 08:30, 14. Apr. 2016 (CEST)
Sicher richtiger Hinweis. Aber wäre es nicht spannender, die Unterschiede zwischen HSS-E und HSS-G zu diskutieren ...? *rofl*
Nö, die Profis diskutieren mittlerweile eher die Beschichtung. Allerdings wird HSS heute fast nur noch bei Bohrern und Sägeblättern im Privatbereich und im Handwerk verwendet. In Lohndreh/fräs-ereien kommen mittlerweile selbst beim Bohren heute Systemwerkzeuge mit Schneidplatten bzw. speziell beim Bohren auch sogenannte Schneidkronen zum Einsatz. HSS konnte bei den Schnittleistungen Hartmetallen bzw. Karbiden und ähnlichem noch nie "das Wasser reichen". --Kharon 10:03, 14. Apr. 2016 (CEST)
HSS hat auch bei den "Profis" noch seine Berechtigung. Dann wenn es darum geht eine Spezialwerkzeug herzustellen. Sobald es aber in die Massenfertigung geht, dann stimmt das mit den Wechselplatten und Beschichtungen. Wenn keine "Standartwerkzeug" ist und es keine passenden Platten bzw. Halter gibt, nimmt man auch heute noch HSS. Weil ein HSS Werkzeug kann man mit normalen Werkzeug noch selber sich zurecht stutzen, bei dem ganzen Keramik-Gedings kannst das vergessen. Wenn irgendwie geht versucht man natürlich schon zu vermeiden -gerade auf CNC-Maschinen-, dass man ein HSS Werkzeug benutzen muss. Aber es gibt noch immer Nischen wo HSS seine Berechtigung hat. Denn je nach Stückzahl rentiert es sich schlicht weg nicht, seine Spezialanfertigung beschichten zu lassen, weil die Lebensdauer auch ohne Beschichtung genügend gross ist. --Bobo11 (Diskussion) 17:06, 14. Apr. 2016 (CEST)
Die Lebensdauer eines Bohrers, sofern er nicht weggeworfen, sondern wieder angeschliffen wird, ist mitunter ein Menschenleben lang und hat nichts mit dem Material zu tun. Der Fachmann nennt, das, was du Bobo11 als Lebensdauer bezeichnest, "Standzeit". Hinweis: Du gehörst auch zu denen, die unbedingt immer was sagen müssen, in der Regel steht es schon (richtig, dem Sinn nach) schon da. Vorschlag: Bleib doch bei der Bahn und dessen Themen, da scheinst du Fachmann zu sein und da wärst du dann auch wertvoll anerkannt.--2003:75:AF10:AC00:856:DC92:2A7:C090 07:49, 15. Apr. 2016 (CEST)

Und was muss passieren, damit man "Hochgeschwindigkeitsstahl" in der Wikipedia findet? (nicht signierter Beitrag von 90.146.53.23 (Diskussion) 02:22, 17. Apr. 2016 (CEST))

Die gängigen Lehnübersetzungen von high speed steel – zu denen „Hochgeschwindigkeitsstahl“ nicht zählt – findest Du im Artikel Schnellarbeitsstahl, auf den Hochleistungsschnellarbeitsstahl, Hochleistungs-Schnellarbeitsstahl, Hochleistungsschnellschnittstahl, Hochleistungs-Schnellschnittstahl und Schnellschnittstahl weiterleiten. --Rôtkæppchen₆₈ 02:48, 17. Apr. 2016 (CEST)

Was für einen Sinn hat die Vorrichtung auf diesem Foto…

📸?

Was sind’n das für schwarze Röhren, die da an dem Zaun befestigt sind; soll da mit Hilfe der Sonnenenergie Wasser erwärmt werden oder für was is’ das gut?🤔😳😜

--Molekularbiologe (Diskussion) 00:45, 14. Apr. 2016 (CEST)

Weitere Informationen gibt es unter Sonnenkollektor und insbesondere Vakuumröhrenkollektor. Das Bild wurde im Dezember 2015 von Benutzer:Johann Jaritz erstellt, bei Detailfragen kann er vielleicht weiterhelfen. Gruß, --Nfreaker91 01:21, 14. Apr. 2016 (CEST)

Danke,Nfreaker91.
Dann war mein Gedankengang ja gar nicht so verkehrt. 🙃☺️😊
Bin mir nur nach wie vor nicht sicher, was die mit Warmwasser so fernab vom Gebäude wollen. 🤔😁
--Molekularbiologe (Diskussion) 22:47, 15. Apr. 2016 (CEST)
Was die genau sich gedacht haben weiß ich natürlich nicht. Eigentlich sollte Wasser auf den paar Metern zum Haus aber nicht nennenswert abkühlen, die Positionierungsentscheidung ist vermutlich pragmatisch. Wo sollen sie das sonst hinstellen? Mitten in den Vorgarten? – das stört doch; die Nachbarwiese gehört ihnen wahrscheinlich nicht; das Dach sähe auch komisch aus, es handelt sich hier wohl um einen Hotelbetrieb. Wenn man das an den Zaun dübelt entsteht einfach nur ein etwas dickerer Zaun. Gruß, --Nfreaker91 10:25, 16. Apr. 2016 (CEST)
So eine Platzierungsentscheidung könnte mit Denkmalschutzauflagen zusammenhängen. Immerhin konnten die Kollektoren über einer wärmespeichernden Mauer angebracht werden. --Sitacuisses (Diskussion) 11:25, 17. Apr. 201

Zeitung gesucht

Guten Morgen Gemeinde, ich hab da mal 'ne Frage. Ich suche den Namen einer Zeitung. An Folgendes kann ich mich noch erinnern: Die gab es Mitte der 1990er (im Supermarkt-Zeitungsregal, müsste 1995/96 gewesen sein) und war voll mit den abstrusesten Geschichten und Verschwörungstheorien. Aufgefallen war mir dieses "Werk" damals, als auf der Titelseite ein angebliches Foto von einem Fischmenschen prangte, den man wohl irgendwo gefunden haben wollte. Die Zeitung war (bin ich mir fast sicher) schwarz/weiß und vom Format her ich glaube etwas kleiner als A3. Das Papier war mMn so ein "Tageszeitungspapier". In welchem Rhytmus das Blatt erschien weiß ich leider nicht mehr, auf jeden Fall hatte ich aber mehrere Ausgaben davon. Das ist leider alles, woran ich mich erinnern kann. Kennt irgendjemand von euch diese Zeitung??? Viele Grüße --Maddl79orschwerbleede! 10:07, 14. Apr. 2016 (CEST)

Keine Ahnung, wie die hieß, aber ich erinnere mich auch daran. Habe nie eine gekauft, aber mal drin geblättert. Der Zeitraum könnte passen - ich hätte sogar noch älter geschätzt. Hab mich damals gewundert, dass es dafür eine ausreichend große Käuferschar zu geben scheint. --Snevern 10:12, 14. Apr. 2016 (CEST)
Zumindest die Existenz kann ich bestätigen,und weiß, dass es die hier am Bahnhofskiosk gab. Müsste erste Hälfte der 90er gewesen sein, da hatte ich die frühen Ausgaben der PC Games dort gekauft. --Magnus (Diskussion) 10:15, 14. Apr. 2016 (CEST)
Ufo-Kurier? --Buchling (Diskussion) 10:25, 14. Apr. 2016 (CEST)
Der hier? Nein, der wars leider nicht. Der Zeitraum kann auch schon 93 gewesen sein, aber nicht vorher. Viele Grüße --Maddl79orschwerbleede! 10:31, 14. Apr. 2016 (CEST)
Die deutsche Ausgabe der Weekly World News hieß Neue Spezial. HTH, --Kängurutatze (Diskussion) 10:37, 14. Apr. 2016 (CEST)
@Kängurutatze: Jaaaaaaaa!!!!! Genau das ist sie! Vielen Dank!!!! --Maddl79orschwerbleede! 10:40, 14. Apr. 2016 (CEST)
Köstlich. "68 Prozent der deutschen Männer täuschen den Orgasmus beim Masturbieren nur vor!" --Snevern 10:56, 14. Apr. 2016 (CEST)
Unglaublich! Die gleiche Frage wollte ich gestern stellen, hatte aber keine Zeit für eigene Vorabrecherche. Das kann doch kein Zufall sein....? Danke an den Finder (Känguru)--Wikiseidank (Diskussion) 12:04, 14. Apr. 2016 (CEST)
Wenn das wahr ist, ist es wirklich ein bemerkenswerter Zufall. --95.223.244.70 15:26, 14. Apr. 2016 (CEST)
Is it, schwöre. Ich weiß, dass die Zeitung im Zeitungsregal des Kaisers (gab nichts anderes zum einkaufen) frühe 90er (kurz nach Wende) lag. Da die Kassenschlangen sehr lang waren, habe ich ab und zu mal durchgeblättert. Ohne Quatsch, ähnliche Themen prankten auch auf den daneben liegenden Bunten Blättern (SuperIllu, BZ und so, ich glaube BILD war damals in Berlin noch nicht so groß?), nur die haben das als echt verkauft. Ich hatte schon Googlesuche durchgeführt "Zeitung, 1990er, Verschwörungstheorie, UFO, Alien, Fake" und wollte gestern hier fragen. Das kann kein Zufall sein! Mindcontrol--Wikiseidank (Diskussion) 16:18, 14. Apr. 2016 (CEST)
Viele Männer sollen ja auch eine Erektion vortäuschen, um ihren Frauen einen Gefallen zu tun.--Optimum (Diskussion) 19:43, 14. Apr. 2016 (CEST)
@Wikiseidank: Das ist natürlich wirklich ein krasser Zufall... Ich wollte eigentlich schon vor 'ner ganzen Weile nach dieser Zeitung fragen, hatte es aber vergessen. Keine Ahnung warum es mir ausgerechnet gestern wieder einfiel... ;) Viele Grüße --Maddl79orschwerbleede! 14:56, 15. Apr. 2016 (CEST) PS: Bei uns gab es das Blatt bei Spar.
Diese Zeitung kenne ich auch mit dem dritten Auge etc. Im Artikel steht das Blatt sei dem Boulevardjournalismus zuzurechnen, aber ich glaube die Käufer dieser Zeitschrift wussten sehr genau, dass nichts von dem was da drinsteht wahr ist, also ist das eher eine Satirezeitschrift gewesen, also eine die sich über die Bild und ähnliche Zeitungen und deren sensationslüsternen Schreibstil lustig macht.--Giftzwerg 88 (Diskussion) 13:07, 16. Apr. 2016 (CEST)

Geeignete Formulierungen beim Thema Shoa

Hallo,

ich sitze momentan an einer Arbeit über Theresienstadt und bin mir in zwei Fällen mit der Formulierung unsicher. Vielleicht könnt ihr mir weiterhelfen.

1. Wenn eine Person 1942 vermutlich an Mangelernährung in Theresienstadt starb, sollte ich dann schreiben, dass sie ermordet wurde? Anders gefragt: Kann bzw. sollte jeder Todesfall in Theresienstadt als Mord bewertet werden?

2. Wenn eine Person 1942 in einem tschechoslowakischen Gefängnis von deutschen Wachen zu Tode geprügelt wurde, kann und sollte sie dann als Opfer der Shoa bezeichnet werden?

Herzlichen Dank für jeden Hinweis.

--131.188.230.53 10:23, 14. Apr. 2016 (CEST)

Wie lautet das Thema der Hausarbeit denn genau? --Heletz (Diskussion) 10:52, 14. Apr. 2016 (CEST)
Es geht um Leipziger Ärzte im KZ Theresienstadt (z. B. Felix Skutsch). Bei der zweiten Person handelt es sich um einen Verwandten. --131.188.230.53 11:30, 14. Apr. 2016 (CEST)
Frage 2 kann man problemlos mit «ja» beantworten; träfe btw auch zu, wenn die Wachen Slowaken gewesen wären. Frage 1 spielt auf die Problematik Mord/Völkermord an. Hier kann man problemlos Mord schreiben, aber auch eine andere Formulierung ist möglich («starb im Zuge der Shoah an Mangelernährung»). Haltze ich ohne weiteren Kontext jetzt erst einmal nicht für so wichtig. --Kängurutatze (Diskussion) 11:38, 14. Apr. 2016 (CEST)
Bei Frage 2 bin ich mir keineswegs so sicher: Es ist nicht die Rede davon, dass die totgeprügelte Person Jude war, und die Shoa wird allgemein nur auf jüdische Opfer bezogen. Es wurden aber auch andere umgebracht, sicher auch in tschechischen Gefängnissen. Ein totgeprügelter nichtjüdischer Kommunist ist kein Opfer der Shoa. --Snevern 12:05, 14. Apr. 2016 (CEST)
Ja, gut, ich war davon ausgegangen, daß es sich um Juden als Opfer handelte, sonst macht der Begriff ja in der Tat keinen Sinn. --Kängurutatze (Diskussion) 12:25, 14. Apr. 2016 (CEST)

Also die Frage mit dem Mord hat hier letztes Jahr hohe Wellen geschlagen. Da die allgemeine Erinnerungskultur diese Opfer zu den Ermordeten zählt, ist in Deiner Arbeit auch quellentreu entsprechend so zu verfahren. Das Problem ist da eher die Zuweisung von konkreten Tätern. Zu 2 - Du bist auf einem sehr umstrittenen Pflaster! Das Problem ist die unterschiedliche Verwendung der Begriffe durch verschiedene Opfergruppen und Opfer, sowie begleitenden Nichtopfergruppen. So kenne ich die Maximaldefinition, nach der alle jüdischen Opfer des Nationalsozialismus und seiner Verbündeten zwischen 1933 und 1945 Opfer der Shoah sind (Großes Unglück). So war das bis Mitte der 70er Jahre, wo durch das allgemeine Bekanntwerden der Umstände des Massenmords und der Verbindung mit dem Begriff Holocaust und dem Symbol Auschwitz dafür (inklusive Vergasen und Verbrennen) eine Fokusierung auf eine bestimmte Teilgruppe der Opfer erfolgte. Schon der Beginn dafür kennt unterschiedliche Daten, alle erfassen jedoch 1942. Es ist schlicht bis heute nicht möglich, die Todesumstände aller Ermordeten zu rekonstruieren, das Schicksal von bis zu 1,5 Mil. Menschen wird immer ungeklärt bleiben. Deshalb wäre es kontraproduktiv, nun bei bekannten Todesumständen eine Differenzierung vornehmen zu wollen, vor allem wenn die Täter eindeutig Angehörige deutscher Militär/Polizei/SS waren. Was der andere Punkt ist, ob zB. Opfer von Kroaten und Ungarn auch Opfer der Shoah sind, aber das ist ne andere Baustelle. Oliver S.Y. (Diskussion) 12:14, 14. Apr. 2016 (CEST)

Ich denke, dass dein Formulierung "So kenne ich die Maximaldefinition, nach der alle jüdischen Opfer des Nationalsozialismus und seiner Verbündeten zwischen 1933 und 1945 Opfer der Shoah sind." weiterhin Gültigkeit hat. Zumindest habe ich auch nach den 70ern das immer als selbstverständlich angesehen. Ob Deutsche oder ihre Vasallen die Mörder waren, das Ziel war das gleiche. Problematisch finde ich eher, dass über die Vasallen in Deutschland ein relatives Unwissen herrscht. Ob das jetzt die ungarischen Horthy-Faschisten oder die kroatischen Ustascha sind. Beide waren in Südost-Europa entscheidend am Holocaust der Nazis beteiligt und waren Vollstrecker vor Ort. Dazu kamen dann natürlich noch Hunderttausende von Opfern unter Serben, Roma, ungarischen/kroatischen Kommunisten und anderen "Staatsfeinden". Was übrigens auch in der Gegenwart immer noch große Wellen schlägt. Aber alle jüdischen Opfer (auch die von Ungarn, Kroaten und anderen) würde ich definitiv der Shoah zurechnen. --87.184.155.35 13:51, 14. Apr. 2016 (CEST)
Wird leider selbst innerhalb der Wikipedia anders gesehen, da entsprechende Quellen andere Auffassungen haben. Geht auch nicht um Einzelfälle, beispielsweise die Opfer der Reichsprogromnacht. Da fast alle Experten den Holocaust als Verbrechen während des 2.Weltkriegs definieren, fallen diese Opfer aus der Defintion, wenn man Shoah mit dem Holocaust gleichsetzt, und letzteren nicht nur als Hauptbestandteil der Shoah betrachtet. Liegt vieleicht auch an den Proportionen der wenigen 10.000 bis 1939 und den Millionen ab September 39.Oliver S.Y. (Diskussion) 14:52, 14. Apr. 2016 (CEST)

Vielen Dank für die ausführlichen und hilfreichen Antworten. Ich werde mich wohl an der Maximaldefinition orientieren. --131.188.230.53 16:40, 14. Apr. 2016 (CEST)

hm, aber "1942 in einem tschechoslowakischen Gefängnis" ist mMn arg seltsam, de facto existierte die ČSSR ja nu nicht mehr und hat deswegen auch keinen Strafvollzug mehr vollzogen? --Edith Wahr (Diskussion) 17:50, 14. Apr. 2016 (CEST)
Evtl. ungenau beschrieben, gemeint ist wohl in Böhmen. Aber wenn ein irgendein Gefängnisinsasse von Wärtern umgebracht ist das erstmal doch nicht Teil eines Völkermords. Dazu müssten die nähren Umstände bekannt sein, es könnte ja auch ein "gewöhnlicher" Verdächtiger/Krimineller gewesen sein. Wer aber im KZ starb, der ist so oder so ein Opfer des Holocaust, egal wie genau er letztlich umkam. @87.184: Wer sollen die "Horthy-Faschisten" gewesen sein. Horthy war Antisemit, ja, aber kein Faschist und er hat die ungarischen Juden ja eben nicht an die Nazis ausgeliefert. Die "Faschisten" hießen dort Pfeilkreuzler und kamen erst 1944 an die Macht (nachdem die Deutschen das Land besetzt hatten)--Antemister (Diskussion) 18:22, 14. Apr. 2016 (CEST)
Zumindenst geschah laut Wikipedia folgendes unter Horthy: "Das erste antijüdische Gesetz unter Horthy – einen speziellen Numerus clausus für jüdische Studenten umfassend – stammte aus dem September 1920. Der jüdischen Minderheit (6,2 % der Gesamtbevölkerung) wurde darin der Zugang zur Universität erschwert und eine Beschäftigung im Staatsdienst praktisch unmöglich gemacht.". Wie auch: "Ab 1940 wurden die Juden aus den ungarisch besetzten Gebieten, wie der Südslowakei und Siebenbürgen, in das Generalgouvernement Galizien getrieben, wo sie letztlich dem deutschen Zugriff ausgeliefert waren." Vor allem der zweite Teil belegt somit die Teilnahme Ungarns am Holocaust schon vor den Pfeilkreuzlern und ob die "Vertreibung" gewaltlos stattfand, ist stark anzuzweifeln. Horthy mag per Definition kein Faschist gewesen sein, die Tendenzen waren jedoch offensichtlich. Ein anderer Punkt, der mir dabei ins Auge fällt, ist das Massaker von Novi Sad bei dem unter dem ungarischen Feldmarschall-Leutnant Ferenc Feketehalmy-Czeydner im Januar 1942 insgesamt bis zu 4000 Menschen zum Opfer fielen, ein großer Teil auch jüdische Zivilisten. Feketehalmy-Czeydner wird Horthy unterstellt gewesen sein, somit gab es tödliche Aktionen von Ungarn gegen Juden (und andere) schon vor 1944. Gehören diese Opfer nicht zum Holocaust? Wäre das solch ein Fall, der von User Oliver S.Y. angesprochen wurde, obwohl er während des 2. Weltkriegs stattfand? Danke.--87.184.154.153 11:14, 15. Apr. 2016 (CEST)
Mord ist eine täterbezogene juristische Kategorie und als solche dann von anderen Tötungsdelikten (Totschlag, KV mit TF) abzugrenzen. Die juristische Einstufung ist dabei länder- und zeitabhängig, weil sich Gesetze halt unterscheiden und verändern. Ich finde, diese Art Wertung hat in historischen Darstellungen überhaupt nichts zu suchen und wirkt immer unfreiwillig komisch (obwohl die Autoren das selbst oft gar nicht bemerken): Hat Caesar die Einwohner von Alesia, die er vor seiner Feldbefestigung krepieren ließ, ermordet? Die Zeitgenossen hätten die Frage wohl für absurd gehalten. Wurden die Einwohner Dresdens durch die alliierten Bombenangriffe im Februar 1945 ermordet? Und was ist der Unterschied zu bei der Shoa ums Leben gekommenen Personen? Wobei höchstwahrscheinlich eine gewisse Anzahl von in KL usw. internierter Personen eben nicht "ermordet" wurden, sondern aufgrund anderer Ursachen (Suizid, Krankheiten, Unfälle, Altersschwäche usw.) starben - auch dann, wenn dieser nicht internierungsbedingte Anteil an den Todesfällen nur klein war, so falsifiziert er die Verallgemeinerung dennoch. Ich würde empfehlen, die Begrifflichkeit zu überdenken und eine wissenschaftliche Terminologie zu verwenden, die auch in hundert Jahren noch Bestand hat.--80.171.153.232 15:54, 16. Apr. 2016 (CEST)
Wie weit will man gehen?
Zum Beispiel: Was ist ein Mörder?
War Adolf Hitler ein Mörder?
Antwort: Hitler wurde nie wegen Mordes verurteilt und war deshalb kein Mörder! Es bleibt schwierig... NfdA (Diskussion) 21:11, 16. Apr. 2016 (CEST)
Soldaten sind Mörder --Rôtkæppchen₆₈ 01:31, 17. Apr. 2016 (CEST)

Alternative zum Internet?

Im Artikel FidoNet steht: Am 30. Januar 2011 wurden viele FidoNet-Nodes reaktiviert, um die zeitweilige Abschaltung des Netzes während der Revolution in Ägypten 2011 zu umgehen. Deshalb meine Frage: Gibt es eigentlich noch andere alternative Netzwerke zum Internet? Es soll z. B. so eine Art Usenet via Packet Radio geben, aber wird es auch entsprechend eingesetzt? (nicht signierter Beitrag von 188.101.74.200 (Diskussion) 11:45, 14. Apr. 2016 (CEST))

Hamnet? Amateurfunkdienst? --Heimschützenzentrum (?) 19:48, 14. Apr. 2016 (CEST)
Gibt es denn bereits solche Netzwerke? Was ist der Stand bezüglich Telefonleitung und alten Usenet-Verbindungen über UUCP?--94.222.212.233 11:03, 16. Apr. 2016 (CEST)
Die „Telefonleitung“, wie es sie bis vor ungefähr vier Jahren gab, gibt es nicht mehr. Dank VoIP und Vectoring bedient sich die Telefonie des Internets und nicht umgekehrt wie vorher. Mehr und mehr bisherige Analog- und ISDN-Anschlüsse werden mehr oder weniger zwangsweise auf ADSL, VDSL und Vectoring umgestellt. Von daher ist das Festnetz keine Alternative zum Internet mehr. Auch sind die ganzen neuen dezentralen ADSL- und VDSL-Verteiler vom öffentlichen Niederspannungsnetz zur Energieversorgung abhängig. Eine Netzausfallreserve per Notstromgenerator und Stützbatterie wie bei den bisherigen Analog- und ISDN-Netzen und den ADSL-Anschlüssen aus den Hauptverteilern (ehemals Ortsvermittlungsstellen) gibt es nicht mehr. Das Festnetz fällt also als Rückfallebene weitestgehend aus. Es bleiben die Mobilfunknetze. Die einzelnen Mobilfunkbasen haben üblicherweise Stützbatterien, sodass sie zumindest eine in Krisenzeiten übliche stundenweise Stromabschaltung überstehen können. Auf ein mobiles Internet würde ich mich in solchen Fällen nicht verlassen. Es bleiben digitale Punkt-zu-Punkt-Verbindungen per Schmalbandmobilfunk. Diese sind in derselben Geschwindigkeitsklasse wie die zu Fidonet-Zeiten genutzten Analogtelefonmodems. Man müsste also das gute alste Fidonet auf 9600-bps-GSM-Schmalbandverbindungen migrieren und hätte dann dasselbe Look-and-Feel wie zu Anfang der 1990er-Jahre, als Internet zuhause noch unbekannt war und eher das Fidonet mit 2400 bps populär war. Ich habe seinerzeit die Fidonet-Nutzung aufgegeben, weil ich mir als junger Student die Telefonrechnung nicht leisten konnte. Außerdem hatte ich ja über die Hochschule Internetzugang. --Rôtkæppchen₆₈ 00:57, 17. Apr. 2016 (CEST)

Berufsbegleitend einen Bachelor machen - Dauer?

Guten Abend, mal angenommen man möchte neben einem Beruf den man zurzeit ausübt auch noch was anderes machen und versucht sich bspw. an einem Bachelor-Abschluss an der FOM Hochschule. Da gibt es bspw. das Modell, dass man Freitag Abends und am Samstag in die Hochschule kommt. Wie lange braucht man so um zum Abschluss zu kommen? Auf fom.de finde ich diesbezüglich keine Angabe. Das wird aber regelmäßig doch länger dauern als die 3 bis 3,5 Jahre eines Vollzeitstudenten oder? --87.140.192.16 21:00, 14. Apr. 2016 (CEST)

Du findest die Studiendauer bei den einzelnen Studiengängen. Normalerweise sind das wohl sieben Semester bis zum Bachelor. --Snevern 21:21, 14. Apr. 2016 (CEST)
Das schon, aber man kann doch nicht das gleiche in 1,5 Tagen lernen was ein Student in einer Woche lernt oder (bzw. wenn das verlangt wird, dann ist es ja deutlich anspruchsvoller). --87.140.194.3 21:35, 14. Apr. 2016 (CEST)
Das sollte heute doch anhand der ECTS errechenbar sein. Ein LP bedeutet 25-30 Stunden Arbeit, ein Bachelorstudium hat regulär drei Jahre bzw. 180 LP welche einer 40h-Woche entsprechen sollte. Was kann man neben einer Vollzeitstelle studieren? 20h/Woche (eher weniger) ist sicher die absolute Obergrenze die eine Hochschule ansetzen kann (auch wenn gerne über Leute berichtet wird, die neben einer vollen Stelle noch Vollzeit studieren). 6+ Jahre für eine Bachelor, dessen nutzen dann doch eher zweifelhaft ist... Da kommen mir die Zweifel, obwohl ich von solchen Weiterbildungskursen eig. keine Ahnung habe. Ist es denn für solche Kurse nicht üblich dass man in dieser Zeit nur Teilzeit arbeitet und nur die Präsenzzeiten auf Freitagabend und Samstag gelegt werden zwecks einfacherer Organissation?--Antemister (Diskussion) 22:35, 14. Apr. 2016 (CEST)
Theoretisch hat man auch beim einem Teilzeit Studium nicht länger. In der Regel wirst du aber 1-2 Semester aussetzen müssen, weil so ein Teilzeitstudium geht an die Substanz. Du wirst pro Semseter nicht weniger lehrnen müssen als ein Vollzeitstudent. Mehr als ein Beschäfftigunggrad von 60% wird, eh nicht empfolen, eher weniger. Wenn die Peräsenzzeiten nicht auf die ganze Woche verteilt sind, erleichter dies natürlich die Organistation, n somit ist es leichter neben dem Studium noch Geld zu verdeinen. Aber eben die perönliche Lehrnzeit ausserhalb der Uni wird daduch nicht kürzer. --Bobo11 (Diskussion) 06:15, 15. Apr. 2016 (CEST)
Nichts von dem konnte ich,selbst nach zweimaligen Lesen verstehen. Wie ich weiter oben schon schrieb, musst du nicht überall mit deinen Ansichten präsent sein. Wenn man sie wenigstens verstehen würde.--2003:75:AF10:AC00:856:DC92:2A7:C090 08:17, 15. Apr. 2016 (CEST)
Der Semesterinhalt ist auch bei einem Teilzeitstudium der selbe. In dem oben genannten Beispiel sind die Vorlessungen in ein verdammt kurzen Zeitraum gequetscht. Vorlessung=Präsenzzeit. Der ganze übrige Kram (Bibliotheksrechersche, Lesen, Schreinkrsm), musst du selber orgsnisieren, dss kannst an den beiden Tagen vergessen. Ein guter Bekannter hat den Ingenier HTL, auch in Teilzeit mit 50% gemacht. Der hatte auch Freitag Abend plus Samstag. Der lehrnte aber mehr oder weniger an zwei weiten Tagen jeweils rund 12 Stunden. Er meinte mal neben den Präsdenzzeit habe er pro Woche ca. 25-30 Stunden aufgewendet. Neben einem 50% Arbeitspensum, blieb da nicht mehr viel Freizeit. -Bobo11 (Diskussion) 08:36, 15. Apr. 2016 (CEST)
Mal ganz ehrlich: Der Mann mit der langen Nummer hat nicht ganz unrecht. Du haust aber gleich wieder Ansichten in einer unbekannten Sprache rein - wäre es nicht besser, vor dem Einloggen einfach noch mal kurz den Beitrag zu überfliegen? Zwecks Lesbarkeit? Vorlessung und Rechersche heitern ja noch auf, aber den Schweinkram "Schreinkrsm" zu schreiben, ist schon an der Grenze, ebenso wie die ohne "i" geschriebene Präsidenzzeit, was zum Fugg das auch immer sein mag. Lehrnen ist eben doch nicht aller Unheile Anfang, auch nicht an der Unnieverssitaett. NfdA (Diskussion) 08:56, 15. Apr. 2016 (CEST)
Schon mal auf seine Benutzerseite geguckt? --Optimum (Diskussion) 09:08, 15. Apr. 2016 (CEST)
Danke für den Hinweis. Dass er Schweizer ist, hatte ich durchaus gemerkt. Es geht auch nicht um das eine oder andere "ß", das wäre hinzunehmen. Trifft übrigens auch nicht so recht zu, denn ein Schweizer, der (Beispiel) mal einen Dürrenmatt liest, findet dort genauso viele "ß" wie ein Deutscher, weil diese Bücher ja einmal für den ganzen deutschen Sprachraum hergestellt werden. Die Juxtaposition von ß-Problematik und "Legasthenie" überzeugt nicht wirklich. "Legasthenie" ist auch insgesamt überholt. Jedenfalls sollte ein erwachsener Mensch, der Schule und Berufsausbildung hinter sich hat, sich dazu durchringen können, den eigenen Text zu kontrollieren, bevor er ihn freigibt. NfdA (Diskussion) 09:23, 15. Apr. 2016 (CEST)
Und was die ohne "i" geschriebene Präsidenzzeit ist, findest Du hier: Präsenzzeit.--Optimum (Diskussion) 13:21, 15. Apr. 2016 (CEST)

BK Lieber Herr Bobo11, ich entnehme dem neuerlichen Beitrag, dass du nur einem Mitteilungsdrang frönen willst und man dich schlichtweg als Troll einstufen darf.--2003:75:AF10:AC00:396F:20CF:F230:F3C6 09:03, 15. Apr. 2016 (CEST)

Ach ja, wenn ich schreibe es geht nicht länger, der Inhalt müsse der gleiche sein bin ich ein Troll. DANKE. --Bobo11 (Diskussion) 09:07, 15. Apr. 2016 (CEST)
  • Die ganze "Normung" des Studiums lässt keinen echten Freiraum mehr. Damit die gegenseitig anerkannt werden können, muss der Inhalt je Semester über einstimmen. Das heisst im Klartext wenn es zum Titel 7 Semester braucht, brauchst immer du 7 Semester. Am einzigen Ort wo die heute noch Freiraum haben, ist bei der Dauer des einzelnen Semesters. Eine "Abendschule" darf ein Semester mit der Dauer von 9 Monaten anbieten, und muss den Stoff nicht in 5-6 Monate quetschen.--Bobo11 (Diskussion) 09:47, 15. Apr. 2016 (CEST)
Das hat ja dann den Namen Semester nicht mehr verdient, sondern müsste eher Novemester heißen. :) --Snevern 09:54, 15. Apr. 2016 (CEST)
Klar stimmt dann der Name "Semester" nicht mehr wirklich, aber von der Stoff-Stufe her eben schon. Und die ganze Hochschul "Normen", stellen heute auch solche Stoff-Stufen ab. Es gibt genau dafür die Punkte im Bologna-Prozess. Das eben 30 Stunden Vollzeitstudium 1 ECTS Punkt gibt (30 Stunden entspricht 1 Woche Vollzeitstudium). Diese 30 Stunden Studienzeit, müssen eben nicht zwingend in einer Woche erbracht werden.--Bobo11 (Diskussion) 13:01, 15. Apr. 2016 (CEST)
Semester kommt nicht von 6 Monaten, sondern vom halben Jahr, ihr Semigebildeten. --Hachinger62 (Diskussion) 11:14, 16. Apr. 2016 (CEST)
Ach ja? Und Trimester kommt von einem Drittel des Jahres? Weil 3 x 3 gleich 12?
Peinlich, peinlich, das. --Snevern 16:36, 16. Apr. 2016 (CEST)

Dassler, die Schraubstollen und der Regen

Vermutlich stehe ich auf irgendeinem Schlauch. Aber ich frag einfach mal hier.

Man schreibt ja allgemein Dasslers Schraubstollen einen wichtigen Anteil am Sieg in Bern zu.

Aber - wenn ich weiß (oder auch nur glaube), dass eine bestimmte Stollenform auf Matsch viel besser geeignet ist als der gemeine Trockenrasenstollen, dann wechsle ich in der Pause, wenn der Regen beginnt, die Stollen aus. Das geht nur mit Schraubstollen, klar.

Aber nehmen wir mal an, es gäbe noch keine Schraubstollen. Sollte ich dann nicht für meine Mannschaft jeweils zwei Paar Schuhe dabei haben, ein Paar mit Trockenrasenstollen und ein Paar mit Sauwetterstollen?

Dass jeder Spieler seine Sonnenschuhe auszieht und dann seine Regenschuhe anzieht, kann doch auch nicht länger dauern als das Anschrauben auf einem Bein (der neue Film zeigt das Anschrauben übrigens bei ausgezogenen Schuhen).

Gab es irgendeinen praktischen Grund?

Oder sagte das Reglement, dass jeder Spieler nur ein Paar Schuhe besitzen darf? NfdA (Diskussion) 23:57, 14. Apr. 2016 (CEST)

Die hatten damals ein ziemlich klammes Budget. Timm Thaler (Diskussion) 01:54, 15. Apr. 2016 (CEST)
Ganz sicher. Aber gibt es einen Beleg dafür, dass man zwar 18 Spielern, Trainer, Arzt, Masseur und so weiter drei Wochen Hotel in der Schweiz bezahlen konnte, aber nicht weitere 18 Paar Schuhe? NfdA (Diskussion) 01:59, 15. Apr. 2016 (CEST)
Ist ein "Masseur und so weiter" zwingend notwendig? oder hätte in der Not nicht auch die 18 Spieler + Trainer + Co-Trainer + Arzt (also insgesamt 22 Mann) gereicht? Aber vielleicht weiß ja jemand jemand genau, wie groß die ungarische Delegation 1954 war. Jedenfalls dürften auch damals wahrscheinlich 18 weitere Paar Schuhe in etwa so teuer gewesen sein wie 3 Wochen Aufenthalt für eine Person, also nicht sehr hoch im Vergleich zu den Gesamtkosten, aber bei knappem Budget eventuell zu teuer. Man muss bedenken, dass das eine zeit war, zu der noch oft Mannschaften wegen Geldmangel gar nicht Quali teilgenommen haben, oder nicht an der WM teilgenommen haben, obwohl sie qualifiziert waren. Polen hatte sich zwar zur Quali gemeldet, aber auf sein Spiel gegen Ungarn verzichtet, wobei ich nicht weiß, ob das 8auch) finanzielle Gründe hatte. --MrBurns (Diskussion) 02:26, 15. Apr. 2016 (CEST) Nachtrag: en:1954_FIFA_World_Cup_Final#Summary beschreibt die Situation anders (den relevantesten Teil habe ich hervorgehoben): In addition, the Germans were equipped with footwear supplied by Adidas, which had produced a previously unknown design of boot with exchangeable, screw-in studs that could be adapted to any weather. This enabled the German players to wear their regular boots despite the adverse weather. Es scheint also eher so zu sein, dass die deutschen Spieler einen Vorteil hatten, weil sie in ihren gewohnten Schuhen spielen konnten. Zu bedenken ist auch, dass Fußballschuhe damals noch aus normalem Leder und nicht irgendwelchen High-Tech-Stoffen oder sehr speziellen Ledersorten wie Känguruleder waren und daher wohl besser passten, wenn sie "eingetragen" waren. --MrBurns (Diskussion) 02:34, 15. Apr. 2016 (CEST)
Mein posting war nicht komplett: Die hatten überhaupt kein Geld für Schuhe. Sie waren darauf angewiesen, daß Dassler ihnen die zur Verfügung stellt; und Bargeld noch obendrauf. Kürzlich lief ein halbdokumentarischer Film über die Dassler-Brüder im TV. Da wurde gezeigt, daß Herberger mit seinem Ansinnen zuerst zu Rudi Dassler (Puma) ging , dort aber abgewiesen wurde. Timm Thaler (Diskussion) 03:46, 15. Apr. 2016 (CEST)
Ob sie wirklich angewiesen waren, weiß man nicht so genau, aber natürlich musste ein Verein wie der DFB immer schon wirtschaftlich denken, daher wenn man die Schuhe gratis und dazu noch Geld bekommt, wird man das bevorzugen gegenüber einer Schuhmarke, bei der man selber noch was bezahlen muss. Ist ja heute auch nicht anders, heute wird meist die Firma Ausrüster, die am meisten Geld einbringt, zumal die Qualitätsunterschiede zwischen den Spitzenprodukten der einzelnen Sportartikelhersteller eher gering ist. Auch wenn im Einzelhandel die Preise sich oft stark unterscheiden. Oft sind Produkte verschiedener Marken auch identisch abgesehen vom Markenlogo. --MrBurns (Diskussion) 02:13, 17. Apr. 2016 (CEST)

Dassler war halt auch ein Marketing-Fuchs und "man schreibt ja allgemein" ist sicher nicht zufällig passiert, sondern wurde sicher gezielt lanciert. --Studmult (Diskussion) 22:24, 15. Apr. 2016 (CEST)

Veröffentlichung von alten Zeitungartikeln und -Anzeigen vor 1932

Hallo,

darf ich Scans von Zeitungsartikeln aus Mikrofilmen mit Erscheinungsdatum vor 1932 in der Wikipedia in die Artikel einbinden?

Es handelt sich dabei um längere Artikel mit Kürzel des Autors oder kurze Artikel ohne Namensnennung. Und auch Werbeanzeigen.

Die Zeitungen selbst existieren seit dem Ende 2. Weltkrieg nicht mehr.

Grüße und Danke für eine Antwort, --Miklas (Diskussion) 00:16, 15. Apr. 2016 (CEST)

Für Bilder die älter als 100 Jahre alt sind haben wir die Regelung wie in Vorlage:Bild-PD-alt-100 beschrieben ist. Sollte der Autor bekannt sein gilt natürlich die Gemeinfreiheit erst 70 Jahre nach dessen Ableben wenn das Datum bekannt ist. Für Bilder vor 1923 die Vorlage:Bild-PD-alt-1923 die aber erst nach Diskussion auf Wikipedia:Dateiüberprüfung/1923 benutzt werden darf. --Mauerquadrant (Diskussion) 01:00, 15. Apr. 2016 (CEST)
Hallo und danke für die schnelle Antwort!
Die Artikel haben fast alle außer einem gar keine Bilder, teilweise sind es nur gesammelte einzelne Einträge aus dem Lokalteil, also wie z. B. "Neues aus unserem Bezirk XY" oder auch Werbeanzeigen. Ein Kürzel ew. findet sich unter zwei längeren Artikeln mit Fotos, die Bilder könnte ich aber rauslassen. Die Frage ist, welche Artikel darf ich als Screenshot in Originalschrift ohne Bilder zitieren? Oder wenigstens als Text-Zitat ohne Screenshot? Und wie sieht es mit den Werbeanzeigen aus? --Miklas (Diskussion) 01:40, 15. Apr. 2016 (CEST)
Für tiefer greifende Fragen würde ich mich an Wikipedia:Urheberrechtsfragen wenden. Mit Bildern meinte ich auch solche die nur aus Text bestehen und nicht nur Fotos. --Mauerquadrant (Diskussion) 07:10, 15. Apr. 2016 (CEST)
Ansonsten gilt die prinzipielle Regel mit den 70 Jahren nach dem Tod der Verfasser (für jeden einzelnen Artikel). Bei 1932 ist das eher unwahrscheinlich, dass da viele schon 1945 gestorben waren. Einzelne Zitatsätze gehen immer, ganz Texte nicht. Werbung hängt von der Art der Graphik ab, je komplizierter, desto eher nein. --Hachinger62 (Diskussion) 11:19, 16. Apr. 2016 (CEST)

Münze, die sich "vierteln" lässt

Ich habe mal gehört, zur Zeit Karls des Großen gab es Münzen, die extra Bruchlinien hatten um sie in 4 Teile zu teilen, wenn man Wechselgeld sozusagen "klein" rausgeben wollte. Komischerweise finde ich dazu kein Bild im Netz. Kennt jemand ein Stichwort oder ist das ein Hoax? (nicht signierter Beitrag von 94.222.212.233 (Diskussion) 08:08, 15. Apr. 2016 (CEST))

Das man zu der Zeit Münzen klein gemacht hat, ist definitiv kein Hoax. Gerade Edelmetalmünzen wurde gewogen, nicht gezählt. Die Stempellung/Prägung war eher eine Qualitäts-, denn eine Wertangabe. Von daher würde es mich nicht erstaunen wenn es Edelmetall-Münzen mit Markierungen zum Kleinmachen gab. Ich hab das auch so in Erinnerung, aber wo ich mir devinitiv nicht sicher bin ist halbieren vs. Viertel, auch beim Zeitraum könnte ich mich irren. Da mehr oder weniger eine ganz bestimmte Münze gefragt ist, muss ich für eine devinitive Antwort zuerst in einem Buch nachschauen, das ich gerade nicht zur Hand habe.--Bobo11 (Diskussion) 08:25, 15. Apr. 2016 (CEST)
Das englische Stichwort wäre broke money. Deutsch evt. nach "Bruchmünze" suchen. Grüße Dumbox (Diskussion) 08:32, 15. Apr. 2016 (CEST)
Ist das mit "broke"/"pleite" verwandt...? --Eike (Diskussion) 09:51, 15. Apr. 2016 (CEST)
Dazu habe ich eine andere Erklärung gefunden. Geldverleiher haben früher ihr Geschäft auf Bänken vor dem Haus verrichtet (daher auch "Bank" für Kreditinstitut). Wenn jetzt ein solcher Geldverleiher pleite ging, hiess es, sei als Zeichen dafür seine Bank gebrochen worden (ital. "zerbrochene Bank" = banca rotta = bankrott, bankrupt). Die Quelle zweifelt zwar an, dass man das wirklich gemacht habe, aber der Begriff "being broke" kommt jedenfalls von daher, ob das nun gelebte Praxis war oder nur eine Legende. --King Rk (Diskussion) 12:38, 15. Apr. 2016 (CEST)
Wobei ich aus der Geschichte der Schlacht- und Freibank darauf hinweisen muss, das damit häufig auch Tische/Tresen bezeichnet wurden, und nicht wirklich "Bänke" als Stadt- und Gartenmöbel gemeint sind, also "auf" da einen falschen Eindruck vermittelt.Oliver S.Y. (Diskussion) 19:42, 15. Apr. 2016 (CEST)
Werkbank, Drehbank, … --Rôtkæppchen₆₈ 22:11, 16. Apr. 2016 (CEST)
Ich kenn noch die Version (selber erlebt in Brasilien), dass vier 25-Centavos-Münzen mit Klebeband ("Tesa") zusammengeklebt werden und damit ein 1 Real bilden. Kann man bei Bedarf auch aufsplitten, wenn man Wechselgeld braucht. -194.138.39.52 09:56, 15. Apr. 2016 (CEST)

Hallo! Also ich weiß um Hacksilber und das die Kreuze auf Sachsenpfennigen als Teilungslinien verwendet wurden, auch wenn sie nicht dafür gedacht waren. Ansonsten würde ich auf Erzählungen über das 9./10. Jahrhundert nicht allzuviel geben. Eine Übersicht über Münzen des Frankenreichs findest Du auf Commons unter [16].Oliver S.Y. (Diskussion) 10:17, 15. Apr. 2016 (CEST)

Straßentunnel Drosselung der zulässigen Höchstgeschwindigkeit

Warum wird vor und in Straßentunneln die zulässige Höchstgeschwindigkeit stark gedrosselt? Wahrscheinlich, weil darin mehr Unfälle als sonst passieren; nur ...wieso? --93.134.234.86 21:08, 15. Apr. 2016 (CEST)

Ein Aspekt dürfte die abrupte Änderung der Lichtverhältnisse sein, an die sich das Auge nur langsam anpassen kann. --84.59.22.29 21:20, 15. Apr. 2016 (CEST)
Im wesentlichen will man mit Tempo 80/60 an Gefahrstellen erreichen, dass alle Fahrzeuge die Gefahrstelle gleich schnell durchqueren, da dann kaum noch überholt wird und weiterhin die Gefahr von Auffahrunfällen durch sich schnell von hinten herannähernde Fahrzeuge reduziert wird.
Im Tunnel kommt noch der Deckeneffekt hinzu. Durch die Decke kann man Gefälle und Steigung nicht richtig einschätzen und fährt ohne Kontrollblick auf den Tacho sehr sicher zu schnell bergab und zu langsam bergan. LKW-Fahrer haben noch das zusätzliche Problem, beim Bergabfahren im Gegensatz zur freien Strecke, nur einen sehr eingeschränkten Bereich der Strecke überblicken zu können. Sie müssen das durch einen größeren Sicherheitsabstand ausgleichen, in den dann aber auch keine Fahrzeuge einfahren dürfen, weil sie z.B. einem Überholer Platz machen wollen. -- Janka (Diskussion) 21:41, 15. Apr. 2016 (CEST)
@Janka: Deinen zweiten Absatz würde ich ohne Quellen als Theoriefindung einordnen: Der Deckeneffekt ist etwas ganz anderes; ohne Blick auf den Tacho fährt man prinzipiell zu schnell (bzw. immer schneller), weil man sich an die Geschwindigkeit gewöhnt; und das Problem der LKW-Fahrer ist gar nicht nachvollziehbar. --TheRunnerUp 22:02, 15. Apr. 2016 (CEST)
Dann lies halt "Effekt der Decke". Im Übrigen gibt es nicht nur im Bereich der Messfehler Deckeneffekte weshalb der dortige Artikel ja auch einen Verweis auf den Pharmakologischen Ceiling-Effekt hat.
Deine Behauptung, durch fehlenden Blick auf den Tacho würde man grundsätzlich zu schnell fahren, kannst du durch einen Anruf z.B. bei der Betreibergesellschaft des Elbtunnels widerlegt finden. Dort ist es ein tägliches Problem, dass durch Ermüdungseffekte und eben die Fahrt in einem Rohr mit Deckel viele Fahrer im Gefälle zu schnell und dann auf der langanhaltenen Steigung zu langsam fahren. Denn für den Fahrer sieht die gesamte Fahrt durch den Tunnel wie eine einzige lange Aufwärtskurve aus, das kurze andersherum gebogene Stück an der Ausfahrt einmal ausgenommen.
LKW-Fahrer haben eine hohe Sitzposition und können in einem langen, steilen Gefälle im Freien die LKW-Kolonne vor sich überblicken -wenn sie da nicht auch viel zu dicht auffahren. In einem Tunnel geht das nicht, da sieht man nur die Decke. -- Janka (Diskussion) 23:50, 15. Apr. 2016 (CEST)

Es sind optische Gründe, das wurde erstmals in Richtlinien für Stadtstraßen Ende der 30er Jahre so festgelegt, damals noch mit Begründung. Fahrer werden unsicher, wenn die Fahrbahn seitlich hoch eingeschränkt ist. Ohne Leitplanken "sieht" man mehr Fahrbahnbreite, mit Leitplanken wird es scheinbar enger und eine komplette Bebauung wie in Tunneln wirkt noch enger, was die meisten Fahrer verunsichert. --Pölkkyposkisolisti 22:10, 15. Apr. 2016 (CEST) @Janka: Wie viele (längere) Tunnel haben denn Steigung/Gefälle? Das ist doch ganz selten, aus genau dem Grund.--Antemister (Diskussion) 23:32, 15. Apr. 2016 (CEST)

Alle Flussunterquerungen. -- Janka (Diskussion) 23:52, 15. Apr. 2016 (CEST)
Da fallen mir einige ein, z.B. der Heslacher Tunnel hat auf 2300 Meter 39 Meter Höhenunterschied, also 17 ‰ Steigung. --Rôtkæppchen₆₈ 01:06, 16. Apr. 2016 (CEST)
Straßentunnel im Gebirge (Alpen) haben von der Einfahrt bis (mehr oder weniger) zur Mitte eine Steigung und dann ein Gefälle bis zur Ausfahrt. Damit erübrigt oder verringert sich doch das Problem, dass man auf den ersten Kilometern zu schnell fahren könnte. Allerdings ist auch das zu schnelle Ausfahren bei plötzlicher Blendung durch helles Sonnenlicht (oder bei plötzlicher Dunkelheit nach dem erleuchteten Tunnel) ein Problem. Der Grund ist ganz banal der Abfluss von plötzlich eintretendem Wasser. Auch Eisenbahntunnel haben so einen "Scheitelpunkt" irgendwo in der Mitte, damit der Zug bei Wegfall des Antriebs immer noch rausrollt. NfdA (Diskussion) 01:35, 16. Apr. 2016 (CEST)
Jedenfalls ist mir schon häufiger aufgefallen, dass es eine deutsche Spezialität zu sein scheint, bei ausnahmslos jedem, auch dem kürzesten Tunnel, bei dem man von der Einfahrt aus schon die Ausfahrt sehen kann, die Geschwindigkeit auf 80 zu drosseln. In den Niederlanden, die ja auch jede Menge Tunnel unter Flüssen und Kanälen haben, macht man das nicht flächendeckend. --Jossi (Diskussion) 13:17, 16. Apr. 2016 (CEST)
Die Subatlantiktunnel, die die Färöer-Inseln mit einander verbinden oder tiefe Fjorde Islands queren, sind regelmäßig derart steil, dass man weder runter noch hoch über den 3. Gang hinaus kommt, wenn man nicht seine Bremsscheiben pulverisieren oder den Motor abwürgen will. Die neueren haben daher drei Spuren, von denen die mittlere immer für die Bergauf-Richtung frei ist – die älteren nicht. An der norwegischen Küste (z.B. bei Ålesund) gibt’s auch solche Apparate. —[ˈjøːˌmaˑ] 18:51, 16. Apr. 2016 (CEST)

WLAN-Gerät gesucht

Ich suche eine Möglichkeit, idiotensicher (!) in einem Schulungsraum WLAN für viele Teilnehmer zur Verfügung zu stellen. Es ist ein LAN-Anschluß vorhanden, wenn man dort einen Rechner anstöpselt, ist er sofort online - ohne irgendwelche Einstellungen. Kann man dieses Cat-5-Kabel nun einfach in eine Fritzbox stecken und haben dann alle im Raum WLAN? Wie und mit welchem billigen Gerät ist sowas möglich? Bitte auch Amazon-Links zu guten Lösungen. --Fahrradmonteur.de (Diskussion) 00:35, 16. Apr. 2016 (CEST)

Wenn Du die fritzbox entsprechend konfigurierst und das Cat5-Kabel in die Buchse LAN1 steckst, dann sollte das klappen. Eigentlich ist eine Fritzbox für so etwas aber oversized. Da gibt es Lösungen für ein Dreizehntel des Preises, die genau das leisten, was Du brauchst. --Rôtkæppchen₆₈ 00:53, 16. Apr. 2016 (CEST)
Geht nur, wenn man an diesem Port nicht nur einen, sondern über einen Switch auch mehrere Rechner anstöpseln kann. Bei einfachen Vernetzungen mit Billigkomponenten ist das zwar der Fall, aber wenn übergeordnet ein Managed-Switch sitzt, kann der Netzwerk-Admin diese Funktionalität dort abgeschaltet haben. Dann kann man an den Port wirklich nur ein Endgerät dranhängen, keinen weiteren Switch und damit auch keine gebridgden Netze. In diesem Fall müsste man einen Router dranstöpseln und ein privates Subnetz mit eingenem DHCP-Server auf diesem einrichten, denn der übergeordnete Managed-Switch gibt für diesen Port dann ziemlich sicher auch nur eine einzige DHCP-Lease aus. Für den Endanwender im WLAN ist das dann idiotensicher, nicht aber für den Stöpselmeister. Der braucht dazu Fachwissen. -- Janka (Diskussion) 03:33, 16. Apr. 2016 (CEST)
Da sollte es ein Einfachstrouter wie von mir verlinkt (kein reiner Access Point) tun, oder etwa nicht? --Rôtkæppchen₆₈ 11:56, 16. Apr. 2016 (CEST)
Vermutlich. Vielleicht auch nicht. Der OP muss dazu statt eines der LAN-Ports den WAN-Port nutzen, den Uplink auf dem WAN-Port auf IP-Verbindung mit DHCP stellen, sowie auf der LAN/WLAN-Seite den DHCP-Server mit DNS-Übernahme oder wahlweise einen DNS-Forwarder einrichten. Letzteres ist oft das, woran es bei Kaufgeräten fummelig wird oder es beim preiswerten Gerät einer Serie absichtlich nicht geht. Wegen irgendwas sollst du ja das tolle teure Gerät desselben Herstellers kaufen. Bei manchen preiswerten Geräten kann man mit der Original-Firmware nichtmal ein anderes Netz als 192.168.x/24 einstellen.
Ich würde daher am ehesten ein preiswertes Gerät kaufen, wo ich notfalls auch OpenWRT draufspielen kann, wenn die Originalfirmware absichtlich verkrüppelt wurde kann ich das dann immer noch tun und habe keine künstlichen Einschränkungen.
Ich wollte aber eigentlich darauf hinaus, dass der OP hoffen kann, dass er einfach den Router als AP benutzen kann, dann ist es kaufen, einstecken, funzt, dass es aber kein Gerät gibt, dass das ohne jede Einstellung im Router-Betrieb sofort kann. -- Janka (Diskussion) 14:30, 16. Apr. 2016 (CEST)
Es gibt wohl Kisten, die so vorkonfiguriert sind, dass man nur noch die WAN-Buchse mit dem Uplink verbindet und die Clients entweder per WPS PBC (Knopfdruck) oder den auf der Unterseite des AP/Routers aufgedruckten Zugangsdaten mit dem Standard-WLAN verbindet. Diese aus der MAC-Adresse des Routers/AP berechneten Zugangsdaten sind aber oft unsicher und nachvollziehbar und sollten durch eine eigene SSID und ein eigenes sicheres Kennwort ersetzt werden. --Rôtkæppchen₆₈ 01:45, 17. Apr. 2016 (CEST)

Philosophische Ideen von Hannah Arendt

Wie analysiert Hannah Arendt die Gleichschaltung im Nationalsozialismus? Worauf achtet sie dabei? Was denkt sie, welche Herausforderungen es für das heutige Verständnis des Bösen gibt?--Skyscraper1996 (Diskussion) 10:05, 16. Apr. 2016 (CEST)

Komm' jez' ... das steht zu 83,7 % in Hannah Arendt. Den Rest formulierst du analog - und der Lehrer strahlt. Play It Again, SPAM (Diskussion) 10:14, 16. Apr. 2016 (CEST)
Mann, wie das komisch tönt! Okay, ich schaue auch in meiner Dokumentation.--Skyscraper1996 (Diskussion) 15:18, 16. Apr. 2016 (CEST)
Ahrend soll sogar selbst Bücher geschrieben haben, behaupten manche. Rainer Z ... 16:19, 16. Apr. 2016 (CEST)
Just zum angesprochenen Thema jibbet sogar einen durchaus unterhaltsamen biographischen Film. Da muß man also nicht mal selber lesen, um diese Hausaufgabe zu lösen. --94.219.9.168 19:36, 16. Apr. 2016 (CEST)

Anteiliger Kaufpreis bei Rückgabe einer gekauften Ware (BGB)

Situation: Ein neues Handy mit Original-Verpackung (verschweißter Karton) wurde in einem Geschäft gekauft. Der Kunde stellt nach zwei, drei Tagen fest, dass er sich den Umgang mit dem Gerät aufgrund der Software völlig anders vorgestellt hatte und nun unzufrieden ist. Bei der Rückgabe äußert der Verkäufer, dass das Gerät zurückgenommen werden könne und der Kunde sein Geld zurückbekäme, allerdings müsse der Käufer in diesem Fall einen (geringen) Anteil des Kaufpreises zahlen, da die Folie entfernt wurde. Der Kunde möchte jedoch sein Geld in voller Höhe zurückerstattet bekommen.
Ist das Einbehalten eines Anteils seitens des Verkäufers rechtens, wenn die Plastikfolie beschädigt bzw. entfernt wurde? Zumindest habe ich die §§ 454, 455 BGB gefunden, wonach der Käufer ein Recht zum Ausprobieren des gekauften Gegenstands hat. – PsY.cHo (Diskussion) 16:41, 16. Apr. 2016 (CEST)

Wurde ein Kauf auf Probe vereinbart? Wenn nein, muss der Verkäufer die - nicht mängelbehaftete - Sache überhaupt nicht zurücknehmen. Alles Weitere ist Verhandlungssache. 89.13.82.8 17:00, 16. Apr. 2016 (CEST)
Ein Kauf auf Probe wurde nicht vereinbart. Der Verkäufer ist hier bereit, die Sache zurückzunehmen. Das Handy an sich hat keinen Mangel, nur der Käufer ist unzufrieden (z. B. wegen der Handhabung der Software). Die fehlende Plastikfolie um den Karton stellt das Problem dar. Diese Folie musste logischerweise beschädigt bzw. entfernt werden, um an den Inhalt (Handy) des Kartons zu gelangen. Daher bin ich an jenen Paragraphen interessiert, die sich auf das Einbehalten (sofern möglich) eines Teils des geleisteten Kaufpreises beziehen, wenn die Folie nicht im Originalzustand ist. Dennoch danke für deine Antwort. – PsY.cHo (Diskussion) 18:04, 16. Apr. 2016 (CEST)
Wenn der Verkäufer die Ware nur aus Kulanz zurücknimmt, kann er die Bedingungen dafür diktieren. Rechtlich wird ein Vertrag zwischen Verkäufer und Käufer geschlossen, entweder ein neuer Kaufvertrag über einen Rückkauf oder ein Vertrag zur Aufhebung des alten Kaufvertrags. Wenn der Käufer zum Widerruf berechtigt und der Verkäufer zur Rücknahme verpflichtet ist (bei Haustürgeschäften), darf ein bloßes Öffnen der Verpackung dagegen nicht zu einer Minderung des zurückzuerstattenden Preises führen (in Deutschland: § 357 (2 und 7) BGB). --BlackEyedLion (Diskussion) 19:44, 16. Apr. 2016 (CEST)
Wir sind uns wohl einig, dass es kein Kauf auf Probe war (der kommt bei solchen Sachen in der Praxis eigentlich nicht vor), und dass die Sache auch nicht mangelhaft ist, sondern dem Käufer nicht gefällt. In diesem Falle gibt es keinen Anspruch auf Rückabwicklung; wenn der Käufer dennoch bereit ist, einen (überwiegenden) Teil des Kaufpreises zurückzugeben, sollte der Käufer sich freuen. Eine Vorschrift, die regelt, welche Höhe ein gesetzlich überhaupt nicht geregelter Einbehalt haben soll, kann es natürlich nicht geben. Insofern hat BlackEyedLion im Ergebnis recht: Der Käufer kann die Bedingungen diktieren, denn er muss das Gerät ja überhaupt nicht zurücknehmen.
Es gibt übrigens auch keinen generellen Anspruch darauf, eine Ware ausprobieren zu dürfen; auch nicht bei einem Mobiltelefon. --Snevern 19:56, 16. Apr. 2016 (CEST)
So nicht. Allerdings nach den Erfahrungen und AGBs und Verhaltensweisen div. Firmen - sowohl on als auch offline - kann eine gekaufte Ware innert x-Tagen zurückgeben werden können. (Auf was anderes würde ich mich nicht mehr einlassen!) --84.186.127.134 21:10, 16. Apr. 2016 (CEST)
Es gibt wohl solche, solche und solche Händler. Es gibt Händler, die die zurückgenommene Ware neu verschweißen und dem nächsten Kunden als originalverpackt andrehen. Dann gibt es Händler, die die Rückläufer nach Prüfung mit Preisnachlass als „kaum gebraucht“ verkaufen. Und dann gibt es Händler, die die Rücknahme unter vollkommen korrektem Hinweis auf AGB und BGB verweigern. Derartige Ware findet sich dann oft ebenfalls mit Preisnachlass bei den bekannten Onlineversand- und „Auktion“shäusern. --Rôtkæppchen₆₈ 23:35, 16. Apr. 2016 (CEST)
+ 1 zu Snevern: Für mangelfreie Ware, die im Geschäft gekauft wurde, gibt es keinen gesetzlichen Umtausch- oder Erstattungsansprüch. „Gekauft ist gekauft“ (vgl. Pacta sunt servanda) – es sei denn, Käufer und verkäufer haben ausdrücklich etwas anderes vereinbart (hier offenbar nicht der Fall). Im Onlinehandel ist das anders, dort hat man ein zweiwöchiges Widerrufsrecht und bekommt bei Rückgabe den vollen Warenwert erstattet, jedenfalls, solange man nur „Beschaffenheit, Eigenschaften und Funktionsweise der Ware geprüft hat“. Für das bloße Öffnen der Verpackung, Einlegen der Batterien o.ä. darf der (Online-)Verkäufer also keine Wertminderung geltend machen. Diese Online-Konkurrenz veranlasst natürlich zunehmend auch stationäre Händler zu mehr oder weniger kulanten Umtauschregelungen - angefangen vom Umtausch binnen weniger Tage „nur in ungeöffneter Originalverpackung“ und nur gegen Warengutschein bis hin zu geradezu abenteuerlichen Versprechungen wie der von IKEA, wo man seine gebrauchten Möbel auch nach fünf oder zehn Jahren noch umtauschen kann, wenn sie einem nicht mehr gefallen. Wenn es aber keine solche, generelle Umtauschregelung des Verkäufers gibt, bist du bei deinem Handy auf Kulanz angewiesen. Und da kommt es dann natürlich nicht nur auf dein Verhandlungsgeschick, sondern auch auf die Umstände an - ist sonst alles tipptopp und ohne Gebrauchsspuren? Hat der Verkäufer einen guten Tag? Bist du ein guter Kunde? Kaufst du stattdesssen das Ersatzhandy bei ihm? Aber juristisch „müssen“ muss er wohl nix, nicht mal das Gerät überhaupt zurücknehmen oder einen einzigen Cent dafür bezahlen.--Mangomix 🍸 00:31, 17. Apr. 2016 (CEST)

Sympathy for the CV

Der deutsche "Teufelskreis" - diese Spirale ins Schlechtere - ist erst seit dem frühen 20. Jahrh. bekannt (viel früher gab es geografische Gegebenheiten, die so benannt wurden).

Circulus vitiosus / vitiosus circulus gibt es schon länger, etwa seit 1610.

(a) Findet jemand CV / VC noch früher?
(b) Gibt es im Griechischen (oder einer anderen alten Sprache) ein noch früheres Analogon, dass auch "Kreis" oder "Ring" oder ähnliches verwendet? Play It Again, SPAM (Diskussion) 10:19, 15. Apr. 2016 (CEST)
Ich denke, was Suarez damit meint, ist etwas anders: Da geht es um einen Zirkelschluss oder Diallele. Das geht zurück mindestens auf Aristoteles (Analytica Posteriora), ist aber nicht dasselbe, was mit Teufelskreis gemeint ist.--Meloe (Diskussion) 14:00, 15. Apr. 2016 (CEST)
Auf Aristoteles geht auch der (unzulässige) infinite Regress zurück. Wurde der vielleicht auch mal als "circulus vitiosus" bezeichnet? Geoz (Diskussion) 15:15, 15. Apr. 2016 (CEST)
Suarez: Es scheint hierum zu gehen. Nicht im heutigen Sinne des Teufels, aber trotzdem etwas, aus dem man nicht mer rauskommt, wenn man sich in das System begibt. Play It Again, SPAM (Diskussion) 19:31, 15. Apr. 2016 (CEST)
Ehe wir weiter gehen, will nöthig seyn, zu fragen: was dann eigentlich ein Circulus vitiosus auf deutsch heiße. --Pp.paul.4 (Diskussion) 21:48, 15. Apr. 2016 (CEST)
Eine Definition von Teufelskreis findet sich in der Zeitschrift für Religionspsychologie, Band 3-4, 1930, S. 106. Dies zeigt jedenfalls ein Buchschnipsel. Zeitschrift für Religionspsychologie, Bände 3-4, Seite 106: Wir nennen dieses charakterologische Gesetz oder, richtiger gesagt, dieses sinnvolle Zusammenwirken von charakterologischen Bestimmungen den Teufelskreis. Autor und Titel des Beitrags zeigt der Schnipsel leider nicht; es ist aber sicherlich nicht der im Wikipedia-Artikel angegebene Paul Watzlawick, der damals erst 9 Jahre alt war. --Pp.paul.4 (Diskussion) 22:54, 15. Apr. 2016 (CEST)

Ich dachte immer, CV meint den Lebenslauf (zumindest in der Sprache dieser Überschrift). --84.135.147.37 10:03, 16. Apr. 2016 (CEST)

Sorry, reimendes Wortspiel. Irgendwoher muss man doch seine Kicks bekommen ... jetzt, wo Drogen so teuer und Frauen so selten sind. Play It Again, SPAM (Diskussion) 13:12, 17. Apr. 2016 (CEST)

Anlieger oder vielleicht auch nicht - reinfahren?

 
Bin ich ein Anlieger - oder vielleicht doch nicht?

Guten Abend, mal angenommen ich möchte jemanden besuchen. Auf der Fahrt dorthin, er müsste so irgendwo hier wohnen, es kann nicht mehr weit sein, komme ich an eine Straße, die nur für Anlieger frei ist. So, nun weiß ich nicht genau, ob der, den ich besuchen möchte, wirklich an dieser Straße wohnt oder ob ich ihn vielleicht auch von der anderen Seite erreichen könnte ohne in den Anlieger-frei-Bereich einfahren zu müssen. Was mache ich denn nun dann? Einen Parkplatz suchen und erstmal per Fuß schauen und dann ggf. das Auto nachholen oder das Gebiet umfahren und von der anderen Seite kommen, weil das Grundstück ja doch nicht mehr im Anliegerbereich liegt (was ich ja aber nicht ahnen konnte, wo ich so vor dem Anlieger frei-Schild stand und nicht wusste was tun)? Sprich, wenn man nicht genau weiß, ob man ein Anlieger ist oder nicht, eine gewisse Wahrscheinlichkeit besteht, darf man dann reinfahren oder nicht? --87.140.194.3 21:58, 15. Apr. 2016 (CEST)

Unser Artikel Anlieger führt sinngemäß aus, dass die Absicht ausreichend ist. --Studmult (Diskussion) 22:10, 15. Apr. 2016 (CEST)
Du darfst rein, wenn du einen Anwohner besuchen willst (= Absicht); du darfst nicht rein, wenn dein Bekannter da nicht wohnt. Die Unsicherheit geht zu deinen Lasten. Freund besuchen = erlaubt; Freund suchen, der woanders wohnt = nicht erlaubt. --Snevern 22:12, 15. Apr. 2016 (CEST)
Hast Du einen Quellenbeleg für diese Behauptung? So auf die Schnelle hat mein Google dazu weder in Kommentaren noch in Beschlüssen irgendetwas gefunden. So lange er da nicht ständig durchfährt oder gemütlich parkt, dürfte das AFAICS unschädlich sein. --84.59.22.29 23:59, 15. Apr. 2016 (CEST)
Es gibt ein Urteil, dass der Anlieger dort wohnen, man ihn aber nicht zwingend treffen muss. Wenn man sieht, dass z.B. sein Auto und damit er nicht da ist, kann man weiter durch fahren. Man sollte also einen Namen kennen, der da wohnt. Oder man schaut(e) auf ein Grundstück, weil man dem Eigentümer ein Angebot zum Kauf machen will. Der Fantasie zu "Anlüger Frei" sind keine Grenzen gesetzt. Die Polizei vor Ort lässt sich selten an der Nase rumführen, weil sie nachprüfen kann und ob man der schriftlichen Zahlungsaufforderung wegen 20 Euro "Durchfahrtsgebühr" auch schriftlich falsch entgegnet, muss jeder mit sich selbst ausmachen. Ich habe das weit kribbligere Spiel nicht erwischt zu werden bisher immer gewonnen. So oft spiel ich das allerdings nicht.--2003:75:AF52:1C00:A9D8:F642:2E:22DC 07:20, 16. Apr. 2016 (CEST)
Nein, ich habe keine Quelle dafür zur Hand, IP 84.59.22.29. Ich habe lediglich die Rechtslage wiedergegeben, die nicht nur für diese Grenze zwischen erlaubt und verboten gilt, sondern ganz grundsätzlich.
Die Polizei hält im Normalfall keinen an, wenn er irgendwo reinfährt, um ihm ein Knöllchen zu verpassen, sondern sie macht Nummernschildvergleiche: Wer fährt rein, und wer davon fährt gleich drauf auf der anderen Seite wieder raus. Nur die werden belangt, und wer von denen dann jemanden nennen kann, zu dem er wollte, der aber leider nicht da war, der kommt damit im Regelfall erstmal durch. Trotzdem schrecken diese Kontrollen die meisten Durchfahrer ab, die dann lieber den erlaubten Umweg nehmen, und damit hat die Polizei ihr Ziel erreicht.
Ich habe sehr viele sehr nette und verständnisvolle Polizisten kennen gelernt. Wer auf so einen trifft und nett bleibt, der kriegt wahrscheinlich auch keinen Strafzettel, wenn er sagt, dass er auf der Suche nach Herrn Dr. Klöbner ist, der irgendwo hier in der Nähe wohnen muss - selbst dann nicht, wenn der Polizist weiß, dass Dr. Klöbner in der Parallelstraße wohnt, die außerhalb der Anwohnerzone liegt. --Snevern 09:14, 16. Apr. 2016 (CEST)
Siehste mal, da werden wir uns ja doch noch einig:) Es ist alles eine Sache des Ermessensspielraums. --94.219.9.168 09:45, 16. Apr. 2016 (CEST)
Wieso - waren wir uns denn nicht einig!? Nicht, dass das von Belang wäre, aber mir war ein Dissenz mit einer IP 94.219.9.168 in dieser Frage bislang nicht bewusst. --Snevern 12:54, 16. Apr. 2016 (CEST)
Dein erster Beitrag war in seiner Absolutheit irreführend. Der zweite Beitrag hat die Aussage des ersten angemessen relativiert. Gesetzestexte sind das eine, deren Auslegung in der rechtlichen Praxis das andere und neben der Rechtsanwendung gibt es sogar noch die Rechtsfindung. --94.219.9.168 15:19, 16. Apr. 2016 (CEST)
Danke für die Aufklärung! Wenn wir dich nicht hätten... --Snevern 16:26, 16. Apr. 2016 (CEST)
… hätte Dein Ego mehr Platz, um sich auszubreiten. Schon klar;) --94.219.9.168 19:30, 16. Apr. 2016 (CEST)
Neeeein, da irrst du: Du stehst meinem Ego nicht im Wege. Aber danke für die humorige Einlage! --Snevern 19:57, 16. Apr. 2016 (CEST)
Ach wer redet denn von im Weg stehen, das sind bloß Präventivmaßnahmen, damit hier niemandem das Ego zu sehr anschwillt. Wir wollen ja nicht, daß noch ein Unglück passiert.;) --84.59.232.210 11:17, 17. Apr. 2016 (CEST)

Alter Fall des Staatsrechts: Reiseverbot in den Ostblock

Hallo, ich erinnere mich einen Fall gehört zu haben, in dem deutsche Gerichte einem Bürger die Reise in die Staaten des früheren Ostblocks untersagten. Der Mann in Frage stellte sich auf Marktplätze oder ähnliches und gab lautstarke anti-kommunistische Vorträge. Die Richter begründeten ihren Entscheid damit, dass die Sicherheit des deutschen Staats in diesem Fall individuelle Rechte überwiegt.

Ist dieser Fall historisch belegt?

--92.250.199.120 13:55, 16. Apr. 2016 (CEST)

Hört sich zumindest eigenartig an. Was wollte er als Anti-Kommunist denn im Ostblock? Waren die Ostblockstaaten nicht ebenso daran interessiert, Querulanten draussen zu halten? Die mussten ja nur das Visum verweigern.--Optimum (Diskussion) 15:08, 16. Apr. 2016 (CEST)
Da wird es sicherlich einiges gegeben haben. Wobei sich natürlich die Frage stellt, ob dt. Gerichte jemandem überhaupt die Einreise in einen anderen Staaten verbieten können. Sie können sicher die Ausreise gerichtlich untersagen, er könnte aber über einen Dritt-Staat in besagte Länder einreisen (falls diese ihm die Einreise nicht verbieten). Es könnte sein, dass das Gericht eine komplettes Ausreise-Verbot ausspricht. Wenn jemand hochgradig vorbestraft ist, kann ich mir das durchaus vorstellen. Was die Sicherheit des dt. Staates betrifft, naja, er wird doch als Privatperson reisen und eine eigene Meinung haben und nicht den Staat vertreten.--87.184.131.234 15:19, 16. Apr. 2016 (CEST)
Hier werden Ausreiseverbote im Zusammenhang mit dem G8-Gipfel in Genua 2001 erörtert. --Vsop (Diskussion) 17:41, 16. Apr. 2016 (CEST)
Wilhelm Elfes ist mir zumindest bekannt, der durfte nicht ausreisen, das ging bis vors BVerfG. Passt aber wohl von der Handlung her nicht ganz. -- 217.236.175.4 22:54, 16. Apr. 2016 (CEST)
Für Bundeswehrangehörige gab es während des Kalten Krieges ein Verbot, in den Ostblock zu reisen. Das betraf unter anderem meinen Bruder, der während seiner Wehrdienstzeit seine in Sachsen gebliebenen Verwandten nicht besuchten durfte, aber auch an einem Familienurlaub mit Österreich–Ungarn–Jugoslawien–Griechenland-Rundreise nicht teilnehmen konnte. --Rôtkæppchen₆₈ 00:24, 17. Apr. 2016 (CEST)
Ja, das galt auch für verbeamtete Geheimnisträger. Ich vermute, der Fragesteller hatte von dem Fall Elfes gehört. Ansonsten scheint mir alles Wesentliche inklusive der Rechtsgrundlagen und Grundsatzurteile bereits in dem verlinkten Artikel von Helle Martensen für den Arbeitskreis kritischer Juristinnen und Juristen an der Humboldt-Universität zu Berlin zusammengefasst. --2003:45:4666:4E00:F82C:D997:F726:C1E7 00:56, 17. Apr. 2016 (CEST)
Das galt sogar für die Angehörigen von Geheimnisträgern. Unsere Klasse durfte damals (1986 rum) nicht nach Berlin, weil einige Väter in der örtlichen Kaserne geheime Dinge taten und wir nicht durch die DDR fahren durften. --Sr. F (Diskussion) 15:54, 17. Apr. 2016 (CEST)

Bass-Riff bei Raabs Einfahrt zur Crash Challenge

Moin! Eine Frage wie gemacht für den Samstag Abend: Aus welchem Song stammt nochmal das Metal-Bass-Solo, zu dem Stefan Raab gerne bei seinen Shows – vor allem bei der Stock Car Crash Challenge – einfuhr? Nein, ich meine nicht Drop Dead von Rage, das auch gern zum Einsatz kam. Herzlichen Dank! —[ˈjøːˌmaˑ] 18:29, 16. Apr. 2016 (CEST)

Moin! Einen Link hast Du nicht zufällig für uns parat? Evtl. findet sich in den YouTube-Beiträgen zur StockCarChallenge eine Szene mit dem Stück. --Blutgretchen (Diskussion) 11:07, 17. Apr. 2016 (CEST)

Grammatikfrage

Hallo Liebe Wikipedianer! Eine kurze Grammatikfrage: "Diese Messung dient zum Nachweis eines Hämatoms als indirektes / indirektem Zeichen einer Fraktur" Welches Kasus nimmt das adjektiv "indirekt" hier an? Liebe Grüße! 87.155.53.220 20:03, 16. Apr. 2016 (CEST)

Richtig wäre "als indirekten Zeichens", da kongruent zum Genitiv davor. Grüße Dumbox (Diskussion) 20:13, 16. Apr. 2016 (CEST)
Der Nominativ ist ebenfalls zulässig. —[ˈjøːˌmaˑ] 20:16, 16. Apr. 2016 (CEST)
Das stimmt. Leider. ;) Grüße Dumbox (Diskussion) 20:19, 16. Apr. 2016 (CEST)
Also sind sowohl "Nachweis eines X als indirektes Zeichen" (Nom) sowie "Nachweis eines X als indirekten Zeichens" (Gen) korrekt? (Stehe wohl auf dem Schlauch!) 87.155.53.220 20:55, 16. Apr. 2016 (CEST)
...da kongruent zum Genitiv davor. Kann das mal einer erklären? Grüße! --84.186.127.134 20:57, 16. Apr. 2016 (CEST)
Am besten der Verursacher. Ein Hämatom ist ein Zeichen. Gleichsetzung, daher beide Wörter im Nominativ. Wir haben es mit dem Hämatom als einem Zeichen zu tun: Jetzt stehen beide im Dativ. Das ist der Nachweis eines Hämatoms als eines Zeichens: Beide im Genitiv. Es setzt sich aber durch, dass hier, wenn der Artikel fehlt, der Nominativ bevorzugt wird: Nachweis eines Hämatoms als Zeichen (statt Zeichens). Grüße Dumbox (Diskussion) 21:03, 16. Apr. 2016 (CEST)
BK BK Entschuldiung! Habe mich verlesen! --84.186.127.134 21:04, 16. Apr. 2016 (CEST)
Und Dank an Dumbox! Besser erklärt als ich es könnte (ich hatte Dativ im Kopf als ich schrieb. Warum? - ? --84.186.127.134 21:17, 16. Apr. 2016 (CEST)
Das hat wohl mit dem Phänomen des "dativus appositionis" zu tun. Es gibt seit einigen Jahren eine Tendenz, bei anderen Kasus als dem Nominativ instinktiv zum Dativ zu greifen: mit Horst Seehofer, dem Vorsitzenden der CSU (richtig), durch Horst Seehofer, dem Vorsitzenden der CSU (falsch), wegen Horst Seehofers, dem Vorsitzenden der CSU (falsch). Grüße Dumbox (Diskussion) 21:33, 16. Apr. 2016 (CEST)
Der Dativ ist dem Genitiv sein Tod! --62.202.182.81 22:21, 16. Apr. 2016 (CEST)
Alles endet im beliebtesten Obliquus – die Romania hat’s uns vorgemacht! :) —[ˈjøːˌmaˑ] 22:55, 16. Apr. 2016 (CEST)
Und das war? Der Ablativ? Oder doch der Akkusativ (ohne das m natürlich)? ;) Grüße Dumbox (Diskussion) 23:06, 16. Apr. 2016 (CEST)
Das sollten wir unbedingt klären, bevor uns wie unseren Freunden von der Insel auch der letzte Kasus den Präpositionalausdrücken zum Opfer gefallen ist! :D Gute Nacht! —[ˈjøːˌmaˑ] 23:55, 16. Apr. 2016 (CEST)

Die Frage ist unvollständig: Ist Dieses oder Jenes gemeint? --84.135.133.155 11:11, 17. Apr. 2016 (CEST)

Siehe hier. Etvl. ist auch der Dativ angemessen (aber eher nicht in Hochsprache). --Chricho ¹ ² ³ 16:23, 17. Apr. 2016 (CEST)

Gesetzentwurf oder Gesetzesentwurf

In Wikipedia gibt es den Artikel Gesetzentwurf. Im Online-Duden gibt es Gesetzentwurf und Gesetzesentwurf, wobei erstere Form als etwas häufiger gekennzeichnet ist. Ansonsten finden sich im Netz beide Versionen in offiziellen Dokumenten der Bundesregierung und des Bundestages. Ist es wirklich so, dass beide Formen richtig und gleichermaßen verwendbar sind, oder ist es doch eher so, dass eine der beiden Versionen besser oder korrekter ist. Und kann es sein, dass eine Version eher in Österreich oder der Schweiz bevorzugt wird? Danke für Eure Hilfe. --190.148.219.53 02:00, 17. Apr. 2016 (CEST)

Ohne die Fragen beantworten zu wollen, liefere ich hier nur einen Link auf Google Ngram Viewer. Demnach ist Gesetzentwurf eindeutig die häufigere Form. --Rôtkæppchen₆₈ 02:12, 17. Apr. 2016 (CEST)
Nicht nur das: Der Informationsinhalt ist derselbe - bei zwei Buchstaben und einer Silbe weniger! Also hat "Gesetzentwurf" eine höhere Infodichte. Play It Again, SPAM (Diskussion) 13:19, 17. Apr. 2016 (CEST)

unbekannte Emailadressen bei Anmeldevorgang als Vorschlag

Beim Anmeldevorgang bei Amazon passierte eben Folgendes: Mir wurden diverse Emailadressen vorgeschlagen, die mir völlig unbekannt sind (und von verschiedenen Providern gmail, yahoo, web.de). Diese Emailadressen habe ich definitiv nicht in meinen Browserverlauf (Chrome) eingegeben. Was ist hier passiert? (Wird anderen Nutzern von Amazon auch meine Emailadresse angezeigt? Eigentlich ist doch der Speicherort mein Browser ..., oder?) --78.54.172.39 11:15, 17. Apr. 2016 (CEST)

Zur letztem Frage: dein Browser ist EIN Speicherort für diese Daten, aber nicht der Einzige. Gerade Chrome speichert AFAIK standardmäßig Anmeldedaten in der Cloud auf Google-Servern (zur eingesetzten Verschlüsselung kann ich nichts sagen). Grüße, Grand-Duc (Diskussion) 11:39, 17. Apr. 2016 (CEST)

Kaisermühlen Blues: Joschi Täubler

Servus,

kann mir jemand sagen, in welcher Folge Joschi „I glab du stehst a bissl auf mi“ oder „I glab Sie stengan a bissl auf mi“ sagt? Danke --194.118.154.13 13:31, 17. Apr. 2016 (CEST)

Korvettenkapitän vs. Lieutenant Commander

Hallo! Der Dienstgrad Korvettenkapitän entspricht einem Lieutenant-Commander der US-amerikanischen oder der britisch-königlichen Marine. In der englischen Übersetzung soll der Korvettenkapitän im NATO-Rangsystem dennoch mit Commander (Junior grade) übersetzt werden.Der Lieutenant Commander, ist der erste Stabsoffizierdienstgrad und somit das besoldungstechnische Äquivalent zum Korvettenkapitän der Deutschen Marine. Aufgrund der historischen Entwicklung des Dienstgrades kann man den LCdr jedoch nicht direkt mit dem Korvettenkapitän vergleichen, da er eher ein Lieutenant in herausgehobener Dienststellung als der Stellvertreter eines Commanders ist und hat auf seegehenden Einheiten Dienststellungen der Art inne, wie sie in der Deutschen Marine eher Kapitänleutnanten übertragen werden. Eine Dienststellung bezeichnet die tatsächliche Eingliederung eines Soldaten in die Befehls- und Kommandostruktur. Das habe ich ja alles soweit verstanden,aber wenn ein Lieutenant Commander auf seegehenden Einheiten Dienststellungen wie eher ein deutscher Kapitänleutnant hat heißt das dass ein deutscher Korvettenkapitän dienststellungsmäßig über einem Lieutenant Commander steht und auch in der Kommando u.Befehls Struktur?

--2A02:2450:1189:228:8514:3014:B984:4CA4 13:58, 17. Apr. 2016 (CEST)

Hallo, ich habe leider deine Frage nicht genau verstanden. Du versuchst aber anscheinend militärische Dienstgrade zu vergleichen. Dienstgrade werden aber unterschiedlich aufgeteilt: Geschichtlich, nach Ländern, Truppengattungen, Besoldungsgruppen, Verwendungen usw., dazu kommen noch verschiedene (teils lange) Traditionen einzelner Ränge. Alle Vergleiche hinken zwangsläufig. Der NATO-Rangcode ist mMn damit auch keine Grundlage für einen exakten Vergleich verschiedener Dienstgrad-systeme. Er beanwortet nur die Frage was passieren würde wenn z.B ein deutscher Fregattenkapitän und ein britischer Lieutenant Commander sich auf einer Insel treffen nachdem beide schiffbrüchig wurden: Man vergleicht den Rang-Code und der Fregattenkapitän hat Befehlsgewalt. Wenn du Unklarheiten bei einem Artikel in der Wikipedia entdeckt hast, sag uns bitte welchen Artikel du meinst. Gruß, --Nfreaker91 17:54, 17. Apr. 2016 (CEST)

Testen, ob Erhitzen durch Mikrowellen erfolgt

Hallo,

ich habe einen Mikrowellenofen. In dessen Anleitung steht eine Tabelle mit den diversen Erhitzungsprogrammen (Garen, Kombigaren, Grill etc.) und den durchaus erlaubten Zusatzteilen, die pro Modus deutlich variieren. Also habe ich anscheinend ein Kombi-Gerät, das mal mit, mal ohne Mikrowellen arbeitet. Nun möchte ich ein Spargelgericht in Alufolie zubereiten (ohne ist keine Alternative). Daher meine Frage: Wie kann ich auf sichere Art und Weise testen, ob ein bestimmter Zubereitungsmodus (Umluft) mit Mikrowellen oder auf herkömmliche Art und Weise funktioniert? Vielen Dank im Voraus, --217.237.164.210 10:13, 16. Apr. 2016 (CEST)

Darf man erfahren, welches Gerät du hast? Play It Again, SPAM (Diskussion) 10:16, 16. Apr. 2016 (CEST)
LG Modell-Nr. MC8088HL --217.237.164.210 10:23, 16. Apr. 2016 (CEST)
Alufolie in der Mikrowelle: Da wird nur die Folie erhitzt (was nicht ungefährlich ist!), die Mikrowellen gehen nicht durch! Die sehr heiß werdende Alufolie erhitzt das, was darin befindlich ist, nur auf herkömmliche Weise (Wärmeleitung). Gruß -- Dr.cueppers - Disk. 10:40, 16. Apr. 2016 (CEST)
Deshalb interessiert es mich ja, ob die Umluft-Funktion über Mikrowellen funktioniert. Ansonsten muss ich mir eine Alternative überlegen. Gruß, --217.237.164.210 10:59, 16. Apr. 2016 (CEST)
Erster Schritt: Was sagt die Gebrauchsanweisung zu "Aluminium(papier)" oder "Metall" im DuoChef?
Suche zu => Spargel Aluminium(folie) Mikrowelle <= bringt keine nutzbaren Resultate (= "ist ungewöhnlich")
Spargel wird in Mikrowelle meist in dazu geeigneten Behältern gegart. Gefaltetes Backpapier (ohne Gewähr) könnte eine Alternative sein.
Selbstversuch (ohne Gewähr): Ein Schnipsel Alupapier in die Mikrowelle - und beobachten (nochmal: ohne Gewähr). Play It Again, SPAM (Diskussion) 11:10, 16. Apr. 2016 (CEST)

Bedienungsanleitung gibt es hier http://www.lg.com/de/support/service-produkt/lg-MC8088HL --Engie 14:19, 16. Apr. 2016 (CEST)

Umluft-Garen
Drücken Sie auf STOP/CLEAR (STOP/LÖSCHEN) .
Zum Umluft-Garen müssen das Backblech bzw. das Metallrost
verwendet werden.
Drehen Sie den Regler MODE (MODUS), um den Umluft-Ofen
auszuwählen.
Es wird das folgende Symbol angezeigt: “ ” [Ventilator/Propeller]
Drehen Sie den Regler TIME/WEIGHT (ZEIT/GEWICHT), bis die Anzeige
“230°C” erscheint.
Drücken Sie zur Bestätigung der Temperatureinstellung die Taste
TIME/WEIGHT (ZEIT/GEWICHT).
Drehen Sie den Regler TIME/WEIGHT (ZEIT/GEWICHT), bis die Anzeige
“50:00” erscheint.
Drücken Sie auf START/QUICK START (START/SCHNELLSTART).

--Vsop (Diskussion) 15:16, 16. Apr. 2016 (CEST)

Die Umluftfunktion kannst Du in Kombigeräten genauso benutzen, wie in einem Backofen. Mach halt nicht den Fehler, das Programm "Mikrowelle + Umluft" zu aktivieren, sondern nutze das reine Umuft- oder das Ober-/Unterhitzeprogramm. --94.219.9.168 15:25, 16. Apr. 2016 (CEST)

Nimm statt Alufolie gebuttertes Pergamentpapier. Kannst du ähnlich wickeln, nur solltest du es noch mit Küchengarn wie ein Päckchen verschnüren. Rainer Z ... 16:17, 16. Apr. 2016 (CEST)
Warum sollte er das in der Umluftfunktion tun? --94.219.9.168 19:37, 16. Apr. 2016 (CEST)
Üblicherweise macht so ein Kombigerät in den verschiedenen Betriebsarten verschiedene Geräusche, weil eben auch verschiedene Komponenten in Betrieb sind. Im Mikrowellenbetrieb bei maximaler Leistungsstufe hört man das Brummen des Transformators für das Magnetron, den Kühllüfter für Magnetron und Trafo und den Drehteller. Bei Betrieb mit etwa halber Leistung hört man abwechselnd zwei Geräusche. Das eine ist Trafo, Kühllüfter und Drehteller, das zweite ist nur Kühllüfter und Drehteller. Im Umluftbetrieb hört man kein Trafobrummen, dafür aber zusätzlich das Umluftgebläse. Wenn man diese Geräusche auseinanderhalten kann, kann man selbst darauf kommen, ob das gewünschte Automatikprogramm mit oder ohne Mikrowellenbetrieb ist. --Rôtkæppchen₆₈ 19:42, 17. Apr. 2016 (CEST)

Abrupter Klimawandel?

[17] ,[18],[19]. Irgendwie scheint alles schneller und extremer abzulaufen, wenn man den Meldungen glaubt. Aber kann man sagen wir erleben die Anfänge eines abrupten Klimawandels? Danke für eure Inputs. --Baumkletterer (Diskussion) 14:50, 16. Apr. 2016 (CEST)

nö, Klima wird über viele viele Jahre ermittelt... das ist wie mit nem Würfel, der schon dreimal hintereinander ne 1 gezeigt hat (da weiß man auch noch nich ob er gezinkt ist)... --Heimschützenzentrum (?) 14:58, 16. Apr. 2016 (CEST)
Es wird immer mehr untersucht und publiziert. Das erweckt den subjektiven Eindruck, es würde immer mehr immer schneller passieren, wodurch die Nachfrage nach Publikationen weiter anwächst, was wiederum den subjektiven Eindruck vermehrt usw. --94.219.9.168 (15:37, 16. Apr. 2016 (CEST), Datum/Uhrzeit nachträglich eingefügt, siehe Hilfe:Signatur)
Das Klima wird zwar über viele (30, um genau zu sein) Jahre ermittelt. Aber was nutzt diese Definition, wenn wir jetzt ein paar extreme Jahre erleben (ähnlich wie 1998 - das wirklich extrem war, zumindest im Sinne der globalen Temperatur) - aber ohne Aussicht aufs Ende? Wird man erst in 10 Jahren einen stattgefundenen Klimawandel konstatieren?--Alexmagnus Fragen? 16:40, 16. Apr. 2016 (CEST)
Da sind eigentlich zwei Fragen. Zum einen: Wenn es darum geht, ob das reale und tatsächliche Klima sich gerade jetzt schneller verändert, wird das nur in der Rückschau zu beantworten sein, weil das Datenrauschen zu stark ist. Man kann mittels Bayesscher Statistik Schwellen definieren, und kommt dann auf Wahrscheinlichkeiten für beobachtete Extremereignisse. Aber auch unwahrscheinliche Ereignisse können zufällig mal eintreten (wobei nach jedem die Wahrscheinlichkeit für Zufall weiter absinkt), sicher weiß man´s dann aber erst hinterher. Zum anderen: Aus der Mechanik des Klimas errechnen Klimaforscher Schwellenwerte, von denen an sich der bisherige Trend nicht mehr linear fortsetzt, sondern das System in einen neuen Zustand wechselt. Dies wird in der Regel als "tipping point" umschrieben, ein halbwegs aktuellen review etwa von Timothy M. Lenton: Early warning of climate tipping points. Nature Climate Change 2011, doi:10.1038/NCLIMATE1143. Tipping point bedeutet z.B.: bei Erwärmung über einen (im Detail umstrittenen) Schwellenwert wird der grönländische Eisschild als Ganzes instabil. Während vorher bei Erwärmung um einen bestimmten Betrag eine dazu proportionale Eismenge taute, taut nun mehr oder weniger der ganze Rest auf einmal. Das Dumme daran ist: Rückgängig nach dem Überschreiten eines tipping point geht nicht mehr.--Meloe (Diskussion) 19:11, 16. Apr. 2016 (CEST)
Siehe Kleine Eiszeit im Mittelalter (1570 bis 1630 und von 1675 bis 1715) als historisches Beispiel für extreme Klimaschwankungen. --Kharon 03:47, 18. Apr. 2016 (CEST)

Monitorhalterung - welcher Monitor?

Hallo, ich würde einen Monitor gerne per Monitorhalterung am Schreibtisch befestigen, nicht mit den üblichen Standfüßen. Aber auf was muss ich da bei Monitorkauf achten? Oder ist quasi jeder Monitor mit so einer üblichen Monitorhalterung kompatibel? --87.140.195.2 18:26, 16. Apr. 2016 (CEST)

VESA-kompatibel dürfte das Zauberwort sein. --Eike (Diskussion) 18:30, 16. Apr. 2016 (CEST)
en:Flat Display Mounting Interface beschreibt die vielfältigen Möglichkeiten dieses Standards. --Rôtkæppchen₆₈ 22:26, 16. Apr. 2016 (CEST)
Bei den sehr günstigen Modellen wird häufig auf eine VESA-Halterung verzichtet weil dies zu einen entsprechend aufwändigen Standfuß zwingt den man da eben gerne einspart um noch ein paar € billiger zu sein. Gute "Preissuchmaschinen" wie z.B. bei http://www.heise.de/preisvergleich/?cat=monlcd19wide bieten "VESA-Halterung" als Auswahlbedingung für die Suche nach dem Wunschmonitor. --Kharon 03:37, 18. Apr. 2016 (CEST)

Die DGE empfiehlt

Zugegebenermaßen 2003, aber noch immer in unseren Artikeln verlinkt (Ich dachte ich hab den Link aus Pflanzenöle, aber das wars nicht), gesättigte Fette nicht zum Braten und Fritieren zu nutzen, weil "deren Oxidationsanfälligkeit sehr groß" sei. Wat?? Also Palmin, Kokosfett, Palmöl (unser Artikel: Dabei wird Palmöl aufgrund seiner ausgezeichneten Hitze- und Oxidationsstabilität [...] zum Kochen, Braten und Frittieren eingesetzt) und Ghee sind schlecht fürs Braten? Dachte man das 2003 wirklich und wie sieht das heute aus? --92.202.112.15 16:31, 17. Apr. 2016 (CEST)

Ich glaube nicht, dass man das tatsächlich dachte. Da ist irgendetwas durcheinandergeraten, denn gerade gesättigte Fette zeichnen sich durch hohen Rauchpunkt und Oxidationsstabilität aus. Der Fettsäureanteil von Palmöl besteht zu fast der Hälfte aus Stearinsäure. --Rôtkæppchen₆₈ 21:33, 17. Apr. 2016 (CEST)

Notebooktastatur kaputt

Gestern die Tasturtur meines Notebooks feucht abgewischt wie jeden Samstag, und seitdem funktioniert sie nicht mehr. Das gerät ist grade mal 4 Monate alt. Ist das ein Garantiefall? Ich mein, so eine Tastatur sollte doch mehr aushalten als etwas zu feucht abgewischt zu werden, etwa wenn man ein Getränk verschüttet. Wenn ja, wie läuft das ab. Ausgebaut habe ich sie schon (sonst könnte ich hier gar nichts schreiben). Kann ich die Tastatur einzeln einschicken, oder muss es das ganze Gerät sein? Zwei Wochen ohne Rechner vs. 50 € Lehrgeld...--Antemister (Diskussion) 20:36, 17. Apr. 2016 (CEST)

Schau mal in die Anleitung, da steht bestimmt was über Wasser drin... --Eike (Diskussion) 20:39, 17. Apr. 2016 (CEST)
+1 – Doku fragen. Mir ist mal Tee in meine Thinkpadtasterthur geschwappt (die einklich auch einiges aushalten sollte), anschließend ging die Cursor-Rechts-Taste nicht mehr (man glaubt vorher nicht, wie oft man die braucht, und es war ausgerechnet am Anfang des Urlaubs). Neues Schlüsselbrett kostete dann 25 Euro in der Strombucht. – Ohne genau zu wissen, um welches Gerät es sich bei dir handelt, ist jede präzisere Aussage Kaffeesatzleserei --Kreuzschnabel 20:43, 17. Apr. 2016 (CEST)
Vergaß es: Ist ein Medion Akoya E7225[20]. Ob Garantiefall oder nicht dürfet aber vom Hersteller unabhängig sein.--Antemister (Diskussion) 20:48, 17. Apr. 2016 (CEST)
Sowas ist normalerweise kein Garantiefall. Hol Dir die Ersatztastatur und schau bei Youtube nach, wie man die einbaut. Bei meinem Samsung NP-N140 musste ich drei Schrauben lösen und das wars. Allerdings hatte ich zuerst die falsche Tastatur, von der der Händler behauptete, sie würde auch für den NP-N140 passen. Tat sie aber nicht. Amazon hat mir aber problemlos das Geld erstattet. Hab dann bei einem anderen Händler gekauft. Ich hatte die Tastatur bei einem Reinigungsversuch mit Fensterreiniger getötet. --Rôtkæppchen₆₈ 21:18, 17. Apr. 2016 (CEST)
Ich hätte eher dazu geraten, gleich ’ne Externe USB-Tastatur anschließen zum Weiterarbeiten und dann die Garantiehotline anrufen, statt selbst am Gerät herumzuwerkeln. Mit viel Glück kommt man dann an einen kompetnten Ansprechpartner, der einen nicht erst mit Standard-Blabla à la „versetzen Sie den Laptop in den Auslieferungszustand“ abspeist, sondern fachgerecht ausschließt, dass es nicht vielleicht doch nur ein Einstellungs- oder Treiberproblem ist (z.B. wäre ja denkbar, dass die interne Tastatur beim Abwischen bloß deaktiviert wurde). Sollte sich aber wirklich ein Hardwaredefekt herauskristallisieren, wäre ich mir ziemlich sicher, dass du für die Reparatur nichts bezahlen müsstest. Juristisch vielleicht schon, sofern du das Ding tatsächlich mit zuviel Feuchtigkeit gekillt hast, aber das müsste man dann schon sehr offensichtlich sehen (Stichwort Beweislastumkehr in den ersten 6 Monaten der ges. Gewährleistung) - und selbst wenn: ich musste mal einen recht neuen Medion-Rechner einsenden, den ich selbst beschädigt (fallengelassen) hatte, er wurde trotzdem kostenfrei repariert. Allerdings wirst du in aller Regel das Ding komplett einsenden und 1-2 Wochen drauf verzichten müssen (wobei ich zumindest darauf bestehen würde, vorher die Festplatte auszubauen), sprich, selbst Ersatz kaufen und einbauen kann unter’m Strich die bessere Wahl sein (sofern man sich das zutraut) und erspart einem auf jeden Fall das übliche Generve mit Hotlines.--Mangomix 🍸 01:31, 18. Apr. 2016 (CEST)
Ach ja, ob Garantiefall oder nicht hängt sehr wohl (und zwar ausschließlich) vom Hersteller ab. Denn im Gegensatz zur gesetzlichen Gewährleistung (die man gegenüber dem Verkäufer hat), ist eine Herstellergarantie freiwillig und kann sehr unterschiedlichen Bedingungen unterliegen. In der Praxis geht sie aber meist deutlich über die gesetzliche Gewährleistung hinaus, ausgeschlossen sind erfahrungsgemäß wirklich nur die Fälle, wo der Kunde den Schaden ziemlich offensichtlich selbst verursacht hat.--Mangomix 🍸 01:49, 18. Apr. 2016 (CEST)

Ist ein Bagger geerdet?

Hi!ä Ist es für den Bagger-Fahrer eigentlich gefährlich, wenn seine Schaufel ein Stromkabel berührt, nachdem die Isolierung abgeschabt wurde? Oder ist der geerdet? (Ich habe aber noch nie ein Kabel vom Bagger zum Hydranten oder so gesehn...) Oder ist der Bagger-Fahrer von der Schaufel isoliert? Thx. Bye Arne --Heimschützenzentrum (?) 22:40, 12. Apr. 2016 (CEST)

Es kommt auf das Fahrgestell an. Gummireifen sind schlechte Isolatoren. Ein Raupenfahrwerk gibt eine wenn auch schlechte Erdung ab. Üblicherweise ist die Fahrerkabine aber aus Metall, sodass sich außer bei sehr hohen Strömen keine Potentialdifferenz im Bereich der Fahrerkabine ergeben kann. Es besteht gefahr, wenn der Baggerführer gleichzeitig seinen Bagger und Gegenstände oder den Erdboden außerhalb der Fahrerkabine berührt. Es ist daher ratsam, von einem die Stromleitung berührenden Bagger abzuspringen, um nicht beim normalen Absteigen die Beine durch eine Schrittspannung gekitzelt zu bekommen. Dito bei entgleisten elktrischen Bahnen. --Rôtkæppchen₆₈ 22:46, 12. Apr. 2016 (CEST)
Berührt denn ein Baggerfahrer im Normalbetrieb überhaupt Metall? Die ganze Kabine ist doch Kunststoff, und da bräuchte es schon Hochspannung... Bzw. dass Bagger Kabel durchtrennen ist doch ganz normal - sind da entsprechende Unfälle bekannt?--Antemister (Diskussion) 23:00, 12. Apr. 2016 (CEST)
Ja, es sind entsprechende Unfälle bekannt. Shit happens. Ich vermute, vor allem bei Baggerfahrern, die nach Erfassen einer Stromleitung beim Absteigen vom Bagger sowohl diesen als auch den Boden berühren. --217.86.69.141 21:56, 15. Apr. 2016 (CEST)
1. meinst du „Gummireifen = gute Isolatoren“? 2. aber ist nicht bei Wechselspannung das Aufladen auf das Maximum und das anschließende Entladen auf das Minimum ausreichend für n HI (=Herzinfarkt)? besonders wenn man gegen ne 110kVeff Leitung kommt? 3. aber in den Zeitungen steht scheinbar immer, dass die Baggerfahrer unbeschadet davon gekommen sind... nur die Kollegen, die draußen in der Nähe stehen, brauchen wohl manchmal einen „Helo“... --Heimschützenzentrum (?) 23:08, 12. Apr. 2016 (CEST)
Ein Stromunfall löst per se keinen Herzinfarkt aus, sondern stört das Reizleitungssystem des Herzens. Ursache für einen Herzinfarkt ist eine Durchblutungsstörung des Herzmuskels z.B. durch Verstopfung eines Herzkranzgefäßes. Das kann zwar zufällig auch im Rahmen eines Stromunfalls auftreten, aber hängt nicht ursächlich damit zusammen. Zu den Umstehenden: siehe Schrittspannung. Der Helikopter wird eher "Heli" abgekürzt, aber das mag was persönliches sein, bitte. Ein Helikopter ist aber kein zwangsweises Rettungsmittel bei einem Stromunfall. --217.86.69.141 21:56, 15. Apr. 2016 (CEST)
Nein. Durch den Kohlenstoffanteil sind Autoreifen schlechte Isolatoren. Energienetze über 1000 Volt Nennspannung sind üblicherweise nicht geerdet, sondern gelöscht, sodass Kontakt mit einem einzelnen Leiter ungefährlich ist, sofern keine kapazitiven Blindströme fließen. Es gibt aber Ausnahmen. Bahnstromnetze haben 15000 bis 25000 Volteff, sind aber üblicherweise geerdet, ebenso ältliche Fernsehapparate mit Allstromchassis, in denen bis zu 25000 Voltabs gegen Erde anliegen können. --Rôtkæppchen₆₈ 01:51, 13. Apr. 2016 (CEST)
Warum ist ein Raupenfahrwerk ein schlechter Isolator? --Ip80.123 (Diskussion) 02:27, 13. Apr. 2016 (CEST)
Es besteht hauptsächlich aus Stahl, einem Werkstoff, dessen Bestandteile fast alle elektrisch leitend sind. --Rôtkæppchen₆₈ 02:30, 13. Apr. 2016 (CEST)
Während meiner Zeit in der gymnasialen Oberstufe wurde die Straßenbahnlinie durch den Ort der Schule im laufenden Betrieb komplett umgebaut. Dabei wurden lose, nachlässig verlaschte Behelfsgleise verlegt. Es kam vor, dass die Haltestelle je nach Baustellenfortschritt jeden Tag woanders war. Ebenso wurde eine Behelfsendhaltestelle mit Wendeschleife eingerichtet, an der es häufig zu Entgleisungen kam. seither weiß ich, wie man sich bei entgleisten elektrische Bahnen verhält. --Rôtkæppchen₆₈ 02:54, 13. Apr. 2016 (CEST)
  • Hier kommt es wirklich auf die Spannung und Stromstärke an (und wie das angebaggerte Kabel abgesichert ist). Aber generell gilt, solange der Fahrer sich in der Kabine (mit einschränkunen auf offener Bedienstand) befindet ist es (relativ) sicher (Stichwort Faradayscher Käfig). Ausserhalb kommt es eben wirklich darauf an ob eine gefährliche Schrittspannung erreicht wird oder nicht. Oder ob eben dabei ein Lichtbogen entsteht, der auch indirekt zum Beispiel durch die Hitze gefährlich wird. Darin besteht übriges die Hauptgefahr für die Fahrzeuginsassen, dass das Fahrzeug wegen dem Lichtbogen in Brand gerät. Nur so als Hinweis auch Raupenfahrzeuge sind beim Einsatz unter/neben einer nicht ausgeschaltete und geerdeten 15kV Fahrleitung immer selber geerdet. Ganz Laienhaft gesagt; Man muss dem Strom immer einen besseren Weg (guter Leiter) zur Verfügung stellen so, dass er gar nicht auf die Idee kommt deinen Körper zu durchfliesen. Solange an deinem Körper keine Spannungsdiverenz anliegt, geht es dir wie den Vögeln auf der Hochspannungsleitung, es passiert nichts weil kein Strom fliesen kann. Und genau hier kommt eben die Spannung zum tragen, den; Je höher die Spannung desto grösser die Gefahr, dass trotz vorhanden sein eines gutem Leiters, ein Teil des Stroms über deinen Körper fleisst. Und es ist die Stromstärke die tötet, nicht die Spannung.--Bobo11 (Diskussion) 06:28, 13. Apr. 2016 (CEST)
1. „auch Raupenfahrzeuge [...] selber geerdet“: wie wird das technisch gemacht? wird da n Erdungskabel mit nem Hydranten und dem Raupenfahrzeug verbunden? oder reicht der Kontakt zum Boden? 2. das mit den „Vögeln auf der Hochspannungsleitung“ verstehe ich immer nich, denn: die müssten doch Kondensator-mäßig immer auf- und entladen werden, so dass n fetter Strom fließt... oder haben Vögel gut isolierte Füße? Menschen haben nur 1kΩ (also bei 230Veff (=625Vpp) sind 's 600mA, was wohl bereits zuviel ist...)... --Heimschützenzentrum (?) 10:29, 13. Apr. 2016 (CEST)
In Eisenbahnanlagen wird das Erdseil üblicherweise mit einer Schraubklemme an einer Schiene befestigt. --Rôtkæppchen₆₈ 16:11, 13. Apr. 2016 (CEST)
Rotkäppchen, wo du herkommst, hat die Eisenbahn dort Raupenfahrzeuge? --62.202.182.81 16:25, 13. Apr. 2016 (CEST)
Raupenfahrzeuge sind als Baumaschinen weit verbreitet. Diese Baumaschinen werden auch von Eisenbahninfrastrukturunternehmen und von diesen beauftragten Bauunternehmen zu Bauarbeiten an Eisenbahnanlagen verwendet. --Rôtkæppchen₆₈ 16:44, 13. Apr. 2016 (CEST)
Es gibt sogar Schienenfahrzeuge mit Raupenfahrwerk. So zum beispiel die MFS der Vanoli AG, hier auf dem linken Bild auf Raupen im Einsatz.--Bobo11 (Diskussion) 19:37, 13. Apr. 2016 (CEST)

Die sich bewegende Schaufelbaggerbaggerschaufel trennt das Kabel und fertig ist. Wenn der elektrische Kontakt mit der Schaufel bestehen bleibt, was sehr unwahrscheinlich ist, hat man entweder beide Drähte des Kabels erwischt und es gibt einen Kurzschluss, dann geht der Strom nicht durch den Bagger zur Erde. Oder man hat nur einen Draht erwischt, dann geht der Strom sicher nicht durch den Baggerfahrer hindurch (Wie auch? Von der einen Pobacke zur anderen?). Dass die Leitung noch nicht abgeschaltet worden sein wird wenn der Fahrer auf die Idee kommt abzusteigen, und dabei einen spannungsführenden Bagger und den Boden gleichzeitig berühren könnte, ist sehr unwahrscheinlich. --62.202.182.81 12:31, 13. Apr. 2016 (CEST)

„@noch nicht abgeschaltet“: Mittel-, Hoch- und Höchstspannungsleitungen sind wie oben bereits geschrieben gelöscht, also nicht geerdet. Im Falle eines Erdschlusses, z.B. durch Baggerbiss, wird die Leitung nur kurzzeitig abgeschaltet, um einen ggf. entstandenen Lichtbogen zu löschen und dann gleich wieder eingeschaltet. In der Leitwarte läuft dann eine Erdschlussmeldung auf und das war’s. Die Leitung kann bis zur Reparatur weiterbenutzt werden. --Rôtkæppchen₆₈ 12:47, 13. Apr. 2016 (CEST)
in meiner Vorstellung, fließt bei Berührung nur eines ehemals braun/schwarz ummantelten Leiters bereits bei 230Veff ein erheblicher Strom... wie beim Kondensator... immer auf und ab... 50× pro Sekunde... jede Sekunde... bis endlich die Sicherung das Elend beendet... --Heimschützenzentrum (?) 16:40, 13. Apr. 2016 (CEST)
Da sitzt du eben einem klasischem Irrtum auf. Wenn der Stromkreis nicht geschlossen ist, fliest gar kein Strom. Und solange du mit deinem Körper keinen eigenen Stromkreis bildst fliest auch kein Strom. Und zwar egal mit wie viel Volt du eigentlich Aufgeladen bist. Erst wenn eine Spannungsdiverenz besteht kommt es zu einer Entladung. Ob die gefährlich wird oder nicht ist sehr direkt von der Stromstärke abhängig.--Bobo11 (Diskussion) 19:00, 13. Apr. 2016 (CEST)
Hier haben sich die Sicherheitsanforderungen in den letzten Jahrzehnten verschärft. Früher galt eine elektrische Betriebsstätte mit isolierendem Fußboden als schutzisoliert und Betriebsmittel dort mussten nicht schutzgeerdet sein. Es kam vereinzelt aber dennoch zu Stromunfällen, wenn der Verunfallte nämlich vorher wässrige Flüssigkeiten verschüttete und so der Fußboden einen großen Kondensator gegen Erde darstellte. Beim Berühren einer stromführenden Leitung stellt der Verunfallte einen Stromkreis mit kapazitivem Blindstrom her. Dieser kapazitive Blindstrom kann wohl fallsweise gefährliche Höhen erreichen. Im Falle der auf der Freileitung sitzenden Vögel kommt es ebenfalls auf die Kapazität zwischen Vogel und den anderen Leiterseilen (gelöschtes Netz) bzw Erde (Eisenbahnoberleitung) an. --Rôtkæppchen₆₈ 20:54, 13. Apr. 2016 (CEST)
Klar das mit dem kapazitivem Blindstrom kann ein Problem werden. Aber eben es auch hier eben das Problem, dass ein Spannungsdiverenz entsteh, und nicht die Höhe der Spannnung na sich. Aber die pysikalischen Gesetze sind eben auch so, dass je grösser die Spannung ist, desto wahrscheinlicher ist eine gefährliche Spannungsdifernez. Wenn bei 230 Volt eine Spannungsdiferenz von 1% endsteht, dann sind das eben nur 2,3 Volt, entsprechend schützt einem der Körperwiederstand noch vor einer zu hoher Stromstärke. Bei 15 kV sind 1% eben schon 150 V, und somit in einem Bereich wo der Körperwiederstand alleine keinen ausreichenden Schutz mehr vor zu hohen Strömen bietet, bzw. der Körperwiederstand ist für diese Spannung zu wenig hoch um den Stromfluss wirksam zu begrenzen. Deswegen ist die Erdung ja so wichtig, damit auf dem alternativen Leitungsweg -sprich dem menschlichen Körper- keine so hohe Spannung aufgebaut wird, dass auf ihn ein Strom fliest der gefährlich werden kann. Es gibt also immer zwei Schutzziele. Eines ist zu verhindern, dass überhaupt fleisst Strommfiessen kann (=Isolierung). Das zweite, dads wenn ein Strom fleist (infolge der Zerstörung der Isolierung), dass der so abgeleitet wird dass er gegen über der Umgebung keine weiteren Spannungsdiferenzen erzeugt. Und das geschieht eben durch eine ordunggemässe Erdung. Wenn die Gefahr der Annäherung an eine Strom führende Hochspannungsleitung besteht, erdet man die Gerätschaften so, dass gegenüber ihrer direkten Umgebung keine Spanunngsdiverenzen endstehen können. Bei der Bahn wird dafür die Baumaschiene eben an die Bahnerde gehängt, womit eben -im Fall der Fälle- gewährleistet ist, dass der Strom den direkten Weg ins Unterwerk findet. Eine gute Erde auch zur Folge, dass es auch wirklich ein satter Kurzschluss ist, und nicht als Lastspitze missinterpretiert werden kann. Ein satter Kurzschuss wiederum sorgt dafür, dass die Sicherungsmittel für Kurzschlüsse ansprechen, und nicht nur die für Überlast. Und diese Sicherungsmittel reagieren in der Regel um einiges schneller als Überlastsicherungsmittel. Wo wir eben bei einen weiter wichtigen Punkt sind. Neben der Stromstärke ist eben auch die Wirkdauer ausschlaggeben, ob es gefährlich wird für den menschlichen Körper oder nicht.--Bobo11 (Diskussion) 22:09, 13. Apr. 2016 (CEST)
nochmal: man stellt sich vor, dass die eine Kondensator-Platte der Boden ist und die andere Kondensator-Platte ein Metall-Teil im Bagger: dann wird der Kondensator doch ständig ge- und entladen... oder: wer möchte schon auf Gummisohlen an einen Lichtschalter greifen, in den ein Wassertropfen getropft ist, weil der komische Landrat den in der Nähe eines Wasserhahns zu haben beliebt? ich nich... was das mit Blindstrom zu tun haben soll, weiß ich auch nich... --Heimschützenzentrum (?) 02:12, 14. Apr. 2016 (CEST)
Da im vorstehenden Artikel sehr viele Eventualitäten angeführt sind, möchte ich ergänzend noch auf folgenden Sachverhalt hinweisen: Wenn ein Bagger, egal ob mit Gummireifen oder Raupenfahrwerk, die Olympiaschanze in Garmisch-Partenkirchen hinabfährt, ist er vom Verlassen des Schanzentisches bis zum Aufschlag in der Aufsprungbahn NICHT geerdet. Es ist daher während der Flugphase besondere Vorsicht hinsichtlich eines Stromschlages geboten! Dies bitte ich zu bedenken. LG --79.225.105.3 15:41, 16. Apr. 2016 (CEST)
Da hier ziemlich viel wirres Zeug geschrieben wurde, sehe ich mich veranlaßt, das mal ein bißchen geradezurücken. Das Gefährliche an einem Stromunfall ist die elektrische Durchströmung des Körpers, oder auf Deutsch gesagt: Strom, der durch den Körper fließt. Von der Größenordnung her sind Ströme im Bereich weniger Milliampère (mA) fühlbar bis unangenehm, einige zehn mA schmerzhaft und ab ca. 100 mA gefährlich, in doppelter Hinsicht: Der Anteil des Stroms, der durch das Herz fließt, kann es aus dem Takt bringen (Arrhythmien, Kammerflimmern) und dadurch einen plötzlichen Todeseintritt herbeiführen, und der durch die Muskulatur fließende Strom läßt diese verkrampfen und führt dazu, daß ein umfaßter spannungsführender Gegenstand nicht mehr losgelassen werden kann und auch Helfer Probleme damit haben, den Verunfallten davon zu trennen. Für eine Durchströmung ist eine Potentialdifferenz zwischen der Ein- und der Austrittsstelle des Stroms am Körper erforderlich, ihre Höhe hängt von der Differenzspannung und dem wirksamen Körperwiderstand - i. a. einige Kiloohm, unter ungünstigen Umständen (feuchte Haut, großflächige Kontaktstellen) auch darunter - ab. Der kapazitive Verschiebungsstrom ist hingegen i. a. gering: Ein Gegenstand mit ca. einem Zentimeter Durchmesser hat gegenüber "unendlich" eine Kapazität von etwa einem Picofarad, entsprechend weist der menschliche Körper rund 0,1 nF Kapazität auf; da bedarf es schon ziemlich hoher Spannungen, bis dadurch nennenswerte Stromstärken zustandekommen.
Es kann durchaus vorkommen, daß eine Baumaschine bei der Arbeit in leitenden Kontakt mit spannungsführenden Leitern kommt und selbst hinreichend gut isoliert steht, daß sie dadurch keinen Kurz- oder Erdschluß herbeiführt. Dann steht die Maschine zunächst einmal selbst unter Spannung. Das führt für die Insassen nicht direkt zu Schäden, weil an ihnen i. a. zunächst einmal keine Spannungsdifferenz auftritt und damit auch keine gefährliche elektrische Durchströmung. Es gibt dabei aber zwei Probleme: Wenn ein Insasse auszusteigen versucht, kann er dabei mit Händen oder Füßen die Potentialdifferenz zwischen spannungsführender Maschine und geerdeter Umgebung überbrücken und sich so elektrisieren, und zum anderen kann, besonders bei Hochspannungsanlagen, der vergleichsweise geringe Ableitstrom durch die isolierende Aufstellung in Verbindung mit der hohen Spannung diese so stark erwärmen, daß sie zu rauchen und zu brennen anfängt, z. B. die Reifen der Maschine. Dafür reicht auch schon der in gelöschten Netzen verbleibende Reststrom aus. Der beginnende Brand kann dann Insassen dazu zwingen, die spannungsführende Maschine verlassen zu müssen.--80.171.153.232 14:42, 17. Apr. 2016 (CEST)
ach je... 1. „Stroms, der durch das Herz fließt“, „verkrampfen“: das gilt wohl nur für niederfrequenten Wechselstrom...
logo - darum geht es ja wohl, niwohr?
Gleichstrom und hochfrequent ist anders... aber auch eklig... stromunfall... 2. in nen 100pF Kondensator können sogar 10A fließen...
Schon mal mit dem Realitätscheck versucht? 100 pF haben an 50 Hz ca. 32 MOhm - für 10 A braucht man da also schlappe 320 MV - wo kommen die denn vor? Nuhr?
bei ≈1kVpp (also Steckdosen-Spannung) und 1kΩ kommen wir also leicht auf 200mA... deswegen möchte ich auch nich in Gummistiefeln auf dickem PVC-Boden stehend den braunen respektive schwarzen Draht berühren... auch wenn's Dr. Landrat egal respektive recht ist...
Auf PVC-Boden stehend an 0,4-kV-Außenleiter anpacken passiert genau gar nichts - selbst ausprobiert im VDE0100-Lehrgang. Ist aber nichts für plan- und ahnungslose Chaoten-Persönlichkeiten: Strom macht nämlich klein, schwarz und häßlich - mal merken, das!
3. also sollte man am Besten Drohnen einsetzen... --Heimschützenzentrum (?) 23:33, 17. Apr. 2016 (CEST)
Gute Besserung!--80.171.153.232 01:02, 18. Apr. 2016 (CEST)
1. „Gute Besserung“: WP:KPA? 2. „32MOhm“: vllt stimmt ja mit dem Modell vom Menschen was nich... son Körper ist ja nich einfach nur n Stück Kohle... nur weil die Spannung relativ langsam (20msec) von -325V auf +325V wechselt, heißt das ja nich, dass die Umladungen der diversen Strukturen ohne Ströme erfolgt... 3. „PCV“[sic!]: kann ich mir gar nich denken... da hilft es auch nix, wenn da zusätzlich zum PCV[sic!]-Boden noch ne Plan-volle/Ahnungs-volle/unchaotische Persönlichkeit am Werk ist... also: erst reicht n PCV[sic!]-Boden und dann kommen noch diverse Nebenbedingungen (VDE0100), die wohl genauson Unfug sind... früher hieß es mal, dass man in der Nähe vom Wasserhahn keine Steckdose/Lichtschalter haben will... --Heimschützenzentrum (?) 02:16, 18. Apr. 2016 (CEST)
Könnten Sie an Ihren persönlichen Problemen bitte unter Inanspruchnahme dafür qualifizierter Fachkräfte arbeiten? Hier ist dafür nämlich nicht der geeignete Ort, und es trägt zum Thema auch nicht bei, wenn Sie hier absonderlichen Privattheorien frönen. Danke für Ihr Verständnis. Die Tür finden Sie selbst?--80.171.153.232 07:49, 18. Apr. 2016 (CEST)
das liegt doch nich an mir, dass das alles keinen Sinn ergibt... Bsp: reicht nun also, dass Eintritts- und Austrittstelle gleich sind (insb. wegen isolierendem Untergrund)? oder muss man dabei noch ne bestimmde Geiteshaltung haben, damit die angeblichen „±10µA“ keinen Einfluss haben? LOL da stimmt doch wieder was nich... --Heimschützenzentrum (?) 09:35, 18. Apr. 2016 (CEST)
Er findet sie offenbar nicht. Wäre mal jemand mit den entsprechenden Rechten so freundlich, ihn zum Ausgang zu begleiten, aber ganz rasch?--80.171.153.232 14:05, 18. Apr. 2016 (CEST)
1. sonst was? 2. kann mal jmd n toten Froschschenkel auf ne Glas-Scheibe legen und dann n braunen Draht dranhalten? sollte mich sehr wundern, wenn da bloß ≤±10µA fließen... --Heimschützenzentrum (?) 14:29, 18. Apr. 2016 (CEST)

Begriffliche Unklarheit

Welche Bezeichnung ist für die Beschreibung einer Grundlagen- und Klinik-verknüpfender Medizin korrekt, bzw. wieso ist die jeweils andere nicht korrekt? Es geht um die beiden Begriffe translational und translatorisch. --77.180.24.48 21:53, 14. Apr. 2016 (CEST)

In der translationalen Medizin geht es um Verknüpfung oder auch Übertragung, wie Du es zutreffend beschreibst. Translatorisch bezieht sich dagegen auf eine gerichtete Bewegung, wie in Translation (Physik) beschrieben. Mehr dazu auch unter wikt:Translation. --84.62.226.132 22:54, 14. Apr. 2016 (CEST)
So, den Artikel mal in Form gebürstet.
"Translatorisch" findet man auch (von reputablen Institutionen), aber seltener. So ganz abweging halte ich es nicht, da ja von hier (Labor) nach da (Klinik) auch lokal gerichtet "übertragen" wird. Aber es wäre natürlich nett, wenn sich die implizierten Wissenschaftler auf einen Terminus einigen könnten...
<Stichelspassmodus> Dass da vor allem Mediziner drinstecken, kann man daran sehen, dass sie darauf bestehen, den Begriff groß zu schreiben. </Stichelspassmodus> Hehehehe! Play It Again, SPAM (Diskussion) 09:24, 15. Apr. 2016 (CEST)
"Übertragen" aber schon im übertragenen Sinne, darum eher nicht mechanistisch zu verstehen. Just my 2 cents;) --94.219.9.168 10:04, 16. Apr. 2016 (CEST)
Nein, direkt verstanden. Medizinische Forschungslabors sind in den meisten Fällen logischerweise von Patientenbetten getrennt.
Und selbst wenn sie im selben Gebäude sind, Beispiel MGH, liegen sie auf verschiednen Etagen. Play It Again, SPAM (Diskussion) 13:09, 17. Apr. 2016 (CEST)
Hömma, Opi, sowas geht heutzutage über rechnergestützte Netzwerke. Da muß kein Bote mehr geschickt werden;) --94.219.120.56 10:01, 18. Apr. 2016 (CEST)

Impressum bei Inseraten/Profilen von Prostituierten

Hallo, die meisten werden mit dem Thema sicherlich überhaupt nichts zu tun haben und ich bin auch nur aus Recherchelust und Wissensgier darauf gestoßen, aber im Internet gibt es Websites, auf denen Prostituierte ihre Dienste anbieten und inserieren können. Nun sind das Prostituierte und wollen für ihre Dienste logischerweise Geld sehen. Damit hat das was Geschäftliches/Gewerbliches. Aber müssten die Damen dann nicht auch ein Impressum angeben? Bspw. auf Facebook ist es ja so, dass wenn man dies geschäftlich macht, bspw. ein Handwerker hat dort ein geschäftliches Profil, dann müssen da auch die Angaben rein wie Name, Adresse, usw (bwz. ein Link dahin). Ist das bei Prostituierten nicht so, juckt das einfach keinen oder was übersehe ich? --87.140.195.1 00:58, 17. Apr. 2016 (CEST)

Relevante Rechtsquellen sind das Telemediengesetz und der Staatsvertrag für Rundfunk und Telemedien, hier z.B. § 6 TMG und § 55 RStV. --Rôtkæppchen₆₈ 01:23, 17. Apr. 2016 (CEST)
Siehe Impressumspflicht#Informationspflichten_nach_dem_Telemediengesetz, wobei „Entgelt“ m.W. in der Praxis sehr weit ausgelegt wird, es genügt schon ein Werbebanner auf einer ansonsten rein privaten Homepage. Ich vermute, das von dir beobachtete Phänomen ist eine Grauzone nach dem Motto „Wo kein Kläger, da kein Richter“. Selbst für etablierte Web-Plattformen wie XING ist ja noch nicht abschließend geklärt, inwieweit dort nicht nur für den Betreiber der Plattform selbst, sondern auch für die einzelnen User-Profile (die ja im Fall von XING durchaus kommerziell genutzt werden, z.B. von Freiberuflern oder selbstständigen Gewerbetreibenden) eine Impressumspflicht gilt. Hinzu kommt, dass Erotik-Websites oftmals aus dem Ausland betrieben werden, wo bezüglich Jugendschutz usw. weniger strenge Vorschriften gelten als in Deutschland. Wenn sich das Angebot an deutsche Kunden richtet, gilt dann zwar auch die Impressumspflicht, ist aber in der Praxis nicht durchsetzbar oder abmahnfähig.--Mangomix 🍸 01:35, 17. Apr. 2016 (CEST)
Gut, der Websitebetreiber mag ja vielleicht im Ausland sitzen und ist so für die deutsche Justiz möglicherweise nicht greifbar. Das ändert aber ja nichts daran, dass es die Prostituierten die in Deutschland ihre Dienste anbieten (bspw. mit Angabe der Telefonnummer und der Adresse an der sie momentan besuchbar sind) erreichbar wären. Und darauf kommt es ja an. Facebook wird auch nicht aus Deutschland heraus betrieben und trotzdem müssen dort gewerblich Tätige auch ein Impressum (bzw. ein Link dorthin) anbieten.
Ich denke auch, dass es nach "wo kein Kläger, da kein Richter" funktioniert und eigentlich ein Impressum angegeben werden müsste. Aber das wäre quasi der Todesschlag für das Inserieren von sexuellen Dienstleistungen im Internet, denn kaum eine wird wohl ihren richtigen Namen und Wohnadresse angeben wollen. Ist das ein Argument (quasi das faktische Berufsverbot bei Impressumsforderung), so dass dieses eben nicht durchgesetzt (werden kann)? Welches Recht steht da höher? Denn andererseits würden Prostitutionsgegner dieses Impressumsproblem ja ausnutzen können und so Prostituierte zum Aufhören zwingen können... --87.140.192.1 02:06, 17. Apr. 2016 (CEST)
Im Ernstfall müsste die Sexarbeiterin einfahc nur eine Kapitalgesellschaft, z.B. eine 1-£-Ltd oder eine Unternehmergesellschaft (haftungsbeschränkt), gründen und die ins Impressum übernehmen: Anbieterkennzeichnung: „Menschliche Wärme“ Unternehmergesellschaft (haftungsbeschränkt), Postfach 666666, 66666 St. Wendel. --Rôtkæppchen₆₈ 02:55, 17. Apr. 2016 (CEST)
"ausländisches Internet...";o)--Wikiseidank (Diskussion) 13:42, 18. Apr. 2016 (CEST)

Polizisten: Äußeres

Seit wann dürfen deutsche Polizisten Bärte (auch sog. Drei-Tage-Bärte), Pferdeschwänze und Ohrringe tragen? Dies ist mir erst in den ca. letzten 10-15 Jahren aufgefallen. Es gibt bei Wikipedia den Artikel "Haarerlass" mit u.a. folgendem Inhalt: Bei den Polizeien von Bund und Ländern waren Bart- und Haartracht teilweise durch Erlass geregelt. In Nordrhein-Westfalen wurde 1972 ein Erlass eingebracht, der aber 1980 wieder abgeschafft wurde.[Link nicht mehr verfügbar] Eine Wiedereinführung bei der Bundespolizei wurde im Januar 2006 durch den Bundesminister des Inneren Wolfgang Schäuble angeregt. Schön und gut, aber was dieser Erlass aussagt steht dort nicht. Es gibt auch keinen Link, noch konnte ich online etwas finden. Weiss jemand etwas?--87.184.131.234 12:13, 16. Apr. 2016 (CEST)

Es gibt wohl ein Bundesverwaltungsgerichtsurteil von 2006 (Bericht im lawblog) wo etwas Ähnliches entschieden wurde. Tenor: Persönlichkeitsrechte des Polizisten dürfen nur mit guten Gründen eingeschränkt werden. Dieser Trend dürfte sich die nächsten 10 Jahre fortgesetzt haben. Ergänzung: Link zum Urteil Gruß, --Nfreaker91 12:44, 16. Apr. 2016 (CEST)
Vielen Dank. Also stimmt meine Beobachtung mit den Haaren. Für die 1970er und -80er auch eher unvorstellbar. Geschweige den davor. Da scheint Deutschland heute diesbezüglich weitaus liberaler zu sein. Kann ich mir für das restliche Europa (außer vielleich Niederlande oder Skandinavien) oder USA nicht vorstellen. Was die Bartlänge oder- form angeht, gibt es sicher auch Regelungen in D. Ob es dann den Ausdruck von Autorität (Drei-Tage-Bart) beieinflusst, ist eine andere Geschichte ;-)--87.184.131.234 13:13, 16. Apr. 2016 (CEST)
In den 70ern waren die Haare bei Männern sehr lang und der BMVg hatte ein Einsehen und erlaubte Haarnetze für Männer. --Heletz (Diskussion) 13:17, 16. Apr. 2016 (CEST)
Ok. Wobei, ich glaube, dass die mit den richtig langen Haaren, nicht zur Polizei gegangen sind. Zumindest nicht freiwillig...--87.184.131.234 14:13, 16. Apr. 2016 (CEST)
Gab es damals schon Polizisten, die undercover gearbeitet haben? Die würden kurzhaarig und glattrasiert in bestimmten Milieus ziemlich auffallen.--Optimum (Diskussion) 14:57, 16. Apr. 2016 (CEST)
Ich dachte eher an die Hippies. Als solcher ging man sicher aus Überzeugung nicht zur Polizei. Es sei denn, wie gesagt, man wurde (unfreiwillig) zur Wache gebracht...--87.184.131.234 15:24, 16. Apr. 2016 (CEST)
Ich glaub auch damals was nicht jeder Mann mit langen Haaren ein Hippie. --MrBurns (Diskussion) 16:04, 18. Apr. 2016 (CEST)
+1, z.B. auch Rockmusiker, und gaaanz viele "Wannabees" aller Art ... -- Zerolevel (Diskussion) 12:06, 19. Apr. 2016 (CEST)
dabei haben die wannabe-hippies doch ganz kurze haare :-) -- southpark 12:08, 19. Apr. 2016 (CEST)
und das mich wirklich verstörende: das video war näher an der hippiezeit als an heute. -- southpark 12:09, 19. Apr. 2016 (CEST)

Morgen werden in Zürich seltsam gekleidete Herren der lokalen Bourgeoisie um einen Scheiterhaufen reiten, in dem eine Sprengladung versteckt ist. Dabei wird ein Marsch gespielt, zu dem im Sommer 1945 die Rote Armee an Stalin vorbeidefiliert ist. Wie sagt man nun der Kleidung, welche die Herren an diesem obskuren Anlass tragen? Ein Frack ist es nicht. --62.202.182.81 13:49, 17. Apr. 2016 (CEST)

 
Umritt, Sechseläuten 2007
Meinst Du diese Uniform, die die Reiter rechts im Bild tragen? --Blutgretchen (Diskussion) 14:03, 17. Apr. 2016 (CEST)
Ich meine das graue Gewand mit Blume im Knopfloch, dazu Zylinder. --62.202.182.81 15:08, 17. Apr. 2016 (CEST)
Hast du wirklich schon alles aufgezählt, was dich an diesem Anlass stört? Laut Photos tragen die Teilnehmer ganz unterschiedliche Trachten, unter anderem auch Fracks. Was meinst du genau, vielleicht ein Justaucorps? --King Rk (Diskussion) 14:07, 17. Apr. 2016 (CEST)
Mich stört nichts an diesem Anlass, jedem Tierchen sein Pläsierchen, die Frage ist wie man dem Kleidungsstück sagt. --62.202.182.81 14:10, 17. Apr. 2016 (CEST)
Der Sechseläuten-Marsch ist nichts anderes als der preußische Marsch der Freiwilligen Jäger aus den Befreiungskriegen. Dieser widerum geht auf den russischen Yegersky-March zurück. Der Kanton Neuenburg war Schweizer Kanton und preußisches Fürstentum, ich vermute, so kommt es zu der Verbindung, weiß es aber nicht genau. Schön ist der Marsch ohne Zweifel. Mit der Roten Armee hat er erstmal nichts zu tun. --Heletz (Diskussion) 14:47, 17. Apr. 2016 (CEST)
Jetzt hätte ich doch glatt vergessen, zum Zürcher Sechseläuten-Marsch zu verlinken, tstsss --Heletz (Diskussion) 08:31, 18. Apr. 2016 (CEST)
Eben doch. --62.202.182.81 15:00, 17. Apr. 2016 (CEST)
Den Marsch gab es schon vor 1815, mit der Roten Armee hat er erstmal nichts zu tun. --Heletz (Diskussion) 15:11, 17. Apr. 2016 (CEST)
Die sture Antwort hat mit der Frage erstmal nichts zu tun. --62.202.182.81 15:13, 17. Apr. 2016 (CEST)
 
Puh, ich musste lange suchen, bis ich unter all den Kostümierten welche mit Zylinderhüten gefunden habe. Das sollen Angehörige der Schneider-Zunft sein und die tragen durchaus Fräcke, allerdings in verschiedensten Farben. Nur die Zylinder sind durchgängig grau. Geoz (Diskussion) 17:38, 17. Apr. 2016 (CEST)
Hmmm, die Schneider reiten aber nicht. In der Eingangsfrage ging es um die seltsam gekleideten Herren der lokalen Bourgeoisie, die um einen Scheiterhaufen reiten. --Blutgretchen (Diskussion) 18:11, 17. Apr. 2016 (CEST)
Die Schneider reiten auch und viele andere Zünfte tragen auch Zylinder. Dazu trägt man meistens Frack. Aber auch Cutaway wäre möglich. Ich fürchte, man kann die Frage so nicht beantworten, wir müssten ein Foto haben. --King Rk (Diskussion) 18:49, 17. Apr. 2016 (CEST)
Bei Zunft (Zürich) sind alle versammelt, vielleicht wird der Interessierte da fündig (ich hab grad keine Lust, alle durchzufieseln). --Heletz (Diskussion) 20:00, 17. Apr. 2016 (CEST)
Auf Commonbs:Sechseläuten gibts Hunderte von Fotos dazu. -- Zerolevel (Diskussion) 11:57, 19. Apr. 2016 (CEST)

Bewertungskriterien in der Flüchtlingspolitik

Gerade eben erklärt mir Monitor in der ARD, die deutsche Bundesregierung ist so zufrieden mit dem EU-Türkei Abkommen, daß jetzt als nächstes die Flucht aus diversen (nord-)afrikanische Ländern (Libyen, Eritrea, Äthiopien, Somalia) in die EU unterbunden werden soll. Sowas geht natürlich nur unter der Voraussetzung, daß man man diese Länder zu sicheren Herkunftsländern erklärt. Wenn ich nun die Situationen in diesen Ländern bspw. vergleiche mit den Begründungen der USA für deren militärische Interventionen im mittleren Osten, dann stellt sich mir eine Frage:

Nach welchen Bewertungskriterien wird unterschieden zwischen sicheren Herkunftsländern, aus denen keine Flüchtlinge aufgenommen werden und solchen Ländern, in denen zum Schutz der Menschenrechte militärische Interventionen des Westens zu erfolgen haben? --84.62.226.132 22:46, 14. Apr. 2016 (CEST)

Das sind politische Entscheidungen, denen naturgemäß häufig wirtschaftliche Interessen zugrunde liegen. Um die Zahl der möglichen Asylbewerber und damit der Flüchtlinge möglichst gering zu halten (und weil man - entgegen aller Beteuerungen - befürchtet, die könnten die öffentlichen Kassen und die Wirtschaft zunächst einmal belasten statt sie zu bereichern), erklärt man möglichst viele Staaten zu sicheren Herkunftsländern. Der Schutz der Menschenrechte spielt dagegen eine weit untergeordnete Rolle - wenn überhaupt, dann die eines Feigenblatts. Militärisch interveniert wird vor allem dort, wo wirtschaftliche Interessen (Rohstoffe) tangiert sind. Die Auswirkung auf das Machtgefüge in einer Region werden meist nicht durchdacht (zumindest macht es den Anschein). --Snevern 23:27, 14. Apr. 2016 (CEST)
"Rein politische Entscheidungen" ist sicher richtig, aber: Letztlich ist das für die Frage ob weiterhin welche kommen wollen oder nicht wenig relevant, denn: Wer es nach Deutschland oder Schweden schafft, wird dort reichlich bezahlt, und dieser Anreiz besteht weiterhin. Die diversen Abwehraktionen erhöhen nur die Kosten für die Reise und schränken den Kreis derer die es sich leisten können eben ein. Momentan ist es ja so dass die Preise stark gefallen sind wegen der etablierten Fluchtrouten, die sollen wohl wieder erhöht werden.--Antemister (Diskussion) 23:57, 14. Apr. 2016 (CEST)
"wird dort reichlich bezahlt" - da habe ich aber ein bisschen was anderes gehört und auch selber erlebt. Oder würdest du für 85 bis 143 Euro im Monat dein Leben riskieren?--141.30.184.50 17:16, 15. Apr. 2016 (CEST)
"Bezahlt" ist eine menschenverachtende Verzerrung der Sachverhalte bis zur Unkenntlichkeit. Einfach unerträglich. --2003:45:4643:F500:6DCA:B258:A44B:697B 20:11, 15. Apr. 2016 (CEST)
Zu Sinn, Effektivität und Effizienz der Maßnahmen habe ich mich gar nicht geäußert. Hab' ich auch nicht vor. --Snevern 06:43, 15. Apr. 2016 (CEST)
Bitte die Fragestellung beachten: Es geht um Bewertungskriterien zur Situation in den jeweiligen Ländern und nicht(!) um die Frage, ob weiterhin welche kommen wollen oder nicht. Das wäre eine andere Frage für die ggf. ein eigener Thread zu eröffnen wäre. --84.59.22.29 12:05, 15. Apr. 2016 (CEST)

Ist natürlich alles deutlich zu komplex für so eine kurze Auskunft. Aber um ein Beispiel zu nennen: die verkündete "Rote Linie" in Syrien waren Chemiewaffenangriffe gegen die eigene Bevölkerung und auch in Libyen war die Lage so, dass die eigene Luftwaffe Innenstädte angriff - das scheint mir doch ein qualitativer Unterschied zur Marokko und Tunesien zu sein. -- southpark 19:35, 15. Apr. 2016 (CEST)

Aktuell geht es wohl um Libyen, Eritrea, Äthiopien und Somalia. Das scheint mir ebenfalls ein qualitativer Unterschied zu Marokko und Tunesien zu sein. Ich frage mich ein bißchen, ob es zukünftig so sein wird, daß der Westen munter Waffen in Krisengebiete liefert, nach Bedarf überall einmarschiert und militärisch interveniert, wenn es passend erscheint, während gleichzeitig die Zivilbevölkerung daran gehindert wird, das Land zu verlassen, damit die nicht in die EU flüchten und dort Asyl beantragen. Zumindest habe ich Monitor so verstanden, daß es jetzt vorrangig darum gehen soll, Menschen konsequent daran zu hindern, ihr Heimatland zu verlassen. Der Moderator hat am Ende des Beitrags noch angemerkt, daß es sich hierbei nicht(!) um eine Satire handelt. --84.59.22.29 21:15, 15. Apr. 2016 (CEST)
Es wird zukünftig so sein wie bisher, dass der Westen (aber nicht nur der) munter Waffen in Krisengebiete liefert, unter Umständen auch mal irgendwo einmarschiert (das wird nur seeeehr ungern gemacht: teuer und unbeliebt, kostet Menschenleben) und militärisch interveniert (schon wesentlich beliebter), wenn es passend erscheint. Allerdings wird die Zivilbevölkerung nicht am Verlassen von Kriegsgebieten gehindert - das wäre nicht nur unfein, sondern ist auch völkerrechtlich verboten (im Gaza-Streifen passiert's trotzdem). Wenn sie aber schon ihr Land verlassen, dann doch bitteschön nicht in Richtung Europa, ja?
Im übrigen wird natürlich unterschieden zwischen Flucht vor Krieg und politischer Verfolgung einerseits und vor Armut andererseits. Armut kann (theoretisch zumindest) ein Grund für einen mit humanitären Überlegungen begründeten militärischen Einsatz sein; ein Asylgrund ist es deswegen aber noch lange nicht. Das wäre ja noch schöner: Sollen die doch erstmal selbst ihr eigenes Wirtschaftswunder hinkriegen, genau wie wir, dann brauchen sie auch nicht unsere üppigen Zahlungen in Anspruch zu nehmen.
So kompliziert ist Politik gar nicht, stell' ich grad fest... --Snevern 22:22, 15. Apr. 2016 (CEST)
Naja, das mit dem leidigen Völkerrecht soll wohl in der Art umgangen werden, daß man kurzerhand behauptet, in dem Land gäbe es nichts, was eine Flucht rechtfertigen könnte, bevor man damit beginnt, die gesamte Zivilbevölkerung an der Flucht aus dem Land zu hindern. Zumindest erklärt Monitor das so und ich verstehe die Ankündigung von Merkel auch in dieser Art. --84.59.22.29 00:10, 16. Apr. 2016 (CEST)
Frau Merkel ist es vollkommen gleichgültig, ob 1 Million Menschen Somalia verlässt, um fortan in Äthiopien zu leben. Es geht sie auch nichts an, der deutsche Bundestag hat darüber auch nicht zu bestimmen. Erst wenn die Somalier hierherkommen wollen, wird die Sache für die Europäer interessant. Die Syrer haben die Europäer auch nicht interessiert, solange sie millionenfach in Lagern im Libanon, in der Türkei oder in anderen Nachbarstaaten dahinvegetierten. --Snevern 09:19, 16. Apr. 2016 (CEST)
Das ist Deine höchstpersönliche Meinung, hat aber nichts mit den Ankündigungen zu tun, auf die ich mich beziehe. Den Monitorbeitrag zum Türkeiabkommen und den nord-/ost-afrikanische Ländern kann man hier ab 19:20 in der ARD-Mediathek anschauen. --94.219.9.168 09:54, 16. Apr. 2016 (CEST)

Nochmals zur Ausgangsfrage: Formell ist es sicher so das ein Land als "sicheres Herkunftsland" eingestuft wird, wenn dort politisch wie wirtschaftlich einigermaßen geordnete Verhältnisse herrschen, sodass jeder der aus wirtschaftlichen oder politischen Gründen das Land auswandern will, regulär/ geordnet auswandern kann. Das zu definieren ist immer Ansichts- bzw. Ermessenssache, und aus Land aus dem ohnehin kaum einer Asyl beantragt bzw. diese keinen Ärger machen. Sicherer Herkunftsstaat (Deutschland) schreibt das auch in etwa so, dennoch hat niemand Länder wie Brasilien oder Malaysia auf die Liste gesetzt, weil die keine Rolle spielen. Den Beitrag von Monitor habe ich auch gesehen (das war nicht der erste zu dem Thema), allerdings ist diese ganze Politik ja völlig schizophren - da lädt man die Leute auf der einen Seite ein (schafft durch die hohen Geldleistungen Anreize zu kommen), und will sie dann mit hohem Aufwand wieder abwehren. (Sicher, die Lieferanten...). Die diversen Abmachungen mit Bashir, Efewerki & Co. - nicht nur hochgradige Verbrecher, sondern gleichermaßen unberechenbar und unzuverlässig - ist mir noch rätselhafter, die Typen sacken lassen sich aufwerten und schicken notfalls soagr noch zusätzlich Leute nach Europa. In welchem dieser Länder gab es denn westliche Militärinterventionen? @Snervern: Gerade Äthiopien ist ein Wirtschaftswunderland, seit Jahren unter den am schnellsten wachsenden Ländern[21], trotz einer Wirtschaftspolitik die unter aller Sau ist. Als in den 1980ern, während der großen Hungersnöte, dort wirklich bitterste Not herrschte, da kam keiner nach Europa. Die Leute waren ja arm. Die Verhältnisse dort bessern sich massiv, wie im übrigen in ganz Schwarzafrika seit zehn, zwanzig Jahren. Heißt: Eine immer größere Mittelschicht, die aber weiterhin nur in "prekärem Wohlstand" lebt, d. h. mangels lokalem Sozialsystem immer gefährdet ist wieder abzusteigen, bei Krankheit, Arbeitslosigkeit oder Wirtschaftskrise bzw. vor allem bei persönlichem Versagen. Und gerade im letzten Fall reicht dann das Geld, um im deutschen oder schwedischen Sozialsystem unterzukommen. In Schweden ist man sogar in gewissem Maße Stolz darauf, "humanitäre Supermacht" für solche Leute zu sein, etwa wie darauf in Deutschland "Exportweltmeister" zu sein, obwohl beides gigantische Summen kostet. @141.30: Die 85 bis 143 Euro im Monat gibt es am Anfang bei freier Kost und Logis, später folgen 400 € Hartz IV und eine moderne Wohnung. Das Durchschnittseinkommen in Syrien ist doch den Krieg von 600 € auf 100 €/Monat gesunken, und die Wohlhabenderen dort wissen, dass es auch noch dem Krieg lange so bleiben wird. Da ist Deutschland/Schweden ganz ganz attraktiv (mit Familie gibt es weit mehr Sozialhilfe als was in Syrien zu verdienen ist), zumal einige ja auch Arbeit finden könnten (aber gut, das lohnt sich mit Familie ja nicht). Lebensgefährlich ist der Trip inzwischen auch nicht mehr - auf jeden Fall wiegt das deutsche/schwedische Geld die Lebensgafahr auf. In der Türkei sind sie ja in Sicherheit, aber mit minimalem Lebensstandard.--Antemister (Diskussion) 10:59, 16. Apr. 2016 (CEST)

Wenn die Wirtschaft klein genug ist, sind hohe Wachstumsraten kein Problem - die Aussagekraft solcher Statistiken ist begrenzt.
@IP 94.219.9.168: An welcher Stelle des Beitrags wird gesagt, die Bundesregierung wolle die Einwohner der genannten Staaten am Verlassen ihres Landes verhindern?
Ansonsten bestätigt der Beitrag meine Meinung: Der deutschen Regierung (und nicht nur der deutschen) ist die humanitäre Lage in den genannten Staaten völlig gleichgültig. Beiträge wie dieser könnten die öffentliche Meinung dahingehend beeinflussen, dass sich die Politik ändert, auch wenn das Publikum von Monitor sicher zu klein dafür ist. Bislang kostet es aber mehr Wählerstimmen, Schwarzafrikaner ins Land zu bekommen (oder sie nicht abschieben zu können) als es kostet, ein korruptes, verbrecherisches und die Menschenrechte missachtendes System zu unterstützen. Das kann theoretisch jederzeit kippen, und dann ändert sich auch die Politik der Bundesregierung. Woran man sieht: Die Mitwirkungs- und Einflussmöglichkeiten der Bevölkerung und der Medien beschränken sich in einer Demokratie nicht auf ein Kreuzchen alle vier Jahre. --Snevern 13:35, 16. Apr. 2016 (CEST)
Hör mal genau hin ab 27:55. In Libyen wurde eine vom Westen gesteuerte Einheitsregierung installiert, welche "Migration" "eindämmen" soll; sagt Merkel. Anschließend wird das noch etwas erläutert. --94.219.9.168 15:11, 16. Apr. 2016 (CEST)
Zwischen der Nordküste von Libyen und der EU liegt nur das Mittelmeer. Das ist schwer zu kontrollieren. Es geht der EU und der Kanzlerin nicht um das Verlassen Libyens, sondern um die Einreise in die EU. Das und nichts anderes habe ich gesagt. Es interessiert keinen, wie viele in den Tschad oder in den Sudan wandern. Ich sehe da immer noch nichts, was meiner Meinung widerspräche - aber bitte: Ich beende das hier. Also: Du hast recht, und ich ein ruhiges Wochenende. --Snevern 16:35, 16. Apr. 2016 (CEST)s
Das ist entweder sehr naiv oder blanker Zynismus. Den Menschen bleibt dann im wesentlichen nur noch die Flucht von einem Krisengebiet ins nächste übrig, während die sicheren Länder, welche maßgeblich für diese Krisen verantwortlich sind, hermetisch abgeriegelt werden. --94.219.9.168 19:23, 16. Apr. 2016 (CEST)
In Äthiopien und Eritrea bahnt sich gerade wieder eine große Dürre an. Sollte der Westen nicht verpennen und ausnahmweise mal rechtzeitig helfen. Rainer Z ... 16:39, 16. Apr. 2016 (CEST)
@94.219.9.168: Ja, jetzt hast du es kapiert. Es ist naiv und zynisch. Aber wir haben keine Weltregierung, sondern (jedenfalls hierzulande) eine demokratisch gewählte Nationalregierung, die in allererster Linie an sich (und ein bisschen an das Land, für das sie zuständig ist) denkt, dann kommt lange nichts, dann kommen die Staaten, auf die wir angewiesen sind (Rohstoffe), dann kommt noch länger nichts, und dann kommt der Rest. Und, seien wir ehrlich: Es sind nicht nur die Politiker. Die Menschen indula ticken mehrheitlich nicht wirklich anders.
Ja, das ist kurzsichtig, naiv und zynisch - aber dennoch ist es so.
@Rainer: Der Westen pennt nicht, er schaut weg, bis er zum Hinschauen gezwungen wird. Es funktioniert nach dem gleichen Muster wie bei den Flüchtlingen: Solange es billiger ist (im Sinne von Steuergeldern und verlorenen Wählerstimmen), hungern die Leute dort. Sobald sich das ändert, ändert sich auch das Verhalten des Westens gegenüber humanitären Katastrophen. --Snevern 20:03, 16. Apr. 2016 (CEST)
Ist das Deiner Meinung nach eine angemessene Reaktion, einfach Wegschauen und schmollen? Hier ein paar Gedanken dazu. Die Menschen im Westen ticken auch gar nicht mal mehrheitlich so, sie fühlen sich bloß überfordert, weil ihnen immer wieder eingeredet wird, es sei ja alles sooooo kompliziert und sowieso alternativlos, Aufklärung über Sachverhalte ist auch gestrichen, weil das die Bevölkerung verunsichern würde. Wenn Du dann daherkommst und wegsehen propagierst, unterstützt Du das, was Du selber für falsch hältst, bloß um Deine eigene Passivität zu rechtfertigen. Nichtstun ist eine Sache, Nichtstun propagieren eine andere. --84.59.232.210 11:04, 17. Apr. 2016 (CEST)
Wie bist du denn auf dieses schmale Brett gekommen!? Immerhin hast du doch erkannt, dass ich "wegsehen" für falsch halte, ein Mindestmaß an Leseverständnis ist also vorhanden. Ich propagiere Passivität nicht, ich konstatiere sie lediglich, und ich selbst verhalte mich keineswegs passiv. Allerdings habe ich wenig Vertrauen in die Lernfähigkeit meiner Mitmenschen (bis hinauf in Regierungskreise), und versuche im Allgemeinen deshalb auch nicht, andere zu irgendwas zu überreden: Ich bin kein Don Quixote und kein Kreuzritter. --Snevern 15:33, 17. Apr. 2016 (CEST)
Wir sind der Westen. Wenn Du konstatierst, der Westen schaue weg, bis er zum Hinschauen gezwungen wird, dann klingt das so, als würde Dich das nicht betreffen. Vielleicht würde ein bißchen mehr Vertrauen in die Lernfähigkeit Deiner Mitmenschen dazu führen, daß diese Dir etwas lieber zuhören. Aufklärung ist ein zäher Prozeß. Das hat aber nichts mit überreden zu tun. Am ehesten erreicht man andere Menschen, wenn man ein Thema aufgreift, daß sie interessiert und dann ihre Begeisterung für weitere Aspekte dazu wecken kann. Zumindest funktioniert das allemal besser als die Methode mißlauniger Meckerpot;) --94.219.120.56 10:11, 18. Apr. 2016 (CEST)
Wow, du trägst zumindest zu meiner Erheiterung bei, dafür danke schön! Nicht, dass ich vorher schlechte Laune gehabt hätte - warum auch!? Aber jedenfalls gehört von uns beiden höchstens einer in die Kategorie mißlaunig + Meckerpot, und ich bin's jedenfalls nicht. so viel steht fest, denn das scheitert zumindest am Kriterium "mißlaunig".
Was die Lernfähigkeit angeht: Du hast recht, ich sollte eigentlich jedem eine Chance geben, und du wirst es nicht glauben: Das tue ich sogar. Jedem, dem ich begegne, als Mandant, Kunde, Auftraggeber, Geschäftspartner, Wähler (ja, ich stehe immer mal wieder auf Kandidatenlisten für demokratische Wahlen) oder wo immer ich Leuten begegne, die bereit sind, zunächst einmal zuzuhören. Das verstehen ist für den Anfang sicherlich meistens zu viel verlangt, drum verlange ich's nicht, und das ändern der eigenen Haltung überfordert selbst wirklich kluge Köpfe ganz oft. Anders als manche habe ich aber bis heute nicht aufgegeben, für meine Ideale zu kämpfen, zumal ich Vater bin und damit auch für die nächste Generation Verantwortung mittrage. Das alles hindert mich aber nicht, die Dinge auch mal beim Namen zu nennen, und in diesem Fall hier ist das eben leider so, wie ich es oben mehrfach beschrieb (und das kann ja jeder nochmal lesen, daher brauche ich es nicht zu wiederholen).
Nochmals danke für die Extraportion gute Laune, und auch dir einen schönen Arbeitstag (falls zutreffend)! --Snevern 11:18, 18. Apr. 2016 (CEST)
Schau, das meine ich. Erst Abwertung der Sache, dann Abwertung des Gegenüber und schönfärberische Selbstdarstellung. Die Selbstdarstellung ist mal wieder wichtiger als das Ergebnis in der Sache. Die Sache ist eher nur ein willkommener Anlaß für den Auftritt. --94.219.125.55 14:32, 20. Apr. 2016 (CEST)

@Snevern: Sicher erreichen nur niedrig entwickelte Länder hohe Wachstumraten, weil die Wirtschaft nur linear wachsen kann (heißt Gerschenkron-Effekt), aber dennoch liegt Äthiopien verglichen mit allen anderen Ländern an der Spitze. @94.219: Armen Kriegsopfern bleibt immer nur die Möglichkeit, in das nächstbeste sichere Land zu fliehen, die können sich nicht die tausendee Euro teure Reise nach Europa leisten. In solche Regionen will auch kaum ein Journalist reisen, zu umständlich, unbequem, gefährlich. Im Ostkongo gibt es Millionen Flüchtlinge, bitterarm. Kümmert in Europa niemanden. Zynisch, klar, aber seit jeher gelten die Reichen mehr als die Armen, auch im Krieg. @Rainer: Dürre in Äthiopien zerstört keine Mittelschicht. Für Flüchtlingsbewegungen irrelevant. Bzgl. Libyen ist zu sagen dass es sich hier um einen Sonderfall handelt: Gaddafi hat die die illegalen Einwanderer schon als Waffe eingesetzt, um europäische Regierungen zu erpressen, nach dem Motto "wenn ihr nicht spurt, dann schicke ich euch Millionen Afrikaner". Selbst wenn die europäischen Staaten tatsächlich Afrikaner abwehren wollten (und, sorry, das ist nicht gewollt, wäre es so dann würde jeder der illegal über die Grenze kommt ins Internierungslager gesperrt), dann würde deren Abschiebung viele Milliarden kosten, die vor wohl allem die Südeuropäer zu tragen hatten.--Antemister (Diskussion) 23:30, 19. Apr. 2016 (CEST)

Christi Himmelfahrt 2016

Hallo zusammen. Gerade festgestellt, dass Christi Himmelfahrt dieses Jahr auf Donnerstag, den fünften Mai stattfindet. Ich erinner mich, dass 2005 ebenfalls Christi Himmelfahrt an a) einem Donnerstag war und b) am 5. Mai war. Wie kann man berechnen, wie hoch die Wahrscheinlichkeit ist, dass genau beide Bedingungen erfüllt sind. Hintergrund ist, dass ich am 4. Mai Geburtstag habe und ich 2005 meinen Geburtstag ziemlich groß an einem Mittwoch feiern konnte. Jetzt elf Jahre später macht man das nicht mehr, aber die Erinnerung kam daran zurück. Grüße--93.193.214.101 21:51, 17. Apr. 2016 (CEST)

Du nimmst einen Osteralgorithmus, der den Rest Deines Lebens abdeckt, und berechnest daraus erst den Ostertermin und dann daraus den Termin für Christi Himmelfahrt, das ist immer 39 Tage nach Ostern. Danach einfach zählen. --Rôtkæppchen₆₈ 21:56, 17. Apr. 2016 (CEST) Ach ja, die Wahrscheinlichkeit für a) ist definitionsgemäß 100 %. --Rôtkæppchen₆₈ 21:59, 17. Apr. 2016 (CEST) Das nächste Mal fällt Christi Himmelfahrt 2157 auf einen 5. Mai. --Rôtkæppchen₆₈ 22:05, 17. Apr. 2016 (CEST)
Ach perfekt. In einem Zyklus von 5,7 Millionen Jahren genau 165.300 mal. Keine Sorge hab ich nicht selber berechnet, aber verstanden. Danke--93.193.214.101 22:15, 17. Apr. 2016 (CEST)
Zuvor (neuen Stils) 1622, 1633, 1644, 1701, 1712, 1785, 1796, 1842, 1853, 1864, 1910, 1921, 1932, 2005, 2016 (auch nicht selbst berechnet). --Pp.paul.4 (Diskussion) 22:33, 17. Apr. 2016 (CEST)

Wer mir ein Datum nennt, an dem Himmelfahrt nicht auf einen Donnerstag fällt, bekommt einen Extra-Punkt. --91.60.24.80 09:10, 18. Apr. 2016 (CEST)

91.60.24.80 hat recht. Himmelfahrt ist immer donnerstags (siehe Christi Himmelfahrt, Einleitung)--85.178.140.187 18:47, 18. Apr. 2016 (CEST)
Es ist schön, dass Du diese Tatsache jetzt in der vierten Formulierung hier angebracht hast. Aus den Beiträgen von gestern 21:56 und 21:59 meinerseits und von 91.60.24.80 um 09:10 sollte das bereits ersichtlich gewesen sein. --Rôtkæppchen₆₈ 00:12, 19. Apr. 2016 (CEST)
Zwei Blondinen lesen Zeitung. Meint die eine: "Heiligabend fällt dieses Jahr auf einen Freitag." Meint die andere: "Oh, hoffentlich nicht auf einen dreizehnten."--Optimum (Diskussion) 00:50, 19. Apr. 2016 (CEST)
Ostern fällt ja zufällig meistens auf einen Sonntag, nur dann nicht, wenn der Karfreitag auf Karsamstag verlegt wird, dann ist es am Ostermontag.--Giftzwerg 88 (Diskussion) 17:52, 19. Apr. 2016 (CEST)
Ostern wurde von Konzil von Nicäa auf den Sonntag nach dem ersten Vollmond im Frühjahr festgelegt. Dabei ist es bisher geblieben und so wie ich die diversen Kirchen und Denominationen kenne, wird sich niemals eine Mehrheit zu einer Änderung finden. --Rôtkæppchen₆₈ 21:22, 19. Apr. 2016 (CEST)
mein obiges Posting war nicht ganz ernst gemeint. ;-)--Giftzwerg 88 (Diskussion) 19:27, 20. Apr. 2016 (CEST)

Unter Hitler erlassene Gesetze, die heute immer noch gelten

Gibt es irgendwo eine Liste solcher Gesetze? --166.176.187.40 03:38, 16. Apr. 2016 (CEST)

Mir ist keine bekannt. Das liegt natürlich auch daran, daß Gesetze durch Novellierungen immer mal wieder mehr oder weniger überarbeitet werden, also nie "rein" so geblieben sind. Spontan fallen mir das die StVO, das Waffengesetz, das Rentengesetz und das Gesetz ein, das den 1.Mai zum Feiertag machte. --Heletz (Diskussion) 06:32, 16. Apr. 2016 (CEST)
Ach, ja: Das Bundesjagdgesetz natürlich auch. Wobei die Frage ist, ob eine solche Auflistung etwas bringt. Denn ein Gesetz ist ja nicht automatisch "nationalsozialistisch" nur weil es unter den Nazis erlassen wurde. Das StGB geht auf das Reichsstrafgesetzbuch von 1871 zurück und der jetzt diskutierte Mordparagraph (also nur ein winziger Teil davon) ist noch von den Nazis, was daran zu sehen ist, daß nicht eine Tat definiert wird, sondern einen Charakter ("Mörder ist ..."). --Heletz (Diskussion) 06:44, 16. Apr. 2016 (CEST)
Etliche Vorschriften im deutschen Luftverkehr gehen auf die Nazis zurück, z.B. die Flugleiterpflicht und die Pflicht zum Führen eines Flughauptbuches am Flugplatz (wer ist wann wohin abgeflogen – sollte einst die Flucht von Staatsfeinden erschweren). Ob es irnkwo eine Liste gibt, weiß ich aber auch nicht. --Kreuzschnabel 07:36, 16. Apr. 2016 (CEST)
Fragt sich halt immer, was man mit solch einem Wissen anfangen kann. Der Artikel Straßenverkehrs-Ordnung (Deutschland) nennt ein heute noch existierendes Zeichen (394), das unter den Nazis eingeführt wurde. Ob man sich als Parker unter solch einer Laterne jetzt unwohl fühlt? --Heletz (Diskussion) 07:43, 16. Apr. 2016 (CEST)
Eine Liste der während der Nazizeit neu und zusätzlich erlassenen Gesetze gibt es unter Historikern sicherlich, ansonsten wurden die aus Zeiten davor bestehenden Gesetze erweitert/ verändert. Soweit diese Gesetze und Änderungen nationalsozialistische (faschistische) Ziele verfolgten, sind sie aufgehoben oder (rück)geändert worden und haben in den 70 Jahren seit Beendigung des Naziregimes eine freiheitliche und demokratische Pflege erhalten.
Die Frage kommt aus USA: Nein, in Deutschland gibt es keine Gesetze mehr mit Nationalsozialistischen Gedankengut. Gott sei Dank. --2003:75:AF52:1C00:A9D8:F642:2E:22DC 07:51, 16. Apr. 2016 (CEST)

Ein Gesetz das heute nicht mehr gilt, aber sehr lange in der Bundesrepublik galt war z.B. das Rabattgesetz. Eine umfangreiche Liste von teilweise bis heute gültigen Gesetzen findest Du unter Kategorie:Rechtsquelle (Zeit des Nationalsozialismus) -- - Majo Senf - Mitteilungen an mich 07:54, 16. Apr. 2016 (CEST)

An die IP: Historiker halten Faschismus und Nationalsozialismus feine getrennt! Das ist nämlich etwas völlig Verschiedenes! --Heletz (Diskussion) 08:39, 16. Apr. 2016 (CEST)
Sicher nicht identisch. aber völlig (?) verschieden? --84.135.147.37 10:01, 16. Apr. 2016 (CEST)

Bis 2009 galt noch die Hausratsverordnung, das war ein Führererlass aus 1944, mitten im Krieg. -- 217.236.175.4 10:03, 16. Apr. 2016 (CEST)

+ 1 @ Heletz. Es fragt sich, welchen Erkenntnisgewinn solch eine Liste überhaupt liefern könnte. Das Tierschutzrecht wurde unter den Nazis stark ausgebaut (hauptsächlich verschärft, siehe: Tierschutz im Nationalsozialismus. Erst ab den 70ern wurde diese Gesetzgebung wieder überarbeitet (und zwar noch weiter verschärft). Wenn man da einen Trend konstruieren wollte, könnte man behaupten, die heutigen Tierschützer seien noch schlimmere Nazis, als die Nazis ;-) Andererseits haben die Nazis auch die lateinische Schreibschrift eingeführt. Wäre das allein schon ein Grund zur Current- oder Sütterlin-Schrift zurückzukehren? Geoz (Diskussion) 10:24, 16. Apr. 2016 (CEST)
Sogar noch weitergehend: Soll man den Umweltschutz und Tierschutz aufgeben, nur weil er von Nazis "erfunden" worden ist ...? --Heletz (Diskussion) 12:28, 16. Apr. 2016 (CEST)

Mitunter sind nach 1945 auch nur einzelne Begriffe ausgetauscht worden, um den Gehalt einer Vorschrift in ihr Gegenteil zu verkehren - so ist in den Gesetzen, die das Beamtenwesen in der Bundesrepublik regeln, davon die Rede, dass Beamte die "Gewähr bieten" müssen, dass sie für die "freiheitlich-demokratische Grundordnung" eintreten; 1933 hieß es im §4 des "Gesetzes zur Wiederherstellung des Berufsbeamtentums" noch, dass er die Gewähr bieten müsse, für den "nationalen Staat" einzutreten. Von daher stammt zumindest die Grammatik solcher Vorschriften 1:1 aus der NS-Zeit. --77.179.12.141 12:20, 16. Apr. 2016 (CEST)

Und weil die Grammatik im Deutschen nun ist wie sie ist ist der Beamteneid nationalsozialistisch ...? --Heletz (Diskussion) 12:28, 16. Apr. 2016 (CEST)
Er meinte sicher den Duktus. Pun intended. -- Janka (Diskussion) 15:31, 16. Apr. 2016 (CEST)
Was am Mordparagraphen tatsächlich (noch) nationalsozialistisch ist, erklärte schon 2013 der Vorsitzende des 2.Strafsenats beim BGH in seinem Artikel Völkisches Recht. --Heletz (Diskussion) 15:58, 16. Apr. 2016 (CEST)
Fischer seinerseits wurde kritisiert im Blogbeitrag von RA Thomas Fuchs Anmerkungen zu Fischers Kritik des Mordparagrafen --Heletz (Diskussion) 16:11, 16. Apr. 2016 (CEST)
Die Personalausweise gehen auch auf das 3. Reich zurück. Erst bei den Juden eingeführt dann beim Rest der Bevölkerung.--217.234.25.73 18:13, 16. Apr. 2016 (CEST)
Wenn das die Reichsbürger / Germaniten mitbekommen ...! --Heletz (Diskussion) 19:19, 16. Apr. 2016 (CEST)
Bei denen sind Fakten nicht so wichtig und können immer in die gewünschte Richtung um"argumentiert" werden. Deine Beispiele oben sind bis auf das StGB übrigens nicht zutreffend. Die wurden allesamt danach ordentlich vom Bundestag und Bundesrat beschlossen und die alten komplett aufgehoben. Natürlich wurden immer wieder einzelne Vorgängerregelungen unverändert übernommen. Man kann aber nicht ernsthaft behaupten, dass diese Gesetze aus der Nazizeit stammen würde. Zutreffender wäre wohl, das die erstmals gesetzliche Regeln in den Bereichen schufen. --Paestaweary (Diskussion) 22:09, 16. Apr. 2016 (CEST)
Aus diesem Grunde haben wir Historiker auch keine solche Liste, sondern diskutieren Genese und weitere Entwicklung je getrennt. Paestaweary (Diskussion, Du solltest bedenken, daß die Formulierung im EP nicht von mir stammt, sondern vom Fragesteller. Es ist mir nicht klar, warum Du die Beispiele als "meine" Beispiele bezeichnest. Nur naive Menschen kommen auf die Idee, ein Gesetz mit nationalsozialistischem Inhalt sei unverändert bis heute gültig. Der Spiegel insinuiert manchmal sowas. Und bei dem von mir angezogenen Mordparagraphen handelt es sich ja nicht um ein ganzes Gesetz, wie in der Fragestellung, sondern, wie von mir schon erwähnt, nur um einen winzigen Teil davon. Man hätte die Frage auch schlicht mit "Nein" beantworten können, was den Fragesteller aber wohl nicht befriedigt hätte. --Heletz (Diskussion) 08:30, 17. Apr. 2016 (CEST)
Hi Heletz, das war kein Vorwurf, aber Du hattest diese Beispiel genannt, deswegen das "Dein". Eigentlich sollten drei Unterscheidungen gemacht werden - wenn man auf ganze Titel abstellt: a) vor Hitler erlassen (StGB), b) in der Zeit erlassen (o.g. Hausratsverordnung) und c) danach erlassen - "Deine" Beispiele. Die StVO als Beispiel wurde in den 70er komplett neu erlassen, was vor allem bedeutet, das ausnahmslos alle Regelungen den grundgesetzlich vorgegebenen Prozess durchlaufen sind, während bei a) und b) u.U. nur einzelne Teile geändert wurden. Sehr wahrscheinlich wurden auch in der neuen StVO einzelne Passagen aus den Vorgängervorschriften übernommen, aber die muss man jeweils einzeln betrachten und kann eben nicht "die StVO" als Ganzes darunter führen. Daher wäre "Mord" sicher ein gutes Beispiel, aber nicht ganze Titel, außer vielleicht die Hausratsverordnung und einige wenige andere, die aber alle überarbeitet oder wenigstens überprüft wurden. --Paestaweary (Diskussion) 19:00, 17. Apr. 2016 (CEST)
@Paestaweary (Diskussion)Ist ok, wir meinen im Grunde das Gleiche! --Heletz (Diskussion) 21:50, 17. Apr. 2016 (CEST)

Es gibt sogar Gesetze, die erstmals im Sachsenspiegel niedergeschrieben wurden und die heute noch gelten, 800 Jahre später. Das mit Hitler ist nicht ungewöhnlich. -Pölkkyposkisolisti 13:35, 17. Apr. 2016 (CEST)

Informative Übersichten ohne Anspruch auf Vollständigkeit findest du hier und hier (besonders zum Strafrecht, neben dem Mord ist da die Nötigung Paradebeispiel). --Chricho ¹ ² ³ 16:38, 17. Apr. 2016 (CEST)

Die ZEIT widmet sich in einem aktuellen Artikel dem Naturschutz unter den Nazis samt der heute empfohlenen Anlage von Hecken für Vögel. --Heletz (Diskussion) 06:55, 18. Apr. 2016 (CEST)
Noch eine Ergänzung: Auf dem Bereich der Freikörperkultur wurde 1942 das Nacktbadeverbot der Weimarer Zeit gelockert. --Heletz (Diskussion) 21:26, 19. Apr. 2016 (CEST)

Ergänzen möchte ich noch Gesetze, die sich auf bis heute bestehende Grenzen von Städten, Landkreisen oder Ländern auswirken, wie das Groß-Hamburg-Gesetz von 1937 oder das Salzgitter-Gesetz von 1941. --91.221.59.29 14:00, 21. Apr. 2016 (CEST)

Tonhöhe

Das wir (wie das Wort ja schon sagt) Tonhöhen mit Höhe und Tiefe assoziieren, ist uns das kulturell durch den sprachlichen Ausdruck eingetrichtert, oder handelt es sich um natürlich veranlagte Synästhesie? Letzteres würde mich insofern nicht verwundern, als Gleichgewichtssinn und Gehör ja auch räumlich nah beieinander liegen. (nicht signierter Beitrag von 134.245.225.55 (Diskussion) 14:04, 16. Apr. 2016 (CEST))

Die Tonhöhe definiert sich durch die Anzahl an Schwingungen pro Sekunde (Hertz). Insofern ist das also eine physikalische Größe; 880 Hz ist nunmal mehr, ist höher als 440 Hz. Das hat mit "kulturell eingetrichtert" nichts zu tun - es sei denn man würde sagen wollen, dass 880 weniger als 440 sein soll. Warum wir zu größeren Zahlen "größer" und nicht "kleiner" sagen, dürfen dir Sprachwissenschaftler beantworten. Das ändert aber auch nichts dran, dass 880 Hz von der Bedeutung her nunmal mehr als 440 sind - selbst, wenn man es "kleiner" nennen will.
Was dagegen "kulturell" geprägt ist, ist die Bedeutung einzelner Schwingungsanzahlen. Z.B. ist es physikalisch nicht zwingend, dass der Ton, der 440 Hz hat, Kammerton heißt oder dass Stoffe, die Lichtwellen mit der entsprechenden Länge reflektieren "rot" genannt werden. Das ist schon kulturell, aber zwangsläufig ist auch diese Zuordnung nicht. Es gibt z.B. Menschen, die Farben hören können und da sieht man, dass es offenbar keine "feste Verdrahtung" einer Tonhöhe zu einer Farbe gibt. Synästhesie trifft es daher mMn nicht. --87.123.16.81 14:19, 16. Apr. 2016 (CEST)
 
Swinging Cladni (1756-1827)
Diametral positioniert: "Tonhöhe" wurde verwendet, ehe man wusste, dass da höhere Frequenzen eine Rolle spielen.
Der Begriff "Tonhöhe" wird in den meisten Sprachen - Altu..., ...alto, Haut..., 音高 etc. - mit "Höhe" verbunden. Das schreit quasi nach einer erfahrungsmässigen Erklärung (aus dem täglichen Leben) - die hier sicherlich auch noch kommen wird.
Cladni (1756-1827) ... aber schon 1674 gab es "hohe" und "tiefe" Töne.[1] Play It Again, SPAM (Diskussion) 14:44, 16. Apr. 2016 (CEST)
Hohe Töne kommen von oben, meinen die hier. Grüße Dumbox (Diskussion) 14:42, 16. Apr. 2016 (CEST)
yes yes yes ... YES YES YES  :-))) Das dort verlinkte paper ist jetzt im Artikel Tonhöhe...Play It Again, SPAM (Diskussion) 14:48, 16. Apr. 2016 (CEST)
(BK) +1 zu Play it, ich denke auch das die physikalische Erklärung nicht schlüssig ist. Denn genauso könnte man über die Wellenlänge argumentieren, hohe Töne haben aber eine kürzere Wellenlänge, 40cm ist "weniger" als 80cm, der Ton ist aber "höher". --Engie 14:49, 16. Apr. 2016 (CEST)

Interessant ist auch der Unterschied zur Lichtwahrnehmung: Dort erscheinen uns tiefe Frequenzen als rot und hohe als violett, was wir obendrein als benachbart im Farbkreis empfinden. Wenn das nicht noch erstaunlicher ist! Rainer Z ... 16:11, 16. Apr. 2016 (CEST)

Er hat Farbkreis gesagt...! Play It Again, SPAM (Diskussion) 18:30, 16. Apr. 2016 (CEST)
Andererseits liefert ein kühlerer Temperaturstrahler, z.B. eine Kerzenflamme, wärmeres Licht (1000–1700 Kelvin) als ein heißerer Temperaturstrahler, z.B. eine Halogenglühlampe (2800-3100 Kelvin). --Rôtkæppchen₆₈ 02:18, 17. Apr. 2016 (CEST)
"Das schreit" also, wie Play It Again, SPAM hier vorgestern geschrieben hat, "quasi nach einer erfahrungsmässigen Erklärung (aus dem täglichen Leben) - die hier sicherlich auch noch kommen wird." Und keiner hat mal kurz zwei Töne geschrieen. Einen ganz tiefen, und dann einen ganz hohen, so eben mal kurz, ganz einfach, Kopfstimme? Aber das ist auch nur Theoriefindung. --MannMaus (Diskussion) 16:18, 18. Apr. 2016 (CEST)
Gerade im Selbstversuch durchgeführt - und von schreckhaftem Kater gekratzt worden.
Kam beides aus derselben Öffnung gleicher Höhe (Mund). Play It Again, SPAM (Diskussion) 20:03, 18. Apr. 2016 (CEST)
Das hat wenig mit physikalischen Begriffen zu tun, denn so wurden Töne schon eingeteilt, lange bevor die Menschen eine Vorstellung von dieser Wissenschaft hatten. Ich vermute eher, dass das etwas mit dem Hervorbringen von Tönen zu tun hat - wie das besonders Sänger an sich selbst erfahren, wenn sie sowohl das eine als auch das andere versuchen. Tiefe Töne singt man aus dem Bauch - während hohe Töne eher aus dem Kopf kommen (Kopfstimme). Besonders Anfänger im Singen versuchen das auch körperlich, wenn sie sich bei hohen Tönen nach oben recken, während sie sich bei tiefen nach unten drücken. Chiron McAnndra (Diskussion) 14:59, 19. Apr. 2016 (CEST)
"Der Begriff "Tonhöhe" wird in den meisten Sprachen - Altu..., ...alto, Haut..., 音高 etc. - mit "Höhe" verbunden." - Ja, stimmt, zumindest, was die Beispiele betrifft, die ich verstehe. Bei denen handelt es sich aber um europäische Sprachen, also aus einem Kulturraume. Daraus erschließt sich also nicht, ob die Assoziation kulturell bedingt oder in der Natur des Menschen verankert ist. Aufschlussreicher könnte die Information sein, ob die Assoziation auch in Sprachen ganz anderer Kulturräume (etwa den Sprachen australischer Ureinwohner) so verbreitet ist. Wäre dies der Fall, hätte man ein deutliches Indiz für die Naturbedingtheit der Sache, und wenn nicht, für die kulturelle Prägung. Da ist dann halt jemand gefragt, der Sprachen aus unserer Sicht völlig exotischer Kulturräume beherrscht.--134.245.225.55 17:34, 19. Apr. 2016 (CEST)
Ich habe mir romanische, germanische, chinesische und slawische Sprachen angesehen. Das reicht mir erst einmal. Jeden neuen Erklärungsansatz, der mit einem Beleg präsentiert wird, werde ich mit Freuden annehmen und durchdenken.
"Die Position der Zukunft" (in visueller Wahrnehmung) ist auch in vielen Sprachen eindeutig (ausser in - wenn ich richtig erinnere - einer Ethnie, die die Zukunft hinten sehen/vermuten [es gibt meistens Ausnahmen]). Play It Again, SPAM (Diskussion) 22:14, 21. Apr. 2016 (CEST)

Einzelnachweise

  1. Erasmus Francisci: Die lustige Schau-Bühne von allerhand Curiositäten. - Nürnberg, Wolffg. Moritz Endter 1674. Wolffg. Moritz Endter, 1674, S. 35 (google.com).